Sunteți pe pagina 1din 494

MECHANICS OF

Composi e Structures

Smll:mlm ~.... ...... , ,., .,111Sl1111m


MECHANICS OF COMPOSITE STRUCTURES

An increase in U1e-use of c-0n1posite materials in n1any areas or engineering


has led to a greater deniand for engineers versed in the design of structures
1nade fro1n such materials. Although numerous books offe.r introduc.tions to
oontposites. few de1nonstrate advanced concepts or emphasize structures.
ll1i~ book addresses U1at need by offerit1g studenB and engineers tools for
designing practical contposite structures. l lte foc us is on fibe.r.re.i nforced coin
posites composed of fibers en1bedded in .a 1natrix. An1ong the topics of interest
to the des igner are stress-strain relationships for a \vide range of anisotropic
mate.rials; be.nding. buckling. and vibration of plates: bending, torsion, buck
ling, and vibration of solid as well as thill\\alled beams; shells: hygrothermal
slr.esses and strains: finite elen\e.n.t fonnulation: and failure criteria. "lbe ent-
ph:asis is on analyses that lead to 1nethod s applicable to a variety of structural
design proble.nts.111e expressions re.suiting fro1n Lhe analyses are e.i ther readily
tL~able or can be Lranslated into a 001nputer algorithn1. f\fore than 300 illustra-
tions, 50 fuJly wol'ked problents, and mat~.rial properties data s.ets are included.
$t)1ne kno,,ledge of 001nposites.. differential equaHons, and 1na1rix algebra is
helpful but not necessary, for the book is self.contained.
l ltis book \Viii be of great practic.aJ use to graduate students, researchers.,
and practicing e.ngineers se.eking to acquire advanced kno,vledge of Lhe
1nechanics of co1nposites and of the applications of composite 1naterials.

Ll.s216 P. Kolli1r is Professor in the Deparil1nent of Architecture-at the Budapest


University of rechnology and Econontics.

George S. Springer is Paul Pigott Professor of Engineering in the Deparunent


of Aeronautics and Astronautics at Stanford University.
C...Af\.IBRJDGE U NJYf!.RSl'rY PRESS
Cambridge, New York, Melbourne, Madrid, Cape Town, Singapore, Sao Paulo

Cambridge Universiry Press


The Edinburgh Building, Cambridge cs2 2Ru, United Kingdom
Published in the United Siares of America by Cambridge Uni\rsity Press, New York
~V\VW. c:an1bridge .o rg
l ntOnnarion on chis cide: \'>'\W'.cambridge.org/978052 1801652

Cambridge U niversity Press 2003

This book is in copyrighr. Subject to scarucory exception and co the provision of


rele\anr collective licensing agreemencs, no reproduccion of any parr n1ay take place
\virhout rhe written permission of Cambridge Universiry Press.

Firsc published in prinr formar 2003

SN IJ 9780 511 -05703- 8 eBook (Adobe Reader)


ISBN IO 0-511 05703- 2eBook (Adobe Reader)

SN IJ 978 0-521- 80165 2 hardback


ISBN IO 0 -521 -80165 6 hardback

C...an1bridge Universiry Press has no responsibility for the persiscence or accuracy of


URLS for excernal or chird ..parry incernet websites referred co in this book, and does not
guaranree chat any concent on such websires is, or will remain, accurate or appropriace.
Contents

!'reface page .xi


Li.<t ofSrmbol.i

1 Introduction
2 Dlsplac:ements, Strains, and Stresses '
2.1 Strain.-Displacetnent Relations 4
2.2 Equilibrium Equations 6
2.3 Stress-Strain Relationships 8
2.-'.I Generally Anisotropic Materilll s
23.2 Monoclinic Ma1erial JI
23.3 Or1ho1ropic Material I~
23.4 Tranovenely Isotropic Material 19
2.3.S Isotropic Material 20
2.4 Plllne-Strnin Conditjon 22
2..4. l Free End - Generally 1-\ nisotropic Ma terial 28
2.4.2 Free End - Monoclinic Material 30
2.4.3 Free End - O rthotropic. rra11s\e1sely lsotro1)ic.
or lso11opic f\<taterial ~
2.4.4 Buihln Ends - Generally A nisotropic l'\1ateriol 35
2.4.S Built.Jn Ends- Monoclinic Material 3()
2.4.6 Built-In Ends-Orthotropic, Traits\1ersely Isotropic,
or IJ01ropic 1'-1a teriaJ 3S
2.5 Plane-Strcss Condition .lb
2-6 Hygrc>1hermal Strains and Stresses 44
26.1 Plane-Strain Condltioo 47
26.2 Plane-Stress Condition 47
2.7 Boundary Condi1ions ~7
2.8 Con1inuity Conditions ~
2.9 Siress and StraJn Tramfornt.atioos 49
2.9. l Stress rransCormation 50
2~9 .2 Strain Transronnation 52
2.9.3 Tran.'l ronnation of the Stiffness and Contplinnce Mauices 53


vi CONTENTS

2.10 Strain Energy 55


2.10.1 111e Ritz Method 55
2.11 Summary 56
2. 11.1 Note on the C.Ontpliance and S tiffness Matrices 56
3 Lammated eomposlles <
3.1 Laminate Code 63
3.2 Stiffness f\.tatrices of Thin Laminates '>5
3.2.1 me Significance of the [A]. [BJ, and [ D] Stiffness Matrices 72
3.2.2 Stiffness fvtatrices for Selected Laminates 74
4 Thin Plates S9
4.1 Governing Equations 90
4.1.1 Boundary Conditions 92
4.l.2 Strain Energy 92
4.2 Deflection of Rectangular Plates 93
4.2.1 Pure Bending and In.Plane Loads 93
4.2.2 Long Plates 94
4.2.3 Simply Supported Plates- Symmetrical Layup LOO
4 .2.4 Plates \\rith Built~ln Edges -Orthotropic and
Symmetrical Layup 107
4.3 Buckling or Rectangular Plates l 12
4.3.I Simply Supported Plates- Symmetrical layup ll2
4.3.2 Plates\\itli Built-In and Sim ply Supported Edges- Ortbotropic
and Symmetrical Layup l 18
4.3.3 Plates with One Free Edge -Ortbotropic and
Symmetrical Layup 124
4.3.4 PJates \Vith Rotationally Restrained Edges - Orthotropic
and Symmetrical Layup 127
4.3.5 Long Plates l32
4.4 Free Vibration of Rectangular Plates 141
4.4.1 Long Plates l~J
4.4.2 Simply Supported Plates - Symmetrical Layup 144
4.4.3 Plates with Built-In and Simply Supported Edges- Orthotropic
and Symmetrical Layup 149
4.5 Hygrothermal Effects LSI
4.5.l Change in n\ickness Due to Hygrothermal Effects 161
4.6 Plates 'vith a Circular or an Elliptical Hole t63
4.7 lnterlan1inar Stresses l 66
5 Sandwich Plates I 69
5.J Governing Equations 170
5. 1.1 Boundary Co1tditions 171
5 ..1.2 Strain Energy 173
5.l.3 Stiffness l\tatrices of Sand\vich Plates l74
5.2 Deflection of Rectangular Sand,vich Plates 178
5.2.1 Long Plates 178
5.2.2 Sin1ply Supported Sand,vich Plates - Onhotropic and
Symmetrical Layup 182
CONTENTS vii

5.3 Buckling of Rectangular Sandwich Plate.s 185


5.3.l Long Plates 185
5.3.2 Simply Supponed Plates - Onhotropic and
Symmet rical Layup 187
5.3.3 Face Wrinkling 190
5.4 Free Vibration of Rectangular Sand\vich Plates 196
5.4.l Long Plates 196
5.4.2 Simply Supported Plates - Onliotropic and
Symmetrical Layup 199
6 Beams 203
6.J Governing Equations 203
6. 1.l Boundary Conditions 205
6. 1.2 Stiffness Matrix ~05
6. 1.3 Compliance Matrix 209
6.l.4 Replacen1e.nt Stiffnesses 210
6.2 Rectangular, Solid Beams Subjecte-d to Axial Load
and Bending 210
6.2. l Displacements -Sy1n1netric.aJ Layup 211
6.2.2 Displace1nents - Unsy1n1netrical Layup 213
6.2.3 Stresses and Strains 214
6.3 Thin-Walled. Open-Section Orthotropic or Syn1me.trical
Cross-Section Beams Subjec.ted to Axial Load and Bending 217
6.3.l Displace.ments ofT-Bean1s 217
6.3.2. Displacements of L-Bea1ns 221
6.3.3 Displacements of Arbitrary Cross-Section Beams 226
6.3.4 Stresses and Strains 233
6.4 1hin-Walled, Closed-Section Onhotropic Bea1ns Subjected to
Axial Load and Bending 243
6.5 l'b rsion of .Thin-\Valled Bea1ns 248
6.5.l Thin Rectangular Cross Section 248
6.5.2 Open-Section Orthotropic Bean1s 250
6.5.3 Closed-Section Orthotropic Beams - Single Cell 252
6.S.4 Closed-Section Orthotropic Beams- Multicell 200
6.5.S Restrained Warping - Open-Section Orthotropic Bea1ns 261
6.5.6 Restrained Warping - Closed-Section Orthotropic Beams 2~
6.6 Thin-Walled Bearns '"ith Arbitrary Layup Subjected to
Axial Load. Bending, and Torsion 265
6.6. L Displacen1ents of Open- and Cosed-Section Bean1s 267
6.6.2 Stresses and Strains in Open- and Cosed-Section Beatns 268
6.6.3 Centroid 271
6.6.4 Restrained \Val'ping 271
6.7 1ransversel)' loaded ll1in-\Valied Beanis 274
6.7. l Beams \vith Orthotropic Layup or '"ith Sym1netrical
Cross Section 276
6.7.2 Beains 'vith Arbitrary Layup 280
6.7.3 Shear Center 283
6.8 Stiffe ned Thin-\Valled Bean1s 28S
viii CONTEHTS

6.9 Buckling of Bean1s 290


6.9. L Beams Subjected to Axial Load (Flexura1-1o rsional
Buckling) 291
6.9.2 Latera1-rorsional Buc.kling of Orthotropic Beams with
Sy1nn1e.trical C...Toss Section 296
6.9.3 Loc~I Buckling 300
6.10 Free Vibration of Beams (Flexural-10rsional Vibration) 306
6.10.1 Doubl>' Symmetrical Cross Sections .iOb
6. 10.2 Bea1ns \\'itJ\ Synunetrical Cross Sections 309
6.10.3 Bea1ns \\'itb Unsymmetrical Cross Sections 309
6.11 Summary 3!2

7 Beams wtth Shear Deformatton 313


7.1 Governing Equations 314-
7.l. l Strain- Displac.e1ncnt Relationships J iS
7. 1.2 Force-..'\traio Relationships 315
7. L3 Equilibrium Equations J20
7. 1.4 Su1nn1ary of Equations 320
7.1.5 Boundary C..onditions 321
7.2 SriCfnesses and Con1pliances of Beaots 321
7.2. l Shear Stiffnesses a nd Co1nplia11ces of T hinWalled
Open-Section Beams 322
7.2.2 Shear Stiffnesses and Compliances ofll iinWalled
Closed-Section Bea1ns 325
7.2.3 S6ffnessesofSand\vich Beams 316
7.3 Transversely Loaded Beams 329
7.4 Buckling of Be.ams _;34
7.4.l Axially Loaded Bean\S \Vill\ Doubly Syn\melrical aoss
Sections (Flexural a nd Torsional Buckling) 335
7.4.2 Axially Loaded Bea1ns with Symn\etrical or Unsyn11netrical
Cross Sections (Flexural-lOrsional Buckling) 341
7.4.3 Latera1-rorsional Buc.kling of Beams \\'ith Sym1netrical
Cross Section 345
7.4.4 Summary 346
7.5 Free Vibration of Bean\s ~47
7.5. l Beams \\'ith Doubly Symme.trical Croos Sections 347
7.5.2 Beams \Vith Sy1nmetrical or lJnsymmetrical Cross Sections 356
7.5.3 Summary 359
7.6 Effect of Shear Deformation .359

8 Shells 365
8.1 Shells of Revolution with A.'\.'isynunetrical Loading 367
82 Cylindrica l Shells 368
8.2.1 Membrane lbeory 368
8.2.2 Built-In Ends 370
8.2.3 Temperature- Built-In E nds 379
8.3 Springback 380
S.3.1 Springbac.k of Cylindrical S hells 380
8.3.2 Doubly Curved S hells 384
CONTENTS ix

8.4 Buc,kling or Shells 384


8.4. l Buc.kling of Cylinders 387
9 Finite Bement Analysis 395
9.1 '111ree-Din1ensional Elen1e.nt 396
9.2 Plate E lement 3'17
9.3 Bea1n Eiement 397
9.4 Sublaminate 398
9.4. 1 Step 1. Elements of (J) due to l n-Pla neStresse.s 400
9.4.2 Step 2. Elements of (J) due to Out-of-Plane
Normal Stresse.s 40J
9.4.3 Step 3. Elements of (J] due to Out-0f-Plane Shear Stresses 405
9.4.4 Step 4. 'l'he Stiffness Matrix 407
10 Failure Criteria 411
10.J Quadratic Failure Criterion 413
10.'l.l Orthotropic f\.1aterial 4 14
I0.1.2 li'ansversely Isotropic ?vtaterial 420
10.1.3 Isotropic Material 421
10.1.4 Plane-Strain and Plane.Stress Conditions 4"'"
10.1.5 Proportional L.oadlng - Stress Ratio 423
10.2 "l\.1aximum Stress"' failure Criterion 425
10.3 ''Maxi1nutn Strain'' Failure Criterion 426
10.4 Plate with a Hole or a Notch 430
I0.4.1 Plate \Vith a Circular Hole 431
I0.4.2 Plate \Vith a Notch J34
I0.4.3 O \aracteristic Length 434
11 Mlcromechanlcs 436
J 1.1 Rule or ~tixtures 436
11.1.1 Longitudinal Young Modulus 1 438
11.1.2 liansverse Young ~1odulus Ei 439
11.1.3 Longitudinal Shear Modulus G12. 439
11.1.4 'li'ansverse Shear ModuJus (;2.1 440
11.1.5 Longitudinal Poisson Ratio 1112 441
11.1.6 Uansverse Poisson Ratio "23 442
11.1.7 rh ennal Expansion Coefficients 443
11.1.8 "11oisture Expansion Coefficients 445
11.1.9 111ennal Conduc.ti\lity 446
11.1.U) Moisture Diffusivity 447
11.1.11 Specific Heat 448
11.2 Modified Rule of Mixtures 448
J 1.3 Note on the Micromechanics Models 449
Appendix A. ernu,Sectional Properties of Thin-Walled CMtPostte Beams 453
Appendix B. Buckling loads and Nall.ral Frequencies of 011hotmplc Beams
with Shear lletonnation 461
Ajtpendlx C. Typical Material Properties 464

Index 469
Preface

The increased use of con1posites in ae rospace, land, and 1nnrine applications has
resulted in a growing den1and (or e nginee rs versed in the design of st:ructures
n1ade o f fiber.re inforced composite materials. To satisfy this demand. a nd l o in ..
troduce engineers to the subject of composites. numerolL~ excellent texts have
been published dealing with the mechanics of composites. These ICJ<IS deal with
those fundamental asperu needed by engineers new 10 lhe subjccL Our book
addresses topics nol generally covered by existing texts but that arc necessary for
designing prac1ical suucaures. Among the topics in this book of special interest to
the designer. but thnt usually are not included in standard texts. are stre,s.s...strain
relationships for a wide range of anisotropic materials: bending. buck.Jing. and
\ibration of plate5: bending. torsion. buckling. and vibration of solid as \Veit as
thin\\alled beams; shells: hygrothe nnal stresses and strains: and 6nite element
formulation. The material is presented in s ufficie nt detail to cnnble the reader
to follow the developments leading to the finaJ results. The expressions resulting
from the anolyses a re e ither readily usable or can be translated into a computer
algorithnl. Thus.. the book should be useful to students and researchers \\ishing to
acquire knowledge o f so1ne of the ad\'anced concepts of the mechanics of con1pos
ites as \\'e ll as to e ngineers e ngaged in the design o f structures 1nadc or con1pos ite
n\atc ria ls.
The en1phasis is on analyses built on fundamental concepts thnt are applicable
10 a variety of structural design problems. lo presenting 1hc material 've have
strived to follow lhe outline commonly used in tex1s dealing wi1h the analysis
of structures made of isotropic materials. \\'e have consciously omit1cd empirical
approaches.. Test results are certainly of \TaJue to the engineer. Ho\\ever. for com
po.sites.. the~ mostly apply only under specific circumstances and cannot readily
be generalized to different materials and different applications. We have included
material properties data lo help the designer perform calculntions 'vithout the
need to search 1hc literature.
The book i self-contained. Nevertheless, the reader will find it helpful to have
a background in n1c.chanics nnd in compos ites and son\c kno,vledgc of differential

xi
xii PREFACE

equations and matrix algebra. Weha\1e made a.n effort to keep the notation as uni
form as practicable and reasonably consistent 'vith accepted usage. The principal
syn1bols are summarized in a list ofsytnbols.
\Ve are grateful to Professor Istvan HegedUs for his constructive comme nts.
We thank Dr. Rita Kiss. Gabriella Tarjan, and Aniko Pluzsik fo r proofreading
portions of the manuscript, Gabriella Tarj3n for preparing the illustrati~ns, and
E.r ic. Allison a nd Sarah Brennan for their help in co1upiling t he index.

Laszlo P. Kollar
Budapesl
George S. Springer
Sta11ford
List of Symbols

\Ve have used, \\'ht revcr possible, notation standard in elasticity, structural anaJy..
sis.. and composite materials. We tried to avoid duplication, although there is son1e
repetition of lhose syn1bols that are used only locally. In the following list \Ve have
not included those symbols that pertain only to the local discussion. BeJo,v. 've
give a verbal description of each symbol and. when appropriate. the number of
1he equation in \vhich the symbol is first used.

Latin let1er1
A area
Jti tensile stiffness of an isotropic laminate (Eq . 3.42)
[AJ. A;1 tensile stiffness of a laminate (Eqs. 3.ll\, 3.19)
(a l "i inverse or the [Aj matrix for symmetric laminates (Eq. 3.29)
(BJ, ~' stillness or a laminate (Eqs. 3.18. 3.19)
(CJ,C11 30 stiffness matrix in the x 1, x1 x.i coordinate system (Eq. 2.22)
fCJ.c,, :\D stiffness matrix in the x, )', l coordinate systc1n (Eq. 2.19)
c moisture concentration (Eq. 2.154); core thickness (Fig. 5.2)
IDJ. D11 bending stiffness of a laminate (Eqs. 3.18. 3.19)
JDJ'.Di, reduced bending stiffness of a laminate (Eq. 4.1)
Di* bending stiffness or an isotropic laminate (Eq. 3.42)
D. D. i5 parameters (Table 6.2, page 222, Eq. 6.157)
(dj . it, inverse or the I DJ matrix for symmetrical laminates (Eq. 3.30)
d. ,r."" distances for sand"ich plates (Hg. 52)
Ei. ,. , Young's moduli in tbe x1 x1 x, coordinate system (Table 2.5)
[) stiffness matrix in tbe FE calculation (Eq. 9.4)
0. tensile stiffness or a beam (Eq. 6.8)
f[I bending stiffu._.. of a beam (Eq. 6.8)
f[I_ warping stiffness of a beam (Eq. 6.244)
F,' F,; mcngth parameters in the quadratic failure criterion (Eq. 10.2)

xiii
xiv LIST OF SYMBOLS

/;; constants in the quadratic fa ilure criterion (Eq. 10.25)


/ , /;; fre.quency (Eq. 4.190)
f,, J,. J,, b<Xly forces per unit volume (Eq. 2.13)
G13 , Gu, G 12 shear moduli in the xh x 2, XJ coordinate system (Thble 2.5)
Ci, torsional s tiffness of a beam (Eq. 6.8)

h b, /tl" plate thickness


distances of the botto1n and top surfaces of a plate fron1 the
reference plane (Eq. 3.9)
i 111 polar radius of gyration (Eq. 6.340)
[J] inve rse of the material stiffness ma trix (EJ (Eq. 9.16)
K nu1nber of layers in a laminate; nun1ber of \Vall segments;
stiffness para mete r of a plate (Eq. 4.153)
k rotational spring constant (Eq. 4.149)
k equivalent length factor (Eq. 6.340)
L.x. Ly din1ensions of a plate
L length; number of cells in a multicell beam (Eq. 6.222)
I.;. I!; load a nd failure load (Eq. 10.42)
Ix . I~ half lmckling length (Eq. 4.142), hal! buckling length
corresponding to the lo\\est buckling load o( a long plate
(Eq. 4.173)
bending and n.vist moments per unit length acting on a
la mina te (Eq. 3.9)
hygrothermal moments per unit length (Eq. 4.247)
bending moments acting on a beam (Fig 6.2)
bimoment acting on a beam (Eq . 6.232)
in plane forces per unit length acting on a laminate (Eq. 3.9)
inplane compressive forces per unit le ngth (Eq. 4.109)
hygrothermal forces pe r unit length (Eq. 4.246)
buckling load of a uniaxially loaded plate (Eq. 4.141)
axial fo rce acting o n a beam (Fig. 6.2)
buckling load and buckling load due to bending deformation
(Eq. 6.337)
buckling load in the x- z and x- y planes, respectively
(Eqs. 6.337, 7.110)
N..T,, buckling load under torsional buckling (Eqs. 6.337, 7.1JO)
[P], [P] s tiffness matrix of a beam (Eqs. 6.2, 6.250). Without bar refers
to the centroid; \.\'ith bar to an arbitrarily chosen coordinate
systen1
p transverse load per unit area~ distance bet\veen the o rigin and
the tangent of the wall of a beam (Eq. 6.190)
axial and trans\'erse loads (per unit length) acting on a bean1
(Fig. 6.1); surface forces per unit a rea (Eq. 2.166)
[QJ, Q;; 20 plane-stress stiffness n1atrix in the x1, x2 coordinate system
(Eq. 2.134)
LIST OF SYNllOlS XY

(QJ, Q.; 20 plane,stress stiffness matrix in the x. y coordinate system


(Eq. 2. 126)
Q., buckling load resulting in lateral buckling (Eq. 6.359)
q shear ftow (Eq. 6.189).
R s1iffness parameter (Eq. 3.46)
ii s1ress ratio (Eq. 10.42)
R,,. R,., R. 1 radii of curvatures of a sbell (Eq. 8.1)
I Rj. R.1 compliance matrix under plane~strain condition in the X1 .r2
ooordinale syste m (Eq. 2.79)
IRJ, R., compliance n1atrix under pJanestrain condiLion in the x. y
coordinate sys1em (Eq. 2.65)
ISJ.S1, 30 compliance matrix in the x1, xi , X3 coordinate system
(Eq. 2.23)
[SJ. s,, 30 compliance matrix in the x. y. z coordinate system
(Eq. 2.2 1)
Si, shear stiffness of a beam, i. j = z. y. w (Eqs. 7.13. 7.36)
s., shearstiffness of a plate, i , j = 1, 2 (Eq. 5.15)
;;, shear compliance of a beam. i. j = z. y . ., (Eq. 7.38)
si. st. sj lensile strengths (Eq. 10.13)
s~. s;. s; compression strengths (Eq. 10.13)
SZJ. SIJ. SIZ shear Slrengths (Eq. 10.15)
f 1orque acting on a beam (Fig. 6.2)
f~ rosirained warping-induced 1orque (Eq. 6.235)
T.<oJ SainiVenan1 torque ( Eq. 6.239)
IT. J 20 mess transformation matrix (Eq. 2.182)
lt.J 30 mess transformation matrix (Eq. 2.179)
(r. J 20 Slrain transformation matrix (Eq. 2. 188)
(1; J 30 strain transformation malrix (Eq. 2. 185)
lorque load acting on a beam (F'ig. 6.1)
thicknesses of the top and bouom facesheets (Eq. 5.26)
strain e nergy (Eq. 2.200)
displacement in the :c direction: varies '"'ith the x and y
coordinates only (Eq. 2.50)
II dilplaeement in the x direction
u displacement of the reference surface in the x direction
111. Uz. llJ displacements in the x1. x2 and x3 directton
v displacement in the y direction~ \aries .ith the x and y
coordina1es only (Eq. 2.51)
\'Olume of fibers, matrix, and void
ou1-of-plane shear forces per unit length (Eq. 3.10)
1rans verse shear forces acting on a beam (Fig. 6.2)
displncement in they direction
v" displaccnlent of the reference surface in they direction
volunle fraction of fibers, 1natrix. and void
xvi LIST OF SYMBOLS

d isplacement in the t direction~ varies with the x and y


coordinates only (Eq. 2.52)
(II'], (II'] compliance matrix of a beam (Eq. 6.17). No bar refers to the
centroid; bar to an arbitrarily chosen coordinate system
UI deHection in the t d irection
;;, maximum deHection in the x direction (Eq. 4.29)
w deflection of the reference surface in the t direction
1.11 8 , wS deHections due to bending and shear deformations (Eq. 7.85)
)'c:, Z: coordinates of the centroid of a beam (Eqs. 6.54, 6.73)
Ys.:~ coordinates of the shear center of a beam (Eq. 6.311)
l.k. tk -l coordinates of the top and bottom surfaces of the k~h ply in a
laminate (Eq. 3.20)

Greek letters
parameter describing shear deformation (Eq. 7.253)
paran1cter desc.ribing shear deformation, i = u;, 1/1, 1V, (JJ
(Eq. 7.244)
[a].<>1; compliance matrix of a laminate (Eq. 3.23)
OI , fJ parameters describing buckled shape of a shell (Eq. 8.78)
ti;; compliances for closed-section beams (Eq. 6.156)
"ii;. 'CX;; thermal expansion coe ffkients (Eqs. 2.153, 2.158)
{J. A parameters in the displacements of a cylinder (Eq. 8.30)
[fl), /11; compliance matrix of a laminate (Eq. 3.23)
Ji;; compliance of symmetrical cn10ssec tion beams (Table 6.2)
P1; compliance of c losed-section beams (Eq. 6.156)
'ii,. 'iii; n1oisture expansion coefficie nts in the x , y. t directions
(Eqs. 2.154. 2.159)
property of the cross section (Eq. 6.360)
shear strain in a beam in the x- y a nd x- z planes (Eq. 7.2)
engineering shear strain in the x. y, t coordinate system
(Eq. 2.9)
YlJ, Yu . Y12 engineering shear strain in the x1 x 2, x.l coordinate systen1
t.h change in thickness (Eq. 4.282)
t.T temperature change (Eq. 2.153)
lJ, ~.; compliance matrix of a laminate (Eq. 3.23)
'ir; compliance of c lose d-section beams (Eq. 6.157)
7_,_., . . average strains in a sublaminate (Eq. 9.14)
~_,_. , f y , Eit engineering normal strains in the x . y . .z coordinate syste m
) , 21 4'3 engineering no rmal strains in the x1 , x 2 , x3 coordinate syste m
(~. ~. y;~ strains of the reference surface
0.hl f<i.bl .,.o.ht hygrothermal strains in a laminate (Eq. 4.250)
X y t T J:!f

parame ter of restraint (Eq. 4.152)


polar moment of mass (Eq. 6.411)
UST OF SYMBOLS xvH

ply orientation
rate of twist (Eq. 6.l)
rate of twist due to bending and shear deformation (Eq. 7.5)
cuntures of the reference surface (Eq. 3.8)
hygrothermal curvatures of a laminate (Eq. 4.250)
load parameter (Eq. 4.109); buckling load parameter
(Eq. 4.121 ); eigenvalue (Eq. 4.225)
l l1Ji , Gi , .Si parameters in the calcuJation of natural frequencies
(Eqs. 6.398. 6.400, 7.203)
Poisson s ratio
coordinates attached to the wall of a beam (Fig. 6.13)
parameters in the expressions of the buckling loads of plates
with rotationally restrained edges (Eq. 4.151)
potential energy (Eq. 2.204)
"
/>..,. P,.., P:: radius of curvature in the y-z , x- z. and x- y planes (Eq. 2.45)
p,. frl , p, radius of curvature in the x 2-:c.,. x 1- x;h and x 1-x.2 planes
(Eq. 2.53)
Pi:Qmp. Pr, Pm densities of composite, fiber. and 1natrix
p n1ass per unit area or per unit length
normal stresses in the x 1 x2 , .~'J coordinate systen1
normal stresses in the x , )''. l coordinate system
average stress
r:u. r13, r12 shear stresses in the x1, xi , XJ coordinate system
shear stresses in the x, y, t coordinate system
rotation of the normaJ of a plate in t he x- x and x- y planes
(Eqs. 3.2 and 5.l)
rotation of the cross section of a bea1u in the :c- y and x- x
planes (Eq. 7.2)
angle of rotation of the croos section about the beam axis
(twist) (Fig. 6.3)
bending stiffness of an unsymmetrical long plate (Eq. 4.52)
potential e nergy of the external loads (Eq. 2.203)
circular frequency (Eq. 4.190)
circular frequency of a beam due to bending and shear
deformation (Eq. 7.1 98)
circular frequency of a freely vibrating bea1n in the :c- z and
x- y planes. respectively (Eq. 6.398)
circuJar frequency of a freely vibrating beam under torsional
vibration (Eq. 6.400)
dislances bet\\een the ne"' and the old reference surfaces
(Eqs. 3.47, 6.105. 6.107, A.3)
CHAPTER ONE

Introduction

In this book '"e focus on fiberreinforced composites composed of fibers en1bed~


ded in a 1natrix. TI1e fibers may be s hort or long, continuous or discontinuous. and
n1ay be in one o r in n1ultiple directions (Fig. 1.1 ). Such materials offer advantages
over conventional isotropic structural n1aterialssuch as steel, a luminum, and other
types of metal. These advantages include high strength, low weight, and good fa.
tigue and corrosion resistance. In addition. b) changing the .arrangements of the
fibers, the properties of the material can be tailored to n1eet the requirements of
a specific design.
The excellent prope-r ties of composites are achieve d by the favorable char
acteristics of the t1A'O major constituents, namely the fibe r and the maui..x. In
lo\\'perfonnance con1posites, the reinforceme nts, usually in the form of short
or chopped fibers (or particles), provide some stiffeni ng but very Jiule stre ngth
ening; the load is: n1ainly carried by the matrix. ln high..perforn1ance con1posites,
continuous fibers provide the desirable stiffness and s trength, \Vhe reas the matrix
provides protection and support for the fibers, and, importantly. he lps redistribute
the load from broken to adjacent intact fibers.

Long-fiber 001nposite Sbon-f'ibcr con1posite Partic:ulate composite

Unidirectional Jan1in:a ( J>ly) \Vo,en fabric \Vovcn &bric:


(b1a.x1al \\'Ca\e-) (triaxlal weave)
Figure: 1.1: Composite n1aterial S)'Slcms.

1
2 INTRODUCTION

l.an1iMc fonning a Jan1inatc Laminate


Figure 12: Lamjnatcd cumposit~.

The arrangen1ent of the fibers in a structure is governed by the structural


requiren1ents and by the process used to fabricate t he part. Frequently, though
not always. composite structures a re n1ade of thin layers called laminae or plies.
Within each lamina, the fibers n1ay be aligned in the same direction (unidirectional
ply, Fig. 1.1) or in different directions. The latter configuration is produced, for
example. by \Veaving the fibers in t\\'O or more directions (\\oven fabric). The
lamina may also contain short fibers either oriented in the san1e direction or
distributed randomly. Seve ral laminae are then con1bined into a laminate to (orm
the desire d structure (Fig. 1.2).
The mechanical and thermal behaviors of a structure depend on t he prope rties
o( the fibers and the matrix and on the amount and orientations of the fibers. In
this book, we consider the design steps from micromechanics (\\hich ta kes into
account the fiber and n1atrix properties) through macromechanics (which tre.ats
the properties of the composite) to structural analysis. These steps are ilJustrated
in Figure 1.3 for a structure made of laminated oon1posite.

~-Mattix
~--- Fiber
I
~1icronH.:chanks

'+'
l aLn1ns 1plyJ

I
f\lacrom<:chan ics
'+'
~Laminate
I
StructuroJ analy.s1s

iJ? Stn1cture

figure J.3: The le\cl:> of an aJysts or a structure made of lam.in atc-d composite.
CHAPTER TWO

Displacements, Strains,
and Stresses

\Ve consider cornposite ma terials consisting of continuous or discontinuous fibe rs


embedded in a matrix. Such a composite ls heterogeneous. and the properties
vary from point to point. On a scale that is large Ylith respect to the fiber diam-
eter. the fiber :ind matrix properties may be averaged. and the matcri:il may be
treated as homogeneous. This assumption, commonly employed in m:icromechan
ical analyses of composites, is adopted here. H ence. the material is considered to
be quasi-homogeneous, \\ilich implies that the propenics a.r e taken to be the same
at every point. These properties are not the same as the propenies of ei1her the
fiber or the matrix but are a combination of the properties of the cons1ituents.
ln this chapter. equations are presented for calcul:uing 1he displacements.
stre.Mcs, 3nd strains Y+'hen the structure undergoes only small defonnations and
the inateri:il behaves in a linearly elastic manner.
Continuous fibcr-rcinforced composite n1aterials {and s tructures 1nade of s uch
n1ateriab) ofte n have easily identifiable preferred directions nssocinted \Vi th fibe r
orientations or s:ym1netry planes. It is there fore convenient to c1nploy two co-
ordinate systc1ns: a locnl coordina te system aligned, a t a point, either \Vith the
fibers or '"'ith axes of symmetry, and a global coordinate system attached to a fixed
refere nce point (Fig. 2.1 ). In this book the local and global Cartesian coordinate
systems are designated respectively by x1 x2 xl and the x.y. z axer.. In the x. y. t
directions the di.splBcements at a point A are denoted by u. u, w. and in the x1 Y2.
x, directions by 111. 111 , 111 (Fig. 2.2).
ln the x. y.:: coordinate system the normal stresses are denoted by a,. a.-. and
" and the shear stresses by r,,. r,,. and r,, (Fig. 2.3). The corresponding normal
and shear strains are' t,. '~and Yr~~ y,.~, r ..,.. res:pecti,ety.
In the x 1 X?. XJ ooordinale system the normal stresses a.re tie noted by ai. a1.
and .,., and the shear stresses by r 13, ~13 and r 12 (Fis- 2.3). The corresponding
normal and shear strains are t1 ?t2. 3. and YD. yu, Ytz respectively. TI1e s:yn'lbol y
represents engineering shear s train that is t\\~ce tJ1e te nsorial shear str::iin. Yti = 2;;
(i. j = x. y. t"' f. I = I, 2, 3).

3
4 DISPLACEMENTS, STRAINS, AND STRESSES

z x,
figure 2.1 : The global x . y. z a nd toe.al xs. xi. .t) coordinate S)'SlC-m"-

A stress is taken to be positive \vhen it acL~ on a positive face in the positive


direction. According to t his definition. alJ the stresses sbo\VJl in Figure 2.3 are
positive.
The preceding stress and strain notations, re ferre d to as e ngineering notations.
are used throughout this book. Other notations, n1ost notably tensoria l and con..
tracted notation~ can frequently be found in the literature. The stresses and strains
in d iffe rent no tations a re summarized in Tables 2.1 and 2.2.

2.1 Strain-Displacement Relations


We consider a ti,_x longsegn1ent that undergoes a change in length. the ne \v length
being denoted by 6.x1 From Figure 2.4 it is seen that

11 + llx, =ax+ ( 11 +-a;ax


au ) ' (2.1)

\vhere u and u + ~ax a re t he displacements of points A and B. respec.tively, in


the x d irection. Accordingly, the normal strain in the x direction is
ll.x' - 6.x au
f.t = = -. (2.2)
!lx ax
Simila rly, in they and t directions the normal strains are

v =
av
- (2.3)
. ay
aw (2.4)
<,=~'

\Vhere v and u; are the d isplacen1ents in t he y and z directions. respectively.

x
figure 2.Z.: The :r. y. z and x1, x2 .t) coordinate syste ms an d the. corresponding displacements.
2.1 STRAll+-OISPLACEMENT RELATIONS 5

l'"
r,.
a,
4

F'igure 2.3: The stre.o;ses in the global x. y, z and the local x1, .f'l, X'.\ coordinate ::1yste.ms.

For angular (shear) deforn1ation the tensorial shear strain is the average change
in the angle between two mutually perpendicular lines (Fig. 2.5)

a+ {J
fxy = - - (2.5)
2
For small deforn1ations \Ve have

(v+ ~ 6x) - v av
a ~ tan a= =- . (2.6}
ax ax
Similarly fj = 3u/ 3y 1 and lhe xy con1ponent of the tensorial shear strain is

fxy =!_(~"+a u). (2.7)


2 ay ax
In a sin1ilar n1anner \Ve obtain the follo\ving expressions for the fy.;. and E.t;:
oon1ponents of the te nsolial shear s trains:

f
,., -
1
- -
2
(avaz + aw)
- -
ay x= :(a" +aw)
' 2 az ax
(2.8)

Table z.1. Slress notations


Normal stress Shear stress
:r, y , z coordinate system
Tensorial stress a.~ 11.i t
"n' "" ""'
U 1.li

Engineering sLre$.S "r a, r,, ,,,


Contrac1ed nolation "'a, a,
t ;rt
a,
a.1 " "'
x 1, x i .t .i coordjnale systein
Ten.>:o riaJ Slres.s O'J ~

Engineering. srress "" "" f1::J O'?J


""
Conlr.1cted nolalion "' "' cr2 f ?J

"
Ttl. fJ !

"' "' "' "' "'


6 DISPLACEMENTS, STRAINS, AND STRESSES

Table z.z. Strain notatloos {the engineering and cootracted


notatioo shear strains are twice the tensorial she"' slrain)
Normal strain Shear strain
x, y , z coordinate S)'Sle m
Te.nsoria1strain
''" E)y f,.~ .,,
., ,, t"<lt
"' .,1'\v
Engineerins strain
Contr.:i.cted n olation
<,

"
., " ..
Yy, Y,, ?

'
coordinate system
.t_1 , x2, .l)
Tensorial strain
Engine.e ring strain ,,'",, Y?.'
El;! f.U .,. El} f }1

f '!
l'H YIJ
C..ontracted n otation
" " " " "
The engineering shear strains are t\\~ce the tensorial shear strains:
av a 1i;
y,.. = :U,. = -
- - at + -ay (2.9)
au au;
Yx:o
. = ~x , = -at + -ax (2.10)

au au
Y.11y = ~xy = -ay + -ax (2.11)

Jn the x1 , x 2 , x3 coordinate system the strain-displaceme nt relationships are


also given by Eqs. (2.2)- (2.4) and (2.9)- (2.11) with x, y, z replaced by x 1, x 1 , x3
the subscripts x , y, z by 1, 2, 3. and u , v~ w by u,, u2 , U3.

2.2 Equilibrium Equations


The equilibrium equations at a point 0 are obtained by considering force and
mome nt baJanc.es on a sn1alJ 6.x6.y~z cubic ele.m ent located a t point 0 . (The
point 0 is at the center of the eleme nt, Fig. 2.6.} \Ve-relate the stresses a t one face
to those a t the opposite face by the Tuylor series. By using only the first term of
the Taylor serie~ force balance in the x direction gives

y .dx'

u
A' B'
('JU
.4 B " + - .d:r.
Lb: ax

Figure 2.4: Displacement o f the AB line segment.


2.2 EQUIUBRIUll EQUATIONS 7

C'

L "c
L B'r l
:
il: --o ---
v A'
iJ.:
11+-Ll.r
():c
A Llx B

x
Figure 2.5: "Displace ment of the ABC scgme nl.

where j, is the body force per unit volun1e in the x direction. After simplification,
this equation becomes

aax ar)'.l iJ'C:,t f 0


-
ax+ -ay+ -az+ .\'. -- . (2. J3}

By simila r argun1ents, the equilibrium e quations in they and l directions are


&r,, ~ ar9 . f _
ax + ay + az + -' - 0' (2.J4)

ar,, ar,., a.,., f 0 (2. J5)


ax + ay + ax + ' = '
where fr and f.. are the body forces per unit volume in they and t directions.
A mon1ent balance about an axis paralle l to x and passing through the center
(point 0) gives (Fig. 2.7)

(2.1 6}

Figure 2.6: Stresses on the 6.t 6y6,z cubic c lcme nL


''
fl z
,.~ :

,'
''
.. , ,,,,_'' ........
' o r 7
l'l

y
i1T~
~ - -fly
&g .

,, +-- fl x

:&
'' '"
fly

Figure2.7: Strc~<>.<>C$ on the L\x&y L\.t cuhic element that appear in the ntoment balance about an
axis paralle-1 to x and passin~ through the center (point ()).

Byon1ittinghigherorder terms., \\hich vanish in the limit 6x ~ 0, 6.y_,, 0.tJ.z- 0,


this equation becomes

(2.17)

Sin1ilarly, \\'e obtain the follo,ving equalities:

fxy = 1yx (2.18)

By virtue of Eq~ (2.17) and (2.18), the three equilibrium e quations (Eqs. 2.13-
2.15) contain six unkno\vns, nan1e ly. the three normal stresses (ax, ay, a.-,) and the
three shear stresses (ry~ t'.t:= r.1 .v).
In the x,, X2,X3 coordinatesysten1 the equilibrium equations a.re also given by
Eqs. (2,13)-(2.15) with x , y, t replaced by x, , Xz , X3 and the subscripts x. y, t by
I. 2. 3.

2.3 Stress-Strain Relationships


ln a con1posite mate rial the fibers may be oriented in an arbitrary manne r. De-
pending on the arrangen1ents of the fibers, the materiaJ may behave d ifferent ly
in different d irections. According to their -b ehavior, composites may be charac ..
terized as generally anisotropic, n1onoclinic, orthotropic., transversely isotropic,
or isotropic. In the follo\ving, \Ve present the stress- strain relationships for these
types of m aterials under linearly e lastic conditions.

2.3.1 Generally Anisotropic Material


When there are no sy1nn1etry planes with res_pect to the alignn1ent of the fibers
the material is referred to as generally anisotropic. A fiber-reinforced composite.
1naterial is. for e xan1ple, generally anisotropic \vben the fibers arealigned in three
nonortl1ogonal directions (Fig. 2.8).
2.3 STRESS- STRAIN RELATIONSHIPS 9

."
Figure 2.$: Example of a g.c.nerally anisotropic nlaterial.

For a generally anisotropic Jine arly elastic material, in the x. y. z global coor
dinate system, the stress-strain re latfonships are

<1.t = Cufx + C12E,. + Cu::. + C 14Yy::. + CtsYx~ + C16Yxy


<1y = C21l.t + CuE"y + C2JE::. + C24Yy.; + C1srA::. +Curx..
d::. = C11fx + C32Er + CJJE":: + c34Yy::. + CJsYx::. + CJ6Yx1
(2.19)
f'y.; = C41E;r + C..2fy + C.t..lf;: + C.uYy::. + C4sY:r::. + c46Yxy
T:u: = Cs1Ex + Cs1Ey + Cs3E:: + C.S4Yy<> + CssY.t::. + Cs&y.-J'
1'.1y = C61x + Ct12fy + C6JE; + CwYy:: + C6sYxz. + c66Yxy

Equation t2.19) may be writte n in the Corm

<1x Cu C 12 C1J c" Cu c,. ...


<fy c,. c,, C23 c,, C2s c,. fey

": = c" C~n


C32 c.,. C.lS c,. <;
(2.20)
Ty;: c.. c42 c43 c... C4s c46 Yy::
'fx:: Csi Cs2 C53 c,. Css Cs6 Yx::.
f'xy C61 c., c63 c.. C6s c,. y .T}'

where C;1 are the e lements of the stiffness matrix (CJ in the x, y, z coordinate
system.
Inversion or Eq. (2.20) results in the following strain- stress relationships:

Su s.. Sis s,.


"
<y S21
S12

S22
S11
s,, S24 S25 s26 ""
<fy
s,, s36
ss"... s., s"
S1..
s., s... ":
S32
"
Yyz
= 1 :S,u 544
S.s1 s~" s~s s,.
r,. (2.21)

ss".. s62
Yxz Ss2 1'.,~

Y.1y s 6] s... s.. s,. fxy

where Sii are the e ten1ents or t he compliance n1atrix [SJ in the x. y, z coordinate
system and are defined in Table 2.3 (page 10). In this table tests are illustrated
that, in principle, couJd provide means of determining the differe nt con1pliance
matrix e lement".
10 DISPLACEMENTS, STRAINS, AND STRESSES

Table 2.3. The elements of tl1e compliance matrix [SJ in tl1e x, y, z


coordinate system, The elements s, (without bar) i1 t11e x,, x,. x,
coordinate system are obtained by replacilg x, y, z by 1, 2, 3 on the
right-lland sides of tl1e expressions,
Test E~ments of the compliance mabix

ti'

a ,,.
S11
S21
= E,./a,,
= Ey /<T,.
531 = f:/<1,y
s41 = y,~/u...
S;, = r.::/CFt
SM= Y;ry/a.,
Su= E,./ay s.,~ = Y.~?fa,.
~'t:=':> 522 = f y/a,_ Sn = Y.~tfa,..

t~"' S,2 = E~/ay


S1:i = E.:/u,
S23 = f ,./at
Sr.i = Y.ry/a1

Su= y,,fat
Sn = ~1?/at
~ "'~
S,, = E,/ a ( S63 = Yryfa t

tov ~ = E,,/r,.,
S2.i
S,,
= E,./r,.1.
= E1,/ Tyt
~.u = Y>tfft1.
s~ = Y.11./r)'I.
SM= Yx1/ r,t
'T,,_-z
51; = EN/r,,: S45 = y,.: /t.x:
~~~ S2; = Ey/fJ.:
S.1; = E:/T.r:
s$5 = r.,:/r1:
s~ = Y.1y/T.l:..

'T;c'?/ Su;= Ex/rJ.,. S46 = Y1:frxJ


S26 = Ey/fq
SJ.;= E:/ TJf)'
s~ = Y.1:/r1,
Sf..6 = YJ1y/T.vy

In the x 1, x2 , x.l coordinate syste111 the stress- strain relationships a re


Cu c,. C1s c,.
"' c,.
rC11
C 12
Cu ez, Cis ez. ,,
ft

"" C24 ~,

c,, c,. c,.


"'
r,, = l2:: c,,C42
C33
C.u
C34
c.,, c,, c,. Y!.'
(2.22)

t't.l Cs1 Cs.1 Cs" Css Cs Yu


t'f2 C.1 C 62 c., q,, c., c Yl2

\Vhere cj, are the elements of the stiffness matrix CJ in the r


X2. X3 coordinate x,.
systen1.
By inve rting Eq, (2.22) we obtain the following strain-stress rela tionships:
S11 Su s,. S16
,,"
S12 S" O'J
Szi ..~24 s,, s,.
,,
S21
s.l,
S23
S31 .s:" s,. "'
Yl.l
= s.,
S32

S.i2 s., s..


S3.i
s.,, s,,. "'"
r21
(2.23)

Yu S51 Ssi S53 Ss.a Ss.< s,. r 11


Yl2 S.1 S;, .1;;, s.. .1;;., s.. r 12

\vhere S;; are the ele ments of the compliance matrix [S] in the x i, x 2, x3 coordinate
system.
2.3 STRESS-STRAIN RELATIONSHIPS 11

Plan< of symmruy

x, .,

..
~

figure 2..9: 1'"'1attOM o( fibcr-rc1n(orccd moooc.hruc matcruh. The fibcn arc only tn planes
Jralkl 10 t~ X'l- .r'l pane of S)""ft:tmc-try ltop). only perpcodKUb.r lo the: pbnc of S)1'11mctry
(middle). and 1.n the plane of symmetry and perpc:n<hcular to the plane of 1ymmclry(N>llom).

his evident from Eqs.(2.20)- (2.23\ that thecomplianoemotrix ISJis the inverse
of the stiffness ma trix ICJ:

[SJ= (CJ- 1 (2.24)

It can be sho,vn (sec Sectio n 2.11.1} that for an elastic mr.terial the sti(fness and
co1n pliance ma Lriccs are syn1me trical in both the x. y. z and ,\' 1, x2 "'coordinate
systems as follo,vs:

S.; = S ; 1 C, 1 =C11 1.J = 1, 2.. .. ,6.


(225)

Beca\15e of this symmetry. in both the (SJ and the (C( matrices ooly 2 1 of the 36
elements a.r e independent.

2.3.2 Monoclinic Material


When there is n symmetry plane \Vith respect to the aJignmcnt of the fibers, the
mate rial is referred to as monoclinic. ExampJes of monoclinic fiberreinforced
con1posilcs nre shown in Figure 2.9.
For a monoclinic 1nnter inl \Ve spe cify the co 1npliance lSI and stiffne ss lC]
n1atrices in an ,\' 1~ .t'2 .\';1 coordinate system chosen in such a way that x 1 nnd x 2 are
in the plane O( syn11n ctry. whereas X3 is pe rpe nd icular to this plane.
12 DISPLACEMENTS, STRAINS, AND STRESSES

Generally anisotropic Monoclinic

Plane of synunctry

y't"
"'l. ;f".'..t' "
~ .: . :l}L _.: - >

Figure 2.10: The normal stre.ss a 1 causes shear strain Yu tn a generaJJy anisotropic material ( le(t}
and no shear strain in a monoclinic material (right).
The elements of the compliance matrix for a n1onoclinic n1aterial are ob ~
tained by 1nodifying t he-con1pliance n1atrix o( a generally anisotropic n1ateriaL
We observe that in a generally anisotropic mate rial a normal stress a 1 causes an
out..of-planeshear strain YIJ (Fig. 2.10 left), but in a monoclinic mate rial subjected
to a normaJ slTess O'l (<Yt being in the plane of symmetry) the out~ofplane shear
strain }'13 is zero (Fig. 2.10 right). Consequently. for a n1onoclinic material the S's1
element of the co1npliance matrix is zero. By sin1ilar arguments it can be sho,vn that

Table 2.4. The elements of the compliance matrix for monoclinic


materials. For orthotroplc, transversely Isotropic, and Isotropic materials
S,= S.,, = 0, S,. = Ssi = O, S,. = S., = O, S., = S,. = O
Test Bements of the compliance matrix

ti'
er,
Sn = 1 /a 1
Si1 = z/a1
Sn = f'J. /a 1
Sn= y,,/u1 =0
~1 = Yu/<r1 = 0
Sri1 = Yu.fa,

CT;i, <72 S1.?: = f1/a i ..?: = Y?J/Gz = 0


<-~-+ S2.?: = 2/ai 5.:-2 = Y1J/<T2 = 0
s.'.?: = f:'J./ a 2 '*! = Y11/ai
to-, Su= 1 /a~ S.u = Yi;;/a.1 = 0

~J. Si~= fz /aJ


=
s~'J. f.,/aJ.
~~ = Yu/ a;; = 0
.Sis'!> = Y12/a.'
0'3
<-
...., S14 = 1/T2~ = 0 S.u = YlJ/ri.~
Si = E2/rv = 0 ~4 = YJJifl'J.
to =
S3' f.l/Tl'J. = 0 $s. = Yn/ f2'J. =0
t~~.,.,, s. ~ = 1 / ru = 0
Sti = f'z/ r u =0
Sis = '11,}/ft'
~~ = Yuiru
s~ = f'J./r u = 0 ~ = Y12/f13 = 0

... 7 12
Su. = f 1 /rt~
Su,= f2 / r 12
s.. = Yn/ll? = 0
S.V.. = yu/fu = 0
~
-+ S3r. = f'J. / r 1: ~ = Y12/f1 ?
2.3 STRESS-STRAIN RELATIONSHIPS 13

for a monoclinic material the S.11 S.12 S51, S:i 3, S51 , S64 , .~ elements a re a lso zero.
(Since the con1pliance matrix is scyn1metrical the elements S1.t, 52<1~ Szs, SJ4, Sis,&!,,
S56 are a lso zero.) The e lements of the compliance n1atrix are listed in Ta~Je 2.4.
111e elements of the con1pliance n1atrix may be expressed in tern1s of the
engineering constants defined in Table 2.5. ln Tables 2.4 and 2.5 the types of tests
are also illustrated that. at least in principle~ could provide the elements of the
001npliance matrix and the engineering constants. The relationships between the
elements of the compli:ince n1atrix and the engineering constants are sho,vn in
Thbles 2.6 and 2.7.
The nonzero and zero elements of the compliance matrix can best be seen
when the m atrix is \.\'ritten in the forn1
S11 S12 s., 0 0 s,.
S 12 Siz S'-' 0 0 Si.
Su S1.1 sl., 0 0 s_,.
IS] = 0 0 0 s,. s,, 0
(2.26)
0 0 0 S.u s,. 0
s,. s,. S3a 0 0 ......

Table 2.5. The engineering constants kJr monocllnk: materials. For ortholropk:, transversely
isotropic, and isotropic materials v1e = ve1 = 0, v26 = vs2 = O, v36 = 1.153 = 0, vs = 1.154 = O
Test Engineering oonstantJ
Y-0ung's modulus in lhe x 1 direciion 1 = 0 1/1
Poi..;oson's r.itio in the x1- .r2 plane 1'12' = -E-2/El
Poi.;;son's ratio in lhe .-q -.r3 plt:ine lllJ = -f-.i/ E t
Poisson parameter v ii;= Y1z/E-1
Young's modulus in lhe .t"l direction Ez = 02 /E?
Poi..;oson's r.itio in thex?-.r1 plane l /'21 = - f1/ E2
Poisson's ratio in lhe .t"l-.fJ plane 11'1.; = - f.;/1
Poisson par:uneter 11'2f = yu /E2
Young's modulus in lhe XJ direction 3 = OJ/f-J
Poisson's ratio in lhe XJ-Xt plane l':JI = -E1/EJ
Poisson's ralio in lhe XJ-X2 pla ne 11'J2' = -E2/EJ
Poisson parame ter 11J(J = Y1z/ EJ

Shear modulus in the x2-x.t p1ane Gu = "!?J/ Yl'J


Poisson parameter = yu/ ~J
1'.i!\

Shear modulus in the .t1-x.l plane Gu= r u/yu


Poisson parameter 1"$4 = ~J/)'13

Shear modulus in the .t1-.t? p lane G12 = n z./'Yl?


Poisson parameter llt.1 = E-1 /yu
Poisson paramete.r 11c;? = f2/ n2
Poisson paramete.r l 'to.> = E3/ Y12
''n 11r
; = cl! r,J = I , -? , .3)
('
14 OISPIACEMENTS, STRAINS, AND STRESSES

Table 2.6. Bements of Ille compliance matrix in terms of Ille engineering


constants ID< monoclinic materials. The expressions are also valid tor
orthotropic, transversely isotropic, and isotropic materials with S. 6 = S.1 = o,
S,. = S,2 = O,S.,, = S,. = 0, S., = S.. = O.
Su = E1 / a 1 = E1/ (LE1) = T, So = 0
Si_1 = E2/<11 = Ei/ (1E1) = - "')..~ s~, = o
SJ1 = E3/a1 = E.i/(E1t1) = - ;: -%1 = Ys2/a1 = Y'lli( 11) = "~~
Su= t1 / a i = E1/( 2t2) = - i~ St? = 0
Su = E2/a1 = t1/(1'<2) = i s!'!. =O
SJz = E3/<T? = E;;/ (it2) = ::~ ~a: = Y12/'1? = Yu/ ( /i2) = ,.~
11

S1J. = E1 / <TJ. = E1 / (3EJ ) = -~ s~> =O


Si.; = E2/<1) = E:i/ ( E:;t:;) = - ~ S~; =O
SJ;. = EJ/'1.~ = E3/(3EJ) = -t; Slil = Y12/<1J. = YI!/{ JfJ.) = ",.~
Sit =0 s~ = )'!:;/ r?1 = YlJ/ (Gi;;y._;,) = d:,
S iA = 0 s~ = YJJ/T?.l = Yu/(Gi.iY.n ) = ~~~
s,. =0 s.. =0
S1~ =0 &.,. = YrJ/r u = yi.1/ ( G1;;yu ) = ~~'
Si:> = 0 S~ = Yti/ TJJ = Yl.l/( G1.1Y1J) = it
.lj, = 0 s,. =o
S111 = E1/ r 11 = E1/ (Gun2) = ;~~ s.. =o
Sll. = El IT l! = = ""<;61:z
~j (G U )'ll) s,. =o
SJ6 = 3/ru = J/(G1?YU) = ~:: .S(.e. = YU/ TJl = Y11/(G12Yu) = d,?
The stiffness n1atrix is obta ined by inve-rting the con1pliance matrix as follows:
Cu C12 C,, 0 0 c,.
C12 Cn Ci.1 0 0 c,.
re] = c,,
0
c,, c,, o o c,.
0 0 ~ ~ 0
(2 27)
.
o o oc._,c"o
c,. c,. c,. 0 0 c..
Because the [SJ and [CJ matrices are symmetrical (Eq. 2.25) only I 3 of the
elements are independent (Table 2.8).

2.3.3 Orthotropic Material


When there a re thre e mutually perpendicular symn1etry planes with respect to
the alignment of the fibers the material is referred to as orthotropic (Ftg. 2. JI).
Examples of orthotropic fiberre.i nforced 001nposites are shov"n in Figure 2.12. For
an orthotropic material 'Ne specify the stiffness and compliance matrices in the x 1
xi, X J coordinate systen1 defined in such a way t ha t the axes are perpendicular to
the three planes of symmetry (Pig. 2.11 ).
\Ve apply a norn1al stress cr1 (Fig. 2.13). Because 11 1 L~ in the x 1-x2 symmetry
(orthotropy) plane the outofplane shear strains are zero, ( y13 = Yl.1 = 0); and
2.3 STRESS-STRAIN RELATIONSHIPS 15

Table 2.7. The compliance matrices in terms of the engineering constants for
monoclinic, ort11olropic, lransversely Isotropic, and isotropic materials

- .L _!'.ll !:!l !a -

"
- r.1
''I?
.,
J..
E! -

- e,
1:;
' ''.I!
0
0
0
0
Gu
<iu
..."'
- 0"' .,0
111~ ':.i J.. 0 0
-Ti
(SJ=
0 ..L .,. "''0 monoclinic:::
<>u CiU
''4 ~ ..L
() 0 0 0:> GJ!
0
!Ji ~
'ii' 0 0
t
-"
E:' !

J.. ''!! II.JI


0 0 0
- ! - 1
_2! " f: i 1.'!
I
-~
E;
0 0 0
- '1.'. "!'

*
~, - 6.l 0 0 0
(SJ= orlhotropic
0 0 0 +.
<n
0 0
0 0 0 0 ~ 0
..L
-
- J.
0

-17
0 0

_.::u.
0 0 au_

/:'1

- Ti ''U .,
J.. -L;_
"~
"J!
0
0
0
0
0
0

(SJ=
-~ ,, -~
f:i .,
.L 0
20+:;t)
0 0
trans\ersely isotropic
0 0 ()
~ 0 0
0 0 0 0 ..L 0
G1.
0 0 0 0 0 ..L
G1 ~

. -e 0 .
"

-i
I
-, ~
T: -i
I
0
0
0
0 0
-~ -~
f: 0 0 0
[SJ= " iO + ) isoLropic.
0 0 0
0 0
0
0
0
0
0
0
0
0
llit~
0 -.-
~ l+v)
0

because O'l is also in the x 1- x 3 symmetry plane. the y12 = 0 shear strain is zero.
This implies that St.i- S15 S16 are zero. By similar argun1ents it can be sho\\n that
for an orthotropic material the Su, Sis- S26, S34, $l5. SMi. St.s. S46, Ss6 elements are
also zero. Accordingly~ the con1pliance matrix is

Sn Sn s,, 0 0 0
S 12 S22 S23 0 0 0
S t.l S23 S:u 0 0 0
[SJ = (2.28)
0 0 0 Su 0 0
0 () 0 0 S5s 0
0 0 0 0 0 ~

The elements of the compliance matrix are listed in Table 2.4. In tern1s of the
engineering constant~ the compliance n1atrix is given in Table 2.7. The stiffness
16 DISPLACEMENTS, STRAINS, AND STRESSES

Table Z.& The nonzero engineering constants for monoclinic, ortllotroplc,


transversely isotropic, and isotropic materials ( = Tf T, ,
see Table 2.7)
Nonzero eng;ineerilg constants
f\laleria.l Independent D ependent
f\lonoc.linic 1, E~, EJ
Gi.i,Gu , G12
vu. ''IJ ~lJ
' ' 16 ''bh \l.j.j, l'.~

Orthotropic 1. E'1 . 1
Gn : Gu. Gu
'-'!:!. vu. \i23
rrans ...crsely ,, Ei
iso tropic Gu
\'12, ''!J
Isotropic E, (= E) ="
Ei = E.; = E. 111) = vn
vu {= v) G?! = Gu = G1i = t<~">

Figure 2. l J: f\fate.rial " 1th t hree planes or symmetry.

+
figure 2.12: Ulustrations or
6her reinforced orthotropic.compo...;itcs.1'be fibers are oriented in
4

three mutually perpc.odicular directions {le.fl): the fibers are distri buted e quall)' in the + Ci and
-0 directions in planes parallel to the x 1- xi plane ( rii ht).
2.3 STRESS-STRAIN RELATIONSHIPS 17

"fu = O -ru= O '"ta:~= O


Pigure2.l 3: A normalstTCs.c;a1 applied ln the.ti -xi and x1-.\) symn1c try pianes of an o rlhotropic
ntaterial.

n1atrix is obtained by inverting the con1pliance n1atrix. The nonzero tern1s of the
stiffness mat rix are
Cn 0 () 0
re"c,, C 12
C22 C21
Ci.1 C33
0
0
0
0
0
()
(2.29)
0 0 c,. 0 0
ICJ =l1' 0 0 0 Cs5 0
0 0 0 0 c..
In the (SJ and [CJ
matrices of the 12 nonzero eleme nts only 9 a re independent
(Table 2.8). Equation (2.29) can be written in the form

[CJ =
(LJ
[~ ~ ~] (2.30)
O 0 OJ
0 0 0 [MJ
[0 () ()
The submatrices (L] and (M] are give n in Tables 2.9 and 2.10 in terms of the e n
gineering constants..
With the compliance matrix given by Eq. (2.28), t he strain-stress relationships
(Eq. 2.23) become
,, Sn S12 () 0 0 .,.,
,, S12 S22
S 11
S:u 0 0 0 <>'2
,, s., s1.., S.u 0 0 0
"'' (2.31)
= 0 0 0 s,. 0 0
Y2.1 1'23

Yl.l 0 0 0 0 S.ss 0 1'13

Y12 0 0 () 0 () s.. ~"


This equation sho'vs an important fea ture of orthotropic mate.rials, natnely.
that normal stresses do not produce shear deforn1ations \Vhen these stresses are
in the x 1, xi .\) o rthotropy directions. Note, however, that normal stresses applied
in the x , y, t directions (which do not coincide with the x1, x1 , x3 orthotropy
directions) result in shear de(ormations, as illustrated in Figure 2 .14. ln this case
(in the x. y, z coordinate sy.sten1) none of the elen1e nts of the con1pliance and
stiffness mat rices i~ zero.
18 DISPLACEMENTS, STRAINS, ANO STRESSES

Table 2.9. The [LJ submatrix In Eq. (2.30) for 0<1hotropic, transversely isotropic,
and isotropic materials
Orthotn1pie
2 (\'Jl + ~ V,,t'l))
Ez (1 - ~vf3 }
1) (v23 + ft1,121ru)
_ 1':1 Czf.J:- 11fu.rt1!.j-~:! 1:J-Z111:1u;.~f:2f.~-'ii".!"2~
D- 1~1'.:, 1'"1,

Tran.o;,crsely bulropic
Et( l -1) ,v,,( t + 1~,J
[L] = ~ Ei v12 (l+ 1>v) E2(l -~vf?)
[
/iv12 (I+ >i.1) Ei (v?J + * vf2 )
D= 1- ~;;-2(1 + vv)~vf1
Lo;utn1pie
v
(L) = Cl+"K'- "'' [ ' : v l -v
I' v

Table 2.10. The [M] s.ibmatr!x In Eq. (2.30) fol orttiotrop~


transversely isotropic, and isotropic materials
Orthutrc).pic

(M) =
Gn0 G,,0 O
0
J
[ 0 0 Gu

[
Triln.'llcrstly isotropic

[M) = ~0
:!(l +V) 0

0
G.,
0 JJ
0
"
!{1+11)
00 ]
0 '
2(J +l')

,~~
6, '
......
Figure 2_14: Ortbo tropic material subje.cted Lo a normal stress. 'fhcr<:- IS no shear strain ' ''hen
the. str ess ts applic-d jn one o f the o rthotropy dircc.tions (le.ft). but t he.re is shc-ar strain when th e
stres,.<; is not a long a n ort hotrop)' direction (right) .
2.3 STRESS-STRAIN RELATIONSHIPS 19

\
Plane uf isotropy
Figure 2.15: Ex.antplc of a tlber-reinforccd. transvcrscly isotropic: composite. matc lial.

2.l.4 Transversely Isotropic Material


A transversely isotropic material has three planes of symmetry (Fig. 2.11) and, as
such, it is o rthotropic. In one of the planes of symmetry the ma te.ria l is treated as
isotropic. An example of trans\'e.rsely isotropic ma te rial is a composite reinforced
with continuous unidirectional fibers \vith a ll the fibers aligned in the x 1 direction
(Fig. 2.15). In t hL< case the material in the plane perpendicular to the fibers (x 2- x,
plane) is treated as isotropic.
for a transversely isotropic material \\'e specify the s tiffness and compliance
n1atrices in an x 1, x2 x3 coordinate syste1n chosen in such a \Vay that the axes are
perpendicular to the planes of syn1metry and x 1 L" perpendicular to the plane of
isotropy (Fig. 2.15). ln this coordina te syste m, because of 1nate rial sy1nmetry four
=
of the Poisson ratios are zero (vu1 ~'26 = IJJ6 = ""!i
= 0). furthern1ore, because
of isotropy the follo\ving engineering consta nts are re lated:

E, = ,, G,, = Gn. (2.32)

For an isotropic mate rial the shear 111odulus is 1

E
G=--- (2.33)
2 (1 + v)
Correspondingly. for a mate rial that is isotropic in the x 2- x3 plane '"e \Vrite

c,_J = 2 (I E,
+ .,,_,) (2.34)

Equations (2.32) and (2.34), together with the expressions in Table 2.6
(page 14), yie.ld the con1pliance matrix in tenns of the engine.e ring cons tants. The

1 E. P. Popov, Et11;irieeririg .\1ecl1a11ics of S()fids. PrcnticcHaU. Englewood Cljffs., New Ji:rsey, 1990,
p. 151.
20 DISPLACEMENTS, STRAINS, AND STRESSES

results a re given in Table 2.7 (p age 15). The zero and nonz-ero eleme nts o f the
con1pliance matrix are

S11 Su S 11 0 0 0
S 12 S12 Su 0 0 0
S 12 S23 Sn 0 0 ()
(SJ = (2.35)
0 0 0 2 (S22 - Su) 0 ()
0 0 0 0 s.. ()

0 0 0 0 0 s..
The stiffness n1atrix is o bta ined by inverting the compliance ma trix. The zero
and nonze ro e le n1ents of t he stiffness matrix are

re,.c,, C 12 C 12 0 0 0

l1' C11 c,, 0 0 0


Ci~ C22 0 0 0
C!!-Cr. (2.36)
0 0 -2- 0 0
ICI = 0 () 0 c 0
0 () 0 0 c..
Jn te rms of t he e ngineering constants, the e.le ments of the stiffness matrix are
given by Eq. (2.30).
In both the con1pliance a nd stiffness ma trices, o f the 12 nonze ro ele1uents only
5 a re independe nt (Table 2.8, page 16).

2.3.5 Isotropic Material


ln a n isotropic 1naterial there are no preferred directions and every pla ne is a
plane of synunetry. For exa mple, a composite containing a large nun1ber of ran
domly o riented fibers be haves in an isotropic ma nne r. For a n isotro pic ma terial
the coordina te system n1ay be chosen arbiua rily. He re, \Ve present the compliance
and the stiffness matrices in the x 1, x2 , and .t'J coord inate syste m.
Because o f material symmetry fo ur of the Poisson rat ios are zero (vu; = '126 =
11., 0 = V.t.'i = 0). Also, because of isotropy some of the e ngineering constan ts are
related as fo llows:

Ei =E,=E,=E
(2.37)
= l.'J] = 1112 = v
l.'23

G= E (2.38)
2(1 + v)

Equations (2.37) a nd (2.38), together with the expressions in Table 2.6


(page 14), give the compliance n1atrix in te nus of the engineering constants. The
2.3 STRESS-STRAIN RELATIONSHIPS 21

results are in Tub le 2.7. The e Je.ments of the compliance matrix are

rs11 s,, S12 0 0 0


S,2 S11 0 0 0

' '" l1
S12

S11 S11 0 0 0
0 0 2 (Sn - S12) 0 0
0 0 0 2 (S11 - Sn) 0
0 0 0 0 2(S11 - S11)
(2.39)

The stiffness n1atrix is obtained by inverting the con1pliance mauix. TI1e eJe..
ments of the stiffness matrix are

Cu c,, Cu. 0 0 0
c,, C11 Cu 0 0 0
c., C12 C11 0 0 0
[CJ = 0 0 0 C11 -C1~
--,-- 0 0
(2.40)

0 0 0 0 -,-
C11 - C1:
0
0 0 0 0 0 -,-
Cu-C1 ~

In te.rms of t he e ngineering constants. the elements of the stiffness matrix are


given by Eq. (2.30).
In both the compliance and stiffness matrices, of the 12 nonzero eleiuents only
2 are independent (Table 2.8, page 16).

2.1 Exan1ple. Calculate the elc111ei1rs of the stiffness anti corupliance n1arrices of a
graphite epo.'<y unidirectional ply. The engineering constants are given as E1 =
148 x 109 Nim'. 2 = 9.65 x 109 Nim', G 11 = 4.55 x 109 Nim'. v11 = 0.3, and
1'23 = 0.6.

Solution. For a transversely isotropic n1aterial the co1npliance matrix is given


in Table 2.7 (page 15, third row). By subotituting the engineering constants into
the expression in Table 2.7, and by using th e condition that Vi j/ E; = v;;/ Ei (see
Table 2.8, page 16) we obtain

6.76 - 2.03 - 2.03 0 0 0


- 2.03 103.63 - 62.18 0 0 0
2
- 2.03 - 62.18 103.63 0 0 0 10- 12!!...
[SJ=
0 0 0 331.61 0 0 N
0 0 0 0 219.78 0
0 0 0 0 0 219.78
(2.41)
22 DISPLACEMENTS, STl\AJNS, AND STRESSES

Figure 2.16: The x. y. z and the x 1, xi. x~ coordinate sys1cms.

The e len1ents o( the stiffness matrix are obtained by inve rting the con1pliance.
matrix
152.47 7.46 7.46 0 0 0
7.46 15.44 9.41 0 0 0
7.46 9.41 15.44 0 0 0 N
[CJ= rsr' = 0 0 0 3.016 0 0
JO-.
m'
0 0 0 0 4.55 0
0 0 0 0 0 4.55
(2.42)

2.4 Plane-Strain Condition


The.r e are c ircumstances \Vhen the s tresses and stra ins do not vary in a certain
direction. This direction is designated by e ither the x3 or the t ax; s (Fig. 2.16).
A lthough the stresses and stra ins do not vary a long X3 (or t), they n1ay vary in
planes perpendicular to the x3 (or t) axis. This condition is refe rred to as plane.-
strain condition.
\Vhen plane-strain condition exists in a bod}' made of an isotropic material
the x 1 - x2 (or x- y) planes of the cross se.c tion remain plane and perpendicular to
the x~ (or t) a.x;s. In a body n1ade of a n anisotropic n1aterial these pla nes do not
necessarily remain plane.
Plane-strain condit ion may e xist far from the edges in a long body with constant
c ross section \vhen both the material properties and the applied loads are uniform

~t t<-

~
~t t<-
~t J.
Body t<-
Force
~t t<- ! Body Force

figure 2.1 7: Surface a nd body forces lhal may be applied under planc:. strain condition. The
applied fore.es must he uniform a long the longitudinal axis a nd ntusl be in equilibrium for each
seg.mc:nt.
2.4 PLANE-STRAIN CONDITION 23

~,,

''[~'

Figure 2.18: Possihlc c.nd supports for structures a nai)'ZCd by plane-strain condition.

along the longitudinal axis and, in addition. the loads are in equilibriun1 on any
plane segment (Fig. 2.17).
When the afore1ne ntioned plane~strain condition exists:. the threedimensional
analysis simplifies oonside.rably. For an isotropic material, the normal strain f .l (or
E~) in the axial direction (x.l or z) and the out-of~plane shear strains Yt~ and y 23
(or Yx~ and Y>'z) are zero. For fiber~reinforced con1posites these strains are not
necessarily zero. Nonetheless, as is discussed in this chapte r, plane-s train condition
inuoduces s in1plific.ations that facilitate the analysis.
<Jeon1etry. The cross section perpendicular to the axis and the 111aterial prop~
erties must not vary along the length. Both ends of the body n1ay be built~in or
n1ay be free, o r one end may be built~in '"hile the other one is free (Fig. 2.18).
\\'hen both ends are built .. in, the longitudinal axis (x3 or z) remains straight and
its length remains constant. \Vhe n one or both ends are free. the longitudinal axis
n1ay become curved and its length may change.
Fiber orientatio11. On the basi."i o( fiber orientation, the body is analyzed as
generally anisotropic.. n1onoclinic, orthotropic. transversely isotropic, or isotropic
(Fig. 2.19).
lo;otropic Transversely Orthotropic
isotropic

I [Q]
~'(onoclini c Generally
anisotropic

Figure 2.19: IUus trations of pos:>iblc. fibcr orientations for plane-strain condition.
24 DISPLACEMENTS, STRAINS, AND STRESSES

Out-of-plane In-plane

Axjal s urface Normal surface: Tangcn1ial surfac,e

AxiaJ body NonnaJ body

Figure 2.20: llluslrations of surface and body forces that may be appbcd under plane-strain
condition.

In addition to the fiber arrangen1ents sho\\'D in Figure 2.19, the con1posite


may contain any combination of the fiber arrangements shov.'11 in this figure. The
in1po rtant fact is that the analysis must be performed according to the most com
plex fiber a rrangement inside the body. A body that contains a t least one generally
anisotropic fiber arrange1nent must be treated as generally anis<>tropic: a body that
consists of monoclinic. o rthotropic, and trans\'ersely isotropic fibe r a rrangen1ents
must be treated as 1nonoclinic: a body that consists of ortbotropic and transversely
isotropic fiber arrangements n1ust be treated as orthotropic.
For monoclinic, orthotropic, and transversely isotropic material~ the analysis
sin1plifies considerably when one of the ma te rial symmetry planes coincides v.ith
the .plane of planestrain.
Loads. Nonnal, tangential, and axial forces may ac.t on the surface. and body
forces may act parallel and normal to the longitud inal axis (Fig. 2.20). These

Axial ~fon1cn1 Torque

-> Y
ix,)
!
Figure. 2.21: Poss.ihlc. c.nd loads for plane-strain rondilion.
2.4 PLANE-STRAIN CONDITION 25

Table z.11. Relationships between the stresses and the end toads
Load x, y, z coordinate system
End a:<ial N=fa,dA 1i/ = f ,,,dA
A A
End mo1nent Af1 = f xa,rl A M, = }' x 1a; rl A
A A
Endmomenl M., = J,, y at1IA .~1 = f.t2altfA
A
f'= /(yr,, + xr,,} tfA f = J (x2rt1 +.t1 f'?.~ )1/A
A A

applied forces ( loads) may not vary along the length. In addition, an axial force.
moment, and to rque may be applied at the ends (Fig. 2.21). The forces acting
perpendicular and para lle l to the longitudinal axis are referred to as in plane a nd 8

out-of-plane.
Each of the loads shown in Figures 2.20 and 2.2 1 n1ay act in con1bination. t\ s
stated previously. the only requirement is that the loads be in equilibrium on each
segment (Fig. 2.17) of the body.
The end loads shown in Figure 2.21 a re re lated the to the stresses by the
expressions given in Table 2.11 .
When t\vo o r n1ore types of loads a re applied, the stresses and stra ins can
independently be e-alcula ted for each type of load. The stresses and strains thus
obtained are t he n supe rimposed to obtain the final results.
Displacen1ents. Plane..strain condition requires that the stra ins do not vary
along the longitudinal axis. TI1us., in the x, y, ! coordina te system \Ve ha\'e

a.. = o a., = o a,_ = o


az az az (2.43)
ay,., = o ar,, = o ay,, = o.
lz az az
The {ollo\\ ing displacements satisfy these oonditions2
1

u (x. y) - c,yz - 21 c,z'


,, =

"= V(x. y) + C1xz - ~c,z' (2.44)


w = W(x.y)+ (C,x + C,y + C4)Z,
where U. V, a nd Ware functions that de pend o nly on x and y, and C1 , C,, C.1, a nd
C4 are co nstants. For sma ll displaceme nts \\'e have the folJo,ving re la tionships 3:
(I aw 32u ~2 v
<, = z
o
-Py = - -<Jz2 p, = - v~-
""" (2.45)

2 S. G. Lckhnitski.i. Tl1eoryof Ela.~idty r>fon A11i.u uropk Body. ~tir Pub1ishcrs. 1',fosoow. 198!, p. 104.
;; T. H. G. t.fc.i0n. Aircrof1 S1n1c111resfor ngi11eeri11g. S1uden1.t. 3d edition. Halsted Press. John \Vile)' &
Sons. New York. 1999, p. 2.1W.
26 DISPLACEMENTS, STRAINS, ANO STRESSES

Figure 2.22: Illustration of the rotations of


line clc.mc.nts paraUcl v.ith the x- and y-.axcs.

'vhere E; is the strain along the longitudinal axis: 1/ p 1 , 1/ P.x are the curvatures
of the longitudinal axis in the x-z and y-.z planes., respectively. By virtue of
ay.,, /az = 0 (Eq. 2.43), we have (Eq. 2.11)
(2.46)

(2.47)

As is illustrated in Figure 2.22, i? represents the rate of f\vist of the c.rosssection.


The constants in Eq. (2.44} are determined as follows. From Eqs. (2.47) and
(2.44) we have
01 v a2u
11 - - = - - - =Ci . (2.48)
axaz ayaz
From Eqs. (2.45) and (2.44) we write
au.
f?=
-8t =C4
1 a'11
- = - -a.2 = c, (2.49)
Py "'
t a1 v
f>x = - az' = c,.
\Ve can now write the displacements as
1 1 ,
u = U (x. y) - ~yz - - - r (2.50)
2(>y

u = V(x, y)+llxz - 1 -1 r ' (2.51)


2(>.<
w= W(x.y)+ (2-x+ 2-y+~) z. (2.52)
PJ' l>x
These represent the following five deformations (Fig. 2.23):
Planar - planes perpendicular to the longitudinal axis ren1ain plain and per
pe ndicular to the a.xis
Nonplanar - planes pe rpe ndicular to the longitudinal axis deform out of the
plane
"0f __':'C:::: ) "
"'" "'""
~

~ I Q

""
~
II

~
II II
ll

...,
t>._;"
I
..., . ,,
..;
"~ Q Q
II

\~V ~
II II

" "
~

--------- / "' ...


-1..:
Q;;~_-:_- j~
!l.
-1 ('I
- 1..:
.
-
"-
H

~
Q

~ II

"
I

ll
II

6
..:\ "'
--- 7
"' /' ~

---='
ff

;_~ -------~~~/ - I .:
- I <"
"
<i:
-I ~
\ ..
I Q
--------- II II II
~
" "
~

a
...g ~,'':[]
.
.;

~ ~Fo
~

, Q
s:
~

,', Q

'
II
z I _..;;
... < ;> ~

a ~ ~~
a
Q (}) 0 ''
I o
;.e ;.f
c; 5;;
'\ '' ,' Q

"_,' II I I
"' ~
28 DISPLACEMENTS, STIUUNS, AND STRESSES

Bending - the straight longitudinal axis becon1es curved


Axial - the straight longitudinal axis remains straight and undergoes axial
e longation
Angular - planes perpendicular to the Longitudinal axis rotate about the axis.
The u. u, u; components of the displacements corresponding to each of the
deformations are shown in Figure 2.23.
\Ve again use the .r. y, z coordinate systen1 for generally anisotropic materials.
for monoclinic., orthotropic., transverseJy isotropic., and isotropic n1ateria ls \\'C use
the x 1 t 2 , x 3 coordinate system, v.ith :<;1 being a long the longitud inal axis of the
body (Fig. 2.16). In the :c1. x2 , x3 coordinate systen1 the displac.ements are

I 1 ,
r~1 = U1(xi . xi) - iJx2.fJ - - - xi
2 f>2
L 1 ,
u1=U1(X1.X1) + iix1X3 - -2-:Cj (2.53)
Pt

' .ti) + (-' x1+ -1 x1 + E3 ) XJ,


UJ=UJ(Xt,
/J2 P1

E;
\\/here is the e longation of the longitudinal axis; 1 / pz. 1/Pi a re the curvatures of
this axis in the x 1- .t'.3 and x 1- x 3 planes, respectively; '' represents the rate of t\\rist
of the cross section.
Jn t he fo l.lowing, 1.\'e present the equilibrium equations. the strain-cHsplace1nent
relationships. a.nd the stre.s.s- strain relationships \vhen the aforen1entioned con
ditions of plane~stra.i n are satisfied. 111e analyses are applicable in regions a\vay
from the two encl< of the body.
\Ve treat problems in tv.o groups: (i) \\/hen one or both ends are free and
(ii) whe n both ends are built-in (Fig. 2.18).

2.4.1 Freil End - Generally Anisotropic Material


We consider a generally anisotropic body \\'ith one or both ends free. This body
may undergo every de formation shown in Figure 2.23, and the displac.e ments a.re
the sum of all possible displacements sho,vn in this figure. The strains are then
obtained from the s train- displacement relationships given by Eq~ (2.2)-(2.4) and
(2.9)-(2.11). By introducing Eqs. (2.50)- (2.52) we have

au
( c =-
&.r ' -av
-
ayy -
(2.54)
1 J
E-= - x +-y+ E~ (2.55)
.. Pr Px ..
aw aw
y,,-=
.. -ay +"x Yx
. = -ax _ ,,y (2.56)
au av
Yx
y
= -ay + -
ax (2.57)
2.4 PLANE-STRAIN CONDITION 29

figure 214: The strc:is component.; that appear in the equdihrium equations under planestra1n
condition.

Since the strain con1poneuts are independent o f t. the stress con1ponents are
also independent o f t, and \\'e have

ar,._ _
ai - 0 (2.58)

By utilizing Eq. (2.58), the equilibrium equations (Eqs. 2.13-'2 .15) become

aa, ar,, (- 0
0
oX
+ 0
oy
+ )X - (2.59)

8<,yaa, f, - 0
+.
uX o .V
+y- (2.60)

ar. ary;+ f =.
-ax<z + - 0
(2.6 1)
ay "
The five s tress con1ponents a,.. a1 , l':.ty rYi r,..z that appear in these equilibrium
equations are illustrate d in Figure 2.24.
For a generally anisotropic n1ateriill the stress- s train re lationships (Eq. 2.20)
may be \.:ritten in pa rtitione d form. as follo\\s:

a, Cu C12c" c,.
Cl.t <x c.,
<1y c., Cu c,,. Cis C 26 .,. ell
ry.: = C14 c!4 c.... c.15 c... Yyz + C:w <, (2.62)
f.T<: Cts cl.S c., c5~ c56 Y.T~ c,.
'f.vy c,. Cu c... c,. c,,. Yx.v c36

<x
~y

a,= (Cu c2, C311 c3s z:,.J YJ't + C.u ': (2.63)
Yx:
Yxy
30 DISPLACEMENTS, STRAINS, AND STRESSES

where, we recall, is (Eq. 2.55)


l I
=-x+-Y+<.
Py P.t
0
(2.64)

111e stiffness ma trix [CJ is the inverse of the compliance n1atrix (SJ defined
in Table 2.3 (page 10). The strain-stress rela tionshi!"' are obtained by inverting
Eq. (2.62) as follows:
R11 R," Ris 'R,. <f., c.,
" fy /?.,
R 12
R12 Ru Ris Ru,
""
C23
c,.
Yy;:_ = R1.i R"J R.s R..
R1.i tyz. (2.65)
Y..-;:_
r.,,
Ris Ris R,, Rss Rs6
R,. R,. R.. Rs R..
rx~

r,,
where R; (i. j = I , 2. 4, 5. 6) a re the in-plane elements of the compliance matrix
c,,
c,. "l
under plane-strain condition calculated fron1 the relationship
Ru R,, R,.. R" R1t. Cu c,, c,4 Cis cl6
- I

R 12 R.22 R,, R.2.'i R 16 C12 C11 C24 c,, c,.


R14 R24 R., R._, R.. = c,4 C24 c..,, C4s c.,. (2.66)
Ris Ris R.s Rss R~6 Cis C2s c,, 'Css c,.
R10 R16 R.o R..... R.. c,. c,. c.. c,. c..
2.4.2 Flff End - Monoclinic Material
We consider a body n1ade ~fa monoclinic n1aterial (Fig. 2..9) \vith the plane of
ma terial syn1metry coinciding \\ ith the x1- x1 plan e sho,vn in figure 2.16. One or
1

both ends of the body a re free. The displacements resulting from specified loads
are sun1n1arized in Figure 2.25.
fllplane /,>ads. TI1e body is subjected to the in-plane loads shown in Fig
ure 2.20. Under these loads the body may undergo only planar, bending, and axial
deformations (Fig. 2.25). Accordingly, Eq. (2.53) reduces to (see Fig. 2.23)
1 I
u1 = Ut (xi, xi) - - -x12
2 1>1
1 l '
112 = U2 (x1 , x2) - 2: I>! x.; (2.67)

tl3 = ( -1 X1 + - I Xi. + '") .t).


Pl 1>1
The strain-displacement relationships (Eqs. 2.2- 2.4, 2.9- 2.11) show that for
these d isplacen1ents the strains are

1= -
au, 2= -
au, (2.68)
ax1 axz

(3= -
t
1 x1+-x2+E"(' (2.69)
3
/J2 Pr
Yi.= 0 Yll =0 (2.70)
au, au,
Yt2= - +-
ax-i
. -.
ax1
(2.71)
~0 -----
r; -------- ~
V-------- ___ 1
------
~~) )
----------- / "'
d:;;:_-_-_-:J\
...~ \"

{~-----------
t I
\I
---~~---"
- - ... - - - - - - -
1/ ~
/

. \;'
il
.il
z
c.
g

~
I
I

a
'' f
"
~ .... ~ ......
"

j ' I

"' ' '' ''


~~ :
e - 'li
j~
3 8
"
~
,.. ~
.,,c
0
s
.5 "
ii!
"'
32 DISPLACEMENTS, STRAINS, AND STRESSES

Since Y,.1 = 0 and Yn = 0, Eq~ (2.22) and (2.27) give


f23 =0 r11=0. (2.72)

Furthern1ore. since the strain and stress components are independent of x3 ~


\Ve have

(2.73)

With these stresses the relevant e quilibrium equations (Eqs. 2.13 and 2.14) -
in the x 1 x2 x3 coordinate syste111 - reduce to
T

(2.74)

(2.75)

l11e stress components <Y1, <12, rti:, 'vhich appear in the equilibriun1 equations.
are illustrated in Figure 2.24 (top, right).
By substituting Eq. (2.70) into Eq. (2.22) and by utilizing Eq. (2.27) we obta in
the following stress- strain re lationships (or inplane loads:

(2.76)

" ' = [C,. C,, C.1] { :: } + Ci1<3, (2.77)


Y12

where, we recall, <> is (Eq. 2.69)


1 1 .
~.l = - X1
f>l
+ -l2
Pi
+ ~1
.
(2.78)

The strain- stress relationships are obtaine-d from Eq. (2.76) by inversion

:: } =
{ Y12 [~:: ~: ~::] ({=: } - {~~} ).
R1 & Ru. ~ r12 CJc\
(2.79)

'''here Rt; (i , j = 1. 2: 6) are the in-plane elen1ents of the compliance n1atrix in the
x 1, x2 coordinate system calculated fron1 the relationship

R,.] = [c" c c,.]-t


12
R16
R,,. c,. c,. c..
C12 Cu C16 . (2.80)
2.4 PLANE-STRAIN CONDmON 33

011t-of-pla11e laad.<. The body is subjected to the out-of-plane loads shown in


Figure 2.20. Under these loads the body n1ay undergo only nonplanar and angu
la.r deformations (Fig. 2.25), and the d isplacements (in the x 1 xi. x, coordinate
system) are (see E q. 2.53 and Fig. 2.23)

Ut = - OX2X3
U'l = t?X1X3 (2.81)
u, = U,(x, ,x,) .

For these displacements the st.rains a re (see Eqs. 2.2-2.4, 2.9-2.11)

, , =0 2=0 (2.82)
~J =0 (2.83)
au_, au,
Yl:.l = -. -+1Jx1
ax2
Yl3 = -,ax- -
1
''Xi (2.84)

Yl2 = 0. (2.85)

Only y 23 and Ytl a re nonzero. Correspondingly. the only nonzero s tresses are
r,., a nd r23 T hus, two of the equilibrium equa tions (Eqs. 2.13 and 2.14) become
trhia l, and the third (Eq. 2.15) - in the x1 , x1 , .'(l coordinate systen1 - reduces to

(2.86)

The r.n, r 13 s tress compo ne nts, which appear in t he equilibriun1 equation, are
illustrate d in Figure 2.24 (bottom, right).
From Eqs. (2.22) and (2.27) we obtain the following stress- strain relationships
for out-of-plane loads:

l'"l=[c,, . c.,]Jv"l
'!13 C45 C.ss Y13.
(2.87)

The s tra in- stress relationships are obtained from E q. (2.87) by inversion

I I [s.. s.,JI'"l
y,,
Yt l = S.as Sss r-11
(2.88)

where the compliance mattix (SJ is the inverse of the s tiffness matrix in Eq. (2.87)

[s., s.,]= [c"' c,_.]-'


S.u . Sss C4s Css
(2.89)

E11d axial foods. Axial loads are applied at t he ends (Fig. 2.21 ). Unde r such
loads the body may undergo only planar, bending, and axial deformations
34 DISPLACEMENTS, SIBAJNS, AND STRESSES

(Fig. 2.25), a nd the displace ments are (see Eq. 2.53 and Fig. 2.23)

I I
u1 = U1(x1.x2) - - - x32 (2.90)
2 Pi
. I I
lll = U1 (l'.1. X2) - ?- -XJ2 (2.91)
- P1

UJ = ( - I X1 + - 1 .'( ? + 3) XJ . (2.92)
Pi Pi

These displacements a re the same as those for in-plane loads (Eq. 2.67). He nce,
the s train- displacen1ent, equilibrium, a nd stress-strain rela tionships are the san1e
as those given by Eqs. (2.68)- (2.80).
E11d mome11t. A hending moment is a pplied at each end (Fig. 2.21). Under
this loading the 'body may undergo only planar, bending:, and a xia l deforma tions
(Fig. 2.25), a nd the displacements are (see Eq. 2.53 and Fig. 2.23)

I I ,
111 = U1(x 1, x2) - - -x;
2 Pi .
I I
Uz = U2(x1, x1) - - -x32 (2.93)
1- P1

llJ = ( -I x1 + -1 :e:z + ,.3) X3 .


Pl /JI

These displacements are the same as those for in-plane loads (Eq. 2.67). Hence.
the s train-displacement, e quilibrium, and stress- stra in relationships are the sa111e
as those given by Eqs. (2.68)- (2.80).
Torque. A torque is a pplied at each e nd (Fig. 2.21). Unde r this load the body
may undergo o nly nonplanar and angular d efor1na tions (Fig. 2.25). and the dis
placemen Ls are (see Eq. 2.53 and Fig. 2.23)

113 = l/3(x1 , .n). (2.94)

l11ese displaceme nts are the same as those for o ut-of-plane loads (Eq. 2.81 ).
Hence, the strain-displacement, equilibrium, and stress-strain re la tions hips are
the same as t hose give n by Eqs. (2 .81)- (2.88).

2.4.3 Free End - Orthotropic, Transversely Isotropic, or Isotropic Material


When at le ast one end of the body is free, the expressions: given in Section 2.4.2are
also a pplicable to bodies treated as orthotropic~ transversely isotropic, or isotropic
provided that one of the materia l symmetry planes coincides \Vith the X1- X2 plane
s hown in Figure 2.16. For s uch bodies the following simplificatio ns apply:
Orthotropic

c,. = c,. = c,. =0 R,. = R,. = 0. (2.95)


2.4 PLANE-STRAIN CONDITION 35

Tra11svenely isotropic
c,. = c,. = c. = () (2.96)
C 11 = C 12.

Isotropic

c,. = c,. = c. = 0 R1 = Ru, = 0


C21 = C 11 R21 = Rl1
(2.97)
c.. = ! (C11 -
C12) R. = 2 ( R11 - R12)
C13 = Cn = C11.

2.4.4 Built-In Ends - Generally Anisotropic Material


When both ends o f a generally anisotropic body are built-in (Fig. 2.18), in-plane
as well as out-of-plane loads may be a pplied (Fig. 2.20). The body may undergo
only pla nar and nonpla na r deformations (Fig. 2.23). Howeve r, the longitudinal
axis o f the body remains straight ( l / p,. l / p, = =
0), its length remains constant
(<~ = 0), a nd the body does not twist (~ = 0). The pe m1issible delonnations are
summa rized in Figure 2.26 and t he displacements are (see Fig. 2.23)

u =U(x, y) v=V(x , y) w = W(x, y). (2.98)

These displacements are the same as the displacements given by Eqs. (2.50)-
(2.52) for a body with free e nds, when 1/ p,., l /p, . <~. and 1) a re se t equal to zero.
Thus, the strain-displacen1ent. the siress-strain, and the equiHbrium equations
are obtained by setting 1/Py. t/ p.1 , E:,
and O equal to zero in the equations o f Sec~
tion 2.4.1 . TI1e stra in- d isplacement re la tionships (Eqs. 2.54-2.57) become
au av
<x = ax .,. = ay (2.99)
Ez =0 (2.100)
aw aw
Yy=-
' ay Y.t ~ = a;- (2.101)

au av
. . = -+-
)".~
ay ax (2.102)

The equilibrium equations are identical with Eqs. (2.59)- (2.61).

~o m1ation
L..,d ...__ Planar Nooplanar

Out-of-plane

Fi_gure- 226: Oeformatjons of a generally anisotropic. body under plane-strain conditjon for
different lo ads; buih-in ends.
36 DISPLACEMENTS, STRAINS, AND STRESSES

The stress-strain relationships (Eqs. 2.62 and 2.63) become


Cn c,, c,4 C1s c,. <x
""
"r c,, Cn C24 C2s c,. <y
Ty.;: = Ct, c1A c.. C4s c46 y,.. (2.103)
r.t.::. C1s Cis c. . ..'i Css c,. Y:r .::.
T.vy c,. c,. c... Cs c.. r..,.
...
Er
cr, =(Cu C23 c34 CJs c,.J Yr; (2.104)
Y.r.::.
Yxy

By inverting Eq. (2.103), we obtain the strain-stress re lationships

<x Rn 'R,. Ris 'R,.


~.\' R12
R 12
Ri1 R14 'R" Ru ""
Yyz = R1J RlJ Rw R.,, R.. ""
rf,. (2.105)
Y.T;: R 1:s RB R.,, Rss Rst. r .q:
Yxy 'R,. Ru, R.. R :;f, R.. f.1:y

where [R] is defined by Eq. (2.66).

2.4.5 Built -In Ends - Monoctlnic Material


We oonsider a body made of a monoclinic ma teria l with the pla ne of n1aterial syn1
me tT)' coinciding with the X 1- X1 plane sho'vn in Figure 2.16. Both ends of the body
are built-in. The longitudinal axis of the body remains.s traight (l/ p,. = I/ p.. = 0),
its length remains constant (<~ = 0). and the body does not twist ( 0 = O); hence,
the body may undergo only pla nar and nonplanar d eformations (Fig. 2.23). The
deformations resul ting from specified loads a re s ummarized in Figure 2.27.

D fi
~n Planar Nonplanar
l.-Oad

In-plane F'::'--!V

Out-of-plane
Q-
Figure 2.27: Defo rmations of a monocllnicbod)' under plane-str ain condition for differe nt loads:
bnill-in ends.
2.4 PLANESTRAIN CONDmON 37

frlplane load.<. The b<Jdy is subjected to the in-plane loads shown in Fig
ure 2.20. Under these loads the body undergoes planar deformations (Fig. 2.27).
and the dLplacements are (see Eq. 2.53 and Fig. 2:23)

u.,. =0. (2.106)

These displacen1ents are the same as the displacements of a body with free
ends with 1/ Pi. l / p 1, and r
set equal to zero in Eq. (2.67). Thus, the strain-
displacen1ent, the stress-strain. and the equilibrium equations are obtained by
setting 1/1>1., l/p1 , and lJ equal to zero in the equations of Section 2.4.2. The
strain~isplacement relationships (see Eqs. 2.68-2.71) are

E1=-
au, E-,=-
au1 (2.107)
ax1 - ax1
,, = () (2.108)
Yi. =0 Yu=O (2.109)
au,
ni= -
au,
+- (2.l 10)
IJx2 &x1
The equilibrium equations are identical to Eqs. (2. 74) and (2.75).
The stress-strain relationships (Eqs. 2.76 and 2. 77) become

(2.111)

(2.112)

By inverting Eq. (2.111 ) , we obtain the strain~tress re lationships

(2.113)

The in-plane e lements of the compliance matrix R1; are defined in Eq. (2.80).
Out-of-plane ltiads. The body is subjected to out-of-plane loads shown in Fig
ure 2.20. Under these loads the body may undergo only nonplanar deforn1ations
(Fig. 2.27), and the displacements (in the x 1 x1 x3 coordinate system) are (see
Eq. 2.53 and Fig. 2.2.'l)

U1 =0 U2 =0 (2.ll4)
These displace1uents a re the same as the displace1nents of a bod)~ with free
ends with 11 set equal to zero in Eq. (2.81). Thus, the stra in- displacement and
the stress-strain relations as \vetl as the equilibrium equations can be obtained by
setting 'iJ equal to zero in the e quations of Section 2.4.2. The strain-displacement
38 DISPLACEMENTS, STRAJNS, AND STRESSES

relationships (see Eqs. 2.82-2.85) are

,, = () (2.11 5)
fj =0 (2.ll6)
au, au,
Y2. = -ax, Yl.l =ax, (2.117)

y12 = 0. (2.118)

The equilibrium equation is identic.al to Eq. (2.86).


The stress-strain relationships (see Eq. 2.87) are

J'"I [c..
l T1J = C45 C.s,-,
r"I
c~,] flYu . (2.119)

By inverting this e quation, 've obtain the strain- stress relationships

1I ,,,,}
Yt.l =
[s"' S.s]
S4sS..,s
Ir,, l
r13 .
(2.120)

where t he compliance matrix is given by Eq. (2.89).

2.4.6 Built-In Ends - Orthotropic, Transversely Isotropic, or Isotropic Material


The expressions given in Section 2.4.5 for monoclinic bodies v.'ith both ends
builtin are applicable to bodies treated as o rtho tropic, transversely isotropic,
or isotropic provided that o ne of the n1aterial S)mn1e'lry planes co incides v.1th the
x1- x2 pla ne shov.n in Figure 2.16. Ho\iever, ror such bodies the simplifications
given by Eqs. (2.95)-(2.97) must be e mployed.

2.5 PlaneStntss Condition


lJnde r p lane.stress condition one of the norn1aJ stresses a nd both out~of- plane
shear stresses are zero. \Ve select the norn1al stress to be zero in the z (or XJ)
coordinate direction and the out-of.plane shear stresses to be zero in the x- y (or
x1- x2) plane (Fig. 2.28)

O;; =0 ,,,, =O rx;; =0. (2.121)

Plane...stress condition n1ay approxilnate the stTesses in a t hin fiber-reinforced


composite plat e whe n the fibers are parallel to the x- y plane and the plate is
loaded by forces along the edges such that the forces are parallel to t he plane of
the plate and are d istributed uniformly over the thickness (Flg. 2.29). The plane
stress condition does not provide t he stresses exactly, not even for this thin ...plate
problen1. Nevertheless, for ma ny thin 'vall structures it is a w eful approxima tion.
yielding ans,\ers 'vithin reasonable accuracy.
2.5 PLANESTRESS CONDITIOH 39

-.
"

7:u -= 0
Figure 2.28: 1bc stresses under plane-stress condition.

With thestipulationsofEq. (2.121), theequilibrium equations(Eq~ 2.13-2.15)


beco1ne
aa, ar,, f - 0 (2.122)
ax + ay + x -

ar,, a.., f
17+ay+ .. = .
0
(2.123)

Equations (2.121) and Eq. (2.'21) gi"e

(2.124)

(2.125)

The e lements of the con1pliance n1atrix.S 11 are given in Table 2 .3 (page 10). The
stres..~-strain relationships are obtained from Eq. (2.l24) by inversion

Q,, i?
g12 Q,.;] {<y
<x } , (2.126)
Qi6 Q66 Yr.v

Figure 2.29: In-plane loads applied to a thin plate. resulting in planc.-stress condjtion.
40 DISPLACEMENTS, STRAINS, AND STRESSES

'vhere Q ;i are t he inplane e len1e nts of the stiffness m atrix in the x . y ooo rdina te-
systen1 under plane -s tress condition:

(2.127)

The stress-strain and stra in- stress relationships s implify cons iderably when
the fiber orientatio ns are such that the n1aterial can be treated as n1onoclinic
(Fig. 2.9), orthotropic(Fig. 2.12), transversely isotropic (Fig. 2.15), or isotropic. In
the follo,ving \Ve cons ider problen1s invol\'ing these mate ria l types and use an x 1
:c.2, x3 coordinate syste m with the x 1, x1 coordinates be ing in o ne o f the s ymmetry
plane s and x3 be ing pe rpend icular to thL" pla ne. In this coordin ate systen1, under
plane ..stress condition, we have (Pig. 2.28, right)

rt~ =0. (2.128)

Mo11oc/i11ic maurials. By substituting Eq. (2.US) into Eq. (2.23} a nd by e m


ploying the complia nce m atrix o f a mo noclinic mate rial (Eq. 2.26). we obtain

Yll = 0. (2.129)

With the s tipulations in E q. (2.128), the equilibrium equations(Eqs. 2.13-2.15)


in the x 1, x2 , x 3 coordinate systen1 become

(2.130)

3r11 a,,, f
- + - + -'= 0 .
axa <Jx2
(2.131)

By s ubstitu ting Eq. (2.128) into Eq. (2.23), we obtain t he fo llowing stra in- stress
relationships:

:: } =
{ Y12 [~:: ~ ~::]
S u; 5'.t. 5(,.,,
{:: }
t 12
(2.132)

(2.133)

The e le n1ents o f the con1pliance m atr ix S;i are given in Ta ble 2.7 (page 15). The
s tress-strain re lationships are obtained from E q. (2..132) by inversion

(2.134)
2.5 PLANE-STRESS CONDITION 41

where Q;; are the inplane e lements of the stiffness maui..x in the x 1 , x1 coordinate
systen1 unde r planestress condition:

Q"] = [s"
Q26 S 12
s,, S16S22
s,.]-' . (2.135)
Q.. s,. s,. s..
l11e expressions presented in this section for monoclinic n1aterials also apply
to orthotropic, transversely isotropic, and isouopic materials. Ho\vever. for these
types of 1naterials the stiffuess and con1pliance n1atrices further simplify, as shown
below.
Orthotropic. For an ortbotropic material the follov.ing e len1ents of the com
pliance matrix are zero (Table 2.7, page 15):

s,. = s,. = 0. (2.136)

Also, from Table 2.7 we have

S11 = -
1
E,
s,.,=
-
- ""
-E, s,,- = -E11 (2.137)

Prom Eqs. (2.135) and (2.136) we obtain

Q,. = Q,. =0. (2.138)

Accordingly, the e lements of the stiffness matrix [ Q) in terms of the e ngineering

l
constants are (Eqs. 2.135, 2.137, and 2.138)
t:, 11:.E:
/) /)

[QJ = [ ' f' "'


D

0
0
G12
. v.here D = 1-
E2 ,
-Vj-,
E, -
=t- V12 "'I
-

(2.139)

A \Voven, or filament wound, layer is orthotropic when there is the san1e num
ber of fibers in the +El and - fl directions in the x 1- x2 plane (Pig. 2.30). Por such
a layer the elements of the stiffness n1atrix are calculated from

Q'{'; """ = ~[( Q;;)., + (Q;;)-.,]. i , j = 1. 2, 6, (2.140)

where ( Q;;)e and (Q;; )_., are the eleme nts of the stiffness matrices of plies ori-
ented in the +e and - 0 directions.. respective ly. The elements of the stiffness

-e~~:,
~ <~~
+e
-Figure 2.30: A laye-r consisting of ti hers in the +e and -<-) directions.
42 DISPLACEMENTS, STRAINS, ANO STRESSES

Table 2.12. Elements of the stiffness malriX of a


woven layer with the fibers oriented in the 0
directions
QiJ"""" =C4 Qu + s4 Qu + 2c2s? ( Q12 + 2QM.)
Q"'.?2''11 = s-1Q11 + c"(h: +2c: 2s? (Qi: + 2Q>J
Q~j"11 =<.2s? ( Q11 + Qn - 4~) + (c:'' + .\..}Qi:
' ' (Qu
Q(',r'1" = cs- + Q12 - 2Qu) + ( c-) - 2')~~ Q 66
.'i

Q'j;:v''" =0
Q"; vn =0
s =sine

matrix in the f) direc6on ( Qi;)s are obtained by the transformation describe d in


Section 2.9.3 and summarized in Table 3.1 (page 70). The e leme nts (Q;;)-<> are
calculate d by replacing f) by - f) in Table 3.1 . By performing these ste ps, we obta in
the e len1ents Qi;v.iw:n ghen in Table 2.12.
Transversely isotropic. For a transversel}' iso tropic ma terial \\le have
(Table 2.7, page 15)

s,. = s,. = 0 (2.141)


S,, = S12. (2.142)

By substituting these values, together \vith the expressions in Table 2.7


(page 15), inlo Eq. (2.135). we obtain the [ QJ matrix in terms of the e ngineer
ing constants. The results are identical to those of Eq. (2. I 39).
l.wtropic. For an isotropic material the following relationships hold (Table 2.7.
page 15):

S21= S 11 s,. = s,. = 0 (2.143)


Su = S 23 = S11 S.. = 2 (S11 - S,,).
From Eq. (2.135) we obtain

Q,, = Q11 Q,. = Q,. = 0


I (2. l44)
Q = 2(Q11 - Q12).

The ele n1ents of the stiffness 1natrix in te nns or the e nginee ring constants are

(QJ= - - " 1
E
l - 1,l () ()
[1 " 0()
!;:
]. (2.145)

'
2.2 Exa1nple. Ca/c14/ate the eletuents of the stiffness tnatri:t: of tt graphite epoxy 1111i
directioual ply under plane-stress condition. The e11gi11eering constants are given as
1 = 148 x 10 Nini'. 2 = 9.65 x IO'Nlm2 , G 12 = 4.55 x t0 Nini' , n = 0.3,
and l'2J = 0.6.
2.5 PLANE-STRESS CONDITION 43

Solution. Under plane-stress condition the stiffne.ss matrix is (Eq. 2.135)

[~::
Q,.
~~
Q,.
(2.146)

The elements oflhe compliance matrix are given in Example 2.1 (page 21). By
substituting these values into the e xpression above obtain

[
\Ve

_,
- 2.03
103.63
0
0
0
219.78 ] =
148.87 2.91
2.91 9.71
0 0
o
4.55
0] io2N.
m
(2.147)

The stiffness matrix may also be calculated by Eq. (2.139) with the e ngineering
constants and with D = 1 - ~: v/2 = 0.994. Equation (2.139) gives

,,
"
[QJ= [
"t 0
0
0 ] 109 -N, .
4.55 m-
(2.148}

This is the same result as that given by Eq. (2..147).

2. 3 Exan1ple. Estilnare tile plane-stress stiffness nu1trix and the engineeriug con
stants of a (45) it-oven fabric layer 1nadf of graphite fibers a11d epo.'C.y resin.

SolutioJL We approxin1ate the \\'Oven fabric by a layer 111ade of one 45e and one
- 45 ply. The properties of these plies are take n to be those of the graphite epoxy
unidirectional ply give.n in Exan1ple 2.2. The stiffness matrix of a ply made of this
material under plane-stress condition is (Eq. 2.147)

~ ~
148.87 2.91
(QJ = l .91 9.71 ] 10 (2.149)
[ 0 0 4.55 m

We obtain the stiffness matrix of the \voven fabric by substituting the elements
of this matrix into the expressions in Table2.12(page42) with(~)= 45. The result
is

Q~," = Qr,' = 45.65 x 109 Nim2 Q'lf' = 36.55 x 10 9 Nim2


(2.150)
""'"
~ 16 -- ,....,
\t'.26 -
- 0 ctt' = 38.19 x 10 9 Nim2 .
44 DISPLACEMENTS, STRAINS, AND STRESSES

The compliance n1atrix is

s.,
]~" = [~' ~ ]-l
[ s..
~;'
Su
0
00
s,.,.
Qft'
0

61.03
l2
Qt"
0

- 48.86
1 45

= - 48.86 61.03 0 ] 10- 12 01'- . (2.151)


[ 0 0 26.19 N

The engineering constants of t he y,oven fa bric are (see Eq. 2. 137)


,,,.,_, I 2
9N
"l = T
1 1
= 16.39 x lO -
m
vt;'' =- S12E1 =0.801
2
(2.152)
E;''-' = ...!_ = 16.39 x
- S22
10!::!
m
""" -., = -1 = 38.19 x 10 9 -N'.
..,,,
- S,.,. m

2.6 Hygrothermal Strains and Stresses


An unrestraine d composite may change both its size and shape 'vhen the ten1per
ature is increased or decreased uniformly by 6. T. The conesponding strains in the
x, y. z coordinate system are

i[ =iil(D.T
(2.153)
y1~ = ii>'~a T ,. = -a.1.va T
Yxr

Sin1ila rly, 1nois ture inside an unrestrained composite causes a change in size
and shape. A uni(orm moisture concentration c in the material results in the
rollo\ving s trains:

<; = {J,c
{' - (2.154)
Y; y = fi.l.J'c.

Inside a dry material the moisture concentration c is zero. ln Eqs. (2.153) and
(2.154);; and Pare the te111perature and 1uoisture e xpansion coefficients. respec
tively. (Note that(; and ~ follo'v the same transformation rules a s strains.. Section
2.9.2.) In an unrestrained composite the strains induce d by uniforn1 te111perature
and n1oisture disuibutions (referred to as hygrothermal strains) are

Ci, l,
',.
~
Ci,
.hl <i:
,Yxr.
yt,I = t.T + c. (2.1 55)
.'<
a,~ '$,.
y ot.,~ lx:
bl
axy PJ.:y
2.6 HYGROTHERMAL STRAINS ANO STRESSES 45

For a generally anisotropic material, in the x, Y~ t coordinate systen1, the


strain- stress relationships are obtained by combining the hygrothermal strains
(Eq. 2.155) with the stress-induced strains (Eq. 2.21 ):

3'11 s,, Sis s,. "'


.," slS s,. "
S12 S1J x
S21 s,, S.n '524 aJ' "'
y
s" s" S1s s...
:s33 S3.i
"'
Y)'t
= S.n S.u s,3 s4.. s., s... "' 'Cy.;
+ '
yhl
(2.156)

Ssz Ssl Ss.a Sss s,. y"


Y.t:: s" s(tl 1.x.; hl
.T.;
Y.TJ' Sb1 s,i.l s.., s"' s.,. T.x.v
bl
Y.TJ'

where ' x . . . , Y.t)' are the -actual strains in the composite.


By inverting Eq. (2. I 56) we obtain the following stress-strain relationships:

<fx

dy
C11
C21
C 12
C 22
C13
Cz.1
Ci"
C 211
C 1s
C2s
c,.
c,.
<x
<y
.....,.
y

'" C31 c" c 3.l C 3:1 CJs c36


"'11
r,.::
=
C.n C.u
C -12 c4..
c,.
C4s
~<;5 Cs6
c,. "
Yn Yy::
rx:: c"
c.,
Cs2 Cs.l
c6., c6~ c., c66
Yx::
v"
bl
Y.t::
f".l,I' c62 y,>' xy

(2.157)

For a mo noclinic material, in the ,t1 , x2 , x 3 local ooordinate system (Fig. 2.9)
ten1perature and moisture do not induce out~ofwplane shear strains. Thus, the
temperature and moisture induced strains are

lr =U16.T .:[ = Ci21> T ,_r = &3.ti r


y[, =0
,.
Y13 =0
,. -
Y12 = <r126. T
(2.158)

tf ='J1c ~ -
= fi2c li = P3c (2.159)
Yb =0 Ytt3 = 0 Yfi = P 12C.

where~ and Pare the temperature and moisture expansion coefficients, respec-
ti\e-ly, in the x 1 x2, x3 coordinate system (ti and Pfollo\v the same transformation
rules as strains. Section 2.9.2). The hygrothermal strains are
,.,I
c;, p,
l~l c;, p,
Ebt
l
c;,
l>T +
p, c. (2.160)
yfi = 0 0
yhl
ll
() 0
yhl ti12 # 12
12
46 DISPLACEMENTS, STRAINS, AND STRESSES

"' .,
The stress-strnin relationships are (Eq. 2.22)
......
.,
I
"2 2 1
<13
= lCJ
' (2.161)
t1J Y.?3 0
! i .\ yu 0
r12 Y12 yfil
where [Cl is prescribed by Eq. (2.27). Note that ' " . . , y 12 are the acrua/ strains
W the composite.
By inve rting Eq. (2.161), \\'e obtain the follo\ving s train- stress relall:io nships:
., "'
., I
,~1
"'
,~
"
YJj 0
= [S J
<1J
r21
(2.162)

Yt.l 0 rn
Yll y"
11 r11.

where (SJ is prescribed by Eq. (2.26).


In ortho tropic, transversely isotr opic, a nd isotropic 1naterials tempe ra ture and
m oisture do no t introduce y23 , Y Lh .a nd y 12 shear strains. Thus, for t hese types of
ma terials the te 1n perature ~ and mois ture ~ induced strains are

'T=a,&r ,[ =a2ar ,J = a3aT (2.163)


y~= O y1~ =0 y,~ =0
'1' = ~ ,c ,~=Pie ,~ = P3c (2.164)
Y!J =0 Yf1 =0 Yf2 = 0.

For orthotropic, transversely isottropic., a nd isotropic materials the s tress- st rain


r elationships given a bove for n1onoclinic n1aterial a re valid (Eqs. 2.161 a nd 2.1 62)
with y~ = 0 a nd the [SJ and [CJ matrices specified in Sections 2.3.4 and2.3.5 (see
also Table 2.13).

Table Z.13. The lhermal and moisture expansion coefficients


Thermal Moisture
Material expansion coefficients expansion coefficients
Generally (;.\'a,, (.i~, f.i.H ;;At (.iX\' ft' . 'if,. 'Pt. F,~ F~~ 'if:"
anisotropic
t\'lonoclinic a" i. a~. al? , ,,Ji,.,,
Orthotropic a., 1.a~ -;i, , ,,,
Ttansve.rsely a" ai = a.i , ,-P, ="ii,
iso tropic
lsotr<>pic 1 = <ii = J "ii, =ii,=#,
2.7 BOUNOARY CONDITIONS 47

2.6.1 Plarne-Strain Condition


The hygrothern1al stress-strain re la tionships under plane-s train condition can be
obtaine d by n1aking the folJo,ving s ubstitutions in the stress-strain relationships
(without hygrothe mial effects) given in Section 2.3:

f; ~ ~,. - ii; 6. T - P;c (2.165)


Yii => Yii - a;; 6 T - ft1;<. i ;! j

where the subscripts i j re present x. y, z and 1, 2. 3 in the x. y , t and x 1 , x 2 x3


coordin ate systen1s. respectively.

2.6.2 Plarne-Stress Condition


The hygrothermal stress-stra in relationships under plan e~stresscondi tion e-an be
obtained by replacing the strains<; and y,; in Eqs. (2.124) a nd (2.145) by those
given in E.q. (2. I 65).

2.7 Boundary Conditions


To o btain solutio ns to the equiJibriun1, stress-strain, and strain-displacen1ent
equations, eithe r the displacement or the a pptied force n1ust be spec.ified a t every
point on th e s urface. The re la tionships bef\veen the displacements and the strains
are given by Eqs. (2.2)- (2.4) and (2.9)-(2.1 L). The surface forces pe r unit area are
related to the stresses at the surface as follo\\'S.
We consider a small volume e lement a V (Fig. 2.31 ). A s urface force pe r unit
area p (\vith con1ponents P.i , p_... 11~) is applied on the Ll1\surface of t his e le n1ent.
Force bala nce in the x direction gives

(2.166)

B
11
x
"
Figure 2.3l: Componc.nts of the s urface (orce p on the l.!t.Asurface an d thc slr<:$SCSo n the one.
OAC.~, and OABsurla-ccs.
48 DISPlACEMENTS, STRAINS, AND STRESSES

:r: A
Figure 2.32: 1he angles "''* I.JI_:-, Wt .

By di,;ding bo th sides by ll.A we obtain


1
Pr =
2
.D.A (<1,ll.tll.y + r.,ax ay + r,_.axai) . (2.167)

In the limit, when ll.A, ax, ay, t>t go to zero, Eq. (2.167) becomes
Ps = 1'1.T + H f :,i + lll f y.t (2.168)

\vhere I , 111, 11are the direction cosines of the normal vector of the boundary surface
(Fig. 2.32) as follows:
azay axaz a xay
I = - - = cos('I' ) m = - - = cos('I',.) 11 = A = cos('I',).
2<'.l.A "' 2<'.l.A 26
(2.169)

Similarly, we have

Pr = nu:r,. + Ir>..,.+ urzy (2.170)


p, = 11<1, + Ir,,+ mr" . (2.171)

2.8 Continuity Conditions


When con1posites are made of laminated layers, perfect bonding is assumed be
tween each layer. Accordingly, at the interfac.es of t\vo adjacent layers the norn1al
stresses are equal, the out.of. plane shear suesses are equal, and the displ.acen1ents
a re equal. Thus, a t two adjacent layers, denoted by k and k + I. we have (Fig. 2.33)

(<>',);., = (O'J ; +1.b (r,Ju = (r,,).+1. (r, J;.1 =(r..,);+1.b (2.172}


(11)~1 = (11)<-+l.b (v);_, = (l.tt1.b (w);. 1=(wJ.+l.b (2.173)
The subscripts t and b refer to the top and bottom of a layer, respectively, The
continuity conditions in tern1s of strains can be obtained from the preceding dis-
placement cootinuity co nd itions (Eq. 2.173) together with Eqs. (2.2)-(2:.4), (2.7),
and (2.11 ) as follows:

(<,);.1 = (<., ).o+l.b (y,, )._, = (Y.., J.. 1.b- (2.174)


2.9 STRESS ANO STRAIN TRANSFORMATIONS 49

HI

Figure 2.33: lbe stresses at the top and bottom surfaces o f hvo adjacc:.nl lap:rs thut n1ust match
to satisfy cont1nu1ty.

TI1e dN. <r,. rx.l' s tress a nd the f:., YK<- Yy:. s train components are not necessarily
oon1inuot1s along two adjacent surface~

2.9 Stress and Strain Transformations


\\'e consider t'vo Cartesian coordinate syste ms \Vith axes p, 'l r and Ji. q', r'.
The orientation of the primed coordinate systen1 \vith respect to the unpritned
coordinate syste m is give n by the nine dire ction cosines (r11 , r2 1. r 3 1) . (r12. r11. fJ1).
("3. rn , '3.' ) specified in Table 2.14.
The o.rientation of the prin1ed Ji, q'. r' coordinate system " 'ith respect to
the unpri1ned coordinate systen1 p , q, r can a lso be specified in terms of three

Table 2.14. Definitions ol the direction cosines

r r'

p~
' A
,,
n.,.
~ "1'
q
,,
r u = cos.sl,.p nz= cosQpo) r u =cos!?"'
r ;:i = cos Q1p r 2i = oos n.w ri;.= cosQ,11
r 31 = c:.osO,.., r~ =cos!?,,., r" = cosn,.
50 DISPLACEMENTS, SIBAJNS, ANO STRESSES

r ,.
p: n,.. ., o"
,,
p n.,
q

~
. .e, ~-
e,
q ,, q
' p ~ p ~ p ~

.Figure 2.34: Con.o;ccullve rola tions to <irrr\'c at the coord1nale system ff, q'. and r' .

.angles f-) P eq, er (Fig. 2.34). These angles a re consecutive rotations of t he prin1ed
-coordinate syste1n about the p, q. r axes, as illustrated in Figure 2.34. The a ngles
e,, H., f), are positive in the counterclockwise direction.
The e p t.:+q, <-:+r angles are rela.ted to the direction cosines by the follo\\~ng
-expressions:1:

e. = Atan2( - ' '" jrf1 + rf1)


e, = Atan2(r11/ cos e., rn / cost-)q) (2.175)
fJ, = Atan2(rn/ cos e., r_,,/ cosflq).
"\vhere Atan2(y, x) is a two.argument arc tangent function as ColJo,vs:

Atan2(y. x) = tan - 1 (~) when x > 0 (2.176)

Atan2(y.x) = tan- 1 (.1)


x
+" when x < 0. (2.177)

When fl, = 900, the n fJ, a nd e P are


e, =O e, = Atan2(r12, Tz,). (2.178)

2.9.1 Stress Transformation


T he stresses in the prin1e d coord inate systen1 a re calculated fro1n the stresses in
the unprin1ed coordinate system by t he transfom1atioo
<>'p 'tu 'fn 12 r., 13 'fo14 7;,.15 Ta 16 "r
<>'q 't21 'fn22 T.1.3 7;,24 'fn25 r.,. a,,
<r' T.,.u T.34
'
r~r
=
7;,31
To.st
Ta3!
Ta4! r.., r....
""l5
""45
'fo:16
'fo46
"'
rq,
(2,179)
r' Tosi 'l:ts1 TasJ 7;,54 ""55 r.,56 r,..
"'
r'pq 't6t 4'62 7;,63 r... T a-65 r. .. r,.,

-' J. J. Crnig.. /11t1lduaiot110 R<>horics { AtttJ1a11i" ' a11d Co111rol). 2nd cdi1ion. Addison Wesley, Reading.
~l a.s.s11ch ~ells, 1989. pp. 43-56.
2.9 STRESS AND STRAIN TRANSFORMATIONS 51

Table 2.15. The slress transformation matrices

~,
0 0 0 0 0
)
0 cp s1r 2t (ISp 0 0
0 \.:? c'r -2Lsp 0 0 c11 = cose,..
p
[1':] = 0 -<,,s, C:pS p c:~ -s; 0 0 s,, =sin e ft
0 0 0 0 c, -.r,,
~o 0 0 0 Sp Cp

- (..?
0 ,
..,2 0 2c:.,s., Cl
0 I 0 0 0 0
[1;,] = s'., 0 c' 0 -2c;;.s1 0 c" =Cose,,
0 0 0' c 0 -s., s11 =sine,,

-
~
- <.,,s.,
0

c;'
0
0

s;'
0
0

0
.,
(....,...


0
s
~

0
-.r; ,.0
0

2c.s,
...
<,'
.,\' ! 0 0 0 - 2t,s~
0 0 0 0 0 c:. = co,;(:),
(1'; ] =
0 0 0 c. - s, 0 s, =sin e ,
0 0 0 s, c, 0
'--<.,sr <., s, 0 0 0 c:? - \'l
' ., .

Equati on (2. 179) ca n also be written as


d ' = [?;,IO', (2.180)
where I1;;. ] is t he trans fom1ation matrix, which n1ay be expressed as
[1;.] = [t/] [1;.J[t;]. (2.181)

where ( ?'/], [t,;11, (t;] are given in Table 2.15. The subscript t1 refers to t he stress
transforma tion, and the bat o n T indie-ates that a ll six stress cotnponents are be ing
transformed. The superscripts p, q, and r refer to the transforma tions a bout the
p. q , a nd r a.xes. respectively.
Plane-s train attdplane-stress. tJnder plan e.strain and plane-stress cond itions
we are interested only in the stresses in the p-q and p'- qf planes. In this case
the stresses in the prilned coordina te system a re obtained from the stresses in the
unprin1ed coordinate syste m by rotation about the r wa xis (see Fig. 2.35) as fo llo\\os::

(2.1&2)

c = cose s = sin 0. (2.183)

Equation (2.182) may be written as


d ' = (T,,j O'. (2.184)
52 DISPLACEMENTS, STRAINS, AND STRESSES

Figure 2.35: Rotallon of the. coord1n<1tc S)'Slc-n1 a round


lhc r -a:us.
'
~.
</

111e. subscript a re fe.rs to the stress tra ns fo nnation, and T \vitho ut a hat ind i
c~ues that o nly t hethree in plane stress cotnponents a re trans fonn ed.
4

2.9.2 Strain Transformation


T he s tra ins in t he pri111ed coordinate system a re calculated fron1 t he stra ins in the
unprin1ed coordina te systen1 by

;,
'<'
411 Tc 11
T.21
h.H
7;22
T..u
7;13
T,,,
r,. l 4
T,,.
-,;,.
7; l5
T,25
Tc1ti
'f;zb
'/;Jo
..
<p

y,,"..
'433 '[;35
' = (2. 185)
Y~,. 7;:11 r.~12 f.43 T,44 1;45 'l;ao
Y;, 7;sl r.~2 T.s.l Yc$.t T,,5 7;~6 y,.,
y;"I 7;,,1 T.62 T.tt.1 T.:64 r;,,$ r.~6 Ypq

Equation (2. 185) m ay be wrillen as

<' = [1;] <. (2.186)

\vhere the vector E. re presents e ngine.e ringstra ins. The s train transforn1ati on ma trix
[~] applie.s to e ngineering strains a nd thus is not the sa1ne as t he s tress trans fo rnl:.\
t ion 1n a trix [i;, J. (Tensorial s ua ins tra ns fo rn1 by the san1e trans fo rn1ntion ma trix
a s the s tresses.)
The trnnsfonnntion n1atrix ( '1;J n1ay be expressed as

[1;] =(t/'][t][t.') . (2. 187)

where (it], [7;'1 J, IT.' J are give n in T obie 2.16. Tiie s ubscript < refers to the s tra in
t ransfonna tion. T he hat on T indic ntes t ha t all s ix strnin compooents a re trans
fonned. The superscripts p. q, a ndr re.f er to the transforma ti ons a bout the p .., q,
a nd r ..a xes, respecti\lely.
Pla11esfrai11 and pla11esfress. U nder pJanestra in a nd plane.stress cond itions
\\le are interested o nly in the strains in the p-q a nd p 1- q' planes. In t his case
t he strains in the prin1ed coord inate systen1 a re o btained from the s tra.ins in the
unprimed coordinate systent by rotation about the r ..axis (see Fig. 2.35) a s fo llows:

{:t }
y~1
= [ ::
- 2cs 2cs
;:
2
' -
-~:]
s1
{ :: }
Ypri
(2.188)
2.9 STRESS ANO STRAIN TRANSFORMATIONS 53

Table 2.16. The engineering strain transformation matrices

~,
0 0 0 0 0 .
0 Cl s'~ c,.s, 0 0
\J -c,,s,,
[1;'] = ~ c; 0 0 "." = cose,.
p
-2CpSp 2cps,, c!-s; 0 0 s, =sine,,
0 0 0 0
~o 0 0 0 t:,.

- <! 0 c.,.,q 0
0 0 0 0
~2 0 (.., = ooss,
[?;"] = .0' 0
c,
-cils!)'
0 -s, s9 =sin c~,,
-2c:.,s., 0 c~- s; 0
- 0 s,, 0 c,
- ,.2 si 0 0 0
I
s; c} 0 0 0 -c,s.
0 0 0 0 0 c, = COS(:),
[t.'] = 0 0 0 c. - s, 0 s, =sin e,-
0 O D s , < :, 0
-2c,s, 2<:,s, 0 0 0 c? -si
- ' '-

E quation (2.188) m ay be written as

' = [7; ) E. (2.189)

The s ubscript E< re fers to the strain trans(om1ation. Yr'ithout a hat. T indicates
that o nJy t he three in plane stra in co 1npo nents are t ra nsforn1e d.

2.9.3 Transformation of the Stiffness and Compliance Matrices


The stress-strain relationships in the unprinled and primed coord inate systems
are (Eq. 2.22)
<7= (C] < " ' =(C'] <'. (2.190)

To oblain the relation bet\\een the stiffness n1atr ices in the unprimed lCJ
and prilne d (C'] coordinate syste111s, we n1ul ti ply bo th s ide.s of the s tress-strain
equation :in the unprimed coordinate system lby [t.]

[t. ].,. = [1;, )[C]<. (2.191)

i\ matrix multiplied by its inverse (1;J-' [1;] is a unit matrix. and we may write

tr.1.,. = 1t HCJ rtr' 1t1 (2.192)


o' (C']
"
By compa ring Eqs. (2.1 90) and (2.192), we see that the e le ments of the stiff-
c;
ness n1atr ix (q 1, 2..... C~,) in the primed (p' , qf, r') coord inate system a nd
54 DISPLACEMENTS, STRAINS, AND STRESSES

the elements of the stiffness matrix (C11 C12 , C66 ) in the unprimed (p, q, r )
coordinate system are re lated by the expression

(C'J = [1;,JlCJlt.r' . (2.193)

The e len1e nts of the con1pliance ma trix (Sj 1, 5;1 . . . , S66 ) in the printe d (p'. q' .
r') coordina te system and the elememts of the compliance n1atrix (S1 I S12, . . . . SM,)
in the unprimed (p,q, r) coordina te system are obtained by inverting Eq. (2. 193)
a s follows:

(S'J = [1;JISl[1;,r' . (2.194)

Pla11e-stress a11d pla11e-strai11 co11diti.011s. Under planestress amd plane-


strain conditions \Ve are interested in t he stiffness and compliance matrices in
the p- q and p'- q' planes.
For plane-suess, the elements o f the stiffness n1atrix in the primed and un-
primed coordinate systems are Qi; and Q;;. where i, j = l, 2, 6. The re]ationship
between O';; and Q;; are given by the transformation specified by Eq. (2.193). The
r esult is

[ Q',,
Q',,
o;,
Q'n
g,,]= l J[Q"
Q',, T,, Q,,
Q,,
Q22 Q"]rr.r '.
Q,. (2.195)
Q,. e,. Q'. . Q l6 Qio Q..

where ( T,, Jand [7; Jare (Eqs. 2.182 and 2.188)

(T.J= [ c'
s' 2cs] s-'
cs]
s' c'
- cs cs
- 2cs
cl - sz
IT.I=
[ c'
s' c'
- 2cs 2c.f cl ~:~ . (2.196)

The elen1ents of the c<Hn pliance matrix in the unprimed coordinate system are
o btained by si111ilar reasoning and a.re

[s;,Si, s;,
.lb
s;,] = IT.I [s"s.,
s;,
s,,
S12
s,.]
s,. 1r.r'. (2.1 97)
s:. s,. s.,, s,. s,. .IO.;
Sin1ilarly, under plane~strain cond itions the stiffness and con1pliance 111atrices
are
c;2 C,,] c.,
[CJz
c,, Ci1 c;, = [T.J [C,,
C11
Cu c"]
c" tr.r' (2.198)
CJ6 Cit. C6,; C1() c,. c..

[ K, ,
Ki1 ,,
K,,
K,]
K,, = (7;] [R:,,
R11
R.,
R11
R,.]
Z;: ir. r ', (2. 199)
K,, Ki. ~. R,, R,.
where (T,, Jand [ 7; ] are given by Eq_. (2.196).
2.10 STRAIN ENERGY 55

2.10 Strain Energy


ln t his section '"e define three paran1eters useful in the analyses of plates and
bean1s. Tlhese are the strain energy, the potential of the external fore.es. and the
total potential e nergy.
For a line.arty elastic syste.n1, the strain en.ergy of volume V is defined as

V=~ .! ff (<,<t, +<,.o, +<,.<t, + y,._r,.,+y"~" + y,.,r,,.)dV. (2.200)

111is expression can be \\ritten as

dV. (2.201)

r.c;
r.1y

\Vith the use of the stress--strain rela tionships (Eq. 2.20) this expression can be
expressed in tern1s o f the strains, as follo\vs:

<x
'r
"
Yy::.
Yx;
dV. (2.202)

Yx,.

where [Cl is the stiffness n1atrix in the .r, y, l coordinate syste.1n . The strain e nergy
n1ust be positive ( U > 0) for any nonzero s train.
The potential of ll1e external forces is defined as

R= - .! JI (f,11 + f,.v + f,w)dV - ff (p,11 + p_, u + p,u1)dA


(2.203)

\I/here. \\'e recall, fx. [,.. /:.are lhe body forces (per unit volunte) and p.r. J1y . P::
are the surface forces (per unit area).
The total potential energy of t he system is

(2.204}

2.10.1 The Ritz Method


ln the R itz n1ethod the displace1nents a re assumed to be in t he fonn
K

u1 =
...L, ck'"" (2.205)
56 DISPLACEMENTS, STRAINS, AND STRESSES

The displacen1ents u;. v;. uJk are conveniently chosen kno\\n functions that
must satisfy the geon1etrical boundary conditions. \\hereas Ai, Bj, C1:. are yet
unkno\\n constants.. According to the principle of stationary potential energy,
at equilibrium the potential energy (Eq. 2.204) must satisfy the conditions

~
aA; =0 . i = I. .. I

a:r, = o j = 1, J (2.206)
as,
3trp =0 k= l , ... , K.
ac,
The constants are pro,~ded by the solution of these equations.

2.11 Summary
The equilibriun1 equations and the stress-strain and strain- displacement reJa..
tionships presented in this chapter are summarized in 1hble 2.1 7 for ge nerally
anisotropic n1aterials a nd in Table 2.18 for 1nonoclinic materials.. Equations for
orthotropic, transve rsely isotropic. a nd isotropic. materials are the san1e as for
monoclinic n1aterials (Table 2.18) v.1th the e le n1ents of the stiffness a nd compli
ance m atrices s implified according lo Eq~ (2.95)-(2.97)
The unknowns of interest for generally anisotropic n1aterials are sun1marized
in Table 2.19 and for n1onoc.linic., orthotropic., tra nsversely isotropic, and isotropic
materials in Table. 2.20. It is evident that under plane.s tress and plane-strain con
cflitions the number of unknowns is reduced and the equations are sin1plified.

2.11.1 Note on the Compliance and Stiffness Matrices


[n this section. v.e give proof of the in1portant statement that the con1pliance matrix
(and consequently its inverse the s t!iffness tnatrix) must be positive definite , and
s.yn1n1etrical.
\Ve consider a sn1all cube n1ade of a linearly e lastic material. 111is initially stress
f.ree cube is deformed. During this defom1ation the internal energy (in this case,
the strain e ne rgy) changes. Since initially the cube is stress free, the deformation
results in a positive change in thestr.ain energy5 (aU U > 0). This requiren1ent
may be expressed as (see Eq. 2.202}

<x
'<y' fy

l<x y
'~ Yy.;: Y.T~ y.Ty llC]
"
Vy.:
Y.t;
> 0 when
"
V.r~
Y.t .;:
#0.

Y xy Y.ty
(2.207)

5 L. E. ~iahcrn . /111roduaiori 10 1/u J\/ec/1onics of a Co111i1111{)1t.f A1edirdn. PrcnticcH:i.U. Englewood


Cliffs. New Jcri:c\'. 1969. o. 292.
2.11 SUMMARY 57

Table 2.17. The equations for a generally anisotropic material


Equilibrium Strain-displacement Stres;s...strain
Three-dimensional 2.13, 2.14. 2.15 2.2. 2.3, 2.4. 2.9-2.1l 2.20
Plane-strain
Free e nd 2.59, 2.60. 2.61 2.54 2.56 257 2.62
Built-in ends 2.59, 2.60. 2.61 2.99 2. !01, 2.102 2.103
Plane-stre.ss 2.122, 2.123 2.2. 2.3, 2.ll 2.126

Table 2.18. The equations for a monoclinic material. The equations lor ortllotropic,
transversely Isotropic, and isotropic materials are the same as tor monoclinic materials with
the stiffness and compllan<e matrix elements simplified as specified in Eqs. (2.95H2.97).

E"'itibrium Strain-displacement Stres$-Slrain


Three-dimensional 2.1.l, 2.14. 2.15 2.2, 2.3, 2.4, 2.9-2.J l 2.20
Plane--strain. Cree end
Jn-ph1ne loa<l 2.74. 2.75 2.68.2.71 2.76
End axial load
End moment
Out-Of-pla ne load 2.86 2.84 2.87
Torque.
Plane.-strain. built-in e nds
Jn-ph1ne load 2.74. 2.75 2.107. 2.110 2.11 1
Out-Of-pla ne load 2.86 2.117 2.119
Plane-stre.ss 2.130. 2.13 1 2.2, 2.3. 2.11 2.134

Table 2.19. The unknowns in the equations for generally anisotropic


materials.
OislJlacements Strains. Stresses
Three-d i1ne nsional u. v. w Ex. f >, <Er a,,, a.,, 11t
Ytt )"., ~. Y...y r,.t. r., z T.1.y
Plane-strain
Fret e nd u. v. w E., . f 1 a,., a v
Y,:, Y1:, Y..-, r1: r., : r..,.
Built-in ends u , v.w .,._ E ~ C11., <1.v
Y~ Yxt<, Y1.) l y:. l.~~ T-1y

Plane-s1ress u, " _.. E)' y_._., C11.. <1.v, r_,~


58 DISPLACEMENTS, STRAINS, ANO STRESSES

Table 2.20. The unknowns in the equations fnr monoclinic, orthotrople,


transverse!~ Isotropic, and Isotropic materials

Displacements Strains
Thre.e-d imensional a 1. aha:1
r 2.;, ru, T1i

Pla.ne.-strain. Cree e nd
In -plane. load
E nd axial load u1. tti fl ,~;:. }'I? a1.~. Tl ?
Endmomenl
Ou t-Of-p lane load u_, Y.z3', Yu T;:.;, T u
Torque
Plane-s1rain. built-in e.nds
Jn-plane load u,, u2 f t. E?, YU a i, a1. r 11
Ou t-of-p lane load II_!. Y?.t , YU T?_!., TIJ
Plane-stress llJ. U? f J, t~. Yll a1. a1, ru

The preceding inequality requires that [CJ be positivedefinite.6 (Correspond-


ingly. [3l [CJ a nd (SJ must a lso be positive definite.) The manner in which we
can detern1ine \\he ther or not (C] is positive definite is discussed subse.q ue nt ly
( page 59).
First we show that the compliauce n1atrix must be symme tricaL To this end
we apply loads in four steps (Fig. 2.36) to o ur initially stress-free c ube made of an
elastic ma teria l.
(a) A te nsile load p (pe r unit a re a) is a pplie d in the x direction. As t he result
of this load "' = p is the only nonzero stre.ss in t he n1aterial. The s trains are
(see Eq. 2.21)

(2.208)

The \vork is

W' = 1,.
'<r,,d<,, + 1 ,.. 1
' u, d<y = 2S11 p1 . (2.209)

(b) t\n additional te nsile load p is a pplie d in tbe y dire ct ion. Tile stresses in

.
t he n1aterial becon1e dx = Cfy = p. a.nd the corresponding strains are
-
,, = Sup + Sup
- (2.210)

The 'vork is

(2.21 1)

t E. Krcyslig, Ad\atlCt!lf E11g i11etri11g .\ldtJ1011u1itt 7th Edition. John \ ViJcy & Sons. New York. 1993.
p. 407.
2.11 SUMMARY 59

i" !"
P<-
b ~p
x
+
D
!,, +
P-->D<- 1'
+
D
i,,
(a/ (b) (c-t (d )
Figure 2.36: Hlustralion o f the (our load steps.

(c) An additio na l com pressive load p is ap.plied in the x directio n. The stresses
in the ma te ria l becon1e '1x = 0, a.\) = p, a nd the corresponding s tra ins are

(2.21 2)

The \vo rk is

(2.213)

(d} An a dd itional compressive load p is a pplie d in they direction. The stresses


in the m ate rial beoon1ea., =<Ty = O,and thecorrespo ndingstrains arel; = ,;~ = 0.
The \vork is

(2.21 4}

111e total 'vork done o n tbe cube is

11' = W" + W" + W" + W' = (S,, - S11)p'.


1
(2.215}

Fo r ao elasilc ma te rial tbe total work is ze ro. Accordingly, we 111usi have

(2.216}

By sintilar argun1ents it can be sho\vn tha t

S;i = S ji ;, ; = t. 2, . . . 6. (2.217}

Thus, <h e compliance m a<rix of a n elastic m at eria l [SJ


(and correspondingly [CJ.
(CJ, and (SJ) must be symme tr ical.
Next " 'e d iscuss the condit ions t ha t e nsure a con1plian ce n1atrix is positive
definite. A syn1me tricaJ matrix is positive de finite \Vhen each of its e igenva lues
is positive. 7 Th\L~. to determine ""he the r or no t the m atrix is positive definite \Ve
n1ust exan1ine the e igenvalues o f lhe con1plianc.e (or s t1ffness) m atrix.
For ortho uop ic, tra nsversely isotropic, and isotropic n1ateria ls the re is a sim-
ple r metho d fo r detenn ining \\'he ther or not the con1plia nce (or stiffness) ma trix:

1 F. B. Hilde brand. ,\ft 1!1od.t nf Applied Alar/ie111a1ic.t. 2tld edition. Prcnti.x. Hall. Engle1,1,ood Cliffs.
New Jct'SC-y, 1965. p. 48.
60 DISPLACEMENTS, STRAINS, AND STRESSES

:is positive definite. The sin1pler me thod utilizes the condition that t he n1at rix is
:positive de finite whe n e ve.r y subdeterminant of the main diagonal is positive.8
Fo r an ortho tro pic n1aterial the .compliance 1natrix is (Eq. 2.28)

S11 S12 Su 0 0 0
s,, s,, Sn 0 0 0
s,_, S" S;u 0 0 0
(2.218)
[SJ =
0 0 0 s.. 0 0
0 0 0 0 Sss 0
0 0 0 0 0 s,..
The follo\\'ing subdeterminants n1ust all be positive:

D1= S11 > 0 Di = S22>0 i), =Sn > O (2.219)


D, = S.u > O Ds =S,,>0 D.= S.. > 0

[),3 =Is,,
- Sn ~:I> 0 Dt .J--Is"
Su Sul > 0
Sn
D12=IS11
Su_ Su
s,,, > 0

(2.220)

S11 S12 Su
D113 = s,, S22 Su > 0. (2.221)
Su s ,_, S;u
'vhere I I denotes the determinant. Since th e stiffn ess n1atrix is the inverse of the
complian ce n1atrix, according to the rules o ( n1atrix inversion the e len1e nts of the
:main diagonal o f the stiffness matri:.x a re
D1-, DI.\ D12
C11 = - - C,, = - - C.u = - -, (2.222)
D12l Di 23 D 1:2J

"\vhere D,J , Du, a nd D 12 are positive \\he n Cu . C12, and C33 a re positi\re,
C11 > 0 C.u > 0, (2.223)
.and. at the same time, D i 21 is positive. Thus. the require ment for positive definite-
n ess is n1et when aJI three of t he follo \\ing cond itions a re me t:

the e le n1ents of the n1ain diagonal of the con1pliance n1atrix are positive
(Eq . 2.2 19),
the dete rminant o f the compliance ma trix is positive (Eq. 2.221), and
the e len1e nts or the ma in diagonal or the stiffness ma trix are positive
(Eq. 2.223).
The san1e conditions a pply for t ra nsve rsely isotropic a nd isotro pic ma te ria l" .
.If the relevant eleme nts of the compliance and s tiffness n1atrices a re presented in

K E. Krcysi.ig. Ad11tutce1/ Eri1:i1tttnng 1\l athentaric.t. 7th e-dition. Jo hn \\'ilcy & Sons.. New York. 199J.
p. 407.
2.11 SUMMARY 61

Table 2.21. The constraints on the engineering con.slants.


Ortho1ropic 1 > 0 E'!. > 0 1 > 0
G-i.1 > 0 Gu > 0 Gu > 0
r, 1 1:, 2 1 t \
I - 1'.?.1 I; ~ 1,
- '1: ~ - P11 n1>:_t "1 - ''11""( > 0
, 1.,
Vit < 'l;
!J.
''i1 < T,
!J.
l 'j1 < f.;

Trans"ersely 1 > 0, E: > 0. G1: > 0


isotropic - 1 < ::.< I - l~ f!
l !!.
''1! < t:

Isotropic , > 0
- 1 <Pu <0.S

terms of the engineering constants (see 'Thbles 2.7 and 2.9. pages 15 and 18) the
preceding conditions can be expressed in ltrms of the engineering constants. De
tails of the algebraic manipulauons are not given here. The results are summarized
in Table 221.9
2.4 Example. The e11ginetri11g co11stt1111s of " grophitt rpoX)' u111dir~ctio11ol pl)'
are givm <IS E, = 148 x !09 Nlm2, E2 = 9.65 x 10" Nlm1 G" = 4.55 x 10" N/11i1.
1111 = 0.3, and l 12J = 0.6. D~ttr1111nt \i'htther or not tllu srt of c.onsuuus is va/ld.

Solution. There 3rc t\VO ways 10 find the nns\vcr to t his problem.
Method I. The complinnce ma trix i (Eq. 2.4 1)

6.76 - 2.0:1 - 2.03 () 0 0


- 2.03 103.63 - 62. 18 0 0 0
- 2.03 - 62.18 I 03.63 () 2
0 ()
io-12~ .
ISi = 0 0 0 331.6 1 0 0 N
0 0 0 () 219.78 0
0 0 0 () () 2 19.78
(2.224)

11\e eigenvalues (A )( IO""V) or this 1nat rix arc


0.0065 0.0417 0.1658 0.2 198 0.2198 0.33 16.
Since every eigenvalue is positive. the specified scl of engineering constants is
valid.
Method 2. For a transversely iso1ropic 1naLcnal Lhe engineering constants must
satisfy the inequalities in Table 2.21. In tcnn.s of the cnsmeering constants these
inequalities are

(2.225)

8. ~t Ltmpnere. J\)woca's Rat.o in Onho1te'lipk ~l*11.1b.. A/AA klwMI. Vol. 1. ll.lfr2ll1. l96&.
62 DISPLACEMENTS, STRAINS, ANO STRESSES

or

- 1 < 0.6 < 0.988 0.09 < 15.3. (2.226)

S ince the inequalities are satisfied, the specified set of engineering constants
is valid.
Let us no\\' assun1e that 112 1 = 0.3. In this case 1.112 = ~! v21 = 4 .601 (see
Table 2.8, page 16), and the compllance matrix (Table 2.7, third row, page 15)
is

6.76 - 31.09 - 3 1.09 0 0 0


- 31.09 103.63 - 62.18 0 0 0
(SJ =
- 31.09 - 62.18 103.63 0 0 0 10.- 12 m' .
0 0 0 331.61 0 0 N
0 0 0 0 219.78 0
0 0 0 0 0 219.78
(2.227)
The eigenvalues (J. x 10') are
- 0.0232 0.0714 0.1 658 0.2198 0.2198 0.3316.
One of the eigenvalues is negative; hence. the specified set of en.gineering
constants is invalid. We reach the sanne co nclusion if v.e use the inequalities given
by Eq. (2.225) since two of the inequalities are not satisfied:

- 1 < 0.6 ~ - 1.76 21.2 ~ 15.3. (2.228)

TI1L< example illll<trates that t'3re should be taken to use proper values of the
Poisson ratios. The v12 and v21 Poisson ratios (re fe rre d to as the n1ajor and n1inor
Poisson ratios} must not be intercha nged.
CHAPTER THREE

Laminated Composites

Composites arc frequently made of layers (plies) bonded toge ther to Com1 a lam
inate (Fig. 3. 1). A layer may consist of short fibe rs. unidircctionol continuous
Obcrs, o r woven o r braided 6bers embe dded in a matrix (Fig~ I. I ond l.2). A layer
containing \VOvcn or braided fibers is referred to a.s fabric.
Adjacent plies having the same mate-rial and the sam e. orientation a re referred
to as a pl) group. Since the properties and the orientations are the same ac-r oss
the ply group. ply group may be treated as o ne layer.

3.1 Laminate Code


An x. y. l orthogonal coordinate sys:1em is used in analyzing lamin:ttcs \vilh the
z coordinMc being perpendicular to the plane of the laminate (Fig. 3.2).
TI1e o rientntior1s o f continuous.. unidirectional plies are specified by the angle fo)
(in degree) with respect to the x-axis (Fig. 3.2). The angle 9 is positive in the
cowne rclock\vise direction. The number of pl:ies within n ply group is spec-ifie<l by
a nun1eric;1J sub.~cri1>t. For e xan1ple, the lan1ionte consisting or unidirectional plies
and sho,vn in Figure 3.3 is designated as

[45J/OJ/'>0,/60J.

This laminate contains (our ply groups. the first containing three plies in the
45degree direction. the second containing four plies in the ()..degree direction, the
third containing t\\-'O plies in the 90-degree direction. the founh containing one
ply in the 6().dcgrcc direction.

Symmetrical lamina te. When the laminate is S)'11Ullelrical wilh respect to the
midplane it is referred to as a symmetrical laminate. Examples of symmetrical
lamina tes nrc shown in Figure 3.4. The lamin a tes represented in Figure 3.4 are

63
64 LAMINATED COMPOSITES

Figure. 3.1: Laminated composite.

Figure 3.2: The x, y. z laminate coordinate.syste m, the X1 . xi . X') ply coordina te system. and the
ply angle.

6-0
90
n
0
n
0
45
45
45
Figure 3.3: Description o( t he la)up in a 1.aminatc consL.;ting of unidirectional plies
[45,/0,/9Cl.i/60J.

-45 45
-45 -45
0 -45
0 "
0 45
-45
"
- 45 -45
- 45 45
f-45,IO,J, (4SJ-45/ 4S),
Figure 3.4: E)'.amplcs of symmctncal lamb1ale$.
3.2 STIFfN ESS MATRICES OF THIN IAMINATES 65

- 45 45
., ..,
- 30 '"
lO - 30
90 - 30
90 30
45 - 45
45 45
[45f 90,130/-30/-451] (451-451301- 30),
Figure 3.5: E:tamplcs of lialan ccd laminates.

specified as

(- 452/0,/- 452] l- 452/02],


(45/ - 45,/45,/-45,/ 45] (45/- 45,/45(, .
The subscript s indicates symme try about the 1nidplan e.

Balanced laminate. ln balanced laminates, for every ply in the +0 direction there
is an identical ply in the - f) direction. Examples of balanced laminates are s hown
in Figure 3.5.

Cross ..pl.r laminates. In cross-ply laminates fibers are o nly in the o.. and 90-degree
directions (Fig. 3.6). Cross-ply laminates 1nay be symn1e trical or unsyn1metrica l.
Since ther e is no d istinction bet\\een the +o and - 0 and bet\veen the +90- and
- 00-degree directions, c ross.ply la n1ina tes are balanced.

Angleply laminate. Angleply lan1inates consist of plies in the + f.) and - e di ..


rections. Angle ply laminates may by syn1me trical or unsy1nme trical, balanced or
unbalanced. Examples of angle..ply lan1inates are shown in Figure 3.7.

n / 4 laminate. tr/ 4 la n1inates consist of plies in v.hich t he fibe rs are in the o., 45..,
and - 45degree directions. The numbe r of plies in each d irection is t he san1e
9() .. ,
(balanced laminate). In addition, the layup is a lso S)'lllme trical.

3.2 Stiffness Matrices of Thin Laminates


Thin Jami nates are characterized by three s ti.ffness matrices deno te d by I A), [BJ,
and I DJ. I:n t his section \Ve determine t hese m.a trices for thin, flat lan1inates under
goingsmaill de forn1ations. The analyses are based on the laminate plate theory and
are fom11.Jla ted using the a pproximations that the s trains \'ary linearly across the

[90,!02] (CW90],
l-igurc 3.6: Examples o( cross-ply lam inalcs.
66 IAMINATEO COMPOSITES

-45
-45 30
45 30
" - 30 figure 3.7: Exumplc;S of angle-ply la minales.
- 30
"'
-45 - 30
[- 30,!30,J

laJninate., (out..of-plane) shear deforn1ations are negligible, a nd the outtOf plane


nom1al stress <1~ and the shear stresses 1'.v~. 1'y z are sn1all compared \\~ th the in
plane ax. ay, and Try stresses. These approxin1ations imply that the stress-stra in
relationships under planestress conditions may be applied. The x. y , .z refer to
a coordinate syste m \vith the x and y coordinates in a suita bly chosen :reference
plane, a nd t is perpendicular to this reference plane (Hg. 3.8).
Freque.n tly, though not a lways, fo r convenience the reference plane i:s take n to
be t he n1idplane of t he lan1inate. Unless the Jan1inate is syn1n1etrical 'vith respect
to the reference plane, tbe reference plane is not a neutral plane, and the strains in
the reference plane are not zero und e r pure bending. The strains in the :reference
p lane are (see Eqs. 2.2. 2.3, a nd 2. I I)
(I auo
<x =IJx
- (3.1)

..:vhere" and u are the x , y con1ponents o( the d isplacen1ent and the superscript 0
refers to the reference plane.
\Ve adopt the Kirchhoff hypothesis. nan1ely, that normals to the :reference
surface re main normal and straight (Fig. 3.9). Accordingly, for small deflections
the angle of ro tation of the nom1al of the reference plane X.t t is

(3.2)

..:vhere w 0 is the out-of-plane displacement of the re (erence plane. The total dis
placement in the x direction is
au:(l
1' = tt 0
-
'=U
Z;t..x
0
- z--
ax (3.3)

Sin1ilarly, the total d isplacement in the y direction is


8uJ0
v= 0
tt - ~ --. (3.4)
ay

Figure 3.8: 1bc coordinate S)'Stcm.


3.2 STIFfNESS MATRICES OF THIN LAMINATES 67

\ Xr:

w
I)' .4'

"l . , ~
Reference plillle/
A
Figure 3.9: .Deformation o( a plate jn the :r-z plane.

By definition, the strains are (Eqs. 2.2, 2.3, 2.11)

-- --
.T -
au
ax y -
au
ay Y.,,.= .--+-a
uy X
an
.
au

Substituting Eqs. (3.3) a nd (3.4) into these expressions. we obtain


(3.5)

au 0 a2 w(J
lx = -.- - z--,
ax ax-
ali(J i)21JJU
'=-
' ay - z--
ay'
(3.6)

au 0
au0 2a2 u/ '
= -;;-
YJt~'
Y + 7uX - z-::;--;-
oxoy

These equations can be v.ritten in the following form:

(3.7)

where t;, '~ Y.~~ are the s trains in the refere nce plane (Eq. 3.1), and K., , Ky , and
Kry are the curvatures of the reference pla ne o f the plate (Fig. 3.10) defined as

(3.8)

The inplane forces and mon1ents acting o n a sn1all elen1ent are (Fig. 3.11 )
,,, ,,, ,,,
N., = j a, dz N1 = f a1dz N.t:,. = J fxydl

'"
- /11,
-'" - Ju.
(3.9)

M, = j zr1,dz M, = j ur, dz M.xy = J'"zrJl>'dz.


- - li.
68 IAMINATED COMPOSITES

z
'
Dcformrd
Refere-nee .Plane

y y

Undefomte.d
b Refere-nee P lane b

, _4+/,l- 1.
,...,,, - l
-t ,.;=1>-=0
' ,, ' '
Figure 3.10: 'fbe cunatures K~- .r,.. an d I(~, o f the refere.nee plane.
'"here N and Mare the in ~pla ne forces a nd moments (per unit le ngth), a nd /J 1 and
J,. a re the distances from the reference plane to the plate's surfaces (Ftg. 3.12).
The transverse shear forces (per unit le ngth) a re (Fig. 3. 11 , right)
h, h,

_.f
\I, = \I.,= / r.,dz. (3.10)
,_ r,,dz -/~

\Ve no\\' recall that (or planestress condition the stress-strain rela tionships/or
ead1ply are (Eq. 2.126)

(3. I I)

/
x
-.
~ ... .. .....
.~Jtl!P
- y

Figure 3.1 l: The-in-plan e forces acting at t he refere-nee- plane (left) and !he moments a nd the
L!rnnncn.e-shear forces (right).
3.2 STIFFNESS MATRICES OF THIN LAMINATES 69

I:.,;~------~-----~
1 ~~~~2is
Rcfcrc-ncc Plane
- ---=-
- . -. z,_._ " -

Figure 3.12: Oislances from lhc reference plane.

By introducing the notation

(3.12)

we \\rite the stress- strain relationships for a ply as

:; } = [Q] { :; } . (3.13}
{
'l'x>' Yxy

where (Q] is the stiffness matrix of the ply in the x- y coordinate system. The e le
me n ts of this stiffness ma trix a re obtained fro111 the e lements of the stiffness ma trix
( QJin the x , - xi coordinate system by the transfom1ation given by Eq. (2. l 95).
By replacing ( QJand [Q' I by l Q] a nd ( QJ, respectively. Eq. (2. 195) yields

Q,, Q,.]
Q,, Q,. [T.), (3.14)
Q,. Q..

where [T,,) and [ 7; J are given by Eq. (2.196) and are re iterate d below,

IT.I=[:::: - cs cs
_;~:: ]
c1 - s2
s'
(7; ) = [ :: c'
- 2cs 2cs
cs]
- cs .
cl - ..,.2
(3.15)

and c = cos f), s = sin fl \11th fl defined in Figure 3.2. For an orthotropic ply the
Joe.al coord inates X1. xi are in the o rthotropy directions. For transversely isotropic
plies these local coordinates are parallel and poerpendicular to th e fibers (Fig. 2.15).
For orthottropic and transversely isotropic nlateriaJs the elements of the stiffness
nlatrix io the global coordinate syste.m are giveo in T.i.ble 3.l in tern1s o( the
elements of the stiffness ma trix in the local coordinate systen1.
70 LAMINATED COMPOSITES

Table 3.1. The elements of the (OJmatrix for an


olhotroplc or transversely lsoltoplc ply oriented fn
the +e direction (Ag 3.2)

Q11 = c'Q11 +s'Qu + 2c's1(Qi:+2(1..)


Qu ='Q,. + c'Q,, + 2c's'(Qu + 2(1,.)
Q11 = <'' (Qu + Qu - 4Q..) +(c' +1')Q11
Q.. = c's'(Q,. + Qu -2Qul +(<' - 1 1l' Q..
'Q,. = u(c'Q,. - 1'Q,, - " -s1 )( Q u + 2Q..))
Q,. = <s(s'Q., - <'Q,, + (<' -s')(Q,, +2Q..)l
CecC(ld:0 SSin9

By substituting Eqs. (3.7) ond (3.13) in10 Eq. (3.9). we obtain

= J.. {*"! }+ J. :IQ]dz {"}


_,h.,
(Q)1tz
Y,,
.:
- 1r.. Kxr
(3.16)

= Jz[Q)dz <"1, } + .f.. z'IQldz { , ;


J,. { K }
. (3.17)
- 111+ Y,ly ..Jti. K,ty

Tbe stiffness ma1rices of lhc laminate are defined ns

..
[ Aj = f.. [Q)d~
_
..
(BJ= f.. z(Q]d~
_
(3.18)

..
(DI= f i(Q]dz.
_..
3.2 STIFl'N ESS MATRICES OF THIN LAMINATES 71

The ele111cn1s of lhese matrices are (i. j = l. 2. 6)


...f f.. xQ,,tlx
b,

,,,, = Q,,tlz B;, =


.
(3.19)

D,, =
...f ::'{1,,tlt.

The IAJ. IBJ, nod IDI matricesarethestiffn.essmatricesofthc lamin.1e. and fQI


is 1he s1iffl1css n1a1rix of 1he ply. Since I QJ is constant across each ply.1he in1cgrals
in the cqualion. above (Eq. 3.1 9) may be replaced by summations (Fig. 3.1 2) as
follows (i. j = I, 2. 6):
K
A,i = ~)011l>(t - <:t- 1)

B., = i1..f-
-
- ' - ,-,_,)
L.)Q.,l.(z; ..2 (3.20)

O., = 31..f-
-
- J - ..._,).
L.)Q,,lt(z, _,

-
where K is the total number of plies (or ply groups) in the laminate: :,a.. ~1 are the
distances from the reference plane 10 the two surfaces or the l<lh ply: and ( Q.,l
are the elements of the stiffne.ss matrix of the k'th ply.
With Lhc preceding definitions of the stiffness matrices. Lhc expressions for the
in-piano forces and mo ments (Eqs. 3.16 and 3.17) become

N,, A11 A 11 A1 8,1 Bu 810 ,;


Ny A11 lb2 Ai 8 12 8 22
8 10 B,.
82
Bo.
"
y"'
i'l..y
M,
=
A10 A16
811 812 ""
8,. Du D12 D1 "
K_.
(3.21)

M, 811 Bi. Du o,.


J\1,, Ba. Bi.
8 21
&,.
'"'
D 11

Di. 0,,, 0.. '"


The ''cctors on the left and right hand side repre.sent generalized forces and
strains. Hereafter. we simply refer to these as forces and strains.
By inverting EqL (3.21). we obtain the strains and curvotures in 1erms of the
in-plane forces and moments:

' "" 01! "b /l11 fJu fl1 N,


1;
y,.~.
au:
" 1
= /i11 /b
tt21
U2b .
0'26
.,
p,,
p.,
fJ22
{J.,
{J,.

...,,,.
{J..
N,
lV....11 (3.22)
"'
.."'',,
Jl<,1 611
/l11 /J22 Jl<.2 &11
/J10 /Ji /J.. S1
&12

"22
Si . ...
M..
J\1
M_.,
72 lAMJNATED COMPOSITES

The (a), [Jl], and IS) mat rices are rela ted to the I A], [BJ, and [DJ matrices by
- I

"" "" " fJ11


flu
{J,,
{J,, fJ1
A11
A12
A 12 /\16 B11
812
B,,
B,,
B1
B,_,,
U"12 <>22 /12 Ai{,
""' fJ., A12

A.. B,. Bu. s..


~:j =
C/1 ci,,, Cl"' /J.2 A1 A16

/111 /321 II ~II


fJ., ~1 2
li11 B11 B 12 B,. D11 D11 D1
{J12
{J,.
/121
{Ju, {J.. ~. 821
B,.
8 2&
8..
812
B,.
812
B,.
B,.
a,,,
D 12
D1
f),..l Dz.
Di. 0..
(3.23)

3.2.1 The Significance of the [A), (8), and [OJ Stiffness Matrices
The [A], I BJ, and [DJ matrices represent the stiffnesses of a lamina te a nd describe
the response of the la1ninate to in-plane fo rces a nd n1on1ents.

A;1 are the inplan e stiffnesses that relate the in .. pla ne fore.es N,, , 1".v,
N .xy to
the in-plane deformations ~;, .:~. Y.~'.w
Dt; are the bending stiffnesses that relate the moments ,\(" ,\<fy. J\.(1 >' to the
curvatures K.r . K y, Kx>"

Table 3.2. llustration of the coupling terms A16, 016, 11,6 , Bi 1, 11,,. B,. for
composite materials. When the element shown in the last column is zero, there is
no coupling. (The coupling terms A26 , o,.,
826 , B., can be illustrated in a similar
manner by applying a force N, and a moment M, in the y-z plane.)
C-Oupfing No C-Oupling Bement
E." tc11s:ion- s:l11:mr
N~..:o,.. ~
.. + N
... ..... ......
,.
No
~: ---------.
..

...........
:..

N
:~
'
t

Bcudin~-1 wi~1; . ...

,11g:::~.. ,('iM,
Extension- t.\\ist
N z.---- --7,
+-1,'
' .......... ,.
Nr
,' -+ .Bu.

.811

-~_.-z ; :z------;z;:.!,
.. . ............ , .Bu

~.-z--z:,
........ .. . ..
_}
3.2 STIFFNESS MATRICES OF THIN LAMINATES 73

Table 3.3. llustraUon of the coupling terms A,,, 012 that may be present
both In composite and in isotropic materials. When the element sh<lwn In
the last column is zero, there is no coupling.

Coupling No Coupling Element


Extcu.sioa-extcuslou
~
Iii : o---------.
~-
:-...
-........ .
N
j
'l'D
+-:
./ . .
"fl'
:-.
~

' .!

Bending- bcndiu.g
,., ..... . .. .. ....
M;(....._ji\f, M,,g'. ... _jM, o,,

~; are the in-plane-out-of-plane coupling stiffnesses that relate the in -plane


foroes N.r1 NJ' 1 N.1:y to t he curvatures"'"")'' K.ry and t he n1oments 1\(.0 J\~v 1
M..~y to the inp lane defonuations f;, ' ;~ r:,..

Examination of the [AJ, (BJ, and (DJ matrices shows that different types of
couplings Jnay occur as discussed belov. and illustrated in Tables 3.2 a nd 3.3.

E.xten.\iori-shear coupling. \Vhen t he eleme nts A 16 , A26 are not zero, in-plane
normal forces N1:. N,., cause shear deforn1ation Y:y and a t\\ist force 1V:t}' causes
elongatious in the x and y d irections.

Bending-t,vist coupling. \Vhen the elen1ents D16 , Di,6 are not zero, bending mo
n1ents J\fJ. , J\fy ca\L~e t\\~st of the lan1inate ><.1..v and a t'vist n1oment J\1J.'Y causes
curvatures in the x- z and y-z planes.

Exteru.io11-t" ist and bending-shear coupling. \\'hen the elements 8 16 , 816 are
1

not zero, in ~ plane normal fore.es Nx. Ny cause 1v.~st K.vy. and bending moments J\1x.
11~fy result in shear deformation Y.~y

lnplane-out-ofp lane coupling. \\'hen the e len1ents B;; are not zero, inplane
forces Nx. Ny. N11,, cause outofplane deformations (curvatures) of the lan1inate,
and mon1ents i\1.x. i\11.v, i\1.xy cause in~plane deformations in t he x- .v plane.
The preceding four types o f coupling are Characte ristic of composite n1aterials
and do not occur in homogeneous isotropic n1aterials. The foJJowing l\vo couplings
occur in both composite and isotropic n1aterials (Table 3.3):

Exteru.ion-exteosion coupling. \\'hen the ele ment A12: is not zero, a normal force
f;,
Nx causes elongation in the y direction and a no nnal force 1\J_\. causes elongation
in the x d~ rect ion f~ .

Bending- bending coupling. \Vhen the element D12 is not zero, a bending nton1ent
i\11,. c.a\L~es cunature of the laminate in the y- z plane ><y. and a bending n1on1ent
My causes curvature of the laminate in the x- .t plane Kx.
74 LAMINATED COMPOSITES

::1.2.2 Stiffness Matrices for Selected Laminates


For certain ply arrangen1e nts (layu ps). son1e of the couplings described do not
occur, and the (A), [BJ, [DJ matrices become simpler.
Sy1t1111etrical /a111i11ale. In a syn1metrical laminate the ply located a t a position
+ t is identical to the ply at -t. Ce>rrespoodingly, the stiffness matrix I QJ of the
p ly at +t is identical to the stiffness matrix of the ply at - t:

IQJ(z) = IQJ (- t) . (3.24)

By substiluting t hese stiffnesses into Eq. (3.18), we find that the l BJmatrix is
zero:

( BJ = 0 (symmetrical). (3.25)

Thus., for a symmetrical laminatte there is no in..plane- out-of plane coupling.


.and Eq. (3.21) m luces lo

'} [A" A
{N "] {'; } A 11
JVy = A11 A iz A26 f~ (3.26)
N1.:y A1n A1ft ,i\,(, y~,

{M' } [~'Di6
D 12
M, = ~, Da
D 16] {~y
D,..b
Kx } . (3.27)
A1xy Du. DM 1i:.,J'

\Vhen the laminate is syn1metr icaJ, the compliance n1atrix is generally ex


pressed as

[a"
a 11
& 12
&zz
16
a ] = [a11
&26 012.
0 12
022
a1
a,.] [~"8,2
812
811
8,.] = [d"
826 d 12
d,,
tl12
a,.]
du ,
a,. '126 Ctt-6 O t6 a,. a,,. .St6 {jzo li(,6 d16 d26 d,,.
(3.28)

'vhere

'"fr,
A12
["" a,. ". ] [A"Ai
a 12
a,2 22
16
ll26 =
A
12
A12 Az (3.29)
Ail> 1166
llt6 {/

[d" ~
D12
d 12
d12 th2 d,.
d,.] = [D11
D11 Di2 . (3.30)
d,. d16 d.. Di D26 D,.
3.2 STIFfN ESS MATRICES OF THIN IAMINATES 75

The relationships between the strains and c urva tures and the fo rces and mo
ments (Eq. 3.22) now simplify to

{y1;.E;} [""ll1~
Eu = a1,
ll]l

a22
"'] {,~, }
a2n 1Vy (3.31)
1 1VXJ'

{ Kx }
Ky = [ d11
lf12
d12
d21
d1]
d16
M, }
{:\1y . (3.32)
"'Y d16
d,. d66 ,,,JI)'
Balanced la111i11ate. In a balance d Jan1inate, for every unidirectional ply in
the +f>direction (1neasured co unte rclock,vise fro1n the x coord inate) t he re is an
identical ply in the - H direction. The e lements of the stiffness matrix I QJ are
given in Table 3.1 (page 70). Fron1 this ta ble 've deduce that the e le n1eots of the
stiffness matrices of plies in the +f) and - f> d irections are related as follo \\s:

Q ll(+<>) = Q ll(- <>) Q.?2(+9) = <222(-ft)

Q l1(+0J = 0 12( - 0) Q66(+9) = 'Q.,,._.., (3.33)


Q16(+") = - Q i(- 9 ) 'Q,,,..,..., = - Q,,.,_.,)
By substit ut ing t hese ele ments into the e xpression o( t he stiffness n1atrix in
Eq. (3.20), we find that
A 16 =Au;= 0. (3.34)
111e structure or the stiffness matrix given in Table 3.4 shov.s that there is no
exte nsion- shear coupling in a balanced laminate (Table 3.5). (Note tliat A1. a nd
A26 a re ze ro o nly in the x- y coordinate syste1n.)
Ele1ne nts Au, a nd A26 are zero for syn1n1e tric-al a nd unsynunetrical lan1ina tes.
Correspondingly, for symme tric.al ba lanced la n1ina tes the Ot 6 and a26 e le ments ol
the compHance ma trix are zero (016 = 0 and a26 = 0). However, for unsyrnmetrica]
balanced Jan1inates no ne of the e le n1ents o( t he con1pliance n1atrix is zero (see
Eq. 3.22).
Ortltotropic la111i11ale. In orthotropic lanl.in ates v.e are interested in tv.o mu
tually pe rpe nd icular directions, called orthottropy direct'ions, in the plan e of the
lamin ate. Norn1al forces and bending n101ne nts applied in these directions do
not cause shear or tv.ist of the lamina te. Hence, t he re are no extension- shear,
bendi ng-~'vist, and extension- twist couplings.
A lantina te is orthotropic v.he n every ply is o rt hotropic and the orthotropy di
rect:ionscoincide with the x and )' directions. Fiber-reinforced plies a re o rtho tropic
under the fo llowing conditions:

\Vbe n the ply is n1ade of unidirectiona l fibe rs and au the fibers are aligned \vith
one of the lan1inate's ortho tropy d irections (Fig. 3.13);
76 LAMINATED COMPDSITES

Table 3.4. The [AJ, [BJ, [OJ matrices for laminates. When the laminate Is
symmetrical, the [BJmatrix is zero. Cross-ply laminates are otthotroplc.
[AJ (BJ [DJ
Symmelric.al

[~ ~]
0
[A,'
Au
A"]
A" 0
['' '' D"]
Du
"'"''
An D 22

Bab1nce<l
A.M A" 0
''
o.. D ,. [),.,

["''"'' JJ
0
Au
A,,
0
[Bu
B,.
Bu

B,,,
B"]
B12 Bn lJy,
B w..
Du '' D
[''
o..
o,."]
o,.
D ii
Dir.

Orthotropit

[All A11
1J '' JJ ['''' l]
[Bu Bu Dn
Dn
"''
0
Ai.?
0 0
Bti
0 0 0

Isotropic

["''"'' Au ..~..,] [Bu'' 8,,~~] ['''' Du o.,~~J


0
Au

0 , 0
B12
Bu
0 0
D.,

QuaSiiSOlr<>pi<:

["'' A,, ~] [''


~!
Ari

0
Bi Bu
B,. B,.
Bu
B"]
o,.
o,. [''
''
o..
'DM' D"]
D,.
Du
D,.

Table 3.5. Couplings in selected laminates ("no means that the Indicated element of
the stiffness matrix Is zero and the corresponding coupling does not occu~
In-plane-
Extension-shear Bending-twist Extension-twist out-of-plane
A,. o,. B,. 811
Sym1netric.al 110 no
Balanced no
Orthotropic no 11<> no
Quasi- no
isotropic
Isotropic no n<> no
Layered
Isotropic no n<> no no
single layer
3.2 STIFfN ESS MATRICES OF THIN LAMINATES n

~y

--D - . ,.
--- >

.. ... . 'T:' g . ;r
' -;,s.

:
'
Figure 3.13: l'I)' nrrongcn1cnll11n orthotropic laminates. The pl)''s J)'n1n1etry iixc11 (d1,shcd lines)
must coinc1de with the l11n1inntc's orthotrop)' .r. y axes.

\Vhen the ply is n "'oven fabric and the ply's sy1nn1eLry axes are nligned \Vhb
the laminates orthotropy directions:
\vhen two adjacent unidirectional plies (oriented in different direclions) are
treated as a single layer and the symmetry axes of this layer are aligned \\-rith
the laminatc's orthotropy directions.
For the orthotropic plies described above, the Q1 and 'Q,. elements of the ply
stiffness m otrix are zero (Eq. 2. 138):
(3.35)
With those volues. Eq. (3.20) gives that the 16 and 26 c lements of the I Al. I BJ .
and (DJ matrices ore zero:

A", = Az,. = 0 B,. = Bu. = 0 D1 = D,,, = 0. (3.36)


1\ ccordingly. there is no e xte nsion-shear, bending- twist, or cxtc nsion- t\vist
coupling in an onhoiropic laminate (Table 3.5). On the o ther ha nd, when the
laminate i.s not or1ho1ropic. these couplings are present and rc$uh in unexpected
deformations.
We observe that the 16 and 26 elements of the (A). (BJ. and (D( matrices are
zero only i n the x- ycoordinate system. \\ilere.x and y are the orthotropydirections
(Table 3.4. page 76).
For wuymmetrical onhotropic laminates the complianC'C matrix becomes (see
Eq. 3.23)

~l
0:11 au 0 /In /Jn
,.,, au 0 fJ21 fJ21
0
/Jn p,.
0 a,,.
(J
0
811
0
8 11 P;I (3.37)

"" /Jn
() 0
0
/JM
8 12
0
822
0 i.J
78 lAMJNATED COMPOSITES

\Vhen the layup isorthotropicand syn1metrical, the e len1ents of the con1plia nce
ma trices are (see Eqs. 3.29 a nd 3.30)

11 " 12 0] d11 d 1z O]
0 12 0 12 0 d 12 d21 0 . (3.38)
[
0 0 ... [ 0 0 d,.

Isotropic /an1ina1e. \Ve consider a laminate in \vhich each ply is isotro pic.
(The n1aterial n1ay be dif(erent in each ply.) Since in isotropic n1aterials there is
n o preferred d irection, the [ QJn1atr.ix in the x- y coord inate systen1 is the same as
t he l Q} matrix in t he x 1- x2 coord inate system:

[QJ = [Q). (3.39)

Conseque ntly, the [A], (BJ, (DJ matrices a re inde pe ndent of the coordina te
directions. By introducing the e leme nts of the ( QJmatrix given by Eq. (2. 145) into
Eq. (3.20), we obtain the followi ng .elements of the (AJ, IBJ, and [DJ matrices:

A11 A12 = A 11 A12


A 11 - A 12
A..,= A1 =0 A,. = 0
2
B11 812 = ~ l 8 11
(3.40)
81 1 - 8 12
B,. =
2
B" =O a,. =0
D11 Du= Du Dri
Du - D12
[),,,, = Dio= O Di. =0.
2

Jn isotropic laminates t he re .are no extension-shear. bending- t\vist. or


e xte nsion- twist co uplings (Table 3.5, page 76), but there may be in-p lane - out
o f-plane coupling.
\Vhen the lan1inate consists of a single isotro pic layer, the no nzero ele1uents
o( the [A], (BJ, and (D] matrices are (Eqs. 2.145 and 3.20)

(3.41)

' vhere

[jw = Eh' , (3.42)


12{1 - v2 )
E is the Young modulus.v is the Poisson ratio, an d h is the thickness. The preceding
stiffnesses are identical to the stiffnesses of isotropic plates.. l

3 S. P. T1moshcn.ko nnd S. \Voino.,.,iskyKricgcr, r""'"''ofP/a1tsa11d Slwlls. 2nd edition. J.1cGraw-HiU,


1'1cw Yo rk. 1959. pp. 5 and 81.
3.2 STIFfN ESS MATRICES OF THIN LAMINATES 79

Invers ion of matrices [A] and [DJ yields t he compliance ma trices I) and (d]
(see Eqs. 3 .29 a nd 3.30). The no nzero ele ments a re
2(1 + "} l
a 11 = Eh a12 = - van U66 = Eh =Ch
(3.43)
12 , 24(1 +v) 12
d11 =iii ,... = /Jl = Chi'
_Quasi-isotropic /a111inate. t\ lan1inate is quasi-isotro pic \\'hen

there a re at least three fiber diree-tions;


the o rientat ion (fiber angle H} of each ply is f) = i1r/ I , where i is an integer
=
(i I. 2. . . . , /), and I is the total number offiber orientations ( I ;:: 3};
the number of plies in each fiber direction is the sa1n e~ and
each ply is made of the same n1aterial and has the same thickness.

For eJCample, in "/4 laminates (page 65) the re a re fibers in the O. 45. - 45,
and 9(}' directions, a nd I = 4.
For each ply, the elemenl< of the [ QJ matrix a re obtained by substituting fl =
i180' / I into the expressio ns in Table 3.1 (page 70). Then, by substituting these
eleme nts into the expression for the [AJ matrix (see Eq. 3.20), we obtain the
folJo,ving no nzero elements of the I A] n1atrix::

A:u = A,1
(3.44)
I 3 1
A 12 = Sh (Q11 + Q,,) + 4'' Q12 - zhQ..
A;e, = A11 - A12 .
2
The stiffness matrix (A) may be written as

A11 An
[All= A12 Au (3.45)
[
0 0
\vhere Rand v1110 are paran1eters defined as
3 1 1
R= 8(Q11 + Q,,) + 4Q12 + 2Q66
(3.46)
1
v1"' = R( Q11 + Q,, + 6Q" - 4Q66 ),
8
and his t he thickness of the laminate. It is stated here witho ut proof that Eqs. (3.45)
and (3.46} are vaJid2 for all values o! I as lo ng as I ~ 3.

1 S. W. Tsai and Ji . T. Hahn, /11u{){fucrior1 ro CiHJl/XJSit~ .\ldttrials. Tccbnomic. Lanc.as.tc.r. Pennsylvani.a.


1980. p. 145.
80 LAMINATED COMPOSITES

Ne\\t Refe--renc-e Plane-2

OriginaJ Referenc.c. Plane 1

Figure 3.14: Definition of the new refe re.n.ce plane.

\Ve obse rve that for both quasi.isotropic and isotropic la n1inates, A!? and
Ai;r, are A22 = A 11 a nd Arin= (A 11 - A12) /2. Thus, under inplane forces., q uasi
isotropic la1ninates be have in the same way as isotropic laminates. that is, there is
no extension-shear coupling (Table 3.5) a nd the [ AJ matrix is independent of the
coordinate d irections. The [ B] and [ DJ n1atrices do not si1nplify for q uasi.isotropic
lamin ates.
Elen1ent" A10 and Au, are zero a nd ~ = (Ai 1 - A12) / 2 for synunetrical and
unsymn1etr ic.al quasiisotropic lan1inates. Correspondingly, for symn1e trica l q uasi
isotropic laminates. the a 16 a nd air. e len1e nts of the compliance ma trix a re zero
(a 16 = 0 and a 26 = 0) and au,= 2(c.r 11 - a 12) . For unsymmetrical q uasi..isotropic
laminates, none of the eleme nts of t he con1pliance ma trix is zero (see Eq. 3.22).
Reference plane. The stiffness mat rices [A), [BJ , and [DJ refer to R eference
P lane 1. The stiffness n1atric.e s for Reference Plane 2 (located at a d istance ()
from Refere nce Plane L Fig. 3.14) a re obtained from Eq. (3.20) by replaci ng~
b y(Z - 11) as follows:
K
'\"; = L;<Q;;)k [(z - 11) - ( 4-1 - e)J
;..1
K
= I:<'?1;).C~ -z.-1) =A.;
;..1

(3.47)

~
-- ''(Q>['
K
L. ,,
;..1
-'-_
3
J
,-er, - 2r,_, + e'(:a _ , >]
.
J -'
Zk- 1

= D11 - 2q8;; + e'A,1 .


These equations correspond to the parallel axi' theoren1.
3.2 STIFfN ESS MATRICES OF THIN LAMINATES 81

The superscript e re(ers to Refe rence Plane 2. \Vhe n the laminate is symn1et-
rical and the reference plane coincides \\ith t he midplane, the ma trix I B] is zero.
\Vhen the la n1ina te is unsyn1n1etrical, the n1atrix [ B] is not zero.
In general, the re is no e value that results in a nonzero l BJ n1atrix for an
anisotropi c composite la n1inate. Jn other \\'ords., fo r an unsymmetrical laminate
there is no reference plan e that is also a neutTal plane.
The compliance matrices [a], [ft], and (8 J refe r to Refere nce Plane I. The
eleme nts of these matrices for Reference Plane 2 are obtained by int roducing
Eq. (3.47) into Eq. (3.23). After algebraic ma.nipulations, we obtain

a~ = ai; + '1 (IJ;1 + fJ ;;) + e28i;


p~ =/Ji; + '1~ij (3.48)
li~ = li;; .

The third of t hese equa tions sho,vs that the be nding con1plia nce n1atrix (S.J is
independent of the choice of the reference surface.
C11ned lan1inates. The stiffness and compliance n1atrices derived in this chap ..
ter for fia t laminates may be a pplie d to thin curved lan1ina tes \Vhen the radius of
curvature is large compared '"ith t he thickness.
Nu111erical l alues of the stiff11es.-; and cor11plia11ce 111atrices of selected /anti
1

nates. Be Jo,v, v.e present nume rical values o f the stiffness and compliance ma trices
of lamina~es with d iffe re nt lay ups. The engineering constants used to calcula te the
laminate stiffnesses and cotnpliances are listed in Table 3.6. While the properties
in this table a re not inte nded to depict a particular n1aterial, they are characteris
tic of n1aoy graphite-epoxy composites. The refore, the prope rties in Table 3.6 are
tLed in the examples in the book.

3.1 Example. Cit/cu/ate the sriffness [A], I BJ, [DJ and 1/re compliance [a L [/l J. [~ I
1natrices of a [010/ 451uJ huni11ate n1ade oj'gratJhite epoxy ru1idirectiotrt1/ plies. The
ply properties are given in Table 3.6.

Solution. The stiffne.ss ma trix of a unidirectional ply \\ith the fibers in the o. .
degree direction is I QJ0 = [QJ. T he stiffness matrix [QJ is given by Eq. (2.147),

Table 3.6. Properties of the material used In the examples


(OJ 45'
Longitudinal Young 's modulus (GPa) E, 148 1639
Transverse Young's modulus (GPa) 2 9.65 16-19
Longitudinal shear modulus (GPa) G 11 4.55 38.19
Longitudinal Poission's ratio .,, 0.3 0.801
Thickness (mln)
" O.l 0.2
82 LAMINATED COMPOSITES

and thus [ QJ is

~ ] 109 ~.
148.87 2.91
(Qf' = [Q] = ~91 9.71 012
(3.49)
[ 0 4.55

T he stiffness matrix [QJ of a ply not in the Odegree direction is (Eq. 3. ~4)

(3.50)

where [ T0 J and [T. J are given by Eq. (3.15). For the 45-degree ply c = cos 45' =
0.707 ands =sin 45 = 0.707, and we have

0.5 0.5 1.0] 0.5 0.5 0.5]


(T,,] = 0.5 0.5 - 1.0 [T, [ = 0.5 0.5 - 0.5 . (3.51)
[ - 0.5 0.5 0 [ - 1.0 1.0 0

By substituting Eq~ (3.49) and (3.51) into Eq. (3.50), we obtain the stiffness
matrix of the 45-degree ply as follows:

45.65 36.55 34.79] N


[QJ" = 36.55 45.65 34.79 109 , . (3.52)
[ 34.79 34.79 38.1.9 m

The layup is shown in Fig 3.15. In calculating the [AJ, fBJ. [DJ matrioes we treat
the ten 0-degree plies as one layer and the ten 45-degree plies as anoU1er layer.
T he ( Aj, I BJ. I DJ matrices are (Eq. 3.20)

[AJ = 1-Ql" (l1 - to) + [-"


QJ. (t 2 - t1)
-1 ' ., '
[BJ = (QJ" ' i - t;i + [ QJ" r, - r, (3.53)
2 2
(DJ= IQJ"z-/ ~ i;: + IQ)" z-i - zf.
, 3

I
~= I mm

I
E;.-~-o-
4510 Zi= O -
,.~~.i_,l~,~,.---,-n1m
....
Figure 3.15: 1be.(0 1o/4S 1fl) laminatc.in Ex.ample 3. J.
3.2 STIFFNESS MAIBICES OF THIN IAMlNATES 83

The [ Q] matrices are given by Eqs. (3.49) a nd (3.52). The dista nces (in meters)
are lo= - 0.001, t 1 = 0, z2 = 0.001 (Fig. 3.15). With these values Eq. (3.53) yields

194.52 39.46 34.79]


[Ai = [ QJ0 (o - (- 0.001) I + IQJ"(o.001 - O) = 39.46 55.36 34.79 t o" ~
[ 34.79 34.79 42.74

O' ( 0001)' 0001 ' O' [ - 51.61 16.82 17.40]


[B]=[QJ" - - . - + [QI". - - = 16.82 17.97 17.40 I03 N
2 2
17.40 17.40 16.82

[DJ
= l-Q.l"0' - ( -30.00li + IQJ"0.0013' - 0' [ 643'.84
I.. 15 I138.155
.4 11.6060] N m.
II.
Jl .60 I 1.60 14.25
(3.54)

The compliance matrices are (Eq. 3.23)

' o/3] = [AB


[{JT B]-'.
D (3.55)

Hence, we have

13.44 - 4.85 - 7.14]


[a ) = - 4.85 41.81 - 21.23 10- ~
[ - 7.14 - 21.23 64.95

lfJJ= [- ~~.l Ul6~~ - =~~


I
1.06
=::::] 10- ~
- 24.05
(3.56)

40.32 - 14.56 - 21.41 ] l


[6] = - 14.56 125.42 - 63.68 10- 3 N."'"'
[ - 21.41 - 63.68 194.86 m

The compliance a nd stiffness matrices of [452' - 45 2 /0 12/ - 452 /45.,J,


[- 304 / 15,/0,]., [02/452 / 90if - 451 [,. [456 / 0, J., and [02/452 /02/452 ] laminates are
calculated simila rly. TI1e result< are in Tables 3.7, 3.8, and 3.9 (pages 84-86). Note
that, for S)'lllmetrical laminates, the following: simplifica tions a pply: l BJ = [,8 [ = 0
and [a] = [a ], [o[ = [df.

3.2 Example. Calculate the sriffness [ AJ. lBf, [DJ and the compliance [a L[/3 L(o J
matrices of a [O,.] lmnimrte made of graphite epoxy tt1tidirectio1url plies. Tlie ply
properties are given in Tobie 3.6 (JJage 81 ).

Solution. The unidirectional laminate is symmetricaJ. and the [Bl matrix is zero:

[B) =O. (3.57)


84 LAMINATED COMPOSITES

Table 3.7. The (A] and (OJ matrices for symmetrical laminates. The unit
ol (A] is 1a6 ~ and the unit ol [OJ is N . m. The malerial properties are
given in Table 3.6 (page 81).
[A] [OJ
lOmJ(orthotropic. symme1rical}

297.75 5.82 0 ] 99.25 1.94

[
5.82
0
19.41
0
0
9.IO [
1.94 6.47
0 0 3t]
[451/012/ 45l J (orthoiropic. symmetrical)

215.17 32.74 45.30 19.52


32.74 48.17 19.52 25.26
[ 0 0 [ 0 0

[45,j - 45,i0 1ii - 45,/452] (balanced. symmetrical)

36~0i]
[ 215.17 .\2.74 [ 45.30 19.52
32.74 48.17 19.52 25.26 4.45]
4.45
0 0 4.45 4.45 20.62

[- 30,/15,/02], (symmetrical)

[ 235.54 32.74 - 10.19] [ 65.42 16.29 - 18.93]


32.74 27.79 - 10.19 16.29 11.60 - 7.74
-10. 19 - 10.19 36.01 -18.93 - 7.74 17.39

[01/45 ~/902/ - 451), (quasi-isotropic. symmetrical)

99.95 31.57 34.6 1 4.58


3.34]
31.57 99.95 4.58 12.34 3.34
[ 0 0 [ 3.34 .l.34 5.14

(45,,JO~J.. (symme.trical)

173.88 46.19 41.75] 34.84 22.93 21.7 1]


46.19 62.55 41.75 22.93 28.90 21.7 1
[ 41.75 41.75 49.47 [ 21.71 21.71 24.02

By treating the 20 plies as a single layer, the IA] and (DJ matrices are (Eq. 3.20)

[ A]= ir(QJ [DJ= :~[Q], (3.58)

tvhere Ir= 0.002 n1 is the thickness of the laminate.


By substituting Eqs. (3.20) a nd (2.139) into Eq. (3.58) and by using the en
gineering constants in Table 3.6 (page 81) (1 = 148 x 109 N/m1 , 2 = 9.65 x
3.2 STIFfN ESS MATRICES OF THIN LAMINATES 85

Table 3.8. The lat and [di matrices lor symmetrical laminates. The
unit ol lat is 10-w and the unit ol [d] Is 10-' o';;;. The material
properties are given in Table 3.6 (page 81).
[) [d)
(Oll>J (ortB10Lropic.. symmetrical}

10~89] 32~67]
[ 3.38 - LOI [ 10.14 -3.04
-1.0 1 51.81 -3.04 155.44
0 0 0 Q

[:!:45~ /01 2/ 4SU (orlhoLropic. symmetrical)

5.18 -3.5~ 0 ] [ 33.~0 -~5.59


- 3.52 23.1> 0 -25.>9 >9.37
[ 0 0 27.77 0 0

[45,/ - 45,/0,,/ - 451/ 45, ] (balanced, symmeirical)

5.18 -3.5~ 0 ] [ 33.16 - 25.33 - 1.69]


- 3.52 23.h 0 -25.33 6Cl.51 -7.fiJ
[ 0 0 27.77 - 1.69 - 7.60 50.51

[- 30,/15,/02], (symmetrical)
5.08 - 6.09 -0.29] 26.87 -25.93 17.70]
- 6.09 47.44 I l.70 -25.93 147.76 37.57
[ -0.29 I L.70 3 l.00 [ 17.70 37.57 93.52

[Oi;/451/901/ - 451), (quasi-isotropic.. ,symme1rical)

11.11 -3.51 0 ] [ 31.38 -7.44 - 15.55]


-3.5 1 11.11 0 -7.44 Hl0.06 - 60.17
[ 0 0 29.25 - 15.55 - 60.17 243.70

[456 /0.0], (symmetrical)


7.45 - 2.99 - 3.77] 71.24 - 25.43 - 41.40]
- 2.99 37.81 - 29.39 -25.43 116.82 -82.58
[ -3.77 -29.39 48.20 [ - 41.40 -82.58 153.66

109 Nim', G 12 = 4.55 x 109 Nlm 2, v 12 = 0.3), we obtain

N E2h N
297 .75 x Hf - A,,= = 19.41 x lo' -
m I - vf, e: m (3.59}
N
Ai..= G12/J = 9.10 x to<' -
m
,IJ3
D11 = ( ) = 99.25 N m [),, = - = 6.47 N . m
12 I -
2 E,
V12j - 12(t - v!2 ~;) (3.60}
G12h.l
D.,, = J2 = 3.03 N tn .
86 LAMINATED COMPOSITES

Tabl.. 3.9. The (A], (BJ, (OJ and the (oJ, [BJ, and [6J malrioes fl>r unsymmelrioal laminates. (A] is
in 106 ~. [BJ is in 1031'1, [OJ Is in N. m, [<>J is in 10- W. [.8J is In 10- ~. and [6J is in 10- .,i.;.
The material properties are given in Table :i.6 (page Iii).
[A] [BJ (OJ
(0 10/ 45rn]

[194-.52 39.46 34.79]


3~.46 55.36 34.79
[ - 51.61 16.82 17.40]
16.82 17.97 17.40 [64~
13.15
13.15
1160]
lS.45 11.60
34.79 34.79 42.74 17.40 17.40 16.82 I l.60 I l.60 14.25

(01/4.S2/0,/451]

o .~2
( 77.8 1 15.79 13.92] [ -4 129
l.346
l.392] [ 4 150 0.742]
15.79 22.14 13.92 1.346 l.438 1.392 0.842 l.ISl 0.742
1192 ll92 17.lO 1.392 l.392 1.346 0.742 0.742 0.912

(45~/0.,,)

42~74] -l~.82] [64~


14~25]
[ 194-.52 39.46 [ 51.61 - 16.82 13.15
3!l'.46 55.36 -16.82 - 17.97 13015 18.45
0 0 0 0 0

[a ) [ft) (4]
(Oio/45rn]

(13.~ -4.85 -7. 14] [ 1700


- 6.0l
- 11.06] [ 40.32 - 14.56 -2 1.41]
-4.8~ 41.S l
-7.14 -21.23
-'l64.95"' -6.0J - 5,04 - Jl.06
-I 1.06 - l 1.06 -24.05
- 1456 l2S.42 -63.68
-21.4 1 - 63.68 194.86

(0,/4.S,/0,/451)

[ 17.SS -7.14 -8 79]


- 7.14 96..15 - 69.68
[ 28.37 -9.99
-9.99 - 8.38 - 18.38
-LS.JS] [ .!35
- 134
- 134 - 165]
l 807 - 1 306
-8.79 - 69.68 128.51 -18.38 - 18.38 -39.96 - l65 -1 306 2 410

(45~/0rn]

43~70] 51~60] 131~ll]


[I l.65
-8.58
-8.58
32.94
[-13.97
12.22
l2::22
15.55
[ 34.94 - 25.74
-25.74 98.83
0 0 0 0 0 0
3.2 STIFfNESS MATRICES OF THIN LAMINATES 87

The compliance matrices[" J and [d) are obtained by inverting the [ AJ a nd [D J


stiffness matrices as follo;,..,,s:

(dJ = ior' . (3.61)

Equations (3.59)- (3.61) give

~
1 - 9 Ol
"" = ;," = 3.38 x 10- '"' = Eih = 51.81 x 10 N
- = 109.89 x 10- ~
1
ll12 = - V12llt l = - 1.01 X 10- 9 ~ u.., = -
G11h N
(3.62)

12 3 1 L2 1
I' = 10.14 x 10- -N m
du = -E,,. du= -EI' = 155.44 x 10- 3 N
-
2 r 111

o-'. - N
l
12 . _, 1
d12 = - Vii d11 = - 'n
.l.l.l"t X) ti,,, = G , = 329.67 x JO -N.
m 1211 m
(3.63)

3.3 Example. Ca/en /ate the stiffiress and dJe co111pliance 111atrices of (i) a ltuni1Jatecl
co111posite consisting of nvo lttyers of 45-degree lvoven fabric, nvelve
layers of Q.tfegree uuidirectlonal plies, and flvo layers of 45..Jegree 1vove11 fabric
([45\/012/ 45\)); and (ii) a laminate consisti11g jive layers of 45degree woven
fabric and Im layers ofO.Jegree unidirectional plies ([45\/011)]). The marerifl'l
properties are given in Table 3.6 (JJage 81 ).

Solution. First we conside r the laminate with ([45\/012/45\J) layup (Fig. 3.16}.
The laminate is symn1etrical, and the (BJ mattLx is zero:

[BJ =0. (3.64)

The con1pliance n1atrix of a unidirectional ply \\~th the fibers in the Odegree
direction '5 [Q) = [ QJ. The stiffness matrix f QJ is given by Eq. (2.147). and thus
[Q]0 is

~ ]1oN,
148.87 2.91
[QJ" = [ Q] = ~91 9.7 1 (3.65)
[ 0 4.55 m

__::4,,s,,.'._ _-1.ri=0.6 nun

Figure 3.16: The l:1)'Upoflhc laminate in Ex


ample 3_1. The supcrcripl r denotes fabric. o,,
~ 1 =-0.6 nun

ct- 45, zo =- Jmm


88 LAMINATED CIJMPOSITES

For a 45-degree wove n fabric the stiffness matrix (Q] is (Eq. 2.150)

45.65 36.55 o ] N
(-Qf.r45 = 36.55 45.65 0 109 m' (3.66)
[ 0 0 38:.19
In calcu lating the [A], [ BJ, (DI mat rices we treat the twelve Odegree plies as
o ne layer and each adjacent \Voven fabric as one layer. The j'A] and (Dl n1atrices
are

(A]= [QI'"" (ti - zo) + [Q]0 (ti - ti l + [Q]"" (t, - t i )


(3.67)
l Df = [Q)"" z;' - ttl + [Q] zi - z/ + [QJ"" zj - tf .
3 3 3
The (Q) matrices are give n by Eq s. (3.65) and (3.66). The dista nces (in meters)
are zo = - 0.001 , t i = - 0.0006, ti = 0.0006, and z, = 0.001 (Fig. 3. I 6). With these
values Eq. (3.67) yields
215.1 7 32.74
(A] = 3~74 48 17 0 ] to'!::!. (3.68)
[ 0 36.01 m

~ ]Nm.
45.30 19.52
( DJ = 19i}2 25.26 (3.69)
[ 0 20.62
The compliance matrices [a I and Id) are (Eqs. 3.29 a nd 3.30)
5.18 - 3.52 0 ]
- I - iJ Rl
[a) = [Af = - 3.52 23.1 5 0 JO N (3.70)
[ 0 0 27.77
33.10 - 25.59 0 ]
(d] = [Dr ' = - 25.59 59.37 0 10-> N '. m (3.71)
[ 0 0 48.51

The compliance and stiffness n1al!rices of the [45~/010] Jan1inate are calculated
s in1ilarly. The results are given in Table 3.9.
CHAPTER FOUR

Thin Plates

In practice \VC Cre qucntly e ncounter "thin p lates whose thickness is small com-
pared wit h nil o ther din1e nsions. Suc.h a plate. undergoing sma ll displacen1e nts..
may be annlyzed with the approximations that the strains vary linearly across
the plate. (out-01-plane) shear deformations are negligible. and t he o ut-of-plane.
norn1al stress a: 3nd shear stresses r ._, t,.._ are s maU compared y.ith the inplane.
normal"" a,., and shear Try stresses.
Under c:cnain ronditions, solutions may be obtained ror thin plates either by
the solutio n or 1he differential equations representing equilibrium or by energy
methods. 1 Herc \\"C demonstrate the use of the first method "ia the example of long
plates and the second me1hod via exam ples o f rectangular pla1cs either with sym-
metrical layup or with orthotropic a nd symmetrical layup. (For orthotropic plates
the d irections of orthotropy are parallel to the edges o r lhe plate.) We chose these
three types of problems because (i) they illustrnte tl1e anolyticnl approaches and
the use of Lhc rclc \1on1 equations, (ii) solution.s can be obLuincd \\1ithout exte nsive
nu1ne ricul olgoritluns. and last , but not least. (iii) they are o f practical interest.
Additiona lly, and iinportant ly. these proble n1s provide insights that are usefull
when a nal yzing plotcs by nun1erical methods.
Altho ugh the specification o ( orthotropy may seen\ to be overly restrictive. in
fact it does no t unduly limit the applicability of the anulyse:s. The reason for this is
tha t plates arc often made according to the 10...percenl rule. and such pla tes be have
similarly Lo orthotropic plates.2 There Core, solu tions fo r orthotropic plates provide
good approximations of the deflections.. maximum bending moments. buckling
loads. and natural frequencies of nonorthotropic plates that have symmetrical
layup and are constructed according to the 10..perccnt rule. The 10-percent rule

1 J. ~t \\'h1lncy. Str,tflural Auolysu of Lontinated .Anuo1roptc PlfltO Thchn<,nltt. Lancas1cr.


Penruyl"ani1. J9R7
1 L Veres a nd L.. P Kollllr, Approximate Anal)'Sis of ~lid-plane S)innlctric Rcc1oniular Composilc:

Plalcs. JtJu rnal ofC01t11>fJ.~ll(' .~tir1ena/$. Vol. 36, 67~ . 2002.

89
90 THIN PIATES

r equires that the plate satisfy the C<>llo,ving conditions:


The plate is n1ade of unidirectio nal plies.
There are at least three ply orien tations.
The angles bet\veen tJ1e fibers are a t least 15.
The nun1be r of plies in each fiber direction is at least LO percent of the total
nun1ber of plies.

Plates confo rn1ing to t he 10-percent rule have better load bearing capabilities
t han unidirectional or angle-ply lan1inates for the follo\ving reasons.
Unidirectional plies are stiffer an d s tro nger in the 0 -degree fiber direction than
in the 90-degree direction perpend ic.ular to t he fibers. Thus. laminates made of uni-
directional plies are ills uited to carry load in the 90-degree direction. Angle -ply
lan1inates \vith only l\\'O fibe r d irections do not resist \Veil tensile load s applied
a long the symn1etry axis.. Plates mad e by t he JO.. pe rcent rule n1inimize these short
con1ings.
The s pecification of sy1nme tricaJ layup is less restrictive than it m ay appear
because the analyses of symme trical plates (for which ([BJ= 0) can readily be
e xte nde d to unsymme trical plates (( B] "# 0) witlt the use of the re duced be nd ing
stiffness IDJ, defined as u .s

(D) =(DJ- (B](Ar ' (BJ. (4.1)


The de Hections, n1aximum be nd ing mo 111 e.nt ~ buckling loads, and n.atural fre.
quencies of unsymmetrical pla tes can be approximated by replacing fDJby (DJ>
in the expressio ns de rived for symmetrical plates.

4. t Governing Equations
lln this section we sumn1arize t he equatio ns used in an alyzing thin plates. \Ve
e n1ploy the x. y. t coordinate systen1. The o rigin is a t the n1idpla ne fo r plates
' vith syn1metrical layup a nd a t a suitably chosen reference plane for pla tes \\ith
unsymn1etrical layup.
The strains a nd curvatures of the refere nce pla ne (Fig. 3.10) a re (Eq ~ 3.1, 3.8)

&u0 0 auo au" &vi)


fo=-
:~ ax y =ay
- y(I
Xf
=ay- + -ax
(4.2)
2a2 ...,o
Kxv= - - .- -
ax &y
3 J. flit. Whilney. S1ruc111rt1/ Analy.ds of L1111i1w1ffl AnU01rop1c Platt!L Tecltnonlic., Uincastcr.
Pcnnsyl\1ania . 1987, p. 203.
" E. Rcissner 11nd Y. Stuvsky. Bending and S lrctching of Certain T)'pcs of lictcrogcncous Aclo tropic
EJastic Plates. 11J11J'J11JI of Applied AlttJuut.iC$. Vol. 28, 402-408, 1961.
$ J. E. Ashton. Approxin1a1c So lutio ns for U ns)1n1me1rkally Lnn1inatcd Plates. Journal ofCn111pMite
1\l t11eriills. Vol. 3. 189-191. 1969.
4.1 GOVERNING EQUATIONS 91

Figure 4.J: Forces and loads ac.ting o n an clement of the plate.

where u> and v<> are tbe displacen1ents of the refere nce plane in the x a nd y
directions., a nd wl) is the out-of.plane displacen1e.n t (de.Hection) of this plane. The
force-strain re lat ionships are (Eq. 3.21)

N., A11 A 12 A1 B11 B12B,. <"x


N_,, A12 Ail
"" 812 Bn Bu. f'.;
1\J.yy A1 A26 Ao. s,. B1ti Bo,, y~,
(4.3)
M., = s, , 812 B,. Du D12 v,. 1(,
M,. Bn Bu s,. D 12 /),..2 v,. I(,,
M,,. B,. B i fi B,,,, v,. v,. v Kxy

In the analyses we n1ay e mploy either the equilibriun1 equations or t he strain


energy.
'TI1e equilibrium equations are6

I) N., al\J.yJi
--+--
3x ay
= - p:r;

aNr a1v.T>' (4.4)
a>' + ax = - py
av, av,.
- + - = - p-
ax ay
,, aA(, + -
1'.t = --
aM,,.
- V. _ ~ aM,,,, (4.5)
3.t ay r - ay + ax

where p.1 p,., a nd P:. are the components of the distributed surface load (pe.r unit
area); N.r. 1'J,. and 1\~ty are the in~plane forces (per unit length); \~... a nd Vy are the
transverse shear fo rces (pe r unit le ngth)~ flt& , M>' and A1."'Y are., respectively, the
bending moments a nd the twist moment (per unit length ) (Fig. 4.1).

'' S. P. Ttmos.hcnko nnd S. \Voinowsky-Kricgcr. r1teoryof Plt11esa111/ Sl1~lls. 2nd edition. 1'1cGrnw-HiU,
New York . 1959, p. 80.
92 THIN PLATES

4.1.1 Boundary Conditions


T he conditions along each e dgeo( the pla te n1ust be specified. Boundary conditions
for an edge parallel \Vith the yaxis are given be )o,v.
Along a buili-in edge, the de Aection w, ihe roiation of the edge aw ~/ax, and
the inplane u0 u0 d isplacen1ents are zero:

w0 =0 u" = v> =O. (4.6)

Along a fiee edge. v.here no external loads are applied, the bending n1oment
MK. the replacement shear fo rce7 v1: + aA1Jty/ax. and the in~plane (orces N.,' NJ;)'
are zero:

M,=0 Vx+
ail(,,
=
o (4.7)
ay
Along a simply supported edge, the deflection u/ ', the bending mo:ment J\1x,
a.nd the in-plane forces N.1 , Nx,v are zero:

M, =0 N., = Nxy = 0. (4.8)


\Vhen in plane motions ar e prevente.d by the support, the in-plane forces are
not zero (N., ;/; 0. 1Vxy 0) V"he re as the in-plane displacen1ents are zero:
tl' = () t1 =0. (4.9)

For an e dge parallel with the x-axis, the preceding boundary conditions hold
'vith x and y inte rchanged.

4.1.2 Strain Energy


As we noted pre\1ously, solutions to plate problems may be obtained by energy
me thods that require knowledge of the strain e nergy. For a linearly e lastic material
the strain e nergy is given by Eq. (2.200). Under planestress condition the stress
con1ponenL~ er~. rx~ and ry;: are zero (Eq. 2.121), a nd the e xpression (or the stTain
e nergy sin1plifies to
L., 1., ht

U= ~ J.! .!
0 0 - Jr.
(O'xfx + 11,.<,. + r,,.r.,,.) d ulydx, (4.10)

,..,here h1 a nd hb are the d istances fro111 the re fe re nce plane to t he plate 's surfaces
(Fig. 3.12). The stra in components are (Eq. 3.7)

(4.11)

1
Ibid.. p. 84.
4.2 DEFLECTION OF RECTANGULAR PLATES 93

The stresses and the s ua ins at a point a re re la te d by (Eq. 3. 13)

{"'"'r.vy"' } = (Q] {Yxy


<x }
fy (4.IZ)

By substituting Eq~ (4.1 1) and (4.12) into Eq. ( 4. 10) and by utilizing the definitions
o( the [ Aj. [ 8], [ DJ matrices (Eq. 3.18), we obtain the following expression fo r the
st rain e ne rgy:

T
"x A11 A12 A16 811 811 s.. "x
L, I .., y A 12 A11 A16 8 11 Bi1 8,. y

V=~f J Yx~\ At A,. At.. 81 Bi. B,,, y:, d ydx.


Kx 811 812 Bi D11 D 12 D1 K.x
0
" Ky 811 811 B 16 D,, D12 Di. Ky
Kxy 810 826 s.. o.. D,, Dr,,, K.vy

(4.1 3)

The superscript T denotes t he transpose o f the vector.

4.2 Defl&etion of Rectangular Plates

4.2.1 Pure Bending and In-Plane loads


\\'e consider an unsuppo rted rectangular plate subjected to pure bending and
to inplane loads (Fig. 4.2). The inplane forces and moments are related to the
refe re nce plane's strains and curvatures by E q. (4.3). Six of the twe lve q uantities
appearing in this equation must be specified as folJo,vs:

tV.T or x M, or K,
1Vr or ")'
J\1y or Ky (4.14)
1V.t y or l~y J\(lJ' or ,_;X)"

With si..x of the q uantities chosen (Eq. 4.14), the re.ma ining six may be obtained
by solving the six simultane ous equations give n by Eq. (4.3). Once the curvatures

1-igurc 4.2: Rectangula r plate suhjc-eted to


bending and in-plan e. loads. y
94 THIN PLATES

y A. ~
~ ~
g ~

L,
I
Figure 4.3: m e difere nt types of supports along !he long e dges of a trans\'ersef) l<>aded long
pl ate~

are kno\vn, the deflection of the refe re nce surface w 0 is ca lcula te d by using the
r elationships betv.een the curvatures and the detlection ( Eq. 4.2) as folJo,vs:

(4.15)

T he follo,ving deHection satisfies these relationships:

(4.16)

TI1is expression for w0 does not include the deflection of the re ference plane
d ue to rigid. body n1otion.

4.2.2 Long Plates


We conside.r a long rectangula r pla te whose length L.v is large compared \Vith its
vidlh L,. The Jong edges may be b111iltin, simply supported. or free, as shown in
Figure 4.3. The pla te is subjected to a distributed transverse load p. Neither this
load nor the edge s upports vary a long the longitudinal .Y direction.
\Vben t he length of the pla te is large con1pare d with its '"idth, a\\ay fron1 the
s hort edges the deflected surface nl ay be assumed to be cylindrical (cylindrical
d eforn1ation, fig. 4.4), and the forces and mon1ents do not vary apprecia.bly along
the length. \\'ith these approxi mations the analysis sin1plifies considera b]y. Before
tve undertake the analysis, we es tablish the lengthtO\\idtb ratios (or \Vhtch a plate
may be considered long.
For an isotropic plate the longp late approximation for the deHecti on is re a
sonable \\he o8
L:
~>3 isotropic plate. (4.17)
L,
\vhere [},. a nd L:
are the le ngth and the \\idth of the isotropic plate, res.pectively.
We no'v establish for orthotropic plates the length..towidth ratios a t "vhich the

-" Ibid., p. 118.


4.2 DEFLECTION OF RECTANGULAR PLATES 95

figure 4.4: Cylindrical deformatio n of a long


rectangular pl at~

longplate approxin1ation may be a pplied. To this eod \Ve obse rve that the deflec-
tions of an orthotropic pla te (\\ith length Ly and \vidth L.1) and a n isotropic plate
(wit h length u, and width U,) a re similar when (page 109)

(4.18)

Thus, f rom Eqs. (4.17) and (4.18) we have that the Jongplate a pproximation
is reasonable \\he n the folJo\ving inequality is satisfied:

l ., > 3
l,
JD::
y/5; orthotropic pla te. (4.19)

TI1is formula, v"hich is established for orthotropic pla tes. n1ay also be used as
a guide for plates whose layup is not o rt hotropic.
We now proceed with the an alysis of Jon.g plates in cylindrical bending. The
gene ra tor of this cylind rical surface is para lle l to the longitudinal y..axis of the
plate. The c urva tures Ky and Kxy of the plate are zero

Ky = 0 Kxy = 0. (4.20)

and K, is (Eq . 4.2)

(4.2 1)

1\ way fron1 the short edges the forces an d mome.n ts do not vary a long the
length or the pla te. Thus, from the last of Eq. (4.4} a nd the first o( Eq. ( 4.5) we
have
tfV, O
- (4.22)
dx+ 1>-=
~

dM, _ \I, =O. (4.23)


dx
The lood is pe rpe nd icular to the surrace, and for simplicity we replace P:: by p.
Thus, by s ubstituting V, from Eq. (4.23) into Eq. (4.22) we obtain the equilibrium
equation
t/2 JWx
-;;;;- + p = 0. (4.24)

We oo te that this e quation. representing e quilibriun1. is independent of the


materia l.
96 THIN PLATES

Figure 4.S: T he plate in Bxampk 4.l.

Symmetrical layup. The layup<>( the plate is symmetrical ([ B] = 0). We now


concern ourselves only with the bending mome nt M,, which. from Eqs. (4.3) and
( 4.20). is

(4.25)

The ele ment D11 o( the matrix [DJ is given by Eq. (3.20).
By substituting Eq. (4.25) into Eq. (4.24) and by using Eq. (4.21), we obtain
the fo llowing equilibriun1 equation tor the anisotropic Jong plate:
tfw _ L = long plate
0 (4.26)
dx' D11 symmetrical layup.
The equation governing the de.Hection of a transversely loaded isotropic. beam
is9
d4w p'
-- -=0 isotropic beam, (4.27)
dx' El
\\here E is Youngs n1odulus. I is tile mo1nent of inertia about the y-axis, and p'
is the transverse load per un it length.
By comparing Eq. (4.26) and (4.27), we see that the equations describing the
d eflections of a long plate (syn1rnetr ical layup) an d an isotropic beam a1e s imilar.
Consequently, the deflection o( a long plate (symmetrical layup) with be nding
s.tiffne.ss D11 is the san1e as the de Hection of an isotropic beam \vith be nding
s.tiffness El when the nume.rical values of the loads are equal (p = p' ). (Note
ho,vever that p is per unit area and p' is per unit length.) T hus, the deflection of a
long plate with symmetrical layup can be obtained by replac.ing El/ p' by D 11 / p
in the expression 10 given for the deflection of the corresponding isotropic bea1n .

4.1 Exan1ple. A 0.1-111/011g turd 0.2nt11Jide rectangular plt1te ls nrade of graphite


epoxy. Tire material properties are given ilr Table 3.6 (page 81). Tire layup is
[ 45\/ 012/ 45\). Tire 0-degree plies are parallel ro tire s/iorr edge of rhe plate.
The plate l\ sbnply supported ttlong all fo1'r etlges and l\ subjected to a rtnifornrly
disrributed 1ra11sverse load p = 50 000 1V/nr 2 (Fig. 4.5). Cal11/ate the 1naxi11uuu

"' E. P. PopoY. n~i11eeri111: AleC'ha11its of Solids. Prcntioc~Hall. En~kwood Cliffs. New JJcr:icy. 1990.
p. 505.
to YI.'. 0. PilkC)', Founulos ftJr Sud.set. St1ai11.t. tu1d Su1u.1u111J Alauict!l John Wiley & Sons, New York .
19?4.
4.2 DEFLECTION OF RECTANGULAR PLATES 97

dejfet'tion, the 111axin1u1n bending 11101ne111s. and tire stresses tt11d strains ilf ead1
lt1)1er.

Solution. The bending stiffnesses of the pla te are (Table 3.7. page 84) D11 =
45.30 N m and Du= 25.26 N m. We may treat this pla te as long when the fol-
lowing cond ition is me t (Eq. 4.19):

-
L,> > -
L.r
~"
Di.2
(4.28)

ln the present problen1 the tern1s in this inequality are Ly/L.r = 3.5 and
3:/D11 /D?.i - 3.47. Thus. the preceding condition is satisfied and t he loogplate
expressions n1ay be used.
The maximum deflection of a simply supporte d be<>m is (Table 7.3, page 332}
_ 5 p' L'
UJ = - - -. (4,29)
384 /
The maximum deflection of the pla te is o btained by replacing El / p' by D 11 / p
(see page 96). For the plate under consideratio n Di 1 = 45.30 N m and L., = 0.2 m.
and \Ve have

_ 5 11L~
w= Dii = 0.0230 m = 23.0 mm. (4.30)
384
The bending mome nts are (Eq. 3.27)

A1x = + D t2Ky + D l61(i:y


D11K.t (4.3 1)
J\~" = D12Kx + D12KJ' + D26Kxy (4.32)

For a long plate K y and K:iy are zero (Eq. 4.20}, and M.x and My are

(4.33)

The maxin1un1 bending n1oment Mx. v.hich arises at 4 / 2, is (see Table 7.3_
page 332)

pl; N m
M, = - = 250.00 - - (4.34)
8 m
From E qs. (4.33) and (4.34) we have

K_, = M, = 5.52 .!.. (4.35)


D11 m

From E qs. (4.33) and (4.34) the maximum bending moment M, (at L../2) is

Di1 M, =
Di2Kx = ,,-- N m
M,, =

- 107.75 - - . (4.36)
'-"I t m
0.4~ 1
98 THIN PLATES

, : :

;t
'
45?

o,1 0.6 E.,


0
500 n ,
0
200 a 11
(% )

t 452 ' -1 -1 -1
figure 4.6: 'l'be nonzero strains nnd slre.ss:es across the thickness o( the plate at L_/ 2 in Exam
p ie 4.1. The unit of a 1s 10" N/m2.

For !he long plate (Ky = K_,y = 0) the strains a l L.,/ 2 a re (Eq. 3.7)
fx = Kx Z = 5.52.t
fy =0 (4.37)
Y.vr = 0.

The strain d istribution a t ~ /2 is s hown in Figure 4.6. Thestressesarecalculated


b y (Eq. 3.l I)

(4.38)

The s tiffness n1atrices fo r the fabric and for the unidirectional layer are given
b y Eqs. (3.65) a nd (3.66). TI1e stresses in the lop layer (where z = /J/ 2 0.001 m) =
at L.,J2 are

"" Q11 45.65 251.95 N


{
dy l = [ -Q12] ~.. = [ 36.55] 109 )( 5.52 )( 0.001 = [ 201.73] 106;;;;
l
~1 ~ 0 0
(4.39)

The s tresses in the other layers are e-alculated s in1ilarly. Tue resulls a re s hown
in Figure 4.6.
Unsy111n1etrical layup. The layup of t he plate. is unsynuuetrical. One of the
long e dges must be restrained along the length\vise direction. \Vith the plate t hus
restraine d, the strain in t he longitucilinal y direction is ze ro throughout tJ1e pla te :

(4.40)

furthermore, we take the shear force N.,, to be zero:


N.vy = 0. (4.41)

Equation (4.41) is valid \vhen one o( the long edges of t he plate is free to n1ove in
the length\vise ydirection. It is only an approxin1at ion \\'he n the le ngth\v~se n1otion
4.2 DEFLECTION OF RECTANGULAR PLATES 99

of both long edges of the plate is restricted, as in the top three configurations of
Flgure 4.3.
By substituting Eqs. ( 4.20), (4.40), and ( 4.41) into the third a nd fourth expres
sions of Eq. (4.3), we obtain

IMJ - [811 8,. o,,]{~'.;'" }


0 _ A 16 l\ir, B it. ~
(4.42)

We now select a reference plane at a distan ce e from the mid plane (Fig. 3. 14).
The strain con1ponents and the stiffnesses referred to this reference plane are
identified by the s uperscript e. (The curvature and the 1\ilx con1ponent of the
moment a.re independent of the position o( tihe refere.nce plane and thus do not
need to be identified by the superscript e.) fur the new reference plane Eq. (4.42)
is v.ritten as

(4.43)

The first rov. of this equation gives


A 9
(J'? = - 0.q -1! - I( ....il.. (4.44)
Y.vy .l A6 .t At,
Substil ution of Eq. (4.44) into the second row of Eq. (4.43) yields

M= (8 - A~. 8~) ..... + (o114


.v ll~x
- ~;,) x K (4.45)

In gen.era!, the be nd ing mon1ent M.x depends both on ,~"' and KJ;. Ho\vever.
there is a reference plane for \Vhich the tern1 :in the parentheses in front of ,;;I? is
zero:
A 9
8~ - ~ =0. (4.46)
11..

We recall that the stiffnesses in the midplane (Refere nce Plane I) a nd t he new
reference plane (Reference Pla ne 2) are related by (Eq. 3.47)

A;~ =A;;
8;~- = Br; - i.>1\ 1 (4.47)
D;~ = D;j - 2eB,, +e 2A;; .
Equatfons (4.46) and (4.47) give

811 - ~ -
A 16B16
o ( A11 - Afti)
A.. = 0. (4.48)
100 THIN PLATES

By rearranging this equation, \Ve obtain the position of the reference plane
where Eq. (4.46) is satisfied:

(4.49)

For a re ference plane at e distance fron1the 1nidplane, the n1oment At/., depends
c>nly on Kx as follo\\s:

D~1 (~~ ]K,.


1

A(,= [ - (4.50)

Equations (4.50) and (4.24), together "1th Eq. (4.2), yield the following equi
llibriun1 e quation for an anisotropic long plate:

long plate
(4.51)
unsymmetrical layup,
11\'here t he S)mbol '41 is the. bending stiffness paran1eter

(B )1
'l'=Df, - -'-;- (4.52)
A
B y using Eq. (4.47), IV may be written as

(816 - e A,.) 2
IV = D,, - 2t>B11 + q, A11 - (4.53)
A"'
where(! is given by Eq. (4.49).
By comparing Eqs. (4.5 1) and (4.27), we again observe that the e quations gov
e.rning the deflections of long plates and isotropic beams are similar. Therefore.
tthe deHection of a long plate with unsymn1etric.al layup can be obtaine.d by replac-
iing El/ p' by 'II/ p in the expression given for the deflection of the corr-esponding
~sotropic bean1.

4.2.3 Simply Supported Plates - Symmetrical layup


We consider a recta ngular plate \Vith din1ensions L.v and Ly simply supported
along its fou r edges (Fig. 4.7). The layup of the plate is symmetrical a nd [ BJ = IO).
T he pla te is s ubjecte d to a unifom1ly distributed load p.

Figure 4.7: RecL1ngular stmpl:r supported


(ss) plate subjected to n uniformly dis-
tributed transverse load.
4.2 DEflECTION OF RECTANGULAR PLATES 101

Follov.ing Whitne)', 11 \Ve analyze the deHection of this pla te by the e nergy
me thod. For a simply supported plate (symme trica l layup) subjected to out-of-
plane load o nly, the in-plane stra ins in the midplane are zero, a nd Eq. (4.13)
simplifies t o

D1] {1<., } dydx.


[),_. Ky (4.54)
D66 K.TJ"

By using the re la tionships between the curvatures a nd the deflectio ns given


by Eq. (4.2), we obtain

For the applied transverse load p (per uni t area) the po tential of the exte rnal
forces is (Eq. 2.203)
L, /.,

Q = - ff
0 0
0
(pw )dydx. (4.56)

We use the Ritz method and select an expression for the deHection that sat-
isfies the geon1etrical boundary condit ions. For the sin1ply supported plate unde r
oo nsidera~ion t he geome trical boundary conditions require t ha t the deflection be
zero along the edges (see Eq. 4.8) as follows:

u/, = 0 a l
,..,
x= L"'
0 :sx _::; Lx
and 0:5y.$Ly
and O!i_y!i. L,.
and y=O
(4.57)
O:sx:SL;t and y = L,.
The (o]Jo\\ing deflection satisfies these conditions:
I l
11'X . pry
uJa = " "
L-L-"';;s1n - s1n - . (4.58)
i I ; -1 L.t Ly
where I a nd J a re the nun1ber or tern1s, c hosen arbitrarily. (or the sumn1ations
and w;i a re consta nts a nd are calculated fron1 the principle of stationary pote.n tial
ene rgy (Eq. 2.206) expressed as

(4.59)

11 J. ri.t. Whitney, Suu rtural Aual_1.,J'J: of Lat11iriared A11tlo1ropic Plat1'$. Tcchoornic. Lane.aster.
Pennsylvania. 1987. p. 13.l.
102 THIN PLATES

We now substitute w" (from Eq. 4.58) into the expressions of U and Q (Eqs. 4.55
and 4.56) a nd perform the differe ntia tions indicated above. Lengthy but straight-
f onvard algebraic n1anipulations result in the following systen1 of sin1ultaneous
algebraic equations:

Ii:
I I
/11 = I. 2, 3, . . . , I
I: L
; .. 1 ; - 1
Gm,,ijWij = P'"''
/ , H=J.2.3 ..... J.
(4.60)

For convenience, we introduce the contracted notation

= I. 2. 3. .. . I
l
i
k = (i - 1)1 + j (4.61)
j = 1, 2. 3. . . . . 1

l=(m - l)/+11 jn=l


= 1 2 3 1
111
, 2 , 3, . . . , J .
(4.62)

Equation (4.60) may now be written a~


I x/
L G.,w, = Pt I= I. 2, 3, ... . I x J. (4.63)

-
where Gkl (= G,.) is given in Table 4.1 a nd. for a uniformly distributed load, Pt
is
. <Ip:~ I.-; if 111 and n are odd
p1 =
l n 11111

0 if 1u o r H is even .
(4.64)

Table 4.1 . Theelements of the matrix [GJ

- 2L, L,~' D1.[(i;)'(J; )(t;)r,.,r,. + (f )'(f;)( i;)r~r.,,)

- 2l,.1..~1 Dz.[( t )'(f )(f, )r,.,r 1 + (i; )2 ( i; )(-/:; )r..,r,,)


I if k =I
'" =l0 ilk,</

P-Uf-;I "f ('I - I') lS odd


-
r,, -
I 0
I
if (i - j) is even

k = (i - 1)1 + j l;. = I. 2. 3. . . . I
J = 1, 2. 3,. . . . ./
m=l, 2.3 . ... , /
I = ( m - J)J + n
l n=l . 2. 3, .. .. J
4.2 DEFLECTION OF RECTANGULAR PLATES 103

In expanded form, Eq. (4.63) is

G11
G2,
(4.65)
[
G\l~J)I

II
By inverting this equation, \Ve obtain the coefficie nts 1JJk

G1(/xl/ i-I ~; (4.66)

Gvx J)\lx J ) POx l /

From Eqs. (4.2). (3.27). and (4.58) the moments are

I J
EE w1; ( I: )2 sin Lil
4
s in l!!:f.
i- l j l ' L,
JI(, }
M,. =(DJ
{ Kx }
Ky =ID] ''
I
t-t....
J
~ w; ('') sin
I L1
2
iir.T
L,
.
sin!!!.!.
1--y (4.67)
{ i - 1 1- 1
M.T_v J(:tf
I J . iT .,,
- ."
'- "
I- w;I2!.!.Lcos
L, 1., ~cos
I'! L.l
I.,
1 l 1 I

For a 1> orthotropic plate D16 = [),_. = 0, and Eq. (4.66) becomes

l6p
wk = w;; = >r6 ij[ D11( t )' + 2( D12 + 2 D"')( i; )2( i; )2 + D22(i; J']'
(4.6S)
where i , j =I. 3, 5 .... (wk= w;; = 0 when i or j = 2, 4. 6 .... ).
Once the deflections are kno,vn, the mon1ents can be calc.ulated by
Eq. (4.67).

4.2 Exa111ple. A 0.11t1lo11g and 0.2 ~111 1 vide rectangular plate i..f 111ade of graphite
epoxy. The tuaterial properties are given in Table 3.6 (page 81). The loyup is
(45i/ 012/45i). The 0-tlegree plies lire parallel co 1/ie shorr edge of the plttte.
The plate is shnply supported ttlong all four edges and is sttbjecred to a u11ifor1n/y.
=
distributed 1ra1Jsverse load p 50 CX>O 1\111112 (Fig. 4.8). Ca/en/ate the 1naxin1un1
deflection and the 111axin1111n bendi11g 1110111e111s.

Solution. The deflection of the plate is (Eq. 4 .58)

'' '' .
I

i I ;- 1
J
w~ = L- L- w;; su1 t11'X
-
L-x
. -
sin l"'Y.
Ly

(4.69)
104 THIN PLATES

Figure 4.S: ThC' plate in txample 4.2.

The plate is orthotropic, a nd the bending stiffnesses are (Table 3.7, page 84)
.D11 = 45.30 N m, D,i = 25.26 N m, D12 = l 9.52 N m, D,,, = 20.62 N m). The
rnaxin1um deflection occurs a t the center of the plate, \vhe.r e x = L., /2 = 0.1 m
and y = l y/2 = 0.35 m. From Eq. (4.68). w;; x l03 a re

i \j 2 3 4 5 6 7
I 24.0389 0 2.9945 0 0.6683 0 0.2033
2 0 0 0 0 0 0 0
3 0.1181 0 0.0330 0 0.0148 0 0.0075
4 0 0 0 0 0 0 0
5 0.0093 0 0.0029 0 0.0015 0 0.0009
6 0 0 0 0 0 0 0
7 0.0017 0 0.0006 0 0.0003 0 0.0002.

At t he cente r of the plate the de Hect:ion is

1 1 . .
w = LL wi;sin "r-, stn -pr (4.70)
i- t ;- 1 _. 2.

\Ve c hose to perforn1 the sun1ma tion up to i =


j = 7. For i, j = 2, 4. 6, the sine
is ze ro; for i. j = 1, 5 the sine is unity, and fo r i, j = 3, 7 the sine is min us one. The
r esulting deflection at the n1idpoint is

w = 0.0214 m = 21.4 mm. (4.7 1)

\Ve now assess the. lengtht0\\ idth ratios under \Vhich the long. plate a pproxi
1

rua tion is reasonable. To this e nd , '" e calculated the n1aximum deHect[ons of the
plate, keeping the width l;r the same while changing the length Ly- In figure 4.9
\Ve plot the maximum deflections thus ca lcula te d versus Ly. In this figure \\'e also
included the dellection given by the long-plate a pproximation (Eq. 4 .30). The
r esults in this figure show that, in accordance with Eq. (4.19), the long-plate for
mula approximates the deflection \Ve ll (\\ithin 8 pe rcent) when L.v is gre ater than
3l_,~D11 / /J22 - 0.694 m.
4.2 DEFLECTION OF RECTANGULAR PLATES 105

I 20
,-----------------------------------
Long-plate approx

'
.~ 10
ii
c
~

o o ~~~~~~~~~~~~~~~~~~~~
0.2 0.4 0.6 0.694 0.8
Length, l 11 <m>
1-igure 4.IJ: '.Maximum deflection or the plate in E:ta m plc: 4.2 as a functio n of the plate length.

The !>ending momenls at the center of lhe pla le are (Eq. 4.67)

(4.72)

The twist moment at the corne r o( the plate (x = .v = 0) is (Eq. 4.67)

(4.73)

4.3 Exantple. A 0.2-1n-/01Jg and 0.21n-u ide rectangular plltte is 111ade of graphite
1

epoxy unidirectional plies. The 1na1erial properties are given in Table 3.6 (page 81).
The layup is [0,/452/ 90,/-452),. The plare. simply supported a/o11g the four edges
=
(Fig. 4.10), is snbjected too unifor111/y distributed transverse load p 50 CNX>Nltn 2.
Cfllculate Jhe n1axil1u1111 d eflection tnrd the 1110.rinuun n1on1e1us.

Solution. The layup of the plate is syn1n1etrical but is not orlhotropic. The be.n
= =
ding stiffnesses are Dn 34.61 N m, Du 12.34 N m, D12 4.58 N m, =
y
SS

x
Figure 4.10: The plate-in Example 4.3.
106 THIN PLATES

Table 4.2. The maximum defle.:lion and the maximum bending and twist
moments calculated by the numerical solution and by the orthotropic
approximation for the plate in Example 4.3
M., M.,
... M, M, X= Y = O X= L, Y= O
mm Nmim
numerical 17.60 160.65 64.53 - 69.28 36.47
orlhotropic 16.93 154.07 63.21 - 49.56 49.56
itpJJroximation

D.. = 5.14 N m, Drn = 3.34 N m, Di.= 3.34 N m (Table 3.7, page 84). The
rmaxin1u1n deflection and the maxin1un1 bending and nvist 11101nents n1ust be ca)..
culated fro m Eqs. (4.58), (4.64), (4.66). and (4.67). With the preceding stiffnesses
tthe calculations yield the results give n in Table 4.2 (first row).
The layup follows the JO.percent rule (page 89), and we treat the plate as
o rthotropic. The deflection of the pl.ate is (Eq. 4.58)

Wo ""'
I J
11rx l "Y
= L-L- W;jSll\-SJO--
; . 1 ;- 1 L_,
.

i_.,,
.
(4.74)

Since the plate is treated as orthotropic Du:. = Di6 = 0, and the relevant bend
= =
<ng stiffnesses are D11 34.61 N m, D,1 I 2.34 N m, D12 4.58 N m. D.. = =
5.14 N m (Table 3.7, page 84). The maximum deflection occurs a t the center of the
l!Jla te, where x = = = =
L,/2 0.1 m and y L,./2 0.1 m. From Eq. (4.68) w;1 x JO'
are
i\j 2 3 4 5 6 7
17.3628 0 0.3409 0 0.0314 0 0.0061
2 0 0 0 0 0 0 0
3 0.1439 0 0.0238 0 0.0052 0 0.0014
4 0 0 0 0 0 0 0
5 0.0119 0 0.0030 0 O.OOIL 0 0.0004
6 0 0 0 0 0 0 0
7 0.0022 0 0.0007 0 0.0003 0 0.0001

At the center of the plate the deflection is


7 7 . .

= LL "' '"2
IV
i l f I
IVj; S1n -,s1n -
_.
(4.75)

\Ve per(onn the sumn1ation up t.o i = j = 7. For i. j = 2. 4. 6 the sine is zero.


for i. j = 1. 5 the sine is unity, and for i. j = 3. 7 the sine is mintL~ one. Thus. the
resulting deflection at the center of 1he plate is

,., = 0.01693 m = 16.93 mm. (4.76)


4.2 DEFLECTION OF RECTANGULAR PLATES 107

The bending mome nts a t the center of the pla te are (Eq. 4.67)

~7~
t- ~
, wI/ (!.:!.)
. l sin !..!.
L, .l sin E!.
1 I
LwJ - J),J 1~ ~
/lr-fx _ Du
[ Du
D 11 il 11

i.,.; t,_,
i I f I
w(b.)2 sin "'-sin
l/ L, l
i
1

= 1154.07 1 N m (4.77)
63.21 m

The twist moment al the corner of the plate (.t = y = 0) is (Eq. 4.67)

(4.78)

The n'kaxilnum de.Hection and the n1axin1urn mon1ents thus calculated are in
duded in Thble4.2 (second row). The maximum be nd ing momeots and the maxi-
n1um deftec.tions calculated by the numerical method and the orthotropicapproxi
n1ation are in close agreen1ent, but the maxin1un1 t\\ist mon1ents differ significantly.

4.2.4 Plates with Built-In Edges - Orthotropic and Symmetrical Layup


consider a rectangular plate \\~th length L.x and \vidth Ly built-in along its four
\ \1e

edges (Fig. 4.1 I). The layup is o rthotropic and symme tr ical. The plate is subjected
to a unito:rmly distributed load p.
The po te ntial energy of the pla te is obtained from Eqs. (4.55) a nd (4.56) by
setting Di.6 and Di.6 equal to zero:

11'p=U+ l'l=2J frL.L,[


. (a2w ' (a2w
D11 a.;l + D22 ayi
'
0
)
0
)

0 "
2a -
+ D,. ( -
2
w)' aax' aay.-
axay + 2D.,---
2
w
0 1
w"
2
- pw0 ] dydx. (4.79)

Figure 4.1 1: R.cctans.ular plate \\ith huillin


edges.
108 T.HIN PIATES

The moments at a point x , y are (Eqs. 4.2 and 3.27)


aiwo azwo
M = - D11--- D1.,--
x axz - ar
()2u,,o aZu;<>
M = - D.,-- - D,,-- (4.80)
r 1 ax - ay?
2a 2 u; 0
J\11v
.
= - ~--,
axay
\.Vbere w is the deflection and Di I .D12. Di.2. D66 are the eleme nts of th.e stiffness
0

ma trix in the x- y coordinate systen1.


The displacements and n1oments can be calcula te d \Vhen the plate s be nding
stiffnesses satisfy the follo\ving Huber orthotropy relationship 12 (see Eq. 4.l53
with K = 1):
I --
D66 = 2(J D, , D,, - D.,). (4.81)
Although this relationship may not hold exactly, we adopt it for calculating the
displace1nents and the n101nents.. Possible e rrors introduced by this re la tionship
are discussed on page 11 I.
\Ve introduce the variable
x ' = -,
x (4.82)
Ct
\'.Vhere a is a constant defined as

(4.83)

Equations (4.79)-(4.83) yield the pote.n tial energy and the moments (per un it
le ngth) as follows:
4
2
1
rr = - j'"!L,
0
.
[),_, [(a'
0
-w-)' + (a-'.-.,)'
axe
1
, + -2 ( fly-
1- IA)
' (w1w")'
---
D11 D,,
---
axay

JY.,_,_
+? __
(4.84)
- / D11 D,,

aw
(Du a'ay'w")]
2 0

M, =a' [ - Du - --
&.rrl

M,. = [
fDf: ( - a'w")
-y~
ax'
- a'ayw"] [),,--
2
[),,--
-- 2
(4.85)

Df, i&lw].
Du D,..1 ax1 ay

12 S. P. T1modlcnkoand S. \Voinowi:ky-Kricgcr, nu~(Jryt,f Plo1est11u/ Shells. 2nd edition. McGraw-Hill.


New York, 1959. p. 366.
4.2 DEFLECTION OF RECTANGULAR PLATES 109

We no'v consider a rectangular isotropic plate with din1ensions L: and L1 .


The potential e ne rgy and the n1omen ts (pe r unit le ngth) of an isotropic plate
are obtained by substituting into Eq~ (4.79) a nd (4.80) Du = Dn = D'm, Diz =
v /j, a nd Dr;,= /Y.,(1 - v;"')/2 (see Eq. 3 .41). In the .r'. y coordinate syste m
the res ult~ are

(4.86)

(4.87)

The supe rscript "iso.. refers to the isotropic plate.


From Eqs. (4.84) and (4.86) we see that the expressions for the potential e nergy
for the Hube r orthotropic plate (\vith d ime nsions L., Ly and stiffnesses D1 1. Du.
Di.2 ) and for an isotropic pla te (\vith d imensions ~and L_,) are identical \vhen
r:, = L..fa (4.88)

(4.89)

The deflections of the plate a re obtained fr on1 t he potential e ne rgy. Hence,


when Eqs. (4.88) and (4.89) a re satisfied, the deHection of the ortho tropic plate
at point :c. y is the san1e as the deflection of the corresponding isotropic plate at
point x', y. that is.

w (x. y) = w;,. (.r'. y) . (4.90)

Equations (4.85) a nd (4.87) show that the mome nts (per unit length) o( a Huber
orlhotropi cplate a re re lated to the moments (per unit length) of the corresponding
isotropic Jl'late by

Mx (x, y) = a 2 ,~ (.r', y)

My (.r, y) = M~"' (x', y) (4.91)

A(,y (x , y) = .,,1~; (x' . y).

By the preceding method. the de Hections and moments (per unit length) of
an orthotropic pla te can be obtained fro n1 the deflections a nd n1oments
110 THIN PLATES

Table 4.3. Maximum deflections and maximum moments of rectangular plates with built-in edges
subjected to a uniformly distributed load p (v;" = 0.3). The loeations p, , Pz. and p., are shown in
Flgwe4.12.

L,
w*-..L "t~c..
- 314 o
at P1
Ml..
. = -nea:
at P2
pL j Ml' - -
1 -
at P,
pLf
1:l
o.i ,, -
Ml -
at P1
..
JJLf 4 ... , t1
M1 = t tCs
at P1
L' c, c. ~
1.0
'
0 .484
"'
0.616
"'
0.6t6 0.554 0.554
I.I 0.576 0.697 0.646 0.634 0.554
L2 0.661 0.767 0.665 0.718 0 .547
1.4 0 .795 0.871 0.682 0.838 0.509
1.6 0 .883 0.936 0.685 0.914 0 .463
1.S 0 .941 0.974 0.685 0.962 0 .418
2.0 0 .975 0.995 0.685 0.989 0 .379
1.000 1.000 0.685 I.ODO 0 .300
""
(per unit le ngth} of the corresponding isotropic plate. The calcu lation steps are as
folJO\VS.

Step 1. \ Ve calculate t he equivalent length L~. stiffness [j.~u, and Poisson's ratio
v"" o( the isotropic plate (width Ly):

. ~ .,
v
vso = 75;;li;;_" (4.92)

\vhere

(4.93)

Step 2. \\'e detern1ine the deHections wij{) and moments (per unit leng th) A~~ .
.1w;I), and M7; of the "equivale nt" isotro pic plate. ?vlaximun1 deflections and max
imum bending moments (per unit length) of isotropic plates (width a, length
I>, be nd ing stiffness D) subjected to a unifo rn1Jy distributed load are given by
T in1oshenko a nd \\1oino\vsky Krieger! 3 We 111odified t hese resuJts and adopted
them for Huber orthotropic plates. The resuJting n1aximum de Hections and max
in1um bending mon1ents are given in TabJe 4.3.

Step 3. The deflection and mon1ents (pe r unit length) M.x, My. a nd M.ty o( the
w(I
orthotropic plate are calculated by (Eqs. 4.90 a nd 4.91)

M.xy =a1';,. (4.94)

IJ Jbid., p. 202.
4.2 OEflECTION OF RECTANGULAR PLATES 11 1

:Pi
~ ------ -+- ' ---- - -
f"1gurc 4.l Z: Locations \\'here the moments and the. deflcc ' P3 :P1
tion arc calculated. I

L,
x

The preceding a pproximate procedure yields the de Hect ions and be nding mo-
n1ents o f o rtho tro pic plates \\~th built-in edges \\'ilhin about 10 percent. 14

4.4 Exan1ple. A 0.2-111./ong and 0.211ltvide rectangular plare is 111ade of graphite


epoxy unidirectional plies. The 111aterial properties are given i11 Table 3.6 (page 81 ).
The layup is [02/452/902/- 452],. The plare, built-iu along rlre four edges (Fig. 4.13).
is subjected 10 a u11ifor111ly di.fltibuced trau.sve7se load p = 50 000 N/111 2. Calculate
tire 11raxi11uun deflection and the n1axinu11n be11di11g n10111ents.

Solution. \Ve ueat this plate as Hube r o rt hotropic. The bending stiffnesses are
D 11 = 34.61 N m. lh..i= 12.34 N - m, D 12 =4.58N m, D, 6 =5.14 N m (Table3.7,
page 84). From Eqs. (4.83) and (4.89) we have

JD,; , L.v
a = fi5; = 1.29 N m L, = ";" =0.155 m (4.95)

IY"' = D,2 = 12.34 N m Df,2 . ,-0?22


-- (4.96)
D11 '-'22

From Table 4.3 (at Ly/ L:, = I .29) we obtain the constantsc 1 = 0.724, c2 = 0.816,
C) = 0.673, c, = 0.774, and <s = 0.529. With the values of these constanl< the
n1aximu1n d eflection and the 111axiluum bending mome nts o f the correspo nding
isotropic plate are (Table 4.3)
. 1 pl:'
w" 0 Ial r, = 384
- ~cor.u 1 = 0.0044 n1 (4.97)

.~"'I ,, ,., =
,,,,, --1_.,; C1 = - 81.21 -
pt: N -
m
(4.98)
. n1

M'..~i = pl~
- -C)
Nm
= - 66.96 - - (4.99)
.> a l ft, 12 Ol

. pla. N n1
M'}"I,, I\ = 24x C, = 38.52 --;,-- (4.100)

11'~1 a1 " -- pt:;


! >'
- ?633 N- m
24 Cs -- .. m . (4.101)

u J. \~crc.s and L. P. Kollir, Approximate Analysis of ti.'1idplanc Symmcutc Rc.c.tanguJar C.omposite


Plalcs.JtJ11n1al ofCo1nprui1e ,l1aurinls, Vol 36, 67.' -684, 2002.
112 THIN PLATES

Figure 4.13: l he plate 1n l!x.amplc-4.4.

The points Pt- i'J are shown in F.igure 4.13. The n1a..xin1um de Hection and the
m aximum bending moments of the con1posite plate are (Eq. 4.94)

w
0
J.al l -'i = w
110
L,, I\ = 0.0044 111 (4.102)

~lat~ = <:i2 "'"I.al ~ = - 135.98 ~


A'lx N .m (4.103)
. N m
M,t" ~ = !\.~"l)L.. ,,. = - 66.96 ~ (4.104)

illl1I 2 lfto j N n1 (4.105)


Mr :11 I\ = ex !Wx iit I\ = 64.50--;-
ii.ul
M,. :u ri = ;~I
M,. a1ri = 2633 N n1
. ~- (4.106)

4.3 Buckling of Rectangular Plates.

4.3.1 Simply Supported Plates - Symmetrical layup


We consider a rectangular plate \V:ith din1ensio ns L., and Ly simply s upported
along its fou r edges (Fig. 4.14). The layup of the pla te is symme trical. I BJ = [OJ.
TI1e pla te is subjecte d to unifonn ly distribute d in plane loads N,,u., N,<1. and
Nx)O a round the edges. These loads are increased propo rt ionally, that is,. the loads
are ),Nxo. AJV)(h AJ\~n ~i. \vhe re). is the load p:iran1e ter. For a buckled plate the load
p aran1e ter is. denot~d by )er Follo\ving \Vhitney, 1 ~ we obtain An by the e ne rgy
me thod.
The strain e ne rgy is (Eq. 4.55)
4
I
U = ?'
- .
r'1 (a'w")' + Dii (a'w")'
-,- +
[ D11 (2a'w")'
--,
ax ay-
D,. - -
axay
0 0

a w" a'w" a'w" W'w"1


a'w" 2a'w") ]
+ 1( D"a;z ay' + 0 1 axi axay + Di ay' axay dydx.

(4.107)

l .S J. ~t. Whitney. S1r11c111rol 11110/)'.~U fl/ .Lor11u1tue1/ AriM>tropie. PlttJCJ. Tcchnomic. Lllnc-aster.
Penni.)'l vani:t. 1987. p. LSI.
4.3 BUCKLING OF RECTANGULAR PLATES 113

Figure 4. 14: Rectangular simply supported (ss) plate sub)tcd 10 comprcss:r.c and shear cdg.c
loads.

A.lr a plate subjec1ed only to inplane loads. which do nol vary \vilh x and Y~
the potential or the external forces is 1()

(4.108)

where N,. N,.. N., arc the in.plane forces (per unit length) inside 1hc plate. These
internal in -plane (orces are related to the edge loads >.N_10 ).N>~ AN~,a by

(4. 109)

We use 1hc Ritt me1hod 10 obtain the deOectioo. For 1hc simply supported
plate under consideration the geometrical boundary conditions require that the
deflectio"' be zero along the edges (Eq. 457):

W
0
=0 at
r .r= ,_
O=:;.rsl,
O~x;::L;1
and O=:;y=:; L,.

and y=O
and y= L,.
s
and 0::: )' l,

TI1e Collov,.ing dcllection satisfies these geon1etrical boundary conditions:


(4.110)

o ~~ itrx . jiry (4.11 1>


W : L.,, L.,, w; Siil - SID - .
,_, 1... 1 1 Li L1

y,here I and J :ire the number of terms. chosen arbitrarily. in the sumn1ations~ w,1
are consuanu and are calcuJated from the principle of stalion:.ry potential energy
(Eq. 2106) expressed "-'

a,.,= a(U+n) =O. (4.112)


aw,, aw,,
\Ve no''' substitute w0 (from Eq. 4.111) into the expressions of U and n
(Eqs. 4.107 and 4.108) and perform the differe ntiation indica1cd by Eq. (4. 112)-

16 S. P. Tinlmhcnko :ind J. Gere, 111rory nf Elturic Stab11i1y. 2nd cdilion. Mc<Jrnw-1hll, Now York.
1961. ,.., J.<I!).
114 THIN PLATES

.t\ lgebraic n1anipulations result in the (olJo,ving syste1u o( sin1ultaneous algebraic


equations:

I I
L m = 1. 2. 3, .. . . I
LL (Gmru 1 - Ab1n11ij} Wjj = 0 {
J. 11 =1,2, 3, . . . , 1.
(4.1 13)
i I j I

For convenience. \Ve introduce the contracted no tation

i.= 1, 2, 3, .. . . I
k = (i - 1)1 +j
I J = l , 2. 3, . . . , 1
(4.1 14)

I = (111 - 1)1 + II l m = 1, 2. 3. . . . . I
11=1,2,3, . . . 1.
(4.115)

Equation (4.113) may now be w.r itten as

Ix/ lxJ
L G.,w, = ), L bktwk, I = 1, 2, 3.. . . , I x 1. (4.116)
k I k-l

where C., (= C,.) is given in T.'ble 4.1 (page 102) and b"' (= b1) is

(4.1 17)

T he Kronecker delta t51t and the paran1eter ri; are also given in Table 4.1. In
expanded fonn Eq. (4.11 6) is

G11 Ga(fxJ) ] [ b11

( [
C(fxJ )I Gu x J )(l x J ) - ). bv~1)1
\Vhen the plate is not buckled. the deflection is zero, whereas for a buckled
!Plate it is no nzero. The values of ). for Ute buckled plate (denoted by ),") are
tthe eigenvalues of Eq. (4.118), and. these can be calculated by comme rcial soft
ware. There are J x I eigemalues, of which the lowest gives the lowest buckling
Coad.
\Ve no\\' consider an o rthotropic plate subjected to N.s.0 and 1~0 e dge loads.
T he forces inside the plate are

1\~ = - A1V;fo N.,y = 0. (4.119)


4.3 BUCKLI NG OF RECTANGULAR PLATES , 15

L,= 100 mm
l<-- " - - - ->I
i ii iiiii

I
SS
F'i~urc 4.1~: 1'he platc-ln 11.xample 4_.;. I/
SS SS L,.= 200 mm
SS

1' t t t t t t
/ \l I (J

For o rt hotropic laminates we have (Table 3.4. page 76)

D1 = >,_. = 0. (4.120)
The eigenvalues of Eq. (4.118) can now be calcula ted directly. T he result is
2 4
!. .. = 11' [Dn(f.:) +2(D12+2D.. )({;)'(t)' + D21(t )']_
( a),,
Nxo T. + N,Q (1..)'
{; )'
L,

(4.121)

(;",),; must be calculate d tor different sets of i and ; . (i.; =I. 2 ... . ). The
lo\\est resulti ng value o( (Ac.);; is the value of interest.
Libove 17 s ho\\ed that, (or simply s upported orthotropicplates, the Jo,vest buck
ling load corresponds to a mode that has a half \\'ave in at least one direction. (In
this directtion e ither i o r j is equal to unity.) \Vhe n the plate is subjecte d to uniaxia.I
co 1npressjon, or to co n1pression in one d irection and tension in the o the r direction,
buckling occurs with a half wave pe rpendicu]a r to the compressive load. ig

4.5 Exa111ple. A 0.1-111-long turd 0.21111vide rectangular plate is n1ade of graphite


epox)t The 11111teri11/ properrie.r are given i11 Table 3.6 (page 81). The layup is
[45\/012/45\ J. The Odegree plies are parallel to t/Je short edge of the plate.
The plate is simply supported along all/ 011r edge.< (Fig. 4.15). The plate is s11bjecred
ro u111for1n co111pressive loads along the long edges. Ca/en/ate J/Je buckling load.

Solution. From E q. (4.121), with N,o = Oand with Ulestiffnesses D11 = 45.30N m.
D,, = 25.26 N m, /Jn= 19.52 N m, D,, = 20.62 N m (Table 3.7, page 84) we
have

N,0 (!,");; =tr' Dn )' + 2(Dn + 2l\>) (L')' + D22 (l.')'(L)']


[ (L ;'
= (11 .t8i1 +2.4Sj1 + 004151:) 10-1. (4.122)

n C. Libo \'c, Bmk:lc Paucm for Bia:ici.nlly Con1prcsscd Simpl)' Supported Orihotropic Rectangular
Plates. l tNuna/ of CQ1up1,sire ,itrueriols. Vol. 17, 45-48. 1983.
I$ T. K. Tung and J. Surdcnas. Buckling or Rcctangula:r O nho tropk Pla 1cs unde r Biaxial Loading.
Journal c>f C-tu"pQ.ti1e Af,11eriaf,, Vol. 2 1, 124- 128, 198.7.
116 THIN PLATES

E
~

~-------=-=i=,-.,..,J
lO

>:... 20
-g __ '\____________________ ------
0
~J O
.s long-plate.approxiniatinn
~ o'--~~~~~~~~~~~---~~~~~--'
"' 0.2 0.4 0.6 0.694 0.8
Length, l\. (m)
Figure 4.J6: The IO\\esl buckling load or the plalc in Example-4_<; as a (unciion of the plate
Ccngth.

T he values of N,o (4.)1; x 10- 3 a re

i\ j 2 3
13.67 21.63 36.57 (4.123)
2 47.17 54.66 67.58
3 103.04 110.46 122.99.

The lowest value is N,o ().");; = 13.67 kN/m, which corresponds to i = j = J.


T hus, the lowest buckling load is

(4.124)

\Ve oov" assess the length-to-,vidth ratios under which the long.plate approx
iin1ation is reasonable. To this end, "\Ve calculate the lowest buckling loads of the
plate, keeping the width Lx the san1 e whiJechanging the length Ly. In A gure 4.16
't.Ve plot the buckling loads thus calculated versus Ly. In this figure \Ve also in
elude the lowest buckling load given by tl1e Jong-pla te a pproximation (Eq. 4.1 70).
The results in this figure show t ha t, in accordance with Eq. (4.19), the Jong
p late formula is reasonable when l, is greate r than 3L,:J D11/ D12 = 0.694 m. At
L_, = 0.694 m the long-pla te fo rmula underestimates the buckling load by about
18 pe rcent.

4.6 Exantple. A 0.2111lo1Jg find 0.2-1tllt1ide rect(IJJgular plate is 111ade of graphite


epoxy unidirectional plies. The n1aterial propenies are give11 in Tttble 3.6 (page 81).
T he /(lyup is 145,/-452/012/-452/452]. The plate, simply s11pported aloug four
e dges, is subjected to unifonn co1npre.r.fi.,e loads along all fo ur edges (Fig. 4.17).
De1er111b1e tire b"ckli11g load.

Solution. The layup of the plate is symn1etrical but is not orthotropic. The bend
i ngstiffnesses are D11 = 45.30 N m , D,, = 25.26 N m, D12 = l 9.52 N m, D.,, =
=
20.62 N m, D1 = 4.45 N m, Di 4.45 N m (fable 3.7, page 84). The buckling
Uoad is calculated from Eqs. (4.117) a nd (4.1 18). With the preceding stiffn esses and
4.3 BUCKLING OF RECTANGULAR PLATES 117

Figure 4.17: 'Ole plate in Example 4.6.


_, ~1__i-=i_i ,_-'Y'>
--,,. .-
:
JVii> ~
ss
SS

SS
ss :
<E-
I
L.,= 200 n1n11

x
t t t

with I = J = 7, these calculations i;eld the following results for (N");1 x 10- ':

i \ j 2 3
23.47 45.05 76.62 (4.125)
2 61.25 94.61 132.38
3 119.45 164.43 214.29.

The Lowest buckling Load is

(4.126)

The layup follows the 10-perce ot rule (page 89), a nd we may treat t he plate as
orthotropk With N..o = N,,, = N,,, fro m Eq. (4.1! 1) we have

N ). . . = "'[ D11(i;J' + 2(D,, + 20...)(t,)' (tJ' + Dn(t )' ]


o( "),, (L)' + (l.)'
'~ ,..,
(279.4i 4 + 749.5i 1 j 1 + 155 .8i4 )1oJ
= (4.127)
25i 2 + 25 j'

The va lues of 1\ b (J."),1 x 10- 3 are

i \ j 1 2 3
I 23.70 46.16 78.58 (4.128)
2 61.00 94.78 135.61
3 118.14 160.34 2 13.26.

The lowest value is N,, (J.0 );; = 23.70 kNfm, which corre spo nds to i = j = L
Thus, the lowest buckling load is

Na = (J.0 )11 N,, = 23.70 kN/m. (4.129)

This buckling load, based on the orthotro py approxin1ation, is 'Ni thin 1 percent
of the buc kling load given by Eq. (4.126).
118 THIN PLATES

Pigurc 4.18: Re.c:tangular plates ubjccted to


uniform biaxial compressive ecCge loads.

4.3.2 Plates with Built-In and Simply Supported Edges - OrthOtroplc


and Symmetrical layup
We consider recta ngula r pla tes with le ngth L, a nd width Ly (Fig. 4.18). T he layup
o( the plate is o rtho uopic and syn1metrical. Each edge of the plate is e ith e r sim ply
suppo rted o r bui1tin. Uniformly distributed in-plane loads 1V."o a nd N,'(I act along
the edges.
The edge loads a re increased pro portionally to AN...0 , A1V)"O \\he re A is the load
J>arameter. For the buckled plate t he load pa rameter is denoted by J.,,. We a pply
the Ritz n1ethod to find A0 . 19
The e xpression of the potential e nergy is o bta ined by setting 1V.ty and Di 6 and
0,6 equal to zero (orthotropy) in Eq~ (4.!07) a nd (4.!08)
L,I.,[
fr a2w" 2 2 '

tt = U +Q
1
= -2 . C-) + C-)
D11 ( ax'
-) + J),,
8 2w"
-- ay1
2d 1 w0
D.. ax ay
-

0 0

8 w" .-,-+ N, caw")' caw)']dydx , (4.130)


2 2
+ 2D12 -;;-;-- 8 -.- u1 +1\ I,, -
ox ay- ax . ay
'" here Nx and Ny are the in~plan e lo ads (per unit le ngth} inside the plate, which
are re lated to the edge (orces by

(4.131)

a nd 1JJ0 is the deflection of the n1idpla ne, which is assun1ed to be o f the {orn1

,., = AX;(x)Y;(y), (4.132)

where A is a n unknown a mplitude. For the X,(x)and Y;(y) displacement functions


we a dopt t he shape of a freely vibra ting beam. (The X,(x) function is lllus trated
in Fig. 4.19.) For diffe re nt end supports X,(x) a nd Y;(yl are give n in Table 4.4
( page 119).
By virtue of the principle of s ta tcona ry po te ntia l ene rgy (Eq. 2.206), we have

a" =O. (4.133)


aA
1
~ J. Veres and L. P. KoUilr, Budding of Ortbotropic Plntcs Vlith Diffe rent Edge Supporis..Jounw/ of
Coutpo:~i1e i\la1eru1l1, Vol. 35, 625-635, 2001.
4.3 BUCKLI NG OF RECTANGULAR PLATES 119

;;;; ~ ~ ~ ~
..'<, .----------..., ~

x, .--.. ...____,, ------ -----..


_,..,,
...._...-
=C>.
x, "'7 = = =
c-, -C"'>

1-igurc 4.19:: freely vibratjng beam. The dtsplaccmcn.ls correspond to )(t(x). i = 1. 1, 3.


'CJ

Equations (4.1 30) and (4.133), together with! Eq. (4.132) and the e xpressions in
Table 4.4, yield

011 4 [),..z 4
L a 1 + L' a, + - (D12+ 2D,;,,)
, . _I
vv"' _ I"' ( N,o N,. )] -- 0.
L2 a 4 + ''.as
[ ' y xy ' }
(4.134)
The param eters <>i, a 2, u3 , u4, a 5 a re de fine d in Table 4.5 (page 120). The in
tegrations ind icated in Table 4.5 simplify when .X;(.r) a nd Y;(y) are calcula ted by
the approxin1ate expressions of 1 given in Table 4.4. The resulting approxin1ate
expressions for 0t1 thro ugh Cts are given in Tables 4.6 and 4.7. and a2 is
(4.135)
By rearranging Eq. (4.134), we o btain the following expression for)." :

Du~+ O,_i ~ + 2 ( D12 + 2Dfi6) fill


(4.136)
(i."),, = (" "' + 1"v.\~II!'
fYs>II "' )

This equa.tion applies \vhen each edge is either sin1ply supported or builtin. When
all fo ur edges are simply s upported, Eq. (4.136) simplifies to Eq. (4.121).

Table 4.4. The dl511Jacement function X1 ol a freely vibrating beam. The


parameter 1 ls to be determined either lrom the exact or lrom the approximate
r
expression. (Also: { = f;.)For 1 the same formulas apply with{, x, L,, i
replaced lby r1. y, L 1, /, respectively.

" [,, "' X. = si n ( lrr~}

A';= y; c&( ;O- y, eooh( ;O +s in (, ~)- s inh(/'~)


Lan, - tanh , = 0 (Exact)
[,, "' , ~ (I+ 0.25)H ( Appr():<i1na1e}
~111 ,., --h /f,
Y, = <X'!l'h(~o-<!Jt'''

X, = y1cos(;~} - y, c~h( 1~} +s in( , ~)- sin h( ,~)


cos /'-i cash, = J (Ex-act)
[, , 1 i:::: (i + O.S)rr ( Appro ximate)
- C>.l:Olf, - C(u;h,.,,
Yi - $1n 1t,-f.~inh p,
120 THIN PLATES

Table 4.5. The coefficients o 1 through "''

-
a3 = ~
' 1'
r\ .
<"''
.,...., )'
..
dlf

c,.

Table 4.6. Approximate expressions lor the coefficients a,


and"''
(I represents the numbe< of half waves in tlle x dlrecdon)

""
iir 1.2.3....

(i + 0.25)ll a1(1 - I) I. 2. 3.

4.730 O'J (1 - 2)
(i +O.S)rr a1 (1 - 2) 2. 3. 4....

Table 4.7. Approximate expressions for lhe coefficients'-" and,,.


(/represents the number of half waves In tlle y direction)

I
" 0.5

&Jr jJt il:rl 1.2.J....

CJ~ (/ +0.25)JT O"J(J -1) 1.2.J....

wr.
4.730
(j +0.5)rr
a ; (UJ-2)
Cf3 (ctJ - 2) 2.3.4....
4.3 BUCKLING OF RECTANGULAR PLATES 121

L.,= 200 mm

--- I i i i i y
<--
Figure 4.20: 1'he plate in E:tamplc 4.7.

N,,, _,
x
- t t t
<- L,= 200 mm

TI1e c ritical Load parameter must be calculated for d iffe rent sets of i and j,.
(i. j = 1, 2 .. . . ) . The lo\\est resulting value of A.0 is the value of interest. The
lowest buckling load (as in the case of simply suppo rte d pla tes) corresponds le>
a mode that has a half wave in a t least one direction. (In this direction eithe r i
or j is equal to unity.) \\'hen the pla te is subjecte d to uniaxial con1pression>or to
oon1pression in one direction and tension in the other direction, buckling occ.urs
\Vilh a half \Vave perpe ndicular to t he compressive load.

4.7 Exa111ple. A 0.2tn-long and 0.2111h4de rectangular plttU! is 1nade of graphite


epoxy u11idirectional plies. The nuuerial properties are give1J in Tttble 3.6 (page 81).
The layup is {452/- 452/012/- 452/ 452]. The plare, built-in along tire four edges, is
subjected l'o u11ifor111 coutpressive loads along it/I four edges (Fig. 4.20). Derer111ine
rite buckling load.

Solution. The layup follows the JO-percent rule (page 89), and we treat the plate
as orthotropic. With t he stiffnesses D11 = 45 .30 N m, D12 = 25.26 N m, D, 2 =
19.52 N m , D.. = 20.62 N m (Table 3.7. page 84,) and with N_,o = Nyo = No. from
Eq. (4.136) we ha\'e

/Ji 1f.-' + D.u if' + 2 ( D12 + 2 D66) ffi'i-


'' ( An)
''U i/ = ., '
!!! + !!!. .., (4.137)
~ Lf
where a 1 and "' depend on i (i = I . 2.... ) and a 1 a nd a5 de pend on j (j =
1. 2.. . . ) as give n in Tables 4.6 and 4.7, third TO\\'. Fron1 these tables the values
are
i =l i=2 i= 3

" 4.73
l 2.9l
7.85
45.98
11.00
98.91
"'
j =I i= 2 i=3
4.73 7.85 11.00
"'<>s 12.91 45.98 98.91 .
122 THIN PIATES

- jY
~----"--~
i,

-:=-
-
-> S.'i or built-in Figure 4.2 1: Uniaxially loaded rectan-
:V,,,11=:: SS SS J\irl> gular plate.

'I ss or built-in
~-----~ -->
x

T he values o! t\l, (),"),; x 10- 3 a re (Eq. 4.137)

i\ j 2 3
I 53.80 81.05 122.31
2 I09.16 142.82 188.79
3 185.55 226.18 280.57.
The lowest value is N,, (A,,);; = 53.80 kNlm, which con-esponds to i = j =I.
T hus, the )o\vesl buckJing load is

N" = (Aa) 11 No= 53.80 kN/ m. (4.138)


U11ifor11r co111pressive load i11 flu x direction. We consider a plate 'vith the
-edges parallel to t he yaxis simply supported a nd the edges parallel to the x-axis
-either simply suppo rted o r buihin {Fig. 4.21). The pla te is subjected to a uniaxial
con1pressive load /\1,.it in the x direction. By introducing the notation N x ,a = A.cT. N.o:u
.and by setting N,o = 0, Eq. (4.136) may be rearranged to yield
a~ 1 a~ L! &5
N.w = Du- 2 +Du-, , + 2(D12 + z~.) z ' (4.139)
a 4 L,, a4 1 . Ly
w here '" and '" depend on i (i = I , 2, ... ) and "'' a nd as de pe nd on j (i =
1, 2, . .. ) (Tables 4.6 and 4.7). The values of i and j (and the corresponding values
<>f a1 , U"4, 0>3 , and as) that yie ld the lowest value of N .x,a are de tem1ined as (olJo,vs.
To determine j, \Ve obsene that both a 3 and <ts increase monotonically with j
(Table 4.7). Hence. the righthand s ide of Eq. (4.139) also increases with j . Thus,
the lowest buckling load corresponds to j = l. At t his load the buckled shape
js a hal( \Vave in the d irection peirpeodicular to the load. This is illustrated in
:Figure 4.22 for a simply supported plate.

Figure 4.22: Uuckled shape- of a uni


axially loaded reclangular plate with
simply supportc.d edges .

.x
4.3 BUCKLINGOF RECTANGULAR PLATES 123

Table 4.8. Buckling loads of unidirectionally loaded ptates (ot1hotroplc and symmelrlcaJ
layup) with simply supported, bolll-ln, and rotationally reSlralned edges(/, = L, / /,
=-
I 1, 2, . ( = + .(=1,
f i--10(
.g& ): L,. L, are Ille length and the width, ntSl)eCtively,
and k 1$ lhe spring constant
5'lpports lklcl<ling load 1140

~ (Dut + l),_,~+2(Du+2l\.l)
~ (Dut +2.441Dntt+ 2.3.l(Di: + 2l\.))

~ (''* + 5.1390,, Ji-+ 262(0.2+ 20.+l)

~l ( D117f
,;
+ ( 1 +4.13900.,~. +(2+0.62~ 2 )(D12 + 20,.))

To dercrnline cr 1 nnd tt4 \Ve re call that th e edges parallel to the y-axis are
simply s upported. The corresponding values o f a 1 and o, are given in the first row
ol Table 4 .6 and are

a, = l:r (4.140)

With these values of a 1 and a. Eq. (4.139) becomes

(4.141)

where I, is the lcng1h of the hall buckling w.-e


/.,,
I,= T' (4.142)

111e va lues of a;i a nd a 5 to be used are those give n in rablc 4.7 ror j = I. The
resulting b uckling loads a re listed in the first three ro'vs o f Table 4.8.
111e \'l.:lluc o f JV.,,cr n1ust be calculated (or diffe re nl vnlues or I (i = I , 2, ... ) ,
and. gener nlly t he lowest resulting \alue is of interest.

:!Ll__. JI E
L,

long pl11c appr-oiunution

L,
,.igurc 4.2~ : Buckling lo11ds o f unidirectionally k>adcd rccla ng:ulnr pla1c1w11h 111mpl)' s-upporled
or huihtn C'dgc:11. The plntc's lcngth is L.v
12-4 THIN PLATES

jY
N~ 1 L, lsN~
free

SS
'---"'-----' - ->
L, x

I~ ssl I~
(al (b) <cJ
.Figure 4.24: Supports along edges parallel (lop) and perpendicular (bottom) to the load
direction.

The buckling loads of plates with simply supported o r builtin edges (Fig. 4.2 1)
.are illustrated in Figure 4.23. For each value of i the lo\vest buckling load is the
same as the lowest buckling load of the corresponding Jong plate that will be given.
s ubsequently, in the first three rows of Table 4.11 (page 136).

4.3.3 Plates with One Free Edge - Orthotropic and Symmetrical Layup
We consider a rectangular plate with length L., and width L,. The layup of the
:plate is o rthotropic and syn1metricail.

One Edge Parallel to thex Axis is Simply Su pported; the Other i.< Free
T he plate is simply supported along the y = 0 edge and is free along t he y = Ly
.edge (Fig. 4.24, top). The edges par allel to the y-axis may be either simply sup
:ported or builtin (Fig 4.24. bottom). The plate is subjected to uniform uniaxial
con1pression N.,0 in the x dire.ctioo (Fig. 4.24, top). \Ve 'vish to d etern1ine the
:lowest value of the load at which the plate buckles.

Edges parallel to the yaxis are simply su pported. We consider pla tes whose
.edges parallel to the ya~is are simp ly suppo rted (Fig. 4.24, a). The buckling loads
.are denoted by Nx,a An exact analysis resulting in a transcendental eq,uation for
N_,...c r is given by Whitney.20 Here. \Ve present a.n approximate analysis, which yields
.a closed-fonu expression for Nx.o lo our analysis 'Ne apply the Ritz n1-ethod and
.approximate the buckled shape (shown in Fig. 4.25) by

u.1 . ("x)
= Aysin -
L.T
(4.143)

'vbere 1\ is a constant and w 0 is the deflection, \Vhich satisfies the geometrical


boundary condition ( w = 0) along the simply supported edges.

20 J. tvl. \\'hilocy. SUU<'tural Aual)'.f/.$ ()j Lar11u1t11etl A1uS()tropic Plate.r. Tccbnomic , Lnncnstcr.
Pcons)'lvnnia. 1987. p. 108.
4.3 BUCKLI NG OF RECTANGULAR PIATES 125

y
free

Figure 4.2'>: Suckled shape of a rectangular plate "'1th o ne free- and three simply supporled
edges.

We exp ress lbe potential energy of the plate by setting Ny = 0 a nd N.x = - Nxo
in Eq. (4.130):

;r - -I
P - 2
1'"1"'[ (aax'
'u1")' + ~'-n., (a'-ay'w-")' + De,(2a'
D 11 - - - w")'
-
axay
0 0

a2w
+ 1( D12 [i';i'
0

ay')]dydx - 2l j"'j"[N,.,(;;:;-)
tJ2 w 0

U I)
au/ '
'] dydx. M

(4.1 44)

By introducing Eq. (4.143) into Eq. (4.1 44), we a rrive at t he following expres-
sion fo r th e potential e ne rgy:

ir
P 4
I /}
A' L Di i -1-+
= --~ ir
3 L!
414.<.L ir
4
- - JVo 2
y L; x 3 Li
2
L' ir- } .
2
(4.145)

Table 4.9. Buckling loll<ls of unidirecijonally loaded plates (orlhotJOplc and


symmetrltj!l layup) with one ~ee edge (Ix = L x / I, i = 1, 2,. . ., ( = ;" );
L , , L, are the length and the width, respectively, and k is the spring cO:,.stant
supports 8:uckling load N""

l.?...,;vp + ,. :~>11 + IZ~""


t4 lj, Ly
126 THIN PlATES

Fig:urc-426: Support alongthc.cdgC'sof a untax:ially


loadc.d rcclangular plate.

By virtue of the principleol s tationary pote ntial energy atr,/a A= 0 (Eq.2.206),


and we have
:r 2 Du t2D66
1
J\ x.n = U- + -V' (4.146)
y

One or Both Edges Parallel lo the J'Axis are Built-in

T he buckling loads of plates that nave one or both edges paralle l to the y-axis
built-in (Fig. 4.24, band c) can be derived in a way similar to that ol E q. (4:146).
T he deta ils are not given here; the resulting buckling loads are given in Table 4.9.
One edge parallel to the x axi.1 is b11illi11; the other i.1free. The plate is built
0

i n along the y = 0 edge, is free a long the y = L,. edge, and is simply s upported
along the x = 0 and x = L, e dges ( Fig 4.26). The plate is subjected to uniform
uniaxial 001upression NJ.ii in thex direction. \\'e 'vish to determine the lo\vest value
ol the load at which the plate buckles (Fig. 4.27). An e xact analysis, resulting in
transcendental equations fo r the buckling loads 1Vx ,n is given by Bank and Yin.21
1-\ n approximate expression for Nx.a is22
2
1' D11 1;D,, 12D"'
Nx." = ( 1 + 1.25/ T + f:T ' (4.147)
x y J'

\.vhere 1.i: = L.1 / i is the length of the half buckling '''ave in the x direction and
i = 1, 2, . . . is the numbe r of half \vnves in the x direction.
Equation (4.147) underestimates the buckling loads by less than 14 percent.
The value of Nx.cr nlus t be calculated (or diffe re nt values of; (i I , 2, . .. ) =
and, in general, the )o\vest resulting. value is of interest.
The buckling load is a function ol the length ol the plate L., (Fig. 4.23). For
each value of i the Jo,vest buckling load is the same as the lo\\est buckling load
o( the corresponding long plate given subsequently in the fifth row of Table 4.11
( page 136).

4.8 Exantple. A rectangnlar plare ivirlt /eiJglh Lx = 0.5 111 and width Ly = 0.05 111 is
r.uade ofgrttphite epoxy u11idirectional plies ~virh the fibers oriented along t/Je x-axis
ofthe plate (Fig. 4.28). The 11wterial properties are given i11 Table 3.6 (page 81 ). The
f ayup is (020]. 011e of rhe long edges ;s builti11: rlre other long edge is free. The short
edges are simply supported. The place is s11bjecrcd to 1111iform compres.rive loads i11
tire x direction. Ca/cu/are rhe bucklir~g load.

i:t L. C. Bank. and J. Yin. Budlin~ or 011hotropic Plates wi1h Free and Rot.ntionally Rcsuaincd
Un.loaded Edges. Tl1i1t\l'olled S1nrc111re.f. Vol. 24, 83-96, 1996.
?:Z L. P. Kolli r. Buckling or UnidirtionuJI)' l.ooded Co1nposi1e Plates wit h One Free and One: R etU1
tionally Restrained l Jnloaded Edge. Jour,,ol <1/ S1n1crural Eugi11et!ri111; , Vo l. 128, 1202-12 1l . 2002.
4.3 BUCKLING OF RECTANGULAR PIATES 127

1-igurc 4.27: Buckled shape or a unia.xiall>' loaded re.<tangular plate \\'tlh a built-in and a (rec
longitudinal edge.

Solution. The expression in Table 4.9. fo urth row (page 125), togethe r \\1th the
stiffn esses D 11 = 99.25 N m, D22 = 6.47 N 111, Di 2 = 1.94 N m, D66 = 3.03 N m
(Table 3.7, page 84) gives

tr 2 Di 1 /.~ [),_, 12D66


tV.T.tt = --,- + 1.25-4- + --,-
/; L), L,
-- 919.5p.
t + 1.294 x 106 I,'l + 14 560. (4.148)
x

where I, = L..fi. The buckling loacl< for i = 1, 2, 3. 4, 5. 6 a re

2 3 4 5 6
N,.0 (kN/m) 342.04 111.12 85.78 97.47 125.46 164.60
The lowest buckling load corresponds to i = 3 a nd is 85.78 kN/m.
The buckling loads of plates o f the same "'vidth but with different lengths are
given by solid lines in Figure 4.29. The buckling loads calcula te d by the equa
lions of Bank and Yin13 are also included in lltis figure. The a.pproxin1ate forn1ula
(Eq. 4.148) underestin1ates 1V.t.cr and, hence. is a conservative estimate.
The lo ng-plate expression gives the lowest buckling load of this plate as
90.64 kN/m (Example 4.11, page 138). This value is a lso shown in Figure 4.29.

4.3.4 Plates with Rotationally Restrained Edges - Orthotropic


and Symmetrical Layup
We consider a recta ngula r plate " 1th length L., a nd width L,. T he layup of the
plate is orthotropic and syn1n1etric.al. The plate issin1ply suppo rted alo ng the edges
parallel to the y-axi~ They= 0 edge is rotationally remained, they= L, edge
is e ither rotationally restra ined (Fig. 4.30, left) or free (Fig 4.30, right). Along a
rotational ly restrained edge the bending moment is proportional to the ro tation

23
L. C. Baok and J. Yin, Buckling or Orthotropic Plaitci; "'ith Free. and Rotnticmally Restrained
Unloaded Edges. Tl1 in -\\lalltd Srr11c1ur.-s, Vol. 24. 83-96. 1996.
128 T.HIN PLATES

y
L,.= SOOmm
,.__ _ _ _ _ _ _ _ _ _ _ _I, /\'

J V.rt ..<>

::: " fr<< "'':: IL,= 50mm


-->>-r.,.,.rr,-,.,,.,.,...,,.,.,.,.,..,,,.,..,777?'777'7r<- ~->
x
Figure 4.28: The plate. in Example 4.8.

of the e dge
.,..aw
M =k-, (4.149)
' ay
\.Vhere k is the rotational spring cons tant. The rotational s pring constant is further
discussed in Section 6.9.3.
The plate is s ubjected to uniform unia xial compression N...-0 in t he x d irection .
We \Vish to de te rmine the lo\\est value of t he load a t \Vbich the pla te buckles.

lloth Unloaded Edges are Rotationally Restrained


We conside r a plate that is rotationally restrained along they = 0 and t hey = Ly
e dges (Fig. 4.30, left). An exact analysis resulting in a tra nscendental equation for
the buckling loads 1\1.,.:r is given by Qiao et al.24 Here, \Ve present an approxin1ate
a na lysis, \vhich yie ld~ a closedforn1 expression for 1\~v .n
111e buckling load of a plate with rotationally restra ined edges must be be
tween the buckling load of a simply s uppo rted pla te (Tuble 4.8, fi rst row. p age 123)
a nd the buckling load o f a plate with built-in e dges (Tuble 4.8. third row). We com-
bine the expressions for plates '"ith s in1ply supported and built.in edges and write

N,." = "'
L' ( D11 fi+(1
L; +4. 139S)D,,L'
. t;
+(2+0.620(D12+2D) , , )
y x y

(4.150)
1
"vhere ~ a nd ~ are parameters that depend on the support conditions a long the,
unloaded edges. These parameters are zero for a plate \vith sin1ply supported
unloaded edges a nd a re unity for a pla te '"ith built-in unloa ded edges. The values
of~ and ~ may be approximated by~'

r =~' (4.151)

""h ere~ is the paramete r of restraint


D,,
~=~ (4.152)
kly

P. Qian. J. F. Davalos. and J. \Vang, Local Buckling or Co1npositc FRP Sh11pcs by Discrete Pla1c
2:-1
Analysis.Joun1tYI of Srn 1cl/JJ'ttl Erisineerin.t;, Vol. 127. 24>25.'i. 2001.
ZS 1.. P. Kolltlr, Discussion on the paper or Qiao. P. Oavnlos. J. F. and \Vang. J.: Local Buckling or
Composite FRP Sh11pcs b~ Di!:erctc Plate Analysis. ltJ11nwl tJ/ Suuc1111dl 11gi11eeri111:, Vol 128.,
1091- 1093, 2002.
4.3 BUCKLING OF RECTANGULAR PLATES 129

~
160 :..
"'
~
~
120
>-' ~
'ti
~

2
80
.s"
;;;
~

&i
40
.....
..... LL...._
' ____,I~ .~
l ,.

0 0.2 0.4 0.6


Plate leng:lh, L,r (m>
Figure 4.2\): Buckling loads o( the plate in Example 4.8. 1bc rc.suhs shown are- by Eq. (4.148)
(labeled as -present ..). by B-a nk an d Yins eq ua tions.. and by t he long-plate appro:tin1ation.

With these approximations of~ a nd f, Eq . (4.150) overestimates the buckling


load by less than 3.5 pe rcent whe n 0 < K ::; 1 and by less than 6.5 pe.rcent when
1 < K :5 3, v.he re K is a stiffness paran1eter d efined as

(4.153)

For most pr:ictic.al layups K is less than 3.


Equation (4.150) is included in Table 4.8 (page 123).
The value o f N,," must be calculated for diffe re nt values o f i (i = 1.1, . .. }.
and. gener ally, t he )o\vest resulting value is of interest.
The buckling load is a function of the lemgth of the plate L., (Fig. 4.23). For
each value of i the lowest buckling load is the same as t he Jo,vest buckling load o f
the corresponding long plate given subseque.ntly in the fo urt h ro\v of Table 4.1 1
(page 136).

One Unloaded Edge is Rotationally Restrained; the Other is Free


The pla te is ro tationa lly restra ined along they = Oedgeandis free a long t hey= LJ'
edge (Fig. 4 ..30, right). An e xact analysis resulting in transcendental e quations

j y L, _. t!I l ,

=Ec;:jE 3-) c:JTC'eE N


l.v
I
..
'{\i r()~ SS
~
~
S.'i <f-
~
~~
"
r..,;

x
/\'..-II_,.
~
SS
<-
'
S.'i <E-

~~
fl

x
Figure 4.:lO: U nia.:ually loaded rectangular plate ,.,.ith t-wo rota tionally re$lTained c-dges or v.ilh
one. ro 1a 1ionall)' restrained and one: free edse~
130 THIN PlATES

Figure 4.3 1: 'fbe plate in Example 4.9.

(or the buckling loads A'x.o is given by Bank a nd Yin. l l> Here, \Ve present an
approxin1ate a nalysis, which yields a closedform expression (or N..a
The buckling load or a pla te 'vith a rotationally restrained edge must be be
tween the buckling load of a plate with a simply supported edge (Table 4.9, first
r-ow, page 125) and t he buckling load of a plate with a built-in edge (Table 4.9.
fourth ro\v). \\1e co1ubine the expressions for plates with a simply supporte d and
\vith a builtin edge a nd v.rite
2
( .
1\1 :r D11
ix,n=-1-1-+f
,. 1.251;/Ji,
L4
12D"'
+-;r 4 154)
x 1 y
\vhere /J.. = L.T/ i is the le ngth of the half buckling wave in t he x d irection, i =
1 . 2, . . . is the nun1ber o f half v.aves: in the x direction, a nd {" is a paramete r that
d epends on the supports along the unloaded edge. This parameter is zero for a
plate 'vith a sin1ply suppo rted unloaded e dge and is e qual to unity for a pla te " 'ith
a. built in unloaded edge. The value of~,, n1ay be a pproxin1ated by2;

~-- = 1 +~.12t (4.1 55)


'"h ere~ is the para1neter of restTaint given by Eq. (4.152).
Equation (4.154) may underestimate the buckling load by up to 14 percent.
The value of 1V.r.n nlust be calculated for diffe re nt values of i (i I. 2 . . .. ),=
and, generally, the )o\vest resulting J\~t.cr is the value of interest.
The buckling load is a function of the length of the plate L., (Fig. 4.23). For
each value of i the Jo,vest buckling load is the same as t he lo\\est buckling load
o( the correspond ing long plate given subseque ntly in t he si.xth row of Table 4.11
(page 136).
=
4.9 Exa1nple. A recta11gular platt, l-'.' ir/r length L.1; = 0.5 111 a11d widrlr Ly 0.05 111 is
tnade ofgraphite epoxy u11idirecriont1l plies wirh the fibers oriented along the x -axis
ofthe plate (Fig. 4.31). The mllterial p roperties are given in Table 3.6 (pllge 81). The
layup is [02ol One of the long edges is rotationally restrained 1vlli/e rlle other long
edge is free. The short edges are sinrply supported. Tire rotational spri11g coJJsrant
of the edge is k = 129.4 1\ 1 The jJlate is subjecred to unifor111 co111pressi11e loads iu
tire x direction. Cttlculare the buckling load.

26 L. C. Bank. :ind J. Yin, Budling or Orthotropic Plates v.ith Free nnd Rorntionally Restrained
l JnJoadcd Edges. Tliin\l'alletl S1ntr-11.11e.1, Vol. 24. 83-96. 1996.
27
L. P. Kollar, Buckling or Unidir.li on.nUy l.onded Co1npositc Pls1c.s with One Free-and One Rotn
tionally Restrained Unload<.>d Edge. ltJurriul of Su1~crurol E11.~i11eeriug . Vol 128, l202-121I. 2002.
4.3 BUCKLING OF RECTANGULAR PlATES 131

80

~
~

<
,;
e 40
long-plate approximatlon
""
Q

~
!!
"' :! IL, I~
L,

0 0.2 0.4 0.6 0.8


Platc-lcnglh, L.T (01)
Figure 4.32: Buckling lo.ads o( lhc plate in Example 4.9. 1be resuhs shown are by Eq. (4.156)
(labeled as - present..), by Bank and Yins equations.. and by the long-plate approximatjon.

Solution. The expression in Table 4.9, fifth row (page 125). together with the
stiffnesses. D11 = 99.25 N m, D>..i = 6.47 N m , D12 = 1.94 N m, Dr..= 3.03 N m
(Table 3.7. page 84) gives
2
n" D11 1.25 1;Di2 12 1);,,
'v.T.tr = T + 1 + 4. t2.c L~ + .L~
1 .,
= 979.5/2 + 0.253 x JO/, + 14 560. (4.156)
'
where/., = L.,/; ands is the parame ter of restraint (Eq. 4.152):
Du
s = ...-
kly
= I. (4.157)

The blDckling loads for i = I. 2, 3, 4, 5, 6 are

1 2 3 4 5 6
1V.,." (kN/ m) 81.67 46.03 56.85 81.20 115.04 157.37

The lowest buckling load corresponds to; = 2 and is 46.03 kN/m.


The buckling loads for pla tes with the same width but with different lengths are
given by solid lines in Figure 4.32. The buckling loads calcula te d by the e quations
211
o( Bank and Yin are also included in this figure. The approximate expression
(Eq. 4.15&) underestimates Nw. and. hence, is a conservalive estimate.
The long-plate expression gives the buckl ing load of this plate as 46.16 kN/m
(Example 4.12, page 139). This value is a lso sbown in Figure 4.32.

a L. C. Baok and J. Yin, Buc.kting or Onbouopic Plates v.-i1h Free. and RotationaUy Rcsuained
Unloaded Edges. Tl1i11\\'(ll/td S1ruc1ur.-s, Vol. 24. 83-'96, 1996.
132 THIN PLATES

"y
,{;S,
~ 4 -iV.,r,;

,
L, .__,v.,fJ
-
j
~
j ~ 4 - :V,ro

< - :V.,o

:lo /\i.tO
x
1-'lgure 4.33: Long platc-subjc-cted to a uniform compressive edge load and the different types of
~upports along the long edges..

4.3.5 Long Plates


We consider a long rectangular plaue of constant t hickness \\hose le ngth is large
con1pared with its width.

Unifonn Compressi>'e Load Along the Long Edge

T he length or the plate is L, and the widt h is L, ( L,. L,). T he s upports along
the long edges are as sho\vn in Figure 4.33. A uniform compressive load N.10 is
a pplied along o ne of the Jong edges or the plate. We wish to de te m1ine the lowest
value of t he applied under which the plate buckles.
\Ve treat the plate as Jong and assun1e that it undergoes cylindrical de-
formation a long its length (Fig. 4.4). This a pproxin1ation is reasona.ble when
.L1 > 3L_..!/ D11/ [),_, (Eq. 4.19).
When three inplane loads N.,0 N,.0 , and N.,,.11 act on tbe plate (Fig. 4.14, right),
the equilibrium equations in the x, y, and ~ directions (Fig. 4.1) arel9
Nx + -a'.~u = 0
-()ax ay
aN,. aN.. ,
-ay + ax = 0 (4.158)

()Vx av... dz,,;u dz wo d1 wu


ax + a); = iVxo dxz + N>o dy2 + 2Nxyodxdy,
'''here N.x, Ny, and N.x,. are the in plane forces (per unit length), and Vx and ~\ are
the transverse s hear forces (per unit length) (Fig. 4.1). tvlo1nent e quilib ria about
the)' a nd t a.xes give (Eq. 45)
v._.=>-a-
aM., aM,,.
+-a-
aM,. aM.,,.
II'.=ay- +ax- (4.1 59)
x y
\Vhere Mx and J\111,. are. respectively, the be.nding mon1e nt and nvist mo n1e nt per
unit length. 1\way fron1 t he short e dges the (orces and 1noments do no t vary along

~ S. P. Ttmoshcnko and l Gere. Tll e(Jry of Efosric S1tJl>ili1y. 2nd edition. ~kGrnw-H iU. New York.
1961, pp.333-334.
4.3 BUCKLING OF RECTANGULAR PLATES 133

the le ngth o( t he pla te. Thus, from the last of Eq. (4.1 58) and from the firs t of
E q. (4.159) we have
dV. d2wo
- ' - N,..- 2 = 0 (4.160)
dx dx

dM, - \I,= 0. (4.161)


dx
By substituting \I, from Eq. (4.161) into Eq .. (4.160) we o btain the equilibrium
equation
il1 1W x d1 1n u
-:;-;-
ux-
- N.,. -dx--, = 0. (4.162)

As v.as sho,vn in Section 4.2.2, v.he n the pla te is syn1metrical. the be nding
moment M, is (see Eq. 4.25)

(4.163)

Equations (4.162}. (4.21), and (4.163) yield

Jong plate
(4.164)
syn1n1etrical layup.
The eq uation describing the buckling of .an isotropic beam subjected to an
axial load iVJ;o is30
d4u; ..... tFuJ
4 + N," d
E/ - -
lfX . X2
=0 isotropic beam. (4.165)

The structure of the t\vo preceding equatio-ns is the san1e. There fore, the buck
ling load of a long plate (s)'mmetrical layup) may be obtained by substituting the
value of Di1 for El in the expression for the buckling load of the corresponding
isotropic beam.
It \Vas sho\\11 in Section 4.2.2 that 'vhen the layup o( t he pla te is unsyrnmetTicaJ.
the deHection may be obtained by replacing EI/ p' by l(f/ p in the expression fo r
the deflection of the c.orresponding isotropic beam. By sin1ilar arguments it can
be shown tliat the buckling load of a Jong plate with unsymmetrical layup may be
obtained i>y substitu ting the value of l(f for El in the e xpression for the buckling
load of the corresponding isotropic beam. (l(f is given by Eq. 4.52.)

4.10 Exan1ple.. A 0.1111lo11g and 0.2-n1ll1ide rectangular plare is 111ade ofgraphite


epoxy. Th n1a1erial properties are given in Table 3.6 (page 81). Tiu! layup is
(45~/0 12/45q. Tire Odegree plies are parallel ro rlre .rlrorr edge of r/Je. plare,
- -
111e pla1e fs eirlrer si1nply supported or builtiH along all fo ur edges. Tire plate is
subjecred ro uuifortu co111pressive loads along J/Je long edges (Fig. 4.34). Ca/cu/are
tire bm'klil1g load.

30
Ibid .. p. 2.
/,,-w 700mm L,,=-100 nun

t t t t t t t
Figw-c 4.,14: 1bc plat~ in Example 4.10.

Solutlon. The pla1e may be trea1ed as .. long" (Example 4.1. page 96). The buckling
loads of the corresponding beam a re (Eq. 6.337)

:r'EJ
1V,,, = V- (ss) (4.166)
2
4
Q. - " El (buihin). (4.1 67)
' o - Ll
The buckling loads of the plale are obtained by replacing/ by Du (see page
133) as follows:

:r'D,,
,v," = ~ (ss) (4.168)

N. - 41T' o,, (builtin ). (4.169)
.. o - Ll
,
Wi1h 1he \alue of Du= 45.30 N m (Table 3.7, page 84) and wi1h L, = 0.2 m.
tho buckling loads are

N'"' = 11. 18 kNlm (ss) (4.170)

N'"' = 44.71 kN/m (builtin). (4.171)

r"
IL, l,
lN,,
..
N,,

-- s

ia,I fl bI
SS
Cc~I tdl
c:::;
(Cl I frtt
! fll fr
Cgl~

(h)ls;c ssl Ci) lss Cjl I


..1gurc 4,3!'i; Long plntc sub1ectcd to uniaxial load ( lop) ond lhe Jiup1'H)rt11 ttlong lhc Jong (a tog}
11nd 11hor1 cdgc;c ( h to j).
4.3 BUCKLI NG OF RECTANGULAR PLATES 135

Uniform Con1pressi\e Load ;\long the Short Edge - Orlholropic


and Symmetrical Layup

We conside r a plate with length L, and width/_, ( l, L,)(Fig.4.35). The layup of


the plate is orthotropic and symmetrical. TI1e possible edge supports are shown in
Figure 4.35. The pla1e is s-ubjcc1cd to n uniax:ial co1npressive load A~o (Fig. 4.35_
lop).

Simply supported and built-in edges. The buckling loads of long plates whose
long edges are either simply supported or built -in (Fig. 4.35. a, b. c) are given below.
The buckled shape of a long plate away from the loaded edges is the same as
the buckled shape of a plate simply supported along the loaded edges. Therefore.
the buckling loads oflhe long plate are also given by Eq. (4.141 ). \Ve ue interested
in the lowest buckling load. \Ve denote the buck.ling length corresponding to this
buckling load by r,'. With this notation. Eq. (4.141) becomes
, I !l_r.2 a,
N,.a = Durrl'f + 0,," 2 1{+ 2(Du+20,,.) L( (4.172)

The necessary condition that gives the lowe$1 N,.., is d ( N ..n) / d(t; ) = 0. This
condition and Eq. (4.172) give

/
0
.t
= !!...l,
l . vrn;;,
rs;; (4.173)

Table 4.10. The buckling tenglhs co<respondlng to the


lowest buckling load with unidirectionally loaded longgiates
(orthotropic and symmetrical layup, E = ii\O(, ( - "");

'"
L 1 is the wldlh, and ii Is the spring constant. Plates with one
simply supported and one tree long edge buckle wtth a long
wave In the x direction, as lllustraled In Figure 4.21>.
supports BuckNng tenglll r,
g.cl_ _,,::,__,If iv... 1., fii!,
A'~
~L.I__-'_'_..JI 0.RL,{!!jf
L.l___...JI E A'A 0.6641~/i

f~ L, ~
l.61SL. f/i

0.707L,~
1~ THIN PLATES

Table 4.11. The lowest buckling loads of long plates with olthotropic and symmetrical layup
<~ = , v = zo!ro~ K = ~ . .,,=j
r+h><<=A.u ' ). Ly Is the width, and kis the
, ,/1f;;1Ji l+fl-ZZ- 3 $$")(
spring conslant. The buckling loads of long plates with one simply supported and one free long edge
are given in the first three rows of Table 4.9 (page 125).
Supports Buckling load Nx.~ or N:(y,(t

~I
SS

:i5
IEN.. ;fl2J D11D,.,+2 (D11 + 20,,,))
I 1''s
~I
SS
~(3.125~ + 2.33(011 +20.,,)]

IE!V~,
I ~(4.53~ + 2.62( 0 12 + 2D,.l)
L>

_,1::~:::1 ~<- N
=t
~ ...,......_ ,.._ - ~ ~(2Jl + 4.139tJD11D12 + (2+0.62{ 2) (D11 +20,.)J

Jii" (15. IK~ + 7(1 - K) j when K :; I

J o,,o" ~
Li I b-.l vl - ,,+(K, - 1)6( 1-v) l whcnl < K

J"',!; 0" [ K(q l5.1Jr=7.+(1 - 1)6(1 - 11))+ ~] when K :5 I


" l +-112(

Jt~! "" J111 5. I Jf="i; + (K - q)6( 1 - 11)) when l < K


I.;

JI S.<
SS l(f ~ (13.9,/Du D11 + 11.1 (D12 + 20,.))

-!J,~ (8.125 + 5.045K)


r
-> -> ->
JV,.,, whl!n K :5 l
~
S.<

~,/Du(D1 i+2D<:t,)(J 1.71 + ~) when L< K


SS
~ <(-

1J,~ (15.07 + 1.0SK) when K :: I


;'J / O,,(D12+ 20..)(l8.59+ If,') when I < K

"-:.. / lltl!
z
Figure 4.36: Buckled shape of n lo ng plate with simply supported edges.
4.3 BUCKLING OF RECTANGULAR PLATES 137

As before. t he buckling length 1; is the half wavelength of the buckled shape.


We obtain the lowest buckling load by introdUJcing Eq. (4.173) into Eq. (4.172) as
follO\\"S'.

N.,.cc = ~ (a~JDuD,2 +<>s( D12 + 20..,)). (4.174}

The values of"' and <>s to be used a re those given in Table 4.7 (page 120) fo r
j = l . The resulting buckling lengths and buckling loads are listed in the first three
rows of Tables 4.10 a nd 4.11.

Both long edges are rotationally restrained. TI1e plate is rotationally restrained
along the unloaded long edges (Fig. 4.35. d). The buckling load for a pla te with
arbitrary length is given by Eq. (4.150). The value of I, t hat results in the lowest
buckling load is deno te d by /.~ . When the plate is long, the necessary condition
that gives the lowest Nx.a is d (Nx.") /d (I~) = 0. This condition a nd Eq. (4.150) ,
\Vilh Ix replaced by t.~. give

1-~l JD,;
, - v~ 'vo; (4.175}

where~ is given by Eq. (4.151). Equations (4.150), (4.1 51), and (4.175) yield

N., .cc = ~; [ 2,/1 + 4.139$ ,/ D 11 D,2 + (2 + 0.62$ 2) ( D,, + 2D.,,) l(


4. 176)

This result is included in T.>ble 4.11.

One long edge is imply supported; the other i.< free. The buckling loads are
given in th e first three ro"' of Table 4.9 (page 125) for plates with one of the
edges parallel to the load direction simply s:upported and the other edge free.
These expressions are applicable regardless o( the aspect ratio of the plate and
hence may be used to calculate the buckling loads of long plates with one simply
supported and one free long edge (Fig. 4.35, el.

One long edge is buillin; t he other is free. The buckling loads of plates with
one of the unloaded edges built-in a nd the o the r one free (Ag. 4.35. f) can be
calculated by Eq. (4.147). The value of I, that results in the lowest buckling load is
denoted b)'1;.When the plate is Jong, the necessary condition that gives the lowest
Nx.n is d (N,.") /d (/~} = 0. This condition and Eq. (4.147), with I, replaced by 1; .

Figure 4.37: The plate in Example 4.11.


138 THIN PLATES

l~= 500 mm
!V~
~!-------------~--
free
:V, 0
I
'-+~+--+--l-+--+---i--l---+--+':: L,= ~
1-igurc 4.38: Illustration of lhe plate. in Example 4.12.

give

I"= 1675L
;i y'fD".
[),...1
(4.177)

Equations (4.147) and (4.177) yield

.f7Jtt1J1i . D..,
N.w = 7 ' + 12-L
' . (4.178)
L.v ."
TI1is expression gives a con....;ervative estin1ate of the buckling load and under
estin1ates it by less than 14 pe rcent. Mo re accurate e xpressions, determined by
Koll:\r,31 a re given in Table 4.11 (page 136). TI1e accuracy of these equations is
about 2 pe rcent \\hen 0 < K ~ 1 and is about 5 percent 'vhen 1 < K $ 3.

4.11 Exantple. A rectangular plate ~virlr length 4 = 0.5 "'and width Ly = 0.05 111
is tuade of graphite epoxy tUJidirectioua/ plies 1vitlr the fibers oriented along the
x -axis of t/Je plate (Fig. 4.37). Tire material properties are given hr Table 3.6
(page 8 1). Tire layup is !Ow~ 011e of the /oug edges i.r builti11; the other lo11g edge
is free. The short edges are si1nply supported. The plate is subjecred ro u11ifor111
con1pressive loads iu tire x direction. Calculate the buckling load.

Solution. The stiffnesses of t he plate are D 11 = 99.25 N m, Du = 6.47 N m.


D12 = 1.94 N m , D"' = 3.03 N m (Table 3.7, page 84). The buckling le ngth r; is
(Eq. 4.177)

1.~ = 1.675 L,.j ~~ = 0.166 m. (4.179)

S ince the plate is longer t han the buckling le ngth ( L., > 1; ), we may treat the
pla te as Jong.'" From Table 4.ll (page 136), Kand 11 are

2D..,+ D12 D t2
K= 0.316 v= 0.242. (4.180)
.,/Du D,, 2D,,, + D,,

JI L. P. KolJar, Buckling of Unidirectionally Loaded Composite Plates wilh O ne Free nnd One
RotationnJI)' Rc$traincd Un1oadcd Edsc. }()unurl of S1ni-a ural 11gi11eeri11s,. Vol 128, 1202-1211 .
2002.
4.3 BUCKLING OF RECTANGULAR PLATES 139

Figure 4.39: Long rectangular plate sub-


je-eted to line-a rly var)ing load.

From Table 4.11 (fifth row) we obtain ( K < ! )

./Dii75;i ,.--
N.w = L' ( 15. I K v I - v + 7 (1 - K)) = 90.640 kN/m. (4.181)
y

One long edge is rotationally restrained the other i.< free. The buckling loads of
plates v"ith one of the. unloaded e dges rotationally restrained a nd the other one
free (Fig. 4.35, g) can be C'1lculated by Eq. (4.154). The value of I, that results in
the lowest buckling load is denoted by(~. When the plate is long the necessary
condition that gives the lov.est 1Vx.cr is d (1Vx,i:r) / d (/.~) = 0. This condition and
Eq. (4.154), with I, re placed by /~. give

(4.182)

where~ is the parameter of restraint given by Eq. (4.152). Equations (4.1 54) and
(4.182) yield

,,
J1ix CT -
- 7 ./750l5ll + J? I.A..
_. ' (4.183)
. ) 1+ 4.12s Lf L~

This expression gives a conservative estin1ate of tJ1e buckling load and under
estimates it by less than 14 percent. More accurate e xpressions, determined by
Koll:ir,32 are included in Table 4.11 (page 136). The accuracy o f Ulese equations is
about 2 pe rce nt \\ he n 0 < K .$_ t and is a bout 5 percent when 1 < K 5 3.
1

4.12 Exan1ple. A recuurgular plate with /en grit Lx = O.S 1t1 and 1vid1h Ly = 0.05111 is
1nade ofgraphite epoxy unidirectional plies ~\'ith the fi bers orienrt~d along Jhe :<axis
ofthe plate (Fig. 4.3.~). The material properries are given iii Table 3.6 (page 81). The
layup is (020]. One of the long edges is rotationally restrained vlu'/e the other long
edge is f ree. Tire short edges are sitnply snpported. The rotational spri1Jg constant
of the edge is k = 129.4 JV. The plate is subjected to 111,~for111 corupressive loads in
1/re x direction. Ca/cu/are the bt1ck/i11g load.

Solution. 111e bending stiffnesses o( the plate are D 11 = 99 .25 N m, Dii =


=
6.47 N m, D12 = 1.94 N m, D,,. 3.03 N m (Table 3.7, page 84) and i: I =
J2 Ibid.
1411 THIN PLATES

l 1t:~::::,::::::rr
!V rYJ
-> ->
SS

SS
<- <- <- <- <-
F'tgurc 4.40: Long ree.tangular plate subjec.ted to shear load.

(Eq. 4.157). The buckling l eng th/~ is (Eq. 4.182}

1.~ = 1.675L, ;~(I + 4.12!} = 0.249 m. (4.184)

Since the plate is longer than the buckling length ( L., > /~) . \\'e may treat the
pla te as " long." With the paramete rs Kand v give n by Eq. (4.180), fromTable 4.1 1.
sixtl1 row (page 136) we obtain ( K < 1)

N.w = ~IK(~15.1 ji-:-;;+ (J - 11}6(l - v)] +


lj
7
( l - K)
/1 + 4.12~
I
= 46.16 kN/m, (4.185)

where 11 is (Table 4.11)


1
I/ = =0.369. (4.186)
.;1 + (7.22 - 355,,H

Linearly Varying Load Along the Short Edge - Ortbotropic


and Symmetrical Layup
The layup o f the pla te is orthotropic and symme trical. 111e short edges of the plate
are either sin1ply suppo rted or built~ in, as sho\vn in the botton1 ro'v of Figure 4.35.
Both long edges are si1uply supported. Along the short edges the plate is subjected
to a linearly varying normal load \\!ith maximun1 intensity Nx" (Fig. 4.39). The
lowest buckling load Nx.n at which t he plate buckles is given by Lekhnitskii"
11'2
N, ," = J.i.(13.9/ Du D,, + 11.l(D12 + 2D66 )j. (4.187)

This result is included in Table 4.11 (page 136). The buckling length is given in
Table 4.10 (page 135).

Shear Load - Orthotropic and Symmetrical Layup


The layup o f the plate is orthotropic.and S)'lllmetrical. Both long edges are either
simply supported or built-in (Fig. 4.40). Regardless of the manne r in which tl1e Jong
edges are supported, the short edges may be either simply supported o r builtin.

" S. G. Lckhnitskii. Ani.~or.ropic P/01tt. Gordon and Brca<h Science Publis.hcrs. New York. 196&.
pp. 462-463.
4.4 FREE VIBRATION OF RECTANGULAR PLATES 141

Table 4. 12. The parameter .81in Eq. (4.188) as a


function of K = ~+ 11
11 u

0 ;5 K ;5 1
Simpl)' supported 8. l25 + 5.045K 11.71 + Li'l
.,
Buihin 15.07 + 7.0SK 18.59 + ~~

The plate is subjected to a. uniforn1 shear load 1'1.t>~' \Ve v"ish to de te rmine the
JO\\'eSt value O( the )oad /V.ty. er at \Vhich the plate buck)e.s.
Seydel derived the lowest buckling loads for infinitely long plates. The resulting
expressions, as quoted by \Vhitney,:w are

(4.188)

Seydel gave nun1erical values for fJ1. These values can be approximated within
2 percent by the expressions in Table 4.12.

4.4 Free Vibration of Rectangular Plates


\Vhen a plate undergoes free, undamped vibration, the deHection of the plate is
sinusoidal with respect to tin1e t,

w0 =~sin (wt)= W0 sin (2.Jr/r) , (4.189)

where w is the circular frequency and f is the natural frequency. The period of
vibration Tis
I w=2Tr/= 2ir.
T=- (4.190)
/ T
and U? is the deflection of the plate a t tin1e t = T/4.
In this section \\'e obtain the natural frequencies and, hence, the periods of
vibration, of free.l) vibra ting plates.

4.4.1 Long Plates


\Ve consider a long rectangular pla te whose length L1 is large compared \Vith its
width L., . The mass of the plate is uniJorm. The edges n1ay be built~in, sin1ply
supported, or free (Fig. 4.41 ).

3'I J. l\.1. \\'hilnc.y, Struaurol Analysis of L(IJ11lrut1cd A11isotropic Plotu lCch nontic. Lancas ter,
Pcn n1'ylvnnia. 1987. p. 118.
142 THIN PLATES

,t;;, ~
~ ~
~ i
L,

J:'1gure 4.4l: 1 'hc-ditferent types or supports a long the long edges of a free.I)' vibratin!t long plate..

When L, > 3L,1Dii!Dll (Eq. 4.19}, a laterally loaded plate may be a pproxi
ma ted as a long pla te undergoing cylind rical deformation (Fig. 4.4), a nd the e qui
librium e quations a re (Eqs. 4.22 a nd 4.23)
dV,
dx + p=O (4.191)

dM, - V, =0. (4.1 92)


dx

\Vhere p is the lateral force (per unit area) acting on the plate. In the case o( a
free ly vibrating pla te .pis the inertia force, which, with the use of Eq. (4.189), is

aiw"
p = - p....,
at
= p (2.!r/)2 W" sin (21' f t ). (4.193)

where p is t he mass of t he plate pe r unit a rea. Equations (4. I 91)-(4.193) give

d'M,,+P (2tr f)' W "-0


- . (4.1 94)
d x
When the plate is symme trica l, the bending mome nt is (see Eqs. 4.25 and 4.21)
()2UJU
A1x = DuK.r: = - D11--2 (4.1 95)
8x
W ith t his moment, Eq. (4.194) yields

d'w - L (2.!rf)' w" = 0 long plate


4 (4.196)
dx D1 1 symmetrical layup.

The equation o f a freely vibrating isotropic beam is1s

d'w p' 2 ' isotropic beam, (4.1 97)


dx' - El ( "/) '"=O
'vhere p' is t he mass of t he bean1 per unit length.

-" \ V. \Vcaver, S. P. Tin1oshcnko, and D. H. Young. Vil>rathn1 Pmblet'IS in 11gi11eeti11g. 51h edition.
John \Vile)' & Sons., New York. 1990, p. 417.
4.4 FREE VIBRATION OF RECTANGULAR PLATES 143

Figure 4.42: The plates ln '.':ample 4.13.

From Eq~ (4.196)and (4.197)we observe that the natural frequencies of a Long
plate (syn1n1etrical layup) with bending stiffness D 11 and n1ass p are the san1e as
the natural frequenc.ies of an isotropic beam 'vith bending stiffness El and n1ass p' .
Thus, the natural frequencies of a long plate (symme trical layup) may be o btained
by replacing El / p' by D 1if p in the expression for the natural frequencies of the
corresponding isotropic. bean1.
The natural frequencies or a long plate with unS)'lllme trical layup may be
obtained by replacing E l / p' by \II/ p in the expression for the natural frequencies
of t he corresponding isotropic beam (where 'if is given by Eq. 4.52).

4.13 Exan1ple. A 0.1-111/011g and 0.2. . ntliiide rectangult1r plate lf n1ade of graphite
epoxy. Tire tuaterial properties are given hr the Table 3.6 (page Bl). The laynp is
[45\/012/451). Tire Odegree plies lire parallel ro the slrorr edge of tire plate. The
plate is either situply supported or builtiH ttlong all four edges (Fig. 4.42). The
nrass of the plare is unifor111 (p = 3.2 kg/in 2). Calculate tire circular and natural
frequencies.

Solution. The plate may be treated as "long" (Exa mple 4.1, page 96). T he circula r
frequencies of the corresponding beam are (Eq. 6.398, and 1hble 6. 13, page 308)

w (/~,
, = yp; Li .8 ; =tr.21t.3tr ... . (ss) (4.198)

W ,. p' ~
= ~/ El L'1 .RI = 4.730. 7.853. 10.996, . . . (built-in). (4.199)

Tiie circular freq uencies of the plate are obtained by replacing El/ p' by D11 / p
as follows:

u; =!r.2tr.311', .. . (ss) (4.200)

W , - I/ D11
. - ~; ., = 4.730. 1.s53. to.996, ... (built-in). (4.201)
P L.,1
144 THIN PLATES

Figure 4.43: Rectan$ular simpl)' supported (ss)


~ plate.

W ith 0 11 =45.30 N . m (Table 3.7, page 84) and L, = 0.2 m. tbe circula r fre-
quencies a re

t.IJl = 928 "" = 3 713 W3 = 8 355 )/ S (ss) (4.202)


"'' = 2 104 w1 = 5 801 w 3 = 11 3731/s (built.in). (4.203)

The natural fre que ncies are f = w/ 2:r (Eq. 6.397)


/ 1 = 148 fz = 591 /1= I 330 Hz (ss) (4.204)
! = 335 /1=923 f3 =I 810 Hz (built-in). (4.205)

4.4.2 Simply Supported Plates - Symmetrical Layup


We cons ider a rectangular plate \vith diluensio ns L., and Ly simply s upported
along its four edges (Fig. 4.43). The layup of the plate is symme trical, [ BJ = [OJ.
The n1ass of the plate is uni(orm.
Fo llowing \ Vhitney,30 we obtain the na tura l freque ncies o ( t his pla te by the
energy method. By introducing wl) (see Eq. 4.189) into the expression for U
(Eq. 4.55), we obtain

U= V sin2 (2tr f t) , (4.206)

\Vhere U is defined as
4
-U=-211'1 D11 (a'-ax'"-"')' + >,,.. (a'-ayW-")' + D.,, (2a'
[ -W -")'
axay1

" 0

+2 ( 0 12 .....-
azW
.......-, +
azur aiwo za1iff'
o,.,.-:;-:;-+
azU?
0,6 - . - , - - , -
2-a1wu)]dydx.
ax1 oy ox oxay ay- axoy
(4.207)

The kinetic e ne rgy of the pla te is

i,f J (dw)-
1... z.., '
K = I 71 p dydx. (4.208)
" 0

JI> J. tlil. Whi1ncy. S1tJJ<'-t111ol Analy.,1..~ of Lar11ir1aJe(/ A11iso1ropic Pla1e.1. Tccbnontic. Lane.aster,
Pennsylvania. 1987. p. 166.
4.4 FREE VIBRATION OF RECTANGULAR PLATES 145

Subs tit ut ion o( the deflection, given by Eq. (4.189), into this equation yields
'~ ,"'!
K =&(21f/) ons (21f/ t) / 2 2
j pw" dydx. 1
(4.209)
0 I)

1-\ ccording to the la\v of conservation of energy the c.haoge in strain e nergy
from tin1e / = 0 to tin1e / e quals the change in kine tic energy during this tin1e
(U, - U1. o) = - (K, - K 1. o). (4.210)
Initially. at time r = () the strain energy is zero (Eq. 4.206), but a t time 1 = -Jr
the kinetic e ne rgy is zero (Eq. 4.209). Thus, we have
ul- f, = Kl.I.I (4.211)
Equation (4.211), together with Eqs. (4.206) and (4.209), yields
L , I.>

&(2" n' j .f pw"1dydx = V. (4.21.2)


0 (I

Pron.1 this equation \\'e obtain

(21f f)' = - - - - - -
v (4.213)
1, L,
~ J J pW"'dydx
0 0
We use Rayleigh s ene rgy method~ 7 to o bta in the defle ction. For the s in1ply
supported pla te under consideration the geome trical boundary condit ions re quire
that the d eflections be zero along the edges (Eq. 4.57) as follows:

!'"'
and O::;y::; Ly
x = L" a nd O::;y::; L,. (4.21 4)
ul' =0 at
0 5 :c 5 L,, and y= O
05x 5 L,, and y = L,. .
The follo\\'ing deflection salisfies these geon1etrical bo unda ry conditions:

w = iii" sin (21' ft) , (4.215)


where ~ is
I J ,
'\' '\' 11fX , p ry
iff = L,, L,, IJJij SIU - SID - , (4.216)
;. i ;- i Lt Ly
where I and J a re the nun1ber of terms, chosen arbitrarily, for the summ ations
and u1i; are constanls. Accord ing to the Rayle igh principle the freque ncy of vi
bration of a conserva tive systen1 has a m inim u111 value in the ne ighborhood of the
fundamenla.l iuode.311 \\'e express t his principle in the fo rn1

.!L -o
tluJ;i -
(4.217)

n L. r-.tei.ro\itch, Pritteiplt!.'1 and TI>ch11iqut!.\' of Vibrations. Prcntioc. H all, Upper Saddle River,
New Jcn:l")'. 1997. pp. 5 18--522.
.13 ]bid., p. 520.
146 THIN PLATES

To dete rmine the values of w,;. we substitute Eqs. (4.207). (4.213), and (4.216)
into this expression. Algebraic n1ilnipulatio ns yield the follo\\'ing sy.sten1 of s imuJ ..
taneous a lgebraic equations:
I J
= 1, 2, 3, . . ., I
_L L (GmmJ -
i I ; .. 1
A~mnij) UJ;; = 0
I i. m
j.11=1 , 2.3 .. . ., f ,
(4.218)

'vhere Ais given by


l ,
}, = ;j (2trft pL,L, . (4.219)

For convenience, we introduce the contracted no tation

k = (i - l)f +j j i. = I. 2. 3.. . ., I (4.220)


l J = I. 2, 3. . .'J
I = (m _ t )J + 11 j m = l , 2, 3, . .. I (4.221)
l f l = 1, 2, 3... .. J.
Equation (4.218) may now be written as
I x/ / xJ
L c.,w, = ). L ~kfWk I= I. 2, 3, . ., Ix J . (4.222)
kl k l

where G;t (= G,;) and t he Kronecke r delta J!k (= ~kl) are given in Table 4.1
(page 102). Jn expanded form E q. (4.222) is

G11

([
Gu~1i1
In the case of free vibration the deflection is nonzero. For no nzero deHections.
Eq. (4.223) is satisfied when the determinant of the matrix in the parentheses is
zero. At this condition ), is the eigenvalue of Eq. (4.223). There are J x I eigen-
value~ deno ted by A;; , \Vhich may readily be c.alculated by commercial softv.are.
The natural freque ncies are calcula ted from Eq. (4.219) as follows:

Ji; =; //>~iL,.. (4.224)

For an orthotropic plate 0 16 = 0,,, = 0, and the e igenva luesof Eq. (4.223) can
directly be calcula te d. T he result is

A;; = ~L,L,ir' [ D11 (LY +2(D,, + 2DM) ( L)' (LJ' + o,, (l)'l
(4.225)
4.4 FREE VIBRATION OF RECTANGULAR PLATES 147

Figure 4.44: 'the plate in E.:tamplc 4.14.

Equations (4.224) and (4.225) give the natural frequencies of an orthotropic


plate as follows:

/;; = :; { ~I (L)' + 2(D12 +2~) ( L)' (l)' + o,, (1,) '}


(4.226}

The values of f, 1 must be calculated for different values of i and j.

4.14 Exantple. A 0. 1~111 -/oug and0.2--n1 lt1ide rectangular plate is 111ade of graphite
epoxy. The n1aterial properries are given hr Table 3.6 (page 81). The layup is
(45\f012/ 45j). Tire Odegree plies are parallel to the short edge of the plttte.
Tlie plate is simply supponetl along all four edges (Fig. 4.44). Tile mass of the plate
l'i u11ifor111 (p = 3.2 kg/1112). Cttlculate rhe natural frequencie.'i.

Solution. From Eq. (4.226), with the stiffnesses D11 45.30 N m, 0,2 = =
= =
25.26 N m, D 12 19.52 N m, ~ 20.62 N . m (Table 3.7, page 84} we
have

= / 21830i' + 4 780i'jl+81.11 j 4 . (4.227}

This yields the following values of/;; (Hz):

i\j 2 3
1 163 205 267
2 607 653 727
3 1 346 1 393 1 470 .

The lowest natural frequency corresponds to i = j = 1, and is

/ 11 = 163 Hz. (4.228}


148 THIN PLATES

---~--------------------
long-plate.appruxi1n.ation

o '--~~~~~~~~~~~"-~~~~__,

0.2 0.4 0.6 0.694 0.8


Length. Li; (m)
Figure 4.45: The lo,\C'st natural frequency of the.plate in E:)'.ample 4. l4 as a unction of the.plate
length.

\Ve now assess the lengthtO\\idlh ratios under 'vhich the longplate approxi~
mation is reasonable. To this end. \\'e calculated the lowest natural frequency of the
pla te, keeping the width 4 the same while c hanging t he length L,_ In Figure 4.45
\Ve plot the Jo,vest natural frequency thus calculated versus Ly. l n this figure 've
also included the lov.est natural frequency given by the long-plate approxin1ation
(Eq. 4.204). The results in this figure show that, in accordance \\ith Eq. (4.19), the
lo ng-plate fonuula approxin1ates \vithin 10 percent the natural frequency when
Ly is gre ater than 3L.,:/D 11 / 1),..1 = 0.694 m.

4.15 Exan1ple. A 0.2-inlong and 0.2-tn"ide recu111gular plate is 1nade of graphite


epoxy unidirectional plies. The 111aterial propenies tire given iJJTable 3.6 (page Bl ) .
T he layup is [0,/45,/90,j-452),. T he p late is simp ly supported along the four edges
( Fig. 4.4fi). The mass of tire p late is uniform ( p = 2.56 kglm ' j. Calculate the twttrral
frequencies.

Solution. The layup o f the plate is symmetrical but not orthotropic. The bend
= =
ing stiffnesses a re D11 34.61 N m, D22 I 2.34 N m, D12 4.58 N m, D66 = =
5. 14 N . m, D 10 = 3.34 N . m, D,6 = 3.34 N. m (Table 3.7, page 84). The na tura l
frequencies a re calcula ted from Eqs. (4.224) a nd (4.223). With t he preceding

figure 4.46: The plate in Example 4. IS.


4.4 FREE VIBRATION OF RECTANGULAR PLATES 149

stiffnesses, and with I = J = 7, these calculations yield the following /;1 (Hz)
values:
i \ j 2 3
1 211 446 9 10 (4.229)
2 640 797 1 179
3 I 359 I 569 1 955 .

The layup follows the lOperc.e nt rule (page 89), a nd we may treat t he plate as
orthotropic. from Eq. (4.226) we have

= / 20 850i' + 17 910i2 J1+7 435 i'. (4.230)

The values of /; 1 (Hz) are

i\j 2 3
l 2 15 460 886 (4.231)
2 642 860 1 257
3 I 363 I 566 1 934.
We see that these natural frequencies (v.hich are based o n t he orthotropy
approxin1ation) a re within 8 pe rce nt of the natura l frequencies resulting from the
exact calculatio ns (see Eq. 4.229).

4.4.3 Plates with Builtln and Simply Supported Edges - Orthotropic


and Symmetrical layup
\Ve conside r rectangular plates 'vith le ngth L., and v.idth L y. Each edge of the
pla te is e ither simply supported or built-in. The layup of the plate is o rtho tropic
and syn1metrical. Follo\\'ing Hearmon:.w the natural freq uencies of the pla te are
calculate d by the Rayleigh energy method.
Under free \ibration the deftection o f the plate is (Eq. 4.189)

w0 = W' sin (211' / 1) . (4.232)

By introducing this w 0 into the e xpression of the stra in e ne rgy (Eq. 4.107). and
by setting D1 = Di = 0, we obtain
U = Usin 2 (2tr/1) . (4.233)

Y1 R. E S. Hearn1on. The Frequency of Flexural Vibralion or Rectangultlr Orlhotropie Plates:


with Clamped or Simply Supported Edges:. Journal of Appli~d ,\fecl1llnics. Vol. 26. 537-540,
t 959.
150 THIN PLATES

\vbere U is defined as

-U= -l
2.
rf ' " ' [
(a'iii")- + D-n- (a'iii'
D11 - -
ax'
,
- -')- + D.,, (za2u;
ay'
,
--
0

axa )'
)
2

0 0

+ 2 ( D 12 a'W"
axi ay' a'W") Jd yd.r. (4.234)

By folJo,ving the san1e steps as in Section 4.4.2, \VC arrive at


Ii
(27rf}' = - - - - - (4.235)
! j' f'~ pw"'dydx
0 0
The deHection is assumed to be of the form
W" = AX;(x)lj(y), (4.236)
where A is the, as yet unknown, amplitude. For X,(x) a nd lj(y) we adopt the
shape of a freely '~brating beam. For different end supports these functio ns are
given in Table 4.4 (page 11 9). The variables i and j represent t he numher of half
waves in the.rand y directions, respectively (Fig. 4.19).
By introducing Eq. (4.236) into Eq. (4.235), and by pe rforming the integration,
\Ve obtain

I.;; = ~L,Ly (Du~+ D,,~ + 2(Dn + 2D66) L~~;) (4.237)

\Vhere Ai/ is the eigenvalue, \\hich is defined as

J., 1 = 4
I (211'/;;) ' pL,L,.. (4.238)
The parame ters cri. &2, a3, a4, as are given in Table 4.5. The values of a 1
through as 1nust be calculated numerically. The integrations simlify when X;(x)
and Yi (Y) are calculated by the approx:in1ate expressions of i given in Table 4.4.
The resulting approximate expressions for a , through as are given in Tables 4.6
and 4.7. 'vhereas a2 is
(4.239)
Tile parameters a 1-a5 and the corresponding eigenvalues ).;; n1ust be calcu
lated for dif!erent sets of i and ; , (i, j = I, 2... . ). The natural freque ncies are
then c.alculated by

. - !..1t JpL,Ly
f; , -
'A;; - .!...
- 2..

(4.240)

4.16 Example.. A 0.2ttllo"g and 0.211llvide rectangular plate is 111ade of graphite


epoxy unidirectio11a/ plies. Tire 1nateria/ properties are gil'eJJ in Tobie 3.6
4.5 HYGROTHERMAL EFFECTS 151

Figure 4.47: 1'be plate in Example 4. 16.

(page 81). The layup is IG.i/452/902/ - 452],. Tire p lare is b11ilt-in along the four
edges (Fig. 4.47). Tire mass of tire plate is tmiform (p = 2 .56 kg/111 2 ) . Calculate the
narural frequencies.

Solution. The layup follows the JO-pe rcent rule (page 89), and we treat the plate
as orthotmpic. With t he stiffnesses 0 11 = 34.61 N m, Dii = 12.34 N m, D 12 =
4.58N. m, D06 = 5.14 N . m. 0 16 = 3.34 N. m, 0,. = 3.34N. m ('fable3.7, page84)
Eq. (4.240) yields

/;;= 2'r
1 1[
P at a~ et1 ]
D11 E+ o,20 +2(D12 + 20 .. ) ~I.j

= J214at + 184a~ + 76.3",a s, (4.241)


where a 1 and " ' depend on i (i = 1, 2, . . . ) and " ' a nd as depend on j (j =
1. 2,. . . ) as give n in Tables 4.6 and 4.7, third row. From these ta bles the va lues
are
i= l i= 2 i=3
4.73 7.85 11.00
"' 12.91 45.98 98.91
"'
j = 1 i=2 i=3
4.73 7.85 11.00
"'
"s 12.91 45.98 98.91 .
111e values off;; (Hz) are (Eq. 4.241)
i\j 1 2 3
1 420 712 I 207
2 981 1 222 I 663
3 I 844 2063 2 458 .

4.5 Hygrotltermal Effects

\Ve consider a sn1all e le ment of a thin plate. The ten1pe ra ture a nd moisture con
centration (both of v.hich n1ay vary across the t hickness o( the pla te) a re T a nd c.
152 THIN PLATES

Figure 4.48: H)'grolhcrmal an d mechanical loads


on a ph1te cle.me.nt.

respectively. lnplane forces and n101nents n1ay also act on the e len1enl (Fig. 4.48).
Q\ving to these hygrothern1al and n1echanical loads, under plane.s uess condition,
the strains at a point are (see Eqs. 2.155 and 2.156)

(4.242)

\\/here 6 T is the te1nperature d iffe rence relative to a reference ten1pe.rature

l!.T= T - T,", (4.243)

and (J and fl a re the the nnal expansion and n1oisture expansion coefficients
(Section 2.6). The stress at a point is obtained by inverting Eq. (4.242) as
follows:

{=:}= IQJ({ :: }-tJ.r {f:}-c { ~:. })


f.ty Y xy <x.1>' f3.1J'
(4.244)

By definition. the fo rces and moments a re (Eq. 3.9)

M, } '" {u,}
{ =j
M,
A1xy - Iii.
t " '
fx}'
dt . (4.245)

It is c.onvenient to define tJ1e followi ng equivalent generalized hygrothern1al


forces:

,~t: = f'" IQ] ( {ii~: }+c {p


{N"'} 6.T e: }) dt (4.246)
Nxr - Iii. /3.,J'
<t.1y

{M}: = Jz[ QJ ar ~;. + c !: dz.


Mhl } '" ( {ii } {p }) (4.247)
1\.1.!'y - ltt- CfXf ft JI)'
4.5 HYGROTHERMAL EFFECTS 153

With the definitions o f the [A), [BJ, and [DJ matrices (Section 3.2.2),
Eqs. ( 4.244)-( 4.247) may be combined to yie ld

N, A 11 A.ii A i& 811 812 ,


N"'
~ A 11 A11 Aio 8 12 812 N"'
y
1VJ1y A1 Ai Ai. 81 s,_. N.!1!
= ,
M,;'
M, 811 812 Brn D11 Dn
M,, 8 12 Bn 8 16 Du D,, M~'
y
A(,,. 8 ,. 8,. B,,,, D, Di M"
' Y
(4.248)

au a 12 <Y16 PH N,
0'12 <Y22 &26 /J21 N,
0'16 &26 &'(t() /J61 Nxy
=
fJ11 fJ21 p,,, 611 M,
+
{J,, {J,, p,,, 6,, My
{J,. {J,. fJ 6,. M.1:v
(4.249)

where a;;. /3;;, and S;; are the e len1e nts of thecon1pliance n1atrices (Eq. 3.22). \Ve
define the Collo\\'ing generalized bygrothennal strains a nd curvatures:

au <x12 & 16 /J11


a 12 a22 a 26 P21

=
a 16 a26 a> jJ"' (4.250)
fJ11 fJ21 {J., 611
fJ12 fJ22 (J., 612
fJ1 {Ji. 11.. 6,.
These gene ra lized hygrotherma l strains \vo uld occur in t he re[erence plane of an
unrestrained plate subjected to changes in ten1pe ra ture and moisture content.
Expressions for calculating N'", M'" (Eqs. 4.246 and 4.247) are given in
Table 4.13 for piece,vise linear, linear, and unifor1n te1nperature distributions
(Fig. 4.49).

tlT

Pic-cewise
linear Linear Unifonn

Figure 4.49: Piece\\ise Hne-a r. linc.ar. and uniform lcmperalure dts1ributions acroi;s lhe-plale.
,.. rHINPlATtS

Table 4.13. The hygrotllermal forces N" ,M" for piecewise


linear, linear, end uniform temperature dislrlbullons (Rg. 4.49).
The .<l J'ik.Q and -<l 1i.o0 refer to the top and bottom of lhe kth
layer. for piecewise linear. linear, end uniform moisture
diolrlbutlons .<l T and ii are replaced by c and ii.
Piecewise linear 1e1npcraturc distribution

{ ~}
~
= t. ("-; 1 ({1J.cA7(,.,+A701l {~: }) U u l

With the preceding dcllnition o f the hygrothermal strains Eqs. (4.248) and
(4249) may be written in the following forms;

1V,. Au At? A1 B11 Bu e,. ,. , et.bl


'
N. Au Au A.... Bu Bu Bz. ';' , o.bl
y
NX)'

Al,
M,
= A"
Bu
Bu
A.... A..
8 1? B,.
Bu 8,,, o,,
B,.
Du
B,. 8,,,,
Du o..
0,,,
r:y
K, .......,,
yo.bl

,
e,.
l),J Ky
,.
M,, B,.. &. Dt. 0,,. 0... ....... K.a}'
"'
(4.251)
4.5 HYGROTHERMAL EFFECTS 155

,ox a,, .,,. /111 /112 /J10 N., ,a.bl


,..
y au &zz
Ct12

"2 /121 /322 {J,. /V.1


x
f(l ,hl
)'
y;,. "'Uzr,
" /101 /Joi fJ.. 1\J.yy
Yu.bl
xy
= p., ~l2 s,. +
Kx /311 /321
/311 p,, p.,
~II M,
""'
x
Ky
Kxy p,. flir, {!..
812
~ ..
~12
.S26 ~..
.S26 My
J\(Ty
""'
y
Kht
"Y
(4.252)

Rectangular plat.es 'Willl free edges. \\1e consider a rectangular unsupported


plate, that is. a ll four of tbe edges are free. Tbe plate is subjecte d to pure bend
ing, in plane mecbanical loads, a nd to bygrolhe rmal loads. Sucb a pla te can be
analyzed by Eq. (4.248). In these equations the ten1perature and n1oisture distri-
butions across the plate must be specified as \Vell as 6 of the follov.~ng 12quantities
(Table 4.14):
1V.t or f.~ A(, or K.,..

1Vy or f;: :\~" or " >' (4.253)


1Vxy or f~y A(Ty or Kxy .

111e deHection of t he plate is calcula ted by Eq. (4.16).


When the unsupported pla te is subjecte d only to a ten1pe rature c hange 6 T
and n1oisture c, the 111echanic-al loads are zero (Table 4.14) as follo\\s:
,~, =0 A1xy = 0 (4.254)
M, =O M,, =0. (4.255)
In this c-ase the strains and curvatures are (see Eqs. 4.249 and 4.250)

f; fl),hi

,.. x
fo,hl
y
y'''
xy
y:>,hl

Kx
= '"
"hi
(4.256)
x
Ky Khl

Kxy
hi
l(l:y'
Rectangular plates with b11ilt-i11 edges. \Vhen a plate 'vit h builtin edges is sub
jected only to a te mpera ture c ha nge 8 T and 1noisturec, the strains and curvatures
are zero (Table 4.14) as follows:

" =0
x
0
fy =0 .t y =0
y" (4.257)
Kx =0 Ky = 0 K.ty = 0. (4.258)
The normal forces a nd moments a re calculated by Eqs. (4.246)- (4.248).
Long recta11g11/ar plates. When the plate is long and is subjected only to a
change in ten1perature 6. T and moisture content c, some of the strains, curvatures,
n1oments. and inplane loads are zero. as shov.n in Table 4.14. The response of
the pla te is the n calcula ted by setting equal to zero in Eq. (4.248) tbe quantities
indicated in this table. The de Hections of the plate are calcula te d by Eq. (4.16).
156 THIN PLATES

Table 4.14. Rectangular plates subjected to hygrothermal and


mechanical loads.
Loading Conditions
N, or
or
:E;
N,
N.,, or
,,~\' or "''.,
K
M, or Ky
Al:iy or K1y

,v_, =0 .rv.~ =D N,,,. =0


J\~( =0 ,W,, =0 A/,.) = 0

f: =0 E ~ = 0 y~',, = 0
K, =0 Ky =O Kr;: =
0

~LIT

~ngp~atc !lT
f: =0 E:
= 0 y~~, = 0
K,1 =0 K~ =0 K1.,, =0

Lons plate
!lT
c
.

x K,. =0
:.l' Long plate ,v_,, = 0

4.17 Rxan1ple. A I 1n~long and/ .1nwide plate is n1ade ofgraph ire epoxy unidirec
tional plies. The layup is 1456/0.aJ~. Tire edges of the plate are free. The 1e111perature
of rhe bortom rnrface is raised by 80 c and the rop surface by 120 c (Fig. 4.50).
Estinutfe the ce1nperatureilrduced change in the di111e1Jsio11s of the plate. The ply

80 ' C SO"C !lT,, = 100 "C - 20 'C


Figure 4.SO: Illustration of the plate in Example 4.17.
4.5 HYGROTltERMAL EFFECTS 157

propertles are glven in Table 3.6. (pase 81) 1/ie thernutl expattsiou coefficients are
;;, = =
- 0.7 x 10-<~ w1d ;;, 25 x 1 0- ~.

Solution. The tempe rature disLribution ncross the plate is represented by the
sum o f a constant and a Hnearly varying temperature distribu1ion. as sho\\'ll in
Figure 4.50. The temperature dif(crcnccs shO\\'ll in this figure are

Ll.To= 80+120 = IOO "C Ll. 7; = 120 - 80 = 20 OOl C/m. (4.259)


2
"
where Ii = 0.002 m is the thickness of the plnte. The hygrotherrnal forces are
(Table 4.13. page 154)

(4.260)

(4.261)

N':'} ;J, }
L: zl-2.l._, 1~1. { ;;,
{"">"
K
=fl r, (4.262)
f't/l''
"Y Ali
k-1 ;;
u t

(4.263)

The stiffness mauices [QJ for the zero nnd 45dc~rce plies are (Eqs. 3.49 and
3.52)

148.87 2.91 0 ] N [45.65 36.55 34.79] N


0
IQJ = ~91 9.71 o 10 - ,
m
IQI" = 36.55 45.65 34.79 109 - ,
[ 0 4.55 34.79 34.79 38.19 m

(4.264)

The thennaJ expansion coe(ficienu of the ().degTee plies are

a,.. a, - .
1
-~ } = { ~
- }= { ~
07 } 10- c (4.265)
{

=
\\ilereti1 = -0.7 x 10...... ~ andiiz 25 x 10-~ are the 1hermal expansion coef.
ficients parallel and perpendiculorto the fibers. The thermal expansion coefficients
158 THIN PLATES

z,= lnun
.::2= 0.4 m
Figure 4.51: "fhc layup of the plate- 1n f:x
o, ampk 4.17.
: 1= - 0.4mm
45,
.:-... = - Jmm

transform as the strains. By replacing< by ti in Eq. (2.188) we obtain

(4.266)

where [7;] is given by Eq. (3.51). Hence, in the 45-degree direction the thermal
e xpression coefficie nts are

{- }" [
a, 0.5 0.5 0.5 ti1 12.15 I
ti, = 0.5 0.5 - o.5 -I { ti, } = { 12.15 } 10- c' (4.267)
ii.T)' - 1.0 1.0 0 ] 0 - 25.70

'''here z is the distance from the n1idplane (Fig. 4.51, Z1> = - 0.001 111, .z 1 =
- 0.0004 m. t 1 = 0.0004 m, z, = 0.001 m) a nd K is the numbe r of ply groups
(K = 3). With the preceding values of [QJ and ti t he hygrothe rmal forces and
mon1ents are

{N:' }
,v~ =
{ 10041 } N
31 804 - {,1r,
M'.' 1 = 0 {} (4.268)
l'iht
.Ty :17'
- 16 323 m /\.~~. 111ii
0

{~}
1
{,yhl
1\~ll } =
{,1r,
A1: }
=
{ 16.195 }
5.1 43 t o'N. (4.269)
\ fht
I :ff A1i: !\.lxy A7j - 4.697

The hygrothemrnl strains are given by (Eq. 4.250)

1),h l

"" "" " fJ11 fJ12 fJ1 ..


,yh1
x
,>.ht
,. U12 <>22 Ctu, ~11 /J22 p,. ,y~1
y'i>.ht a,. fJ.. p., p,,. Vht
xy
..
Kht =
/311
"' p..
/J21
Ct..
.Su 812 a,. ' A')'
M" .. (4.270)
,,
Khl
Khl
.~y
/312 fJ22 p.,
{Ju, p..
.S,2
.St6
Jin
bu. ...
()26 ~t
J\J!';,
4.5 HYGROTHERMAL EFFECTS 159

L7mm

t """"
.~~~~~~---->
IOOO mm / ,
! '
/
' IOOO mm
'
f--.,
eE / /
..at"-----
'
I

-------,'
I

x
figure 4.52: 'fhc delormed shape of Lhe plat~- In E.<tamplc 4. 17.

The layup is symmetrical ((/!] = 0), a nd the compliance matrices are (Table 3.8,
page 85)
- 2.99 - 3.77 ]
[ 7.45
[a) = [a]= -2.99 37.81 - 29.39 10- m (4.271)
- 3.77 - 29.39 48.20 N

[ 7 124 - 25.43 - 41.40] 1


1~1 = [dJ = - 25.43 I 16.82 - 82.58 10- ' - - (4.272)
- 41.40 - 82.58 153.66 N m

Equations (4.270), (4.271), and (4.272) give t he hygrothermal strains:

,.h}
" 0.hl
{Y.-y y
(I.hi
.11'
=
{
0.041 }
t.653 10--'
- t.760
{ K~'}.f; {o}
" hi -

6T11 -
0
0
(4.273)

;,.hi }
{""'
y<.hl
11y A7i
= {o} 0
0
(4.274)

The sun1 IJA7ii + (t 6 7i gives the to ta l hygro the rmal strains


,
t'.o.hl 0.041
,.
t'. o,hl
l.653
y(l,hl
'Y
- t.760 10- '.
K ht , = 123.0
<"' 284.2
t(XJI
"bl - 438.6
The deformed shape of the plate is illustrated in Flgure 452.
4. 18 Exaniple. A pl<tte is tuade of graphite epoxy unidirectional plies. T/Je layup
i.r (45./04 ] ,. The 0 -degree plies are parallel ro tire x axis. Tire plate is long in the y
direction, turd oue of the long edges is builtin and rlre orlrer one is free (Fig. 4.53).
171e 1en1perature of rlre bouotu surface is raised by 80 tC and the top surface by
120 "C (Fig. 4.50). uilnate the teurperature~iuduced change in the ditueusions of
tire plate. The ply propenies ore given in Table 3.6 (page BJ), the 1her111a/ expa1Jsio11
coefftcieu1s are fi1 = - 0.7 x JO-fl -2 ttlld fi2 = 25 x 10- 6 1:.
160 THIN PLATES

Solution. The strains in the laminate are given by Eq. (4.249)

,. {J,. N, \i'hl
,.,.
x
"" U12
"'
a16
fJ11
p,, /J22
/J12
{J,,, Ny
I '
1\fhl
"" U22
p,,, p., ~:,
y
Y;1,
Kx
= "'
f!11
a20
p,,
"
p,,, 611 (jl2
{J.,,
liu,
1V:r>'
M., + 1\1.!1
~,.
'
,,~,
Ky /JI? fJ21 /J62 811 ()21 M,,
" -'(l' {J,. p,. p,,. s,. ()26 ~,,, J\1xy J~l
, JiY

(4.275)

The hygrothermal fo rces are given in Eqs. (4.268) and (4.269). The complia nce ma-
trices arc give n in Eqs. (4.271) and (4.272). From Table4.14, sixth row (page 156),
\Ve have

KJ! = 0 K:ry = 0 (4.276)


J\~T.V = 0 M, =0 (4.277)

Equation (4.275) represents six equations that contain the six unL'TlO\VOS 1Y_v,
t\ify. /\.f-"Y f;, Y.~" K.T
The temperature distribution across the plate is represe.nted by the sum of a
constant and a linearly vat'}~ng ten1perature distribution, as sho\\'n in Figure 4.50.
The temperature differences sho\\'D in this figure are

~
= so+2 120 =100 c = 120Ir- 80
LI. ' O LI. 7i = 20 000 0
C/m. (4.278)

The correspo nding hygrothermal forces are given in Eqs. (4.268) and (4.269),
respectively. By substituting the values o( the hygrothermal forces given in these
equations into Eq. (4.275) we obtain

C<.~)ATio = 0.172 X 10- 3 ( y" ) = - 0.475 x 10- 1


.TJI "'7j, (Kx}AT, = 0
(4.279)

- l 1
(K, ).-.7; = 36.71 X 10 . -. (4.280)
m

The sum (),.To + () 4 Ti gives the hygrolhennal strains

.; = 0.172 x 10- 3 r.:, = - 0.475 x 10- 3


"' = 36.71 x 10- 3 - .
1
m
(4.281)

The deformed shape of the plate is illustrated in Figure 4.53.


4.5 HYGROTHERMAL EFFECTS 161

''
'
'' IOO mm ' \ 0 48 x l0_, rad
~ .:
1
' .
::: t :
g+ ... --------"
x
figure 4.53: Oc.formcd shape o( a 100 by IOOmm cleme nt of lhc plate-in Example 4.18..

4.5.1 Change In Thickness Due to Hygrothermal Effects


Ov,.ing to c ha nges in ten1pe rature and moisture content the thickness o( the plate
changes. The change is (Fig. 4.54)

air= j_,,'" <,dz. (4.282)

From Eqs. (2.l33) and (2.165) the normal strain in a ply is

<o = <1 = [Su Sn S,.j { :: } + !> T1i3 + c'if,. (4.283)


't12

In the x1 , x2 coordinate systen1 the ply stresses are

(4.284)
{ :: } = [7;. J { : ; } .
t'tz r..ty

where [T. J is give n by Eq. (2.182). Jn the .r, y laminate coordinate system the

Figure 4.54: Change in thickness of a composile plate nnd the. laminate and ply coordinate
S)'Stcms.
stresses in the ply a re (Eqs. 2. L26 a nd 2.l 65)

{ : : } = [ Q] ({ :: } -
1Jty Yx.v
t> T {; : } - c {
<xxy
~:
fJ...,,
}) . (4.285)

The s tra ins are (see Eq. 3.7)

{ "} { '~ } {"}


fy

YxJ
= ~~
Yx~
+Z Ky

K.t y
. (4.286)

Equations (4.282)- (4.286) describe the c hange in t hickness. For a plate sub -
jected to a uniforn1 6. To change in te mperature these equations n1ay be combined
to yield

"'' = t {is,,
-
(4.287)

\Vhere k is the ply number, K is the total nun1be r o( plies, and z is the coordinate
of the ply (Fig. 3.12). For unifo rn1 n1oisture distribution the change in thickness is
calculated by replacing I> To and ;; by c and '/i, respective ly.
\Vben t he mechanical loads are zero, the stra ins in, and the cunature.s o f the
refere nce surface are due only 10 hygrothermal effects and, from Eq. (4.252), we
have

{
f~ =
}

o
Yxy
,;.h1
{ hi }

y o.ht
xy
{ :: } = {
K.ry
:~: } '
KJtY
(4.288)

4.19 Exan1ple. A rectangular plate is 1nade of graphite epoxy unidirectioual plies.


The layup is [45./0,],. The edges of the p/me are free. The temperature ofthe plate
is roised by 80 C. Esri111a1e tire tetuperau,reinduced cluo1ge in the thickness of rile
plate. The ply properties are given in Table 3.6 (page 81), the 1her,nal expansion
coefficieutsarea1 = -0.7 x10-" ~ aud i:i2 = 25 x io-,,ot.
Solution. The te mperature dL~tribution a cross the plate is uniform a nd is

"To= so c. (4.289)
-.,u I t..J"l lt.U t l l l l l l'I V lllVV....11 VII ............... l l U "-IL.1 1\.1 ..... . u ....

The change in thickness is (see Eq. 4.287)

a1r=t.{1s,, ~J ~nr.1. (<:..-~1llQl ({;IJ -ar. {%:,}J)


+ (:.t - :.t-1) (6 To (ii'l),)}. (4.290)

The stiffness ma1riccs are (Eq. 4.264)

14ll.87 2.9 1 O] N [45.65 36.SS 34.79] N


(QJ"= 2.91 9.7 1 0 109 m2 [QJ"= 36.55 45.65 34.79 LO9 2Ill
[ 0 0 4.55 34. 79 34.79 38.19
(4.291)

11>e thermal expansion coefficienis are (Eqs. 4.265 and 4.267)

~ a..- . .7 l

{ %:, = ~ 10-<> c
}" { 0 }
{-
"
~'
Clx1
}" { =
12.IS
12.IS
-25.70
} 1
10 c'

(4.292)

where l is the distance from the midplane: lo = -0.CXU m. t1 = - 0.()()0.t m. ti =


=
O.OQO.I m. l.J 0.001 m (Fig. 451) and K is the nwnber of ply groups (K 3). =
The stress transfonnation matrix l To] in the 45-degree direction is ob1t1ined from
Eq. (3.51)

IT,, I =
0.5
0.5
o.s
o.s - LO
l.0] . (4.293)
[ - 0.5 0.5 o
111e slrains nnd curvatures o( the n1idplanearegive n by Eq. (4.273) ns fo llows:
.=~
K;r
o.041 x 10- l ~ t.653 x 10- J
: 0 Kv =0
= r.:',
Kxy
= - t.760 x 10- >
= 0.
(4.294)

With 1hese values Eq. (4.290) gives


611 = S.899 x 10- m. (4.295)

4.6 Plates with a Circular or an Bliptical Hole


e consider a plate with symmetrical layup containing either a circular or an
\ \1

elliptical hole (Fig. 4.55). The dimensions of the hole are small eompnred with the
diinensions or Lhe plnte. \\'e assume that a t some distance fro111 the hole 1here is a
region '"here the in-plane forces and strains are nearly uniform. TI1ese "farfield"
forces and strains are taken to be those that v.ould exist in the plate in Ihe absence
o( the hole. We 1Nish to determine the s trains ne ar the hole in tc nns o f the fa rfie ld
forces a nd s trains..
!I

2b p

2a

Figure 4.SS: Elliptical hole in a plate-.

Far fro111 the hole the "farfield" in-plane forces are de noted by 1V,~, ~- tv:f.
and the .;farfield .. in-plane strains are de noted by t;.~:: , ~;= , Y.~;/' . 111ese forces
and strains a re related by (Eq. 3.26)

{ 1v:;
.v~}
~\~ = [ A]
{...~t.:~ } . (4.296)
Yx.1

A t a po int near the hole. the inplane forces are

N., } {N';'} {N; }


N-N~+N
{1V.lr N:f. /\~-"
y - J' y (4.297)

'vhere N;. 1V)~ . N;,, are modification tern1s that depe nd o n x a nd y. Expressions

Table 4.15. The modification terms in Eq. (4.297)


N; = 2 Re Ji <l>I (Z1l +11{<1>; (Z,J)
N; = 2 Re J<l>i (Z,) + <l>i(Z,)J
,v;, = -2 Re ( 1<1>; <Z,) + ,<1>; (Zi)J
. <1>, (Z,) = ""''-'' (k = 1.2)
""+~-.. : - /,: . ' .,,
\k = ,._1,.,,11 (at the interfa ce {,t = e' )
Zt=x+yi1, (k= l. 2)
"s are the roots of the c ha ra<:tt-rlstic. po lynomial ( 1 = 1 i = j14 )
au' - 2a10:1 + {2au + " 66) 2 - 2.uM + au = 0.

whe re Re. re fe rs to lhe real part,; the imagint1ry un it:


is the.complex conjugate. of :
()' rcfc.rs to the.d e rivative \\'ith rc.s pe.c t lo Z-:
a and bare.sho,vn in Figure 4.55~
la;, )= (A.;J- 1 t1re-lhe in-planecomplia nc.es o f the p late.
for these para 1n c 1c rs arc given by Lekhnitskii40 and are presen1ed in Table 4.l 5.
1
The modificatio n te rms depend o n <l>t ( Z.) = 11,~; whe re At represents two un
known complex numbers \Vhose real and im aginary parts cons tilute (our unknown
cons tants. Thc.sc cons1a nts are d etermined from the condition t hat the surface trac-
tion is zero along the s urface o ( the ho le. It su ffices to e nforce this condition only
at l"'t\"O points on the boundary

N, (= tr,' + N;) = 0 at point P


N,. (= ~ + N;,) = 0 at point P
(4.298)
N,. (= N';" + N;) = 0 at point Q
N,,(=~+N;,) =0 at point Q.
TI1e four unknoYln cor1stants in Ak are ca lcula ted by the follo ~ing st eps:

1. TI1e constnnts A,... arc nssumed in the fo rm

A1=C1+C2i Ai = C, + C.i, (4.299)

where I = r-T.
2. The roo ts {ti nnd '"2of t he characteristic polyno miaJ glvc n in Tnble 4.15 are
ca1cu1o.ted.
3. Modification term.~ fl.':. 1V,,
. '
. at point P and N. fl.'~, at point Q a re calculated
for four different sets of C 1. C2 C3 C..
Set I C1 = I. C2 = C3 = C, = 0. resulting in
(N;)f. (N;,li and (N;)r. (N;,)~
Set 2 C2 = I. C 1 = c,
= c, = 0. resulting in
(N;lr (N;,)f and (N;)r (N;,l?
Set 3 C, = I. C 1 = C2 = C, = 0, resulting in
(N;)f. (1\~,)f and (N; lf, (N;,)f
Se t 4 C, = I .C1 = C2 = c,
= 0. resulting in
CN:,>f.

I
(N; lf, (N;,)f and (N;)f,
Fo r arbilrary values of C 1- C4 \Ve have

[(N;)f
I
N;I' (N; )[ (N; )f
N:: (N;,>r (N;,)f (N;v)f
(4.300)
N.,Q = (N-)Q
y t (N')Q
yl
(N vJ
;)Q
N;$1 (N;,)f (N;,lf (f'i;,)f

I
4. Equatioos (4.298) a nd (4.300) give

N:"
~
N':-
[(N;)f (N;)[ (N;)j
+ (N;,)f (N;,)f (N;,)f (N;,)f
(N;)f
(N")Q (/>r.)Q (N')Q (N;l,Q c,
llc,IC, -O
- (4.301)

I,v:;
1 ' 1 y l ' .)
<N:,>P <N:,>f ui;,>f <N;,)'/ c.
5. Equalions (4.301 ) are solve d for the C 1 C2 C3 c, oons tnnt~

.iu S. G. Lckhn1ukil. A11l,1t11t<1plr Plates. G ordon and Brc.nch Science Puhlb hcrs, New Ycwk , 1968,
ll. 31.
166 THIN PLATES

F'tgure 4.56: Orthotropic plalc \Vilh a circular hole.

'''''''''
Next we a pply the preceding equa tions to o rtho tropic plates containing a cir
cular hole with radius R and s ubjecte d lo a unilom1 tensile load N:;' (Fig. 4.56).
T he n1axin1un1 forces (per unit length) at the surface o( the hole ar~4 1

force at point P (4.302)

fo rce at point Q. (4.303)

\Vhere K r is the stress inte nsity factor

K r= 1 + ~
2 ( \ I~
A11 An - .412 + A11A22 - Ai,)
2"'66 - . (4.304)

x and y a re in the directions of o rthotropy, and t he eleme nts of t he stiffness ma trix


are to be calculated in this coord inate systen1. The force distribution N,. along the
x -axi< may be approximated by" (Fig 4.56)

N~
=-t (R)'
:-; + 3 (R)'
:-; - ( Kr - 3) [5 (R) (R)']}
6
N,(.t. O) { 2+ :-; - 7 :-;

(4.305)

4.7 lnterlaminar Stresses


Lamina te pla te theory is formulated on the basis of the assun1ption that the lan1
ina te as well as all the layers a re in a state or plane stre.ss. Correspondingly, all

.ti Jbid., p. 175.


<t? H.J. Konishandl 1'1. \Vbitnc)', Approximatc Slrcssesin nn Ortholropic Plnle Containins a Circular
Hole. l tJttrnal of Con1po.ii1e 1\1a1erials. Vol. 9, 157-166. 1975.
4.7 INTERLAMINAR STRESSES 167

(0190/90/0] [OJ [90]


Figure 4.57: (0/ 90J5 laminalc loaded in lc nsion and the d c.ormalions of unbondcd (). and
90-deg.rc-e ha)'ers.

outof plane stress components are zero (11.: = 0, ry.- = 0, r.c.: = 0). This a.ssump
tion is reasonable in regions a\\ay from free edges. Near free edges. both shear
and norn1al stresses n1ay arise be t,veen the layers. These interlaminar stresses n1ay
significantly alter the stress field existing a\vay fron1 the free edge and, importantly.
n1ay cause separation (delan1ination) of adjacent layers.
We illustrate the stresses near a free edge through the example of a f0/901~
cross..ply laminate (Fig. 4.57). The lan1inate is subjecte d to a unidirectional force
(per unit length) N, with x being in the direction of the ().degree fibe rs. 111e axial
load is shared by the plies., and the axial defom1ation of each ply is the sa1ne
(E~ = E.~). 'Ve no\v consider one of the 0-degree plies and the adjace.nt 9().degree
ply. Their transverse Poisson ratios are different (~.?,.:ft "!";). Hence., if the two
plies \Vere alJO\Ved to n1ove freely, they \VOuld deform by diffe rent amounts in the
transverse y direction (F1g. 4.57). ln reality, the t\VO plies are bonded together, and
their transverse deforn1ations (and transverse strains) are equal. Ob\riously, one
of the plies (in this example the 0-degree ply) must be in tension, and t he other
one (90-degree ply) must be in oomrression.
The stresses on a Odegree ply e le 1uent are s hown in Figure 4.58. A\vay from
the free e dge, on a sn1all e len1ent the ay stresses equilibrate each other, and there
is no interlaminar s hear stress ( r>'~ = 0). At the free e dge a y is unbalanced and is

".v ~~
~~7

u, 7~~~ "
17.v ~ ~ ~ 11,

y y
Figure 4.58: Frccbody diagrams and lhe stress dislnbutions near a free edge.
168 THIN PIATES

equilibrated by the interlaminar s uesses ( r>'~ : 0). Furthe nuore, 1'1y and r,,:: c reate
a mo1nent, which n1ust be equiJibrate d by the interlaminar normaJ stress d;, s hown
in Figure 4.58. The norn1al stress, as 'veil as ry;,. diminis hes in regions away from
the edge.
Care must be take n in calculating the interlan1inar stresses. Under t he ass ump
tion that the n1aterial is linearly elastic, t he calculations result in infinite stresses
at free edges.
CHAPTER FIVE

Sandwich Plates

Sandwich plates~ con..i:isting of a core cove red by faceshccts1 are rrcquently used
instead o f solid plates because of t he ir high bending stiJfness-to"'cight ratio. The
high bending sliffncss is the result of the distance be tween the faceshee t.s. "hich
carry the load, and the light weight is due to the light weight of the core.
Here. we consider rectangularsand\\i.ch plates with racc!lhccts on bo1h sides of
the core (Figs. 5.1 and 5.2). Each fa"'sheet may be an isotropic material or a fiber-
reinfor<Jed composite k1minate but must be thin compared >Aith the core. The core
may be foam or honeycomb (Fig. 5.1) and must have a material symmetry plane
parallel tons midplanc: the rores in-plane stillnesses must be small compared
with the in-plane stiffnesses of the fa=heets..
The bcha"ior of 1hin plates undergoing small dc!onnations may be analyzed
by the Kirchhoff hypothesis.. namely. by the assumptions that normals remain
straight and perpendicular to the deformed reference plane. For a snndwich pJate.
consisting of a core covered on both sides by Caceshects.. the firs t assumption
(nonuals ren1ain s tra ight) is reasonable. However, the second nssuniption 1nay no
longer be valid, bccnusc norn1als do no t necessarily ren1nin perpendicular to the
refere nce plane (Fig. 5.3). In this case the x and y displacements o f a point located
at a distance t (rom an arbitrarily chosen re.fere nce plane are

Lt= fl - zx,: v = v" - tXy: . (5.1 )

v.here rl' and v0 arc the .r and y displacements at the reference plane (v.hcre
z = 0) and x.. ~. X,r: arc the rotations of the normal in the x-z: and )"-Z planes. The
angle x.: ~illustrated in figure 5.3.
As shown in Figure 5.3. the first derivative of the deflection w" of the reference
plane 'vith respect to xis

(5.2)

169
170 SANDWICH PLATES

Figure S.1: lllustration ort he-sandwich plate and the hone)'Comb rorc.

Similarly, the first deriva tive or the deflection w0 or the reference plane v. ith
1

respect to y is

aw
-.- = X1 + y,.. (5.3)
ay .- -

5.1 Governing Equations


The strains a t the refere nce plane are (Eq. 4.2)

'1u 0 ()v0
o-
Yxy -
-ay + -ax (54)

The transverse shear strains are (Eqs. 5.2 and 5.3}

aw 0

Yx .; = ~- Xx:: (5.5)
ox

For convenie nce we define Kx , Ky . and Kxy as

ax.,, ax.,. .
K , -- ax
- -- Ky. -- - - ay (5.6)

\Ve note that K,,, Ky, and Kxy are no t t he curvatures of the re.fere nce pla ne.
They a re tJ1e refere nce pla ne's curvatures only in the absence of she ar deform a
tion.
The three e quations a bove represent the strain- displacement re la tionships for
a sandwich plate.

']:~1
--

.
Jt f
. . R~fe!C'!c.e~pl!,Ul~

Figure S.2: Sand\\'ich-platc-geometry.


.

.
5. 1 GOVERNING EQUATIONS

Figure 53: Deformation of a $.itOdwich plate


ln the x-z plane.

.4

Ne xt we derive the force- strain relationships. The starling point of the analysis
is the expressio ns for t he forces a nd moments give n by Eqs. (3.9) and (3.10)

J
loo ho loo

1V., =
- 1111
cr,dz N, = j <t .d z
-11,,
1 N.cy = J
_,,,,
rJ..,, d z

(5.7)
lo,

f f J
loo ho

,If.,= Z<t,dt M, = Z<fy d Z M.xy = Zfxyd z


- lt. - l lb - lit,

..
J
ho

I', = .r r..,dz
_,,.
v,. = r,.d t, (5.8)
- h.

where Ni, J\1;, and V; a re the inplane Corces, the mon1ents. a nd the transverse
shear forces pe r unit length (Fig. 3.11, page 68), respectively, and 11 1 and " are
the distances from the arbitrarily chose n reference pla ne to the plate s s urfaces
(Pig. 5.2). The stresses (plane-stress condition) a re (Eq. 2.126)

(5.9)

Prom Eqs. (2.2), (2.3), and (2.1 1) together witl1 Eq. (5.1) the strains at a dis
tance z from the re ferenc.e plane are
an au ax.~~ 0

.r=-=
ax -a.r - z-ax-
au 3v
0
"x":.
, = ay = iii - z ay (5.10)

Y.,, -- au au - au" av ( ax.,, ax,, )


0
-
ay + ox
" - ay + o.r
" z Y
., + ax
17~ ::>ANUWlliH flAl t:i

By combining Eqs. (5.4), (5.7). (5.9), and (5.10) and by utilizing tl>e definitions
of the [A], I B], [DJ ma trices (Eq. 3.18), we obtain

~ l
1\f fo - :ix

{}{}I ~v:.
N.tf
=(A) :; + (B)
r;.~
- a:;~
_-2!.u - ~XI~
(5.11)

l
3y ~J:

{ <~ } + (DJ
I
Af.t f.~ - ii.I

M,. } =(BJ !!.!'.!!


- '.:; . (5.12)
{ i\~ty Y:v _ !l!,;, - 3):!!
:ty :J.T

With the definitions in Eq. (5.6), these equations may be written as

N, } { } -~
{ <, }
N,, = [Aj ~ + (BJ Ky (5.13)
{
Nxr Y.1:y K:ry

A~, } = [B] { <.-~~ } + [DJ {


M,.
K, }
Ky (5.14)
{
i\1xy Yxy Kxy

In addition \Ve need the relationships bet\veen the transverse shear forces and
the transverse s he.ar strains.. The re levant expressions are derived in Section 5.l .3.
Here \Ve quote t he resulting expression. \\ hich is 1

1Vy11, I= [s"I 12
s"]jr. . I
522 r,.~
(5.15)

\\'he.re I SJis the sand,vich pla te s s hear stiffne.ss matrix.

In the analyses we n1ay entploy either the equilibrium e quations or the s tra in
energy. The e quilibriun1 equations a re identical to those given for a thin plate
(Eqs. 4.4 and 4.5).

5.1.1 Boundary Conditions


In o rde r to deter1uine the deflection, the condit ions along the four edges of the
pla te must be specified. An e dge may be built-in, free, or s imply supported.
Boundary conditions for an edge paralle l \Vith the y ~axis (Fig.. 5.4) a rc given
beJO\\'.
Along a builtin edge, the deHection w0 , the in-plane displacements u0 , uu, and
the rotations of normals xx~ Xr~ are zero:

u/ 1 = 0 ill) = u0 =0 Xx~= Xy; = 0. (5.16)


Along a free edge, \Vhere no external loads are applied, the bending Mx and
twist M xy mo1uents. lhe lransverse shear force Yx , a nd the in~plane forces 1Vx.
5.1 GOVERNING EQUATIONS 173

Built-in Free \\fuhout \\lath


11idc plutc 11idc plate
~

t= r
%
~
~
~
~ J
~
Figure S.4: Boundary conditwns for an cdJe panllcl to the ,..iaxis..

Nxr are zero:


M, =M, 1 =0 v. =0 N,, =N,r=O. (5.17}

Along a simply supported edge. the deflection w0 , the bending M, and twist
ftlx.. moments, and the in-plane forces N,,. N,, arc zero:

w0 =0 M, =M,, = 0 N,= N,,.=0. (5.18}

When in-plane motions are prcvcn1ed by the support. 1he in-plane forces are
not zero ( 1Vi . 0. N.J;1 ~ 0). \\lhercns the in-plane displacemen1s are zero:

u0 = 0 v0 =0. (5.19}

When there is a rigid pl:ne covcrin!t lhc side of the sand\\'ich plate the norn1al
ca nno t rotate in the y- z plane, ::ind \VC hnve

(5.20}

However. the t 1Ni.st mome nt is not zero (1WN,. :P, 0).


For an edge paralle l with the ,\'n:<is., the cquntions above hold \\~th x and y
interchanged.

5.1.2 Strain Energy


As \Ve noted previously. solutions lo plale problems may be obtained by the equa-
tions described above or via energy n1c lhods. The strain energy (for a linearly
elastic material) is given by Eq. (2.200). The thickness of the sandwich plate is
=
assumed to remain unchanged and. accordingly. f : 0. The expression or the
strain energy (Eq. 2.200) simplifi' to
........
U =~ff
0
J
_,
(a,4, +a,<,+ r.,y., + r.,y., + r,,y,.)d!dydx . (5.21)
174 SANDWICH PLATES

Substitution of Eqs. (5.4)- (5. 15) and Eqs. (5.26)- (5.32) (derive d on pages I 75-
176) into Eq. (5.21) gives
,., J-112 B11 8 11 Bi
A i ,...
f~

r A:?2 B12 Bu Bu, f~


/-l p A,_.
L, I,.

V=~.f f r:)' 1\ 16 A,, At,. B,, B,_. 8,,,, Yx~t


K, B11 B12 B,. D11 D12 Drn I(_,

" 0
Ky
K.~y
LB,, Bi2 B,_. D12 D,, D,,
B1 s,. s..
D,. D,, D,.
Ky

Kxy

+ ly.,_ y,.J -
.... S12
[s;, ~"H I
S21
Yx;
YJ~
dydx, (5.22)

\Vhere the superscript T denotes transpose of the vector.

5.1.3 Stiffness Matrices of Sandwich Plates


The stiffness matrices are evaluated by assuming that the thickness o( the core
ren1ains constant under loading and the in ..plane stiffnesses of the core are negligi
ble. Under these assumptions the I A]. I B], and I DJstiffness matrices of a sandwich
plate are go\erned by the stiffnesses of the facesheets and may be obtained by
the parallel axes theoren1 (Eq. 3.47. page 80). The resulting expressions are given
in Table 5.1. In this table the ( Aj' . [BJ' . j D]' and ( A] 6 . [Bj" , ( D)" are lo be eval
uated in a coordinate syste m \\hose origin is at each facesheefs reference plane.
Whe n the top and bottoLn facesheets are identical and their layup is S)'lllmelri
cal with respect lo each faces heet"s midplane, t he [BJ matrix is zero and the I AJ,
[ DJ matrices simplify, as shown in Table 5. I. (When the layup of each facesheet
is S}'fllme trical. the refere nce plane n1ay conveniently be taken at the facesheets

Table s.1. The (A), (BJ, (OJ stillness matrices of san<lwich plates. The
supersripts t an<I b refer to the t<>p and bottom facesheets. The
distances d, d'. and d are shown In Figure 5.2.
layup of each facesheet
with respect to the facesheet's midplane

Symmetrical
Unsymmetrical (identical faces!leets}
(A] {A)' + (A)" 2 (A]'
[BJ d' [A]'-d"[AI' +(BJ' + [BJ'' 0
[DJ (11' )' I A)'+ ("'>' [A)"+ (D(' + [Df
+ 2t1' (BJ' - 2,f' IBj" \111IA)'+2IDJ'
5.1 GOVERNING EQUATIONS 175

t.'
Fi~ure S.5: She-a r stress distnbution lr: (left}
c in a sandwich pla te a nd the appro:umate dis
tribution (right).

1.

n1idpla ne.) \Vhen the top and bo tto n1 faceshee ts are unsymme trical \\ith respect
to the faces heets midplane but are sym1netrical \\ith respect to t he n1idpla ne of
the sandwich pla te. t he n [A)' = [ AJ, I BJ' = - [BJ. [DJ' = [DJ, a nd the [A), ( B],
[DJ ma trices of the sandwich plate become

IAJ =2 IAJ' (5.23)

IBJ =0 (5.24)

ID) = ~d2 [ Aj' +2[DJ' +2d(B)' . (5.25)

111e s hear stiffness ma trix [S) is de termined as follo\vs. In the core, as a conse
quence of the assumption that the in pla ne s tiffnesses are negligible, the transverse
shear s tress rx~ is uniform. Jn general, in the facesheets the s hear stress d istribu
tion is as s ho\vn in Figure 5.5 (left). \ Ve appro:\;1nate this distribution by t he linear
shear s tress distTibution s ho \\'U in Figure 5.5 (right). t\ ccordingly, the transverse
shear force V, is

(5.26)

where the superscripts c. t. a nd b refer to the core, the top, and the bo ttom
facesheets, respectively. The distanced= c + 11/ 2+1 /2 is s hown in Figure 5.5.
Similarly, we have

V1 = r;.n. (5.27)
"
The s tress-stra in relationship fo r t he core materia l is given by Eqs. (2.20) a nd
(2.27). With the s upe rscript <: identifying the core. t hese equations give

(5.28)

where Ef1 are the e le n1ents of t he core stiffnesses matrix.


We neglect the shear deforn1ation of the thin faces heets. Wit h this approxin1a-
tion the s hear defonnation r.::: of the cross section is as sho\vn in Figure 5.6 (left).
\ \1e a pproximate this deforn1ation by the ave rage shear deformation Yx:: shown in
176 SANDWICH PLATES

11gure 5.6: Shear de.formation o( a sa nd,.,..ich plate.

Figure 5.6 (middle). The relationship between this average shear defonnation and
the core deformation is given by (see Fig. 5.6. right}

Y.,' . _ = ;rx.::
d (5.29)

Sin1ilarly, v.e have

' = d
Y.v.:: ;-r,,:: (5.30)

Equations (5.26)- (5.30) yield the relationship between the transverse shear
forces and the ave rage shear deformation:

l"I=d' [~
\', C' C!CS
(5.31)

By comparing this equation with Eq. (5.15). we o bta in

[~"
S 12
~"]
S21
= d'
c
[Ss5
~
(5.32)

The preceding four e le n1ents of t he n1atrix (C] characterize the core n1ateriaJ.
\\lhereas [SJ is the she ar sliffness n1atrix of the sandv.ich plate. \Ve point o ut that
(3) is 1101 the inverse ofthe [Ej matrix.
Orthntropic sandwich plate. A sand y,ich plate is orthotropic 'vhen both face
sheets as v.ell as t he core are orthotro pic a nd the orthotropy directions are parallel
to the e dges. The facesheets n1ay be d iffe re nt. and their layups n1ay be unsytn
me tric-al. For such an orthotropic sand,vich pla te there a re no extension- shear.
bending- t\vist, a nd extension- l\\'ist couplings. Accordingly, t he fo llov.ring e.le n1ents
o( the stiffness n1atrices are zero:

At = 1126 = 81&= Bu, = D1 = Di = 0. (5.33)

Furthermo re, fo r a n orthotro pic sandv.ich plate the transverse shear force Yx
acting in the x- z plane does not cause a shear stra in Y~ in the y-t plane. This
co ndition gives

s., =0. (5.34)

Isotropic sandwich plate. Jo\ sand,vich plate is isotropic v.he n the core of the
sand,vicb pla te is made of an isotro pic (such as foa n1) or transversely isotropic
(such as honeycon1b) ma te ria l a nd the top a nd botton1 facesheets a re n1ade of
5.1 GOVERNING EOOATIONS 177

Refen:--nce flane = Neutral plane


t'
c/2

c/2
,.
e'} c
- : =~=:\== -~~- r,dl~
..
'-MidplIDlc
..
I

Figure S.7: Neutral plane of an isotropic sandwich plate-.

identical isotropic materials or are identical quasi.isotropic laminates. The thick-


nesses of the top and bottom facesheets ma}' be diffe.rent.
For isotropic facesbeets tbe I BJ matrix is zero ([BJ' = 0). The I AJ a nd I DJ
matrices for tbe isotropic facesbeets are (Eqs. 3.41 and 3.42)

(A)'
.,., [1
11 1:.. r
0]
0 .
I - ('')' ~ '1"
(5.35)

wbere tbesuperscript i refers to tbe top (i = t) or to tbe bottom (i = b) foces beet


(Fig. 5.7) a nd E' a nd vr are the Young modulus a nd tbe Poisson ratio of the
facesheets.
We now proceed to evaluate the IA], [ B}, I DJ matrices for tbe entire sandwich
plate. To this e nd, \Ve choose a reference plane located at the center of gravity of
the l\\'O facesheets. The distance e fro1n the midplane of the core to the ce.nter of
gravity is (Fig. 5.7)

1' (c + 1') - 1(c + 1)


(5.36)
e= 2(1' + 1)

The d is tances ti' and db bet\\een the reference plane (passing through the
center of gravity} and the n1idplanes of lhe facesheets are

(5.37)

By substituting Eqs. (5.35)-(5.37) into the expression for the (BJ ma u'Lx given
in Tuble 5.1 (page I 74) we obtain tha t fo r the entire sand"1ch pla te tbe I BJ ma trix
is zero 'vith reference to the o reference plane. This means that for a sandwich
plate \\'ith isotropic core and isotropic facesheets bending does not cause strains
in this plane. Therefore., this re(erence plane is a "neutral plane:
By substit uting t he expressio ns of d' a nd t1 (Eq. 5.37) into the expressions
given in Table 5. I. we o btain the following (A) a nd I DJ matrices for the sandwich
178 SANDWICH PlATES

Table 5.2. The stiffnesses and the Poisson ratios at Isotropic solid plates and
Isotropic sandwich plates; R Is defined In Eq. (3.46).
Isotropic sandwich plate
Isotropic Isotropic Quasi-isotropic
sotid plate facesheets facesheets
1
A"" .,[},_
1- ..:
1!
(( l + (") J-(M l )Z (r' + rin
c.it)Z1' +P~1t> + (l't'~r,-.t\ E}
lY'"' FY
1!(1- t>!l 1-(ol )!
(r'(d'l2 + 1' (1l'l') R
,,1,,.;) " n + o-. +t>01 - C?~
,.
" "
pla te:

[~' ~, . J [~'
~ l
v' 0
v'
0
(A)= A' l IDJ = lY' l (5.38)

\Vhere Ai:1u and ffa0 are defined in Table 5.2.


When the core is isotropic in the plane paralle l to the faceshee ts from Eq. (2.40)
we have C., = 0, C.., = (C11 - C12)/ 2, and the shear stiffnesses are (Eq. 5.32)

"'
.lJJ
"'
= ..>22 "'
=,,) = d'
c
q, -2 q, s,, = 0. (5.39)

The sand '"ich plate n1ay also be treated as isotropic \\he n the top and bottom
facesbeets are quasi.isotropic lan1ina tes (page 79)consisting or unidire ctional plies
made of the san1e n1a.terial. For such sa.nd,vic.h plates the {BJ n1atrix is negligible.
the (A) a nd I DJ matrices are approximated by Eq. (5.38) (with the te mlS A~ and
fP" define d in Table 5.2), and the e le n1ents of the she ar sti ffness matrix are given
by Eq. (5.39).

5.2 Deflection of Rectangular Sandwich Plates

5.2.1 Long Plates


We consider a long rectangular sand,vich pla te \vhose le ngth is large co111pared
witl1 its wid th (Ly 4). The Jong edges may be built-in. simply s upported, or
free, as shov"n in Figure 5.8. The sandy,rich pla te is subjecte d to a transverse load
p (pe r unit area). This load, as well as the edge supports, does not vary along the
longitudina l y direction.
The deBec.ted surface of the sandwich plate may he assumed to he cylindrical
at a considerable distance fro n1 the short e nds (Fig. 4.4). The generator of this
cylindrical surface is parallel to the longitudinal y~axis of the plate. and hence the
5.2 DEFLECTION OF RECTANGULAR SANDWICH PLATES 179

L,
T.
f igure S.M: 'lhc different types of !iupporls along the long edge~" of Ji long 1111ndw1eh plate.

deflection of the plate ul ' and the ro tation x., z do not ''ary along y:
fJw> <>xx: = o.
-=0 (5.40)
ay Hy
We neglect the shear d efomrntion in tbe y- t plane (y1" = 0). Consequently, the
rotation of the no rmol is zero (Eq. 5.3):

Xg =0. (5.41 )

The equilibrium equ ations are ( Eqs. 4.22 and 413)

dV. 0 (5.42)
dx + p =
dM
tfx' - ".
v, = O. (5.43)

When the sandwich plate is symmetrical with re specl 10 the mid pla ne (I BJ = 0)
from Eqs. (S.12). (S. IS), (5.40), and (5.41 ) , we bave
<~Xx:
M, = - 0 11 - -
l>:r
(5.44)

Equations (S.42), (5.43). a nd (5.44) . together witb Eq. (5.2) . give


sandwich plate, synunetrical layup:
t/ 1 XJC
- Du --+ p = 0 (5.45)

111'''J
x,. S11
- - (dw" - x,-) = 0.
Du --+ (5.46)
tlx' dx
For a transnely loaded isotropic sandwich beam the corresponding equa
tions are (Eqs. 7.83 a nd 7.84)
isorropic sa11d1vic/J h~ani:

d'x+ p'= O
- ifl- (5.47)
d:cJ
o"'x
;;;;+ ~(dw
"JX - x) =0 . (5.48)
180 SANDWICH PLATES

\vhere 'El and ! are the bending and shear stiffnesses of the isotropic sandwich
bean1, respective ly, and p' is the load per unit length.
The equations describing the deflections of long sandwich pla tes and isotropic
sandwich beams are identical when in Eqs. (5.45) and (5.46), D,,. S11. and p
are replace d, respectively, by El, S, and p' . The re fore, the deflection of a long
sandwich plate (symmetrical layup) may be obtained by substituting the values of
Di 1 Su . and p for El. S. and p' in t he expression, given in Section 7 .3, for the
deflection of the corresponding isotropic beam.
\Vben the layup is unsymmetr ical, the e xpression for the mon1ent A1x can be
derive d analogously to the equation of a solid composite plate (Section 4.2.2).
He re \Ve only quote the result, which for sand\vich pla tes is

(5.49)

\Vhe.re Xxz is shown in Figure 5.3. The tern1 in parentheses is the bending s tiffness
parame ter defined by Eq. (4.52). Equations (5.42), (5.43), (5.44, right), and (5.49),
togethe r with Eq. (5.2), give
sandwich plate, t11Jsy111111etrical layup:
d'x
- "' ____:!i 0 +" = 0 (5.50)
dx'
d'x,. + S11
"' dx'
- (dw
dx -
)
x.._ = 0. (5.51)

The preceding e.q uations describing deflections of sandwich plates ( unsymn1e t


rical layup) become identical to the equations of sand\\ich be ams (Eqs. 5.47 and
5.48) when w. Su. and pare replaced, respective ly, by El. f. and /Y. Therefore.
the deflection of a long sandwich plate (unsymme trical layup) may be obtained
by substituting the ''alues of "1, S11 a nd p (or El. S, and p1 in the expression for
the deftectioo of the corresponding isotropic beam.

5.1 E.xan1pl.e. A 0.9ntlong and 0.2>1l1t'ide recta11g11/ar Sftndu1i<:h plate is 111ade of


a 0.02-rntlrick core covered on bot/J sides by graphite epoxy facesheets. The 1t1a
teria/ propenies are given iJ1 Table 3.6 (page 81). The layup of each facesheer is
[45\/012/ 45a and tire tlrickrtess of ead1 faceslreer is 0.002 m (Fig. 5.9). Tire
0-degree plies are parallel to the short edge of the plltte. The plate is either sinrply
srrpponed or brriltirt trlortg all forrr edges (Fig. 5.10). Tire plate is subjected to a

:1=2nun
d=22 mmI -- - . - .. .

---- -
1 c= 20mnl

! t. =2mm
1-..tgurc S.9: The cn>l>S scc.tion of the sandwich plate.an E xample 5.1 .
5.2 DEFLECTION OF RECTANGULAR SANDWICH PLATES 181

Figure 5.JO: The sandvoich plates in Example 5.1.

1u1ifonnly dis1ributed transverse load 500kN!in 2. Ca/cu/ote the 1naxin1ton tleflec


=
rion. The core is isotropic ( ~ 2 x Hl" kN!m2 "' 0.3). =
Solution. The tensile and bending stiffnesses are calculated from Table 5.1 (page
174) as follows:

430.34 65.47 0 ] kN
[A) = 2 [AJ' = 65.47 96.34 0 10-'- (5.52)
[ 0 0 72.02 Ill

l [ 52.16 7.96 0 ]
[D) =zd' [A)1 +2(D]'= 7.96 11.71 0 kNm, (5.53)
0 0 8.76

where [Al' a nd [DJ' a re give n in Table 3.7 (page 84) and d = c + r = 0.022 m. The
shear stiffness matrix is (Eq. 5.32)

[S~" 12
~"] =d'(S' 5]
s,, c ~' q,
=[18615
0
o ] kN .
18615 m
(5.54)

where (see Eq. 2.30and Table 2.10, page 18) q,


- -.
= C'"" = E
l (i ; ..,
'
= 769 231 kN/ m-,
1
Cio;=O.
We may treat this plate as long whe n (Eq. 4.19)

(5.55)

In the present proble m. L,./ L, = 4.5 and 3;:/ D11/ Du - 4.36. Thus, the preceding
condition is satisfied and the Jong plate expressions n1ay be used. The maxin1um
deflectio ns of the corresponding beam a re (Table 7.3, page 332)
_ 5 p' L4 p' L1
(ss) (5.56)
UJ = 384 El + SS

(built-in). (5.57)
182 SANDWICH PLATES

Figure 5. I I: Rectangular simpl)' supported


(ss) sandwich plate subjc.ctcd to trans\'crsc
load.

11le max.in1un1 deflections of the plate are obtained by replac.ing / . S. p' by


D11 , S11 , p (see page 180)

_ 5 pL~ pL!
w = - - - +~ (ss) (5.58)
384 Du 8Su
_ I pl~ pL!
w=---+~ (builtin). (5.59)
384 Du 8S11

With the values of D11 = 52.1 6kN-m a nd S'11 = 18 615~, a nd with L_. =
0.2 m, the maximu1u deflectio ns are

iJi = 0.000 200 + 0.000 134 = 0.000 334 m = 0.334 mm (ss) (5.60)
iJi = 0.000040 + 0.000 134=0.000 174 m = 0.174 mm (built-in).
(5.61)

5.2.2 Simply Supported Sandwich Plates - Orthotropic


and Symmetrical Layup
t.\ sin1ply supported rectangular sandwich plate \Vith dhnensio ns L,, and Ly is
subjected to a uniformly distributed load p (Fig. 5.1I). The layup of the plate is
orthotropic (page 176) and symmetrical wit h respect to the pla te's midpla ne.
Fo r a sin1ply suppo rted syn1n1etrical plate subjected to o ut-of-plane Loads o nly,
the inpla ne strains in the mid pla ne are zero (see Eq. 3.3 I ) as follows:

~.~ =0 Y.x~v =0. (5.62)

Substitution of Eq. (5.62) into the expression of t he strain e ne rgy (Eq. 5.22)
gives

s,,JIrr,...., I} tlytlx.
s,,
(5.63)
5.2 DEFLECTION OF REC:TANGULAR SANDWICH PLATES 183

For o rt hotropic sandwich plates Di = D,, = S 11 = 0 (Eqs. 5.33 and 5.34). With
these values and the expressions in Eqs. (5.5) and (5.6), t he strain energy becomes

U= ~11
' "' [ (ax")- Di +(ax ,,) D, + 2 ax8x. ,ax,,
' 0 1
12
2 8.t 8y I 8 )' l.

+ o(~x. , + ax,_ )1 o+(a: - Xx)1 S11 +(aw"- x.. )1s,,]d)dx.


~ b d ~ .

(5.64)
For an applied transverse load p (per unit area), the potential of the external
forces is (Eq. 4.56)
l., I.,

Q =- I.!
0 0
(pw")dxdy. (5.65)

For a simply suppo rted sand\vicb plate the deflection, bending n1oments, and
rotations of the normals along the edges are zero, resulting in the follo,ving bound
ary conditions:
x =0 and 05y _::; Ly
x= L., and 0 .::; y ::= L,.
Wu =0 at (5.66)
0 5 x _::; Lt and y =0
I 0 5 x _::: L.t and y = L.v

x =0 and 0 ::= y ::= Ly


1'1x = 0 a t
l x = LJ: and 0 ::= y .::; Ly.
(5.67)

o .::; x :;: Lx and y = 0


l 0 .::; x :;: Lx and y = Ly.
(5.68)

x=0 and 0 ::= y.::; L,


Xy;: = 0 at
l x=L, and O:;: y.::;L,.
(5.69)

o .::; x .::; Lx and y = 0


X.t;: =O at
l 0 _::; x 5 LI( and y = Ly .
(5.70)

The {ollo\\~ng deflection and rotations satisfy these conditions:

. LL
u,1 =
I J

'1
.
u. sin 11fX
-
,

sin
L
J 1f)'
- -
.
.

L
i I i- 1 ' Y

I I i;rx . j ;ry
X.t i = L L (X.t i)ij cos T
i I j I .T
sin T
>'
(5.7 1)

I I . i7TX j:ry
Xr
...
= '\" '"' (X1} Sin -
~~ "' 1' L.T
cos -
L}'
.
i I j I
184 SANDWICH PLATES

Table 6.3. ElemenlS ol the coefficient matrix in Eq. (5.73)

Fn = D11 (J; )+2(Du + 2At.) (t-)-?(f,)2 + ~2 ( ~) ..


FJ.1= - D11( ~); - (D11+ 2~} f ( )
2

F,,,=-Du(f,)' -(0.: +2AJ(\;;)' 1;


2 2
F = D (f ) + D(,6 (4;) +S,,
4.i 11

fo = (0.2 + 20,,.) (Z:. )( )

F,,=Du(~)' +0..(1!) +s,,


2

\\/here I and J a re the nun1ber o( tern1s, chosen a rbitrarily, for the su1nn1ations
and w;j, (X.iz.)i; and (Xy~),., a re unkno\\'l\S and are evaJuated by the principle of
stationary potential e nergy expressed ;1s

il(U+ n} =O
i) (X,,)1;

il(U+ l'l) =O
(5.72)
a(xd1;
il(U+ l'l)
--"----'"' = 0.
(}tJJjj

We substitute w. x.. ,. x," (from Eq. 5.7 1) into tbe expressions or U (Eq. 5.64)
and Q (Eq. 5.65} and perfo rm tbe diffe re ntia tions indicate d a bove. Algebraic
manipula tions yield t he (ollov.~ng syste111 ot silnultaneous algebra ic equations:

4
L L
p.lJ p.\'i ] { UJ;;
{ - pL.L,
l 'J,j ,- . .-
}
~ ~J F4$
<L }
Y F34 (Yxz);; = n Q' . (5.73)
[
!'.,,, F..s Fs, (Yr.l1; 0

\\here we have i , j = 1. 3, 5.. . . ( tn;i = (y.i;~)if = (Yy:) ;1 = 0 \\he n i o r j =


1

2. 4, 6 . . . .). The e le ments of the coefficient matrix are give n in Table 5.3 and (vd1;
and (Yr: );; a re defined as

j1f
(y,J
. ,,.. = -w1;
Ly - (x,..)
. I). . (5.74)

For each set of i , j values tbe tbree equations in Eq. (5.73) a re solved simul
taneously (or the three unkno\\ns we;, (Y.T~};; (y,,~};; The de Hec-tion and the rota
tions are then ca lcula te d by Eqs. (5.71) and (5.74).
5.3 BUCKLING OF RECTANGULAR SANOWICH PLATES 185

5.3 Buckling of Rectangular Sandwich Plates

5.3.1 Long Plates


\Ve consider a long recta ngular sand\\~ch plate \\hose le ngth L.v is large compared
with its v.idth L.,. The edges n1ay be built.in, simply supporte d, or free, as s hown
in Figure 5.12. J\ uni(orm co1npressive force Nxo is applied along o ne of the long
edges of the plate. This force, as v.ell as the edge support~. does not vary along
the longitudinal y d irection. \\1e are interested in the load at which the plate
buckles.
111e deflected s urface of t he plate 1nay be assumed to be cylindrical a t a con-
side rable distance from the short edges (Fig. 4.4). TI1e equilibriun1 equations are
given by Eqs. (4. I 60) and (4. 161) and for convenience are repeated here as fo llows:

dV. d 2wQ
' - N_,..., -d l =0 (5.75)
dx .\'
dM, _ I', =O. (5.76}
dx
We no\v consider a sand\vich plate that is symn1etrical \Vit h respect to the
midplane. For this plate the bending mo1nent and the t ransverse shear force are
(Eq. 5.44)

M.v = - Du - ax
ax,. (5.77)

Equations (5.75), (5.76), (5.n), toge ther with Eq. (5.2). give
sa11dwich plttte, syn1111etrical layup:
<f3x diul'
- D 11 - ' - 1VJto --=O (5.78)
dx 3 dx2
d'x - (dw
D11 d x' + Sn dx - x
0
)
= 0. (5.79)

~ J\i"
A ~
L, l ~
~ JV#O

' J -
== ~/\llO

~/\'~
l,
IE 11

"
Figure S.12: Long rc.ctangular sandwich plate s ubjec.ted to a uniform compre ssive edge. load a nd
the d1ffcrcnt types o f s upports along the long e dges.
186 SANDWICH PLATES

Fo r an isotropic sandwic.h beam the correspond ing e quations are (Eqs. 7.1 13
and 7.114)
i sorrop ic sand1vic/J f)eatu:

-d' x - d1 w
- El - . - N,o - =0 (5.80)
d x,, 2 dx

~ dd'x
r-1 x' + ,""(dw
dx - X) = 0. (5.8 1)

\Vhere EI and S are the be nding a nd s hear stiffnesses o( t he sand,vich bean1 and
,VJ.0 is the con1pressive load (per unit length).
The e quations describing buckling of long sandwich plate.s (s ymmetrical layup)
and isotropic sandwich beams are ide ntical whe n in Eq~ (5.78) and (5.79) D11 ,
5;,. and N.,o are re placed, respectively, by/. S, a nd N_,,,. There fore. the buck
ling load (pe r un it length) of a long sandwich plate (symmetrical layup) may
be obta ined by s ubs tituting the va lues of D11 a nd S11 for El and S in the ex
pression for the buckling load of the corresponding isotropic sandwich beam
(Section 7.4}.
ll was shown in Section 5.2. I (page 180) that whe n the layup o f the sandwich
pla te is unsymme trical the defle ction may be obtained by substituting the values
of 'II, 5; 1 and p for El, S, a nd p' in t he e xpression fo r the deflection of the
co rresponding isotro pic sand,vich beam. Similarl)1, the buckling load of a Jong
unsymn1e trical sand \\ich plate 1nay be o btained by substituting the values o f \JI
and S11 fo r El a nd S in the expressio n for !he buckling loa d o f the correspond ing
isotropic bean1 ('vhere \II is given by Eq. 4.52).

5.2 Example. A 0.9111/ong and 0. 2-111~1vide rectangular sand1viclr plate is 111a1fe


of a 0.02-in ..thick core covered on bot/J sides by graphite epoxy facesheets. The
111aterial properties are given ilr Tt1ble 3.6 (page 81). The layup of eaclr fi1 c1.~sheet
is 1 45~/011/ 45~], a11d tire 1lrick11ess ofeaclt f aceslreet is 0.002 m (Fig. 5.9). The
Odegree plies are parallel to the short edge of rite plate. The plate is either siJnply
rnpponed or b11iltit1 alo11g all four edges (Fig. 5. 13). Tire plare is subjected to
1111ifor111 co111pressive loads along the long edges. Ca/culttte the btu:kliug load. The
coreis isotropic ( '"" = 2 x 10" k N!m 1 , v, = 0.3).

L~= 900 nun L11= 900mn1

.J,tt t !. .j. .J, .J, .J, .j. .J, .J,.J,tt i !. .j.


" E<

I~-.,, I~_,
SS y y
SS .SS
SS

ttt tt t t t t tttttt t
x iV1!0
x ..,.
!V,o
figure 5.13: 'fbe sa nd\vich plates in B:ta mple 5.2.
5.3 BUCKLING OF RECTANGULAR SANOWICH PLATES 187

Solution. The pla te may be treated as " long" (Example 5.1 . page 180). The buck
ling loads of the corresponding beam are (Eqs. 7.175 and 6.337)

-
Na=
( L2
rr2/ +f
1)- 1
(ssl (5.82)

- ( 4L2 I)_,
1V0 = !rlEJ + S (builtin). (5.83)

The buckling loads of the plate a re o bta ined by replacing El, Sby D11 , S11
(see page 186} as follows:

N... " = ( 'LD! +


1r ll
S! )-'
ll
(ss) (5.84)

4L2 I ) _,
N.. " = ( ,,., + (builtin). (5.85)
011 511
With tbe values of D 11 = 52.16 kN- m and S11 = 18 615 ':;.',(see Eqs. 5.53 a nd
5.54) and L_, = 0.2 m, t he buckling loads are
1
I l ) -
N.,." = ( 'i21i7o + i8'6i5 = 7 609 kN/m (ss) (5.86)
1
I l ) -
N"" = ( 51 481 + 18 6 15 = 13 672 kN/m (builtin}. (5.87)

5.3.2 Simply Supported Plates - Orthotropic and Symmetrical Layup


We consider a rectangular sandwich plate witb dime nsions L_, and Ly (Fig. 5.14).
The layup of tbe plate is ortbotropic (page I 76) and symmetrical with respect
to the plate's m idplane. All four edges of the plate a re simply s upported. The
sand,vich plate is subjected to uniforn1ly distributed compressive loads Nxi>and
Nyo along the edges. These loads are increased proportionally, that is. the loads are
AN.dh 'J.1\~ 0 , where). is the load parameter. For a buckled plate the load paran1eter
is denoted by An.
The expression for the strain energy is given by Eq. (5.64).
For a plate subjected to in plane loads only, the potential of the external forces
is (Eq. 4.108)

I
I'!= 2. rf /..,Ly

N, ax
,
(aw")- + N,. (aw") ay
2

dydx, (5.88)
" 0
188 SANDWICH PLATES

\Vhere Nx, 1\/_v a re the in-pla ne tensile (orces related to the in-plane con1pressive
forces J..1\~10. ).N_.<1 by
(5.89)
The deflection is assumed to be of the form given in Eq. (5.71). By s ubstitut-
ing Eqs. (5.71). (5.64), (5.88), and (5.89) into Eqs. (5.72) and by performing the
diffe rentiation, after algebraic manipulations we o btain
2 2
44
--
4
([F.F,," F,..F.is] - ).
F.u F.,
p.,, F45 p.,,
["""(\!) +N,11(1;)
o
0 ~ ~])
x
{
w;;
(Yd;; }
= {o} 0 .
(Yf'l;1 0
(5.90)
where (Y.d;; a nd (YY<);; are defined by Eq. (5.74) and F;; a re given in Table 5.3
(page 184). When the load set is under the crit ical value, the deftection of the plate
is zero. \Vhe n the pla te is not buckled, the deHection of the plate is zero, \Vhereas
for a buckled plate it is nonzero. The values of). for the buckled pla te (de noted by
/ , 0 ) are the e igenvalues of E q. (5.90} and a re obtained by setting the detern1inant

of the coefficient matrix: to zero. This gives


F33 F,. Fu
F,, F"' F"
F.s F.s!o F.~s
(5.91)
Ip.,
p._, F.,I
F.~s

\Vhere I I denotes the detern1inant. The values of ();i)cr are calcula te d for di(..
fe re nt sets of i and j, (i , j = 1, 2 . .. . ) . The lo,vest resulting value of (A;i )0 is the
value of intere.s t.
\Vhen t hesand\vich plate is isotropic, \\'e obtain ().;i )cr by re placing Du. Du,
V,,,, in Table 5.3 by (jw, ,,; D"", and ( 1 - v;'") D;w /2 (see Table 5.2, page 178)
11,
and S S12 by S (see E q. 5.39). With these substit ut ions Eq. (5.91) simplifies to

('-)""-
"11 er -
(t.)'+(t )'
2 .
(-1
2 / D.n
+S
--1)-1 (5.92)
N..,, ( t;) + N,<> (f.)
\vhere 1\ 'D. 0 is defined as

(5.93)

5.3 Exan1ple. A 0.9111 /ong and O.b n1lvide rectangular sand1vich plate is 1nade
of a 0.02-in thick core covered 011 both sides by graphite epoxy faceshee1s. The
o.J ou1..N.1nu ur nc.1,,1>1.nuuLAn ~AR u wn..n ruur~ ..
,
L1 000 nim
~-- ->I
i iiiiii ii
. .. "
d= 22nunf
l C
! t l mm
c 20 mm
I t l mm
tttttttt
.v,.
Agurc 5.1.S: 1lac n ndw.ch pl.ate 1n Example SJ.

111attrial pro~rr;t!s art given in Tahlt 3.6 (pagt 81 ). Tht layup of e11cl1 foceshttl
is (4S~/012/ 4SI (. 011d 1/Je t/Jick11ess of each faces/rut is 0.002111. The 0-degree
piles are /Jltrttllel to t/Je s/Jon edge of rite plate. The plate. sh11ply st1ppor1etf along
ttll four edgej' (Flg. S. ISJ. is subjec1etf 10 unifor1n co111pressfve /()(Jds along 1l1e long
edges. Cttlculote the buckliug load. Tire core is isorroplc ( ~ = 2 x lff' kNh11 2 v, =
0.3).

Solution. We SCI N, 0 = 0 in Eq. (5.91) and write

F"
P,..
F,..
F4.1
F"I
Fu
Fn F45 F,,,
(5.94)

I~: ~:I
The poramcl<N F,1 are given in Table 5.3 (pa.ge 184) as follows:

F.u= 011(~:)' + 2(Du+ 20..)(~)'(t )' +ou(t;)'


f,,, = -o,, (!!!..)"
I.,
' - (Di2+20..)!!!..
L, Ly
(i" )'
f,.= - Du (i")'
Ly
- (012+2066)(!!!..)'
L,
/tr
L.,
(5.95)

F., = Di1(~)' +o..(t)' +s11


F45=(Du+20,.) (:) (Z)
F,,= Du(t )' +o..(:)' + Su.
The e lements of the s1iffness matrices are (Eqs. 553 and 554)

011 =52.16kN m Dv. = 11.71 kN . m Di1 = 7.96kN m


D.., = 11.76 kN m
kN
S11=18615-
m
s,, = 18 615-kNm
190 SANDWICH PLATES

By s ubs tituting these s tiffnesses a nd L, = 0.2 m, Ly = 0.9 m into Eq. (5.94),


\Ve obtain the fo llowing values o( 1V.,o. (Ai1) 0 :

i \ j 1 2 3
1 7 965 9 070 11 041
(5.96)
2 13 875 14 491 15 544
3 16195 16668 17 470 .

The s mallest value is N,o (1.n);; = 7 965 kN/m, which corresponds toi = j = 1,
T hus, the buckling load is

N.w = (l.")11 N,. = 7 965 kN/m. (5.97)

In Exa111ple 5.2 \Ve treate d this sandv,.ic.h as a long plate and obtained the
buckling load N= = 7 609 kN/m (Eq. 5.86). This is within 5 percent of the value
given by Eq. (5.97).

5.3.3 Face Wrinkling


We consider a sand\\ich pla te. The top and botton1 faceshee ts are ide ntica l, and
each facesheet's layup is syn1n1etrical 'vi th respect to the facesheet's midpla ne.
The sandwich plate is s ubjected to inplane fo rces N,, N,., N,., (Fig. 5.16, left).
Since the inplane s tiffnesses of the core are ta ken to be negligible, the in-plane
stres:ses in t he core n1ay be neglected 'vit h respect to the inplan e s tresses in the
facesheets. Correspondingly, the io ..plane forces (per unit length) in t he faces heets
are
Nr = i\:lY f = t.b. (5.98)
x.v 2
T he superscript f denotes e it her the- top or the botton1 (acesheet. Under these
inp lane forces the facesheets n1ay beoon1e \Vavy (Fig. 5.16, right). 111ese v.aves
are precursors o( local buckling, a nd the loadset a t which these waves first occur is
taken as the buckling loads. The waves n1ay propagate in t \\'O directions, although
gene rally the v.aves in o ne dire ction don1inate. In our a nalysis \\'e consider only
\Vaves in one direction.
The \Vavelength 21 depends on the material and on the geon1etry of the sand
\Vicb plate. Here, \Ve consider l\\'O cases: (i) t he wavele ngth is "sho rt," such that

N_,,
Figure S.16: Pace ,~rinkling of sandwich plates.
..
,

>

I. and (ii) the wavelength is ..long such that// Ir I (11 is the thickness of
/ / /1
the plate as shown in Fig. 5.2).
111e loads thnt con1ribute to the \.\'aviness of the aceshec1s t\re the inplane
load perpendicular to 1he wave ~ and the norma l load "'~ (corresponding to the
stress o-, ) exerted o n the facesheet by the deforme d core (Fig. 5. 17, lcft). Under
these londs the equilibrium e quation of the racesheet isl
l)duJI a'lwl r
11 (5.99)
"' ae' + Nf ae' = ''

where Wt is the bending stiffness of the faceshee t in the~ d irection - 1hnt is, the
11 e le me nt or the matrix I DI in the E-1' coordinate system. Transformation of the
matrix [DI follows the transformation rule of the matrix (QI given by Eq. (2.195).
Thus.. v;e have

"' = (0.tl;-, = d.t cos' cz + D',, sin4 cz + (20., + 4D'.,.)cos' a sin' a


+ 4 cos' cz sin a V:, + 4 cos a sin' a 0:, f = t . b. (5.100)

where a is the angle bctv.-een the X and Saxes a nd '1., arc the clements of the
bending stiffness matrix of the facesheets in the .r- y coordinate system.
The parameter'"' is the out--0-plane displaceme nt (deHcction) of the aces beet
(Fig. 5.1 7, right).

(5.101)

where I.Vo is the amplitude or the deflection aod I is the hal( buckling \\ avclength. 1

1l1e parameter Nt
i.s the in-plane force in the facesheel in the (direction and is
obtained from the in-plane fore.es J\I.! . .,,.. and N!,
by transformalion. By using the
stress transformat.ion in Eq. (2. 182). we can define /\1
by the fo llowing equation:

JVl =- {N. cos1 a + JV!. sin1 a+ 2~1 cos a .sin a) f = t. b . (5.102)

\Ve are interested in 1.he value of~ at which the 'vavinc-.ss firs t :.rises.
Isotropic con - composit' fatahtns. \Vben the core is isotropic. the out--0!-
plane stress in 1.he core a : varie.s across the. thickness as illustrated in Figure 5.18
(left). When the wavelength is small, the stresso-, varies. as illusiratcd in Figure 5.18
(middle). Since the s tresses vanish away from the facesheeu. the problem may be

1 S. P. l l 1n011hcnkc1;ind J, Otre, Tl1t><JJ'f Of las1ieSmhili1y. 2nd edition. ~1c0raw Hill . New York, 1961,
p. 2.
192 SANDWICH PIATES

F'igurc: 5. 18: 1be slrcss er: d1stribut1on in an isotropic core (left) and 1n an isotropic core with
short WU\'clc.ngth (middle); buckling of a plate on a n elastic foundatjon (right) .

treated as a plate on an infinite e lastic founda tion (Fig. 5.18, right). Then, the stress
at the core faceshee t interface 0'% isl

I a . 1'~
'1 = - - u; 11 sin - where (5.103)
' I I
\Vhere .:and Ve are the Young modulus and Poisson's ratio of the isotropic core.
Equations (5.99), (5.101), and (5.103) yie ld

!(4 . f(~ N.' :r1 . 7"~ (I !(~


\Iii- WI) -
4
Si ll - - ~ u.'o - stn -
2
= - - Wo S ill ~. (5.104)
' 1 I ' 1 I I I
\Vhen the facesheet becomes \\'avy, wo is not zero. The values or NJ: corre-
sponding to uJo 0 are the buckling loads (JVl)cr These buckling loads are given
by the nonuivial solution o( E q. (5.104) and are

I 1fl a Jl
isotropic core
(i\~t, = "'~ 11 + Ttr1 composite faceshee ts (5.105)
s hort 'vave.

\Vhen the v.avelength is long, the solution is obtaine d by assuming that the
stress distribution ":. is uniJonn (Fig. 5.19, n1iddle) and is approxi n1ated as "J
I r ~
" ' = - a= (5.106)
' - l' = -w -c/ 2
where.,= w1/ (c/2) (Fig. 5.19, right). SubstitutionofEqs. (5.101) and (5.106) into
Eq. (5.99) gives

Jr4 . .TT~ I 71'2 . 71'~ , . ]'(~


\Jli- w.,- sin - - N U.'(1- sin - = - - wc1s in - . (5.107)
' I' I ' I' I c/ 2 I
The nontrivial solution is

isotropic core
r rr' ~ I'
(N)
~a
=llf,-+
' /2
- -
c/ 2'lr2 con1posite faceshee ts (5.108)
long \Vave.

2 H. C'.r. Allen. Aualy.,is 1111d Dd i>?JI of Strue1ur11l Sa1ulwicl1 Pa11el.v. Pcrs:unon Press. Oxford. 1969,
p. 158.
5.3 BUCKLINGOF RECTANGULAR SANDWICH PLATES 193

- -
Figure 5.19: The stress at dislritiution in an isotropic core (lcfl): in nn isotropic core \\ith long
W<t\'ck.ngth (middle): buckled shape: (right).

We are interested in the lo\\est value of (JVl)cr This value is obtained by setting
the derivative of ( Nf)o \vilh respect to I equal to zero as follov.s:

d(N/)a _
di - 0 (5.109)

Solution of this equation results in the haJf.\\ravelength lr:r corresponding to


the lo\\est buckling load. Substitution of le into the expressions for (1V:)a gives
(~)er.min T he results a re sun1marized in the top half of Table 5.4.
TI1e value of ( 1Vl)a.min depends on the directiona in \vhich the wave propagates
because \Jlt depends on a (Eq. 5.100). The wave direction is not kno\vn a priori and
must be detern1ined. This is accon1plished by obser,~ng that ~ is a lso a function
o( Ct (Eq. 5.102). The wave will firs t appear in the direction in which the ratio
(.~)er.nun/ Nl
is the sn1allest. This ratio is calcula ted for diffe re nt angles, and the
angle that results in the s1na llest ratio is lhe-desire d et.
Ho11eyco111b core - co111posite facesheeL'i. \Vhen the sand\vich plate consists
o( a hone ycomb core \\ith composite (aceshee.ts, the stress at the core (acesheet
inte rface may be approximated by Eq. (5.106) for both short and long wavelengths.
The buckling loads are identical to those given by Eq. (5. 108) and are

Jr? Ee /2 honeycon1b core


(N!) = -v ...., + - -1 composite faceshee ts (5.1 JO)
0 /- c/2tr
short or Jong waves.

Table S.4. Face wrinkling of sandwich plates with eithe< a hooeycomb or an isotropic core. The
constant a is given by Eq. (5.103) and G, = 11.~~

tsotropic core Honeycomb core

Shortwave Long waw Short or long wave


( " - j2+{11?
Comp<>Sile /\if }c1.1mn = I.) ~ (N!)
l ( f.llltll -
-
2 \ ' "'l ,J?
!is.
faceshe.ets la = j2""'11~ I
.:r=7f T
j+((n

)SOLrOpic
(N')
f .;r n11D -
- I ..'' \1 "'"'"'
J( l- 1j) (J--1t)!0+V.:)
(l'I,) ( l , IUlll
=I
1
j
1.1&
r(l-f)
facesheets ::::: 0.191.~
I "''t;''
er= . 6(1- f)
I
~' = ;r
J14&(1-"f)/- c
Eo ,
194 SANDWICH PLATES

\vhere F..:: is the Young n1odulus of the core perpendicular to the plane of the
sand,vich plate.
l11e lo\\'est value of the buckling load and the direction a in which the \\ ave 1

propagates is de te rmined as previously above fo r pla tes v.ith isotropic core and
con1posite faces heets.
Isotropic core - isotropic facesheets. We now consider sandwich plates v.~th
isotropic core and isotropic faceshee ts. The bending stiffness o( an isotropic face
sheet is (see Eqs. 3.42 and 5.100}

II', t'' ')


= d'" = 12 I - v; (5.111)

\Vhere I is t he t hickness of the faces heet and the s ubscript f denotes the facesheet.
When both the core and the facesheets are isotropic and the faces heets buckle
\vi th short v.aves., the Jo,vest value of t he buckling load is obtained by substituting
Eq. (5.11 1) into Eq. (5.105) and by performing the differentiation indicated in
Eq. (5.109). The result is
isotropic core
isotropic facesheets
s hort '''a\'e.
(5.112)

where G, is t he shear modulus of the core (G, = Ed2 (I + ,,, )). f and c refer to
the facesheets and the core. By the definition o( the inplane force (Eq.5.7) the
tern1 in the bracket is the critical stress.
By neglecting the Poisson ratios (11, = "' = 0). Eq. (5.112} reduces to

isotropic core
isotropic faces beets (5.113)
s hort \Vave.

Hoff a nd 1'.lautner;l obtained this expression with the value of the constant
0.91 instead of 0.79. Ho\\ever, (or practical use they reco1n me nded the value 0.5.
\Vhen bo th the core and the facesheets a re isotropic and the faceshee ts buckle
witl1 long waves, E qs. (5.108), (5.111), and (5.109) give
isotropic core
isotropic facesheets (5.114)
long \\'ave.

For an isotropic facesheet the bending s tiffness does not depend on the di rec
tion. Consequently, buckling \\'aves occur in the direction in \vhich the con1pressive
stress is n1aximum.

l N. J. Hoff and S. E. ~1au tncr. Buck.Lins: of Sandwich l)pc.Pnncls. lt>111?1al of1/Je A t r<Hllutical s~ie11c~s.
Vol. 12, 285- 297. 1945. Sec 11Jso in J. R Vinson, StJ1uJwidJ S1nu.111res t>f f.fOtropic dnd Ct>tU/l'Mit~
it1at1rials. 1Cchnon1k . l ancas1cr. Penns)'lvnnia. 1999, p. 239.
5.3 BUCKLING OF RECTANGULAR SANDWICH PLATES 195

L,=- 900 mm

l = 2mm
d = 22 nun
ID f
c=-20 mm
l=- 2 mm

:r. iV~
Figure S.20: The-sand\\ic:h plate in Example SA.

Ho11eycon1b core - isotropic facesheet,f. Lastly, we conside r a sandwich plate


with hone ycomb core and isotro pic (ace.sheets. The ha lf wavele ngth /0 may be
short or long. In this case Eqs. (5.1 10), (5.111}, a nd (5.109) yie ld

honeycon1b core
( ''' '' ) [ 2 EtE, isotropic facesheets (5.115)
~ er.mm =I 3 c (1 - vf)
s hort or long wave.

This equati on is identical to that give n by Heath.4


The lowest buckling loads and the half wavele ngths correspo nding to these
buckling loads are sun1marized in Table 5.4.
These expressions take into acco unt face \\rinkling in one direction only. They
are accurate when only un iaxial load is a pplied to an o rthotropic plate in one
o( the orthotropy direct ions. In case o f biaxial loading these e xpressions n1ay
overestima te the buckling load. For most practica l con1posite sand\vich plates the
error in t he buckling lo ads give n by these e xpressions is less than 30 pe rcent.

5.4 Exan1ple. A 0.211lloug turd 0.9nJ 1vide rectangular sand1viclr plate is u1ade
of a 0.021n-thick core covered on both sides by graphite epoxy facesheets. The
nrateria/ properties are given in Table 3.6 (page 81). The layup of each fi1cesheet
is ( 45\/012/ 45\1 mu/ tire thickness of eaclr facesheet is 0 .002 111. Tire Odegree
plies are parallel to t11e slron edge of the pltlle. The plare is sinrply supported along
all four edges (Fig. 5.20). The plate is subjected to u11idirectional iHplane loads N,.o.
Estituate tire load at whiclr tire facesheet wrinkles. The core is isotropic ( : = 2 x
106 kN!m 2, "<= 0.3).

Solution. Let us first assun1e tha t the facesheet \\Tinkles paralle l to the y -a xis 'vith
long waves (a= 0, Fig. 5.16). In this direction "11 is (Eq. 5.100)

"'' = Ii,, = D11 = 45.30 N m. (5.11 6)

' \V. G. Healh. Standwich Construction, Part 2:Thc 0ptimum Desi.g:n orFLnt Sandwi.::b Pnncls. Aircut]i
Engi11e.eri11;: . Vol J2, 230-235, 1960. Sec also in J. R Vinson. Stu1dwi('/J Srruaur..s o/ t.fotrop ic and
Co,11posit~ .\tatt'rial.~. Tcchnomic, Ltlnc.aster. Pcnnsyl\'ania. 1999. p. 2.39.
flAlt:i
'"" ~AN.UWltiH

T he value of D[1 is given in Table 3.7 (page 84). T he lowest buckling load and the
corresponding half wa velength are (Table 5.4, page 193)

r
( N,) .
~a. mm
=2 Hi ,
"1; -c
12
= 19 006 -kN
m (5.117)

la= "y.~
T =0.0069 m = 6.9 mm. (5.1 18)

Theassun1ption that the \\'ave is long is valid \vhen/0 is large compared \\ith the
core tl1ickness. Here, In is only abo ut a third of the core thickness and, therefo re,
the long.wave approximation is invalid.
Let us no\\' assun1e t hat the \\'ave is short. With this assun1ption \Ve have
(Table 5.4, page 193)

I 2'4'~ll 2 kN
( N,) . = l.5 ' -
-; er.nun
, - = 34 116 -Ol
rr-
(5.119)

la = / r~:"' = 0.0063 m = 6.3 mm. (5.120)

where (Eq. 5.103)


21' , N
a = (3 -v, ) ( l + vc:) = 35 80 >< 109 2m . (5.121)

111e 'vave may be assumed to be short whe n lcr is significantly sn1aUer than
the core thickness c. Here, the ra tio 10 / c is about one ..third. Thus., the sho rt \\'ave
approximation is unreasonable.
In this proble n1, the \\'ave cannot be t reated as e ithe r Jong or short. 1-\ s a con
servative estimate we take the lo\\er of the two buckling loads given by the Jong
and short-wove approximatio ns. Thus, the lowest buckling load is (see Eq. 5.98)

N.m = 2 ( N,')a.nun
'i
. = 2 >< 19 036 = 38 072 -kN
m (5.122)

5.4 Free Vibration of Rectangular Sandwich Plates


In this section we obtain the naturaJ frequencies f and, hence, t he periods of
vibra tion ( T = 1//) and the circular fre que ncies (w = 21r /) of sandwich pla tes.
In t he analyses that follows \\'e assume that the plate is freely vibrating a nd is
undamped.

5.4.1 Long Plates


We consider a long rectanguJar sand\vicb pla te \Vhose le ngth is large con1pared
'vi th its \\idth ( Ly >> L., ). The e dges may be built in, sin1ply suppo rte d, o r free, as
illustrated in Figure 5.21.
5.4 FREE VIBRATION OF RECTANGULAR SANOWICH PLATES 197

figure S.21: lhc d1ffcrc:n1 types of supports aloog lhc long cdgc1 of long. a.:and-.ich plates undct
gomg free und:ampcd \'lhrahon.

111e dcBcctcd urface of 1he sandwich plale may be assumed to be cylindrical al


a conside rable distance fro1n the short e dges (Fig. 4.4). TI1e equilibriun1 e quations
are Eqs. (4.191)-(4.193) ,
nv,.
-ti.< + 1>(2"/)1 w" = 0 (5.123)

- - - V, = O,
t!M,.
(5.124)
d,t
vlhere ul' i.s the dellection and p is the mass per unit area or the sandwich plate.
We now coraider n sand"ic.h plate that is symmetrical with respect to the mid
plane. The bending moment and the transverse shear force acting on the sandwich
plale arc (Eq. S.44)

M, =-Du ax,, (S.125)


ax
Equation. (S.1 23). (S.1 24), and (5.125), logethe r wilh Eq. (S.2). give
so11dh1ich plall!, syuuuetricol Jayu1>:

tfl X1.: ,
- D11 - d 1 +p(2rr/J-111 =0 (5.126)
x
D,, -rl';<.:
r1.r2
-
+S11 -d
x
(dw" - x,,) =0. (5.127)

Fo.r o vibrating snnd\vich beam the correspondjng cqu:uions are (see


Eqs. 7.178-7.180."' = 2rr /)
isotropic s111rd1vich bta111:
-d' x ,
n-.,
- 1dx-' + p'('lJf =0 (5.128)

fitPx + s('lw - x) = 0. (5.129)


dx' d.r

"'he re El and S:ire the bending and she.ar stiffnesses o f the sand\vich beam and
p' is the inass per uni1 length.
111e preceding set of equations describing the vibration or
long. sandwich
plates (sy1nmc trical layup) and isotropic sandwich bea1ns are identical \vhcn Di 1.
S, 1 nnd p nre re plncccl. respectively. by El. S. nnd p'. Therefore. 1he natural
198 SANDWICH PLATES

Figure 5.22: lbe sand\''ioh plate.'> in Example 5.5.

frequencies of a Jong sand"foh plate (symmetrical layup) may be obta ined by


substituting the values of Du. Su. and p (or i. f. p' in the expression given in
Section 7.5 for the natural frequencies of the corresponding isotropic sandwich
bean1.
The na tural frequencies o( a long unsymmetrical sandwich pla te n1ay be ob
tained by substituting the values of \IJ (Eq.4.52), S11, and p for ifi. S. p' in the
expression for the natural freq uencies of the corresponding isotropic sandwich
be.an1.

5.5 Example. A 0.911llong and 0.2--nllvide recu111gular sand1viclr pltue is 111ade


of a 0.0211tt/Jick core covered on botl1 sides by graphite epoxy facesheets. The
1uaterial prope!rties are given ilt Table 3.6 (page 81). The lttyup of each facesheet
is [45V012/ 45\L a11d the rlrick11ess of etrclr faceslreet is 0.002 m. The ().degree
plies are parallel to the slron edge of the plate. The plare is either situply supponed
or builrill llloug all four edges (Fig. 5.22j. A uu1for1u n1ass is over rhe plate such
that for the courbined 111ass-plate sys1e111 p = 200 kg/nt2. Calcttlate the circul<tr and
tire naruralfrequencies. The core is lfotropic (-..: = 2 x 106 kNhn 2 v<'. = 0.3).

Solution. The pla te may be tre ated as Jong'' (Example 5.1 , page 180). The circular
frequenc.ies of the corresponding beam are (Eq. 7.243. Eq. 6.398, and Table 6.13,
page 308)

W;=
p' L'
(-
p' L'
-.- +=--,
)-1' (5.130)
El,. S" s,
\Yhere

Iii=" 21', 3ir.. (ss) (5.131)


,,.; = 4.730, 7.853, 10.996. . . . (built-in) (5.132)

and

.. . = '" 2tr, 31'. . (ss or built-in). (5.133)


5.4 FREE VIBRATION OF REC:TAHGULAR SANDWICH PLATES 199

L, L,
Figure S.23: Rectangular sandwich plate. \\ith simpl)' supported edges.

The circular fre quencies of the plate a re o btained by replacing EI. S, p' by
D 11 , S11 fJ (see page 198) as follows:

w= (
fJ
-
L'
7 +-=--+
fJ L' )-1. (5.!34)
Du /Ji Su ILS;

With t he values of D11 = 52.16 kN m and S11 = 18 615~ , (see Eqs. 5.53 and
5.54) a nd \\ ith L.t = 0 .2 n1. the first three n1odes of the c ircular freq uencies of the
1

plate a re

w1 =3064 w2=8214 w.1 = 13 344 1/s (ss) (5.!35)


w, =4 233 Wz =8945 w.1 = 13 791 1/s (built in). (5. 136)

The corresponding na tural frequencies are ( f = w/2tr}

/1= 488 /i = 1 3f17 /J = 2124Hz (ss) (5.!37)


/1= 674 j, = 1 424 fi = 2 195 Hz (built-in). (5.!38)

5.4.2 Simply Supported Plates - Orthotropic and Symmetrical Layup


A simply supported rectangular sand,vicb plate 'vith din1e nsions L.t and L1 is con
sidered (Fig. 5.23). TI1e layup of the plate is orthotropic (page 176) a nd symmet
rical \\ith respect to the plate's n1idplane. The plate is undergoing free undamped
vibration. The deflection of the plate is (Eq. 4.189)

w" = w" sin (2tr / 1) . (5.!39)

Analogously, we e xpress the rotations of the norn1als as

X.n x_., sin (2ir / 1)


= (5.140)
x,., x,., sin (2rrf1) ,
= (5.141)

where W", :X.tz and Yyt are as yet unknown funct ions of x a nd .Y T hese func-
tions must be c hosen s uch that W' , Xxz and Xyz- satisfy the boundary conditions
given in Section 5.2.2 (Eqs. 5.66-5.70). To dete nnine t hese functions we introduce
Eqs. (5.139)-(5.141) into t he e xpression for the strain energy given by Eq. (5.64).
This results in

U= V sin 2 (2tr f t) , (5.142)


200 SANDWICH PLATES

'vhere U is defined as

li = _I "- )2Di (uX.v:.


1.,!1, [( oXx:: -ay )' D.. - -
oxx:: uX >'::. D ,
2 !0 ax I + .l + 2 ax ay I.
I)

( ax8)~- + axa.:,-)' fu + (aaxiii" - x.,.)' Su


_ + (a;;;"
ay - x,,) Sn Jdxdy.
2
_
+
(5.143)

Following the s te ps used in the analysis of free vibration of thin plates (Sec
tio n 4.4.2), \\'e arrive at the follo\ving expression for the natural frequency:

(27rf)' = - L,
-L, - - -
u (5.144)
! Pf f pW'~dydx
u (t

\Ve adopt the following expressions for WU. Xx::. X,.::.:

I J , ,
- '\''\' . / 1fX /1')'
w = L., L- wii sin - sm - -
,_, ;- 1 LI( L,.
I J . .
- '\''\' ( 11fX . /1' )'
x.T:; = L- .L- X.i::Jij cos T SID T (5.145)
1..,1 i I X .Y

_ I J . i7rX j71' )'


x,., = LL (x,.J;; sm L
1 1 j I .T
cos L.
y

With these expressions the deHec.tions and the ro tations give n by Eqs. (5.139)-
(5.141) satisfy the boundary given by Eqs. (5.66)--(5.70). The unknown coefficients
w;;, (X,.);; - (X,..);; are determined from the conditions (Eq. 4.217)

a/ =0 (5.146)
a(x.,J,1

1-\ fter algebraic n1anipulations 've obtain

~ ~]) {<:X'.),;}= {~}.


0 0 (y,.,)1; 0
(5.147)

where (Yd;; a nd (y,.),; are de fined by Eq. (5.74), P;; are given in Table 5.3
(page 184), and A is defined as

(5.148)
5.4 FREE VIBRATION OF RECTANGULAR SANDWICH PLATES 201

In the c-ase of free vibration the deflection is nonzero. For nonzero deftec..
lions. Eq. (5.147) is sat isfie d when the determinant o( t he ma trix in the paren
theses is ze ro. At this condition), signifies the e igenvalues of Eq. (5.147), and \Ve
obtain
F.n F,. F.is
f.,4 F" F.s
F_,~ F., Fss L.xL>'
A;1 =
r.
F._,
F.,I
F.~s
4
(5.149)

The values of A;; are calcula te d for diffe re nt sets of i a nd j. (i , j =I. 2, ... ),
o( which the)o\vest value is o( interest.
The natural fre que ncies are calculated from Eq. (5.148):

f;, = ;jp~~. (5.150)

When the s.and,vich plate is isotropic, we obtain ),;; by replacing Du. Dii,
D.. in Table 5.3 (page 184) by /jw, ,,w f>', a nd (I - v"") a w;2 (see Table 5.2.
page 178) a nd S11 , S12 by S (see Eq. 5.39). With these substitutions Eq. (5.149}
sin1plifies to

.. -- L.,4L,.'1r
A,, '[(.!....)'
L
:~
+ (1.)']
L
)'
(N- 1 + s ' )- 1'
D.ij (5.151)

where 1Vv.;; is defined as

No.;; = "'d'" [ (~)' + (l)'] (5.152)

5.6 Exan1ple. A 0.9111-/ong aud 0.2111-1vide recu11rgu/ar satulu:iclr plate is 1nade


of a 0.02nt.fhick core covered on both sides by graphite epoxy facesheets. The
1naterial properties are given hr Table 3.6 (page 81). The layup of each fatesheet
is (45\/012/ 45\t tmd the rlricklfess of each faceslreet is O.<l02 111. The 0-tlegree

CJ I
1 = 2 nun
d = 22 mmI c= 20 mm
t-= 2 mm

Figure S.24: The sandv.ich plate in Example S.6.


202 SANDWICH PLATES

plies are parallel ro the short edge oftire plate. The plate is situply supponed along all
four edges (Fig. 5.24). A u11ifor111 1na..'is is over tire plate such that for the co1nbined
tuass-plate systen1p = 200 kg/1111. Ca/culttu! the circul<tr a11d the natural frequencies.
The core is isotropic ( E.:_ = 2 x JOb k Nltu2, Ve = 0:3).
Solution. The eigenvalues A;; are (Eq. 5.149)

F..u F'.l.t F.1s


f'.,11 F" F4s
f.,s F., F.ss L., Ly
A;; = (5.1 53)
IFF.,.. F,_,I
F.ss
4

The paran1eters F,i are given by Eq. (5.95). The e)e.ments o( the stiffness n1a
trices are (Eqs. 5.53 and 5.54)

D11 = 52.16 kN m D,, = 11.71 kNm D,, = 7.96 kN m


- kN - kN
D.,, = 8.76 kN Ill S11=18615-
m
s,,
--
= 18 615 -
01

and L., = 0.2 m, t.,.= 0.9 m. Wit h these values. Eqs. (5.95) and (5.153) give the
folJo,ving v:ilues o f (An);, x 10- 9:

i\j I 2 3
1 0.0884 0.1007 0.1226
(5.1 54)
2 0.6162 0.6436 0.69(}4
3 1.6183 1.6656 l.7458
The natural frequencies are (Eq. 5.150)

1~
fi; = ;; y-;f,t, (5.1 55)

This yields the following values of fi; (Hz):


i\j 1 2 3
1 499 532 587
(5.1 56)
2 1 317 1 346 1 394
3 2134 2 165 2217
By treating t he plate as long, in Example 5.5 (page 198) we obtained the
following values for the first three natural frequencies: 488: 1 307; and 2 124 Hz
(Eq. 5.137). These natural frequencies are within 3 percent of the values given
above.
CHAPTER SIX

Beams

The response of con1posite bea1ns to loading is more con1plex than that of isotropic
beams, and the analyses of composite beams must take these con1plexities into
account. This requires analyses that are. by necessity, n1ore involved than for
isotropic bean1s but which, nonetheless, result in expressions readily amenable to
numerical con1putations.
In this chapter \\'e treat rectangular solid cross sections as \\ ell as tJ1in ..,valled
1

bean1s that undergo s111all deforn1ations and in \vhich the n1aterial behaves in a
linearly elastic manne r. \\'e neglect shear deforn1ations and adopt the Be.rnoulli-
Navier hypothesis, according to \vhich the origina lly plane cross sections of a beam
undergoing bending re111ain plane and perpendicular to the a.xis of the bea1u.
Axial, transverse. and torque loads may be applied lo the beam (Fig. 6.1 ), re-
sulting io the follo\\~ng internal forces: normal force /\!~ bending mo1nents Al1 , Al:.:
torque f: and the transverse shear forces v,,
V, (Fig. 6.2).

6.1 Governing Equations


The response of a beam to the applied forces is described by the strain-
displacement. force - strain, and equilibrium equa tions. TI1ese equations are given
in this section for conditions in v.hich there is no restra ined \Varping. The effect
o( restrained \Varping is discussed in Sections 6.5.5, 6.5.6, and 6.6.4.
Here, as v.ell as in the folJo,ving analyses.. \Ve en1ploy a n x- y- z coordinate
system with t he origin a t t he centroid. The centroid is define d such that an axial
load acting a t the centroid does not change. the curvature of the axi~ passing
through the centroid. As a consequence of this de-fioition, a bending mon1ent
acting on the beam does not int roduce an axial strain along this axis. Unlike for
isotropic beams.. for composite be.a n1s the centro id does not necessarily coincide
with the center o( gravity of the cross section.
The re are four independent displacements (f1g. 6.3}: t he axial displacement u,
the transverse displacen1ent~ u and u; in the y and t directions, respectively. and
the twist of the cross section I/I. The corresponding axial strain .~, curvatures 1/ f>y
203
204 BEAMS

Figure 6.1: Axial transverse., and torque lo.ads a c.ttng on a section o! a beam .

and l / p, in the x- z and x- y planes, 1 a nd t he rate of twist Ii are define d t hrough


the strain- displacen1ent re lationships
0
= ~
au Ii=-
.
aw (6.1)

.T ax ax
The generalized force-strain re lationship is defined as

I 1~,
NI =
M,
T
[p"p,_,
P 12

P,.

\vhere P.7 are the ele111ents of t he stiffness n1atrix.


111e equilibrium equations for a stra ight bean1 subject ed to the loads shown in
(6.2)

Figure 6.1 are2


af
-= - r
ax
3a~" = -py
av.
ax< = - p, (6.3)

a1ii.
a,\i,, = V.
ax ' ax<= -v,.
The preceding t hree sets of equ ation~ (together with the appropriate bound
ary cond itions) con1ple te ly describe t he d isplacen1ents of, and the forces in, a
con1posite beam.
l11e internal forces N, J\1.v. M;. V,. V;:, a nd fa re de te nuined by the simuha
neous solution of Eqs. (6.1)- (6.3) togethe r with the a ppropriate boundary cond i
tions give n belov". When a bean1 is statically dete rminate, the inte rnal forces can
be obtaine d fro n1 the equilibriun1 e quatio ns. \Vhe n a con1posite bean1 is static.ally
indete rmina te, the internal forces can be o bta ined \\ith the use of replace1uent
stiffnesses in the rele \'ant isotropic beam expressions provided that e ither the
beam is o rtho tropic o r the cross section is sy111me trica l a nd the load is applied
in the plane of syn1metry. The concepts of o rt hotropic bean1 a nd replace1nent
stiffnesses are discussed in Section 6.1.2.

1
T. H. G. fl.tcgimn, Aircraft S rn1c1tirt'S for Eri>!intering S1udenu. 3rd edition. Halsted Press. John
\ViJcy & Sons. New York. 1999, p. 284.
2 B. K. D<inald1;:on, A11a/y1is of ~1in:roft Srrur:t~J'('S. An b11rmluaicu1. ~tcGraw -HiJ I. New York, 1993,
pp. 277- 278.
6.1 GOVERNING EQUATIONS 205

Figure 6.2: 1'hc norma l force f.'i: the bendi ng momc.nls tW.,, .Mt; lhe torque-'f: and the traOS\'erse
shear torccs Py. 11; i nside a beam.

6.1. t Boundary Conditions


At a built~in e nd, the inplan e displacements a nd the slopes are zero. At a sin1ply
supporte d e nd , the io ..pla ne displac.e1ne nts a nd the n1on1ents a re zero. At a free
end, t he m on1ents and tbe transverse shear forces are zero.
When the e nd of the beam is restrained axially, the a xial displacen1ent is zero.
\ Vhen the end is not resua ined axia lly, the axial force is zero.
When the end may rotate, the to rq ue is zero. \Vhen the end is rotationally
rest raine d, the l \\~S l is zero.
The preceding boundary cond itions a re sun1marize<l in Table 6.1 .

6.1.2 Stiffness Matrix


The stiffness ma trix depends o n the geo1netry of the c ross section and on the type
o( ma te rial used in the construction of the bea1n . The geome try (i.e., t he shape)
or the c ross section changes when the bean1 is loaded . \\'e neglect the e ffects of
this change in shape on t he stiffness and evalua te the stiffness matrix for t he cross
section of the unloaded beam.

'

y )/

x
1-igurc 6.3: Displacements of a he.am.
206 BEAMS

Table 6.1. Boundary conditions for beams.


x-zplane x- yplane
Buih in4 u, =0 !!t = 0 IJ=0 i; =0
Simpl)' supporle-d u1=0
'"
l~y =0 u=O 1W: = o
Free ~=0 M, =0 ~=0 M,=o
Axia lly restrained
unrtSLrained
Rotalionally resLralned
unrestrained

For a be am inade of a n isotropic n1aterial ("isotropic be an1') the (orce- strain


relationships are 3

~ I [(EA)
0 0

ltWv
ii,
f
=
0
o
0
(E/1.,) (El,_)
(Ely;} (E/;J
0 0

The terms in parentheses are the tensile EA, bending El y,.. El.,. Et,., (= Et,,.),
and torsiona l G h stiffnesses.
isotropic. (6.4)

\Ve o bserve that for an isouopic beam the re is no coupling between tension (or
con1pression), be nd ing, and torsion. On the other ha nd. for a bea1n made of coin
posite n1aterials.., in general, no ne of the e leme nts of the stiffness ma ui.x is ze ro,
and the re is coupling bet\veen tensio n, be-n ding, and torsio n. Accordingly. ten
sion may cause bending and torsion, torsion n1ay cause tension a nd be nd ing, and
bending n1ay cause te nsion and torsion (see Eq. 6.2). The displace1ne nts resulting
from these couplings are often unexpected a nd are most o f the t ime undesirable.
Fortunately for t he designe r, son1e of the couplings a nd the corresponding dis
placen1ents are no t present \\he n e ither the bean1 s cross section is syn1me trical
or 'vhen the beam is orthotropic.
Sy111111etrical crosssectio11 bea111s. First, \Ve consider a n isotropic be..101 whose
cross section is syn1me trical a bout the t a.xis. 1-\ n axia l load FJ and a be nd ing
moment My (acting in the x-z syn1n1e tT)1 pla ne) are applied to the bean1. For this
beam the force- strain relationships (Eq. 6.4) reduce to

l1ClyNI= [(EA)0 (El,,,)


0 ]! '~ ! * isotropic
symme tric.al cross section.

Next , \\'e consider a composite be am 'vhose cross section is symmetrical about


(6.5)

the zaxis (Fig. 6.4). As a result o( t he symmetry, an axial load N acting at the
ce ntroid does no t int roduce e ither bending or t\\isting of the beam, \vhe reas a

Jo T. H. G. l-,l egson. Airc1oft S1111cr1u1!sfor n1:ineerin,: S111der1ts. 3 rd editio n. Hal1'lcd Press. Jo h n \Vik)'
& Sons.. New York. 1999, pp. 56 nnd 285.
6.1 GOVERNING EQUATIONS 207

1-igurc 6.4: Hlustralionsof composttc. beams \\i1h symmclric.al c ro:;s sec lions subjected to a trans
\'Crsc load in the :c-z S)'mm c!l)' pla ne.

n1oment IJ1 acting in the x- t symn1euy plane introduces o nly bending in this
plane. We d.esignate the elen1ents of t he stiffness n1atrix by lfA and Elyy and \\rite
the stress-stra in relationships as

{MFl>' }= [[[,:\0 OJ {'-/;;~ }


EI,,
composite
symme trica l cross section.
(6.6)

Orthotropic bea111s. 1-\ bea1u is o rthotropic \vhen its 'vall is made of an or-
thotropic laminate and one of the orthotro py axes is aligned "'ilh the axis o f the
bean1. A lan1inate is ortbo tro pic \\lhen every layer is n1ade o f either an isotropic
n1ateria l or a fiber.re inforce d con1posite (page 75). In the latter case, a layer n1ay
consist of plies n1ade e ithe r of wove n fa bric o r of unidirectiona l fibers (Fig. 6.5).
\''oveo fabric plies must be a rra nged s uch that one of t he ply syn1n1etry axes is
aligned with the longitudinal xaxis o f the beam. Unidirectiona l plies n1ust be

*"'
I

---~----
'
1' x
I

---~----
'
Figure 6.5: Layups t ha t resu.11 in no coupli ng bet\\1ee.n te nsion. llcnding. a nd torsion. U nidirc.c
tjonal ply {left); ,~oven fa tiric (middle); l\VO ply layer ( right). For each configuralion. one o f the
S)'mmetry axes must be parallel to the beams longiludinal x -axis.
208 BEAMS

mounted so that all the fibers are either parallel o r perpend icular to the longi
tudinal x~axis or one of t he symme try axes of t\vo adjace-nt unidirectional plies
(treated as a single layer) must be parallel to the beam's longitud inal axis.
It is sho,vn subsequently (Section 6.3.3) that for an orthotropic bean1 P12 =
P13 = P 14 = Pu = P"J.t = 0, and the force-strain relationship is

orthotropic. (6.7)

Jr)
Fron1 the preceding equation \Ve see that the re is no tension- bending- torsion
coupling in an orthotropic bean1.

( P] =
ii
0
[ o
0
o
El,,,
fl,.
o"
o
El,.,
fl..
o" Cl,
o0
0
l
We designate the e lements of the stiffness ma trix by EA El, Cl, a nd write

orthotropic. (6.8)

Principal direc-lion. For isotropic beams, t here is a coordinate systen1 y'- z'
(Fig. 6.6) in which the moment of ine rtia 1,,t is zero (l,'t = 0) . The a ngle bet-
\veen the y' ..axis of this coord inate system and the y ..axis is11
21..
tan2.p= - ' = (6.9)
fry - I:.::

The re lationships De.twee n the mome nts of ine rtia in the y- .z and y'- i coordinate
systen1s a re

(
lyy -
2
I")'+I ' )'Z
(6.10)

lyy - 1")' + I' (6.11)


( 2 )'Z

l,v:: = 0. (6.12)

The directions y' a nd z.' are called principal directions.


As in Eq. (6.9), for a n orthotropic beam we write the a ngle between y a nd y as

2Ell"
tan 2.p = (6.13)

-' E. P. Popo\'. 111;irietriri1-: i\t.l1LJ11ics of So/his. Prcntkc-Hall. Engk\\'Ood Cliffs. New Jersey. 1990.
p..142.
6.1 GOVERNING EQUATIONS 209

figure 6.6: They',:! coordinate S)'Slem in whic-h f.t ts ttro.

By referring to Eq~ (6.JOl-(6.12), we express the bending stiffnesses in the y'- z'
coordinate system in the ronns

(- - )'+Er;,
E /l'f - /._~
2
_,
(6.14)

fl,,+ lfi,,
2 (- - )'+ Er;,
El,., - El..
. 2 "
~
(6.15)

El,., (6.16)
. ' = 0.

6.1.3 Compliance Matrix


\\1ilh respect to the x- y- z coordina te systen1 a ttache d to the centroid, t he strain-
force re la tionships are define d as

il=[i
l tJ
r \\'14
0
\Viz
\\~3
lV2.1
0
IV,,
W1.' w ..
\\~:1
ll'i. l\~14
l!"I
l\) .i J~y
M~
f
.
(6.17)

The \ V11 and \VJl tern1s are zero bec.atL1ie an axial fo rce applied at the centroid
does not cause be nd ing. 'vhereas l Vi 2 and l.Yu are zero because tl1e con1pliance
n1atrix is syn1metrical. The. con1pliance n1atrix ( \\'] is the inverse o f the stiffness
matrix

P~
~ r
0 0 P 12 /'J3
\22 \\'23 w,.l ["P
124 _ Pn P,,

l11~. w,,
l.Y?-4
IV"
w,.
w.,J- Pu
W.., P"
12
P,,
P,,,
P,,
p_,4 p.,
(6.18)

We obtain t he c.01n pliance n1atrix of an orthotro pic beam by substituting


the e le ments of t he matrix [P] given in Eq. (6.8) into this e xpression. The
result is
i. 0 0 0
lit
Elr, - Etn
0 0
lftJJ lft!l-=- (iii):) ? ii,, Et:;..-ci,:r
[IV) = - El ,! E/). orthotropic.
0 0
Et., Et~~ -(Et .;f m.,Et~; -<Et ~r
0 0 0 l
G /1

(6.19)

6.1.4 Replacement Stiffnesses


The paran1eters EA, ifi. Ci, a re refe rred to as re placement stiffnesses. By co n1
paring Eq. (6.5) with Eq. (6.6) and Eqs. (6.4) with Eqs. (6.7) a nd (6.8), we note
the similarity in t he (orce-stTain rela tionships of isotropic and composite beams.
Therefore, the strains (and consequently the displacen1ents) o f an o rthotropic
bea1n and o f a composite bea111 \Vit h syn1n1etrical c ross sect ion can be obtained
by re placing EA, El , G11 by EA. Er, Gi, in the corresponding isotropic beam
solution. For an ortho tro pic beam, the necessary substitutions a re
Isotropic bean1 Orthotropic beam
EA=EA
Elyy. El", El,,
GI,
= El,,.. El,,. El"
=GI,
For a con1posite beam \vi th symn1etrical cross section and loaded io the symn1etry
plane the following substitutions a pply:
Isotropic. bean1 Composite bean1
sy1nme trical cross section svn1me trical cross section
EA. El,., = . tA fl,,

As the preceding discussion indicates, t he displacements of o rthotropic beams


and of composite bean1s 'Nith symn1etrical cross section a re simiJar to the
displace1nents of isotropic bea1ns. However, the stresses are markedly different
in isotro pic a nd in composite beams. In an isotropic beam subjected to ao axial
load and pure bending, there is only axial stress. In a con1posite bean1 subjected
to an a xial load a nd pure bending. in addition to the axial stress, the re are also
transverse no rmal a nd shear stresses. Furthe rn1o re, in a n isotropic.beam subjected
to pure to rque. the re is o nly shear stress. whereas in a composite beam there are
also a:dal a nd ua nsverse nom1al stresses.
In this chapter the stresses are calculated by the lan1inate plate theory, which
does not take into account the inte rlan1inar stresses near free edges (page 166).

6.2 Rectangular, Solid Beams Subjected to Axial Load and Bending


We consider rectangula r lan1inated beams ha,~ng solid cross sections with an
axial force 1V acting at t he ce nt roid a nd a pure bending mon1e nt tffy acting in the
, t b

/J
Figure 6.7: Rc.ctangular laminated beam.

x-z plane (Fig. 6.7). In this section, we develop e xpressions for calculating the
displace1nents and the stresses.
We treat the be am as a narro'v plate a nd build tJ1e an alysis o n the results of
laminate plate t heory presente.d in Cha pter 3. By convention, for plates a long an
edge parallel to the ya.xis, the in ..plane force per unit length is 1V.t a nd the mo111ent
per unit length is M, (Fig. 6.8). For beams, the tota l force Na long a n edge paralle l
toy a.nd the total n1on1ent in the x-z p lane M1 are specified . The total a.xia.1 force.
in t he beam Nconespo nds to b1V., in the plate, a nd the total n1oment in the beam
!Wy corresponds to bMx in the plate (where bis the width). Thus. we c.an a pply the
laminate plate theory expressions to beams by making the folJo,ving substitutions:

lJ
N:r=- (6.20)
b

6.2.1 Displacements - Symmetrical Layup


The layup of the be an1 is syn1me trica l. As noted in the preceding section, \Ve
analyze this be am by the laminate plate theory, according to which t he midplane
strain and curvature of the plate a re (Eqs. 3.31 and 3.32)

plate. (6.2 1)

Bean1 Plate.
Figure 6.8: lntcrnal forces a nd curvatures in a bea m and in lhc. correspondjng plale.
212 BEAMS

\Vhere a 11 and d 11 are the elements of the compliance matrices (Eqs. 3.29 and
3.30). We observe that the curvature of the plate in the .t- l plane K.T corresponds
to the curvature of the beam l / p,. (Fig. 6.8) . If we re placet<., with I/ p,., for a beam
Eqs. (6.20) and (6.21) yield

.!.
p,
= (a") ii,.
b .
con1posite
beam .
(6.22)
~
Mo'::

By comparing this equation with Eq. (6.17), we see that the tenns in paren
theses are the \..Yj 1 and " '22 elen1ents of the con1pliance 111atrix.
For a bean1 n1ade of an isotropic n1aterial, the suain and curvature are (Eq. 6.5)

1 - I 1 - isotropic
E('=-N -=-M,. (6.23)
' EA Py El beam.

It follows from Eqs. (6.22) and (6.23) that the axial strain a nd the curvature
of the axis (and consequently the displacen1ents u and w) of a co1nposite beam
(symmetrical layup) can be calculated by replacing EA and El by .~, and ;;, in
the relevant expressions for the corresponding isotropic beanl.
An isotropic beam subjected to an axial force Nanda bending n1on1ent 1'1,, only
bend~ in the x- t plane. On the other hand, under these loads the cross sections of
a composite bean1 may also t\\i~ t. To determine the an1ount of this t\vist \Ve refer to
the twisting of a plate. The out-of plane curvature of a pla te (symme trical layup)
is (Eq. 3.32)
plate, (6.24)
where K,,. is defined in Eq. (3.8) and is re peated below
2a1u,u () ~
K.w
y
=- - - = - 2--,
axay ax (6.25)

\Vhere w0 is the deflection of the tni<lplane. The expression CJw"/3y in the plate
corresponds to I/I in the bean1 (Fig. 6.8). Thus, we have
J1/f
.,,. = - 2-,- (6.26)
ax
Equations (6.1) a nd (6.26) give the rate of tw ist of the beam as follows:

(6.27)

By combining Eqs. (6.20), (6.24), and (6.27), we obtain the rate of twist of a
bean1 as folJo\vs:
composite
(6.28)
bean1.

\Vhen only ,Q and 19 ,, ac.t, the relevant elen1eots of the con1pliaoce 1natrix are
W11 , W22, Wn. IV,. , IV,. (Eq. 6.17). The eleme nts IV11, IV22, and W,.. a re given
6.2 RECTANGUl.AR, SOI.ID BEAMS SUBJECTED TO AXIAL LOAD AND BENDING 213

'Ncutr31" plane
a
e
~fi dplane

Figure 6.IJ: Unsymmetric.al solid rectangular beam.

by Eqs. (6.22} and (6.28). Elements W14 and w,, a re zero because t he layup is
synunetrical.
When the beam is ortho tro pic, d 16 is zero (Eq. 3.37). a nd the rate of twist is
zero ( 1? = 0) \vhen the beam is subjected to an Jij.v bending mon1enl.

6.2.2 Displacements - Unsymmetrical Layup


The layup of the lan1ina te d bean1 is unsy1nmetricaLThe centroid is locate d at the
a-a plane at a distance '1 fron1 an a rbitrary reference plane. This reference plane
may be chosen as the midplane (Fig. 6.9). The location of this a-a plane is such
that an axial load does not cause a change in the curvature l / Py and a bending
n1on1ent does not catL~e axial strain f.~. Thus. t he a-a plane is analogous to the
neutral plane (\vhich, fo r bean1s made of isotropic ma te ria ls, passes through the
center of gravity}.
We no'v consider a la1nina ted plate (unsyn1me trical layup) on \Vhich an axial
force :V." a nd a moment A(" (per unit le ngth) act in the a-a p lane. For this plate,
the follm1ng relationships hold in the a-a plane (Eq. 3.22):

plate. (6.29}

where at 1 , p~1 , and 8f1 a re evaluated at the a-a "neut ra l.. pla ne. According to the
f:
aforen1entioned definition of t he "neutral.. plane, depends only on Nx. and K.T
depends only on Mx. Therefore. /J~1 must be equal to zero. Thus, 've write (see
Eq. 3.48)

P~, = P11 + e~11 = o. (6.30)

where J:J1 1 and ~1 1 are evalua te d a t t he arbitra rily chosen reference plane that here
we have ta ke n to be the n1idplane. The consequence of Pr, = 0 is t\vofold. The
first consequence is that the d istance e is

e= - -/l11 (6.31}
liu

Tiie second consequence is that Eq. (6.29) reduces to

plate. (6.32}
214 BEAMS

\Ve note again that the curvature of the plate Kx corresponds to the cunature
o( the beam l / Pr Then, fo r a beam, Eqs. (6.20) and (6.32) yield

cotnposite
(6.33)
beam.

By comparing this equation wit h Eq. (6.17), we see that the tem1s in paren
theses a re the lV11 and \\'22 ele1uents of the compliance n1atrix.
II follows from Eqs. (6.33) and (6.23) that the axial strain a nd curvature of
the axis (and conseque.ntly the displacements u and w) of a composite beam
(unsymmetrical layup) can be calculated by replacing EA and El by 1/r
11
and fo.:
11
in the relevant expressions for the corresponding isotropic bean1.
\Vhen a n isotropic bean1 is subjected to an axial force lJ a nd to a be nd ing
mon1ent :\!}., it \viii only bend in the :c-z: plane. The cross section of a con1posite
beam subje~ted to fJ and M1 n1ay a lso l\Vist. To determine the rate of twist of a
con1posite beam. we again refer to a lan1inated plate. The outof-pla.ne cunature
o( a laminated plate (unsymmetrical layup) is (see Eq. 3.22)

pla te. (6.34)

\Vhere flf<i and sr6 are evaluated at the a- a ''neutral .. plane. From Eq. (3.48) \Ve
bave

(6.35)

From Eqs. (6.20), (6.27), and (6.34) we obtain the rate of twist of the beam:

composite
(6.36)
beam.

By comparing this equation with Eq. (6.17), we see t ha t the terms in the paren
theses a re the lV14 and \V24 e len1ents of the con1pliance n1atrix.
\Vhen only lJ and :\1y act, the relevant e lements of the con1pliance n1atrix are
111. II',,, Wu , l\'14, W,. (Eq. 6.17). Elements Wu, W,,, W14. and II',.. are given by
Eqs. (6.33) and (6.36), and Wi.1 is zero.
\Vhen t he bean1 is o rthotropic and one of the orthotropy axes is aligned \\ith
the beam's axis, 816 a nd /3 1 are zero (Eq. 3.37). Therefore. (see Eq~ 6.35 and 6.36),
the rate of t\\ist is zero ('/J = 0) \vhen an orthotropic bean1 is subjected to an axial
force Nanda bending mo1nent Mr

&.2.3 Stresses and Strains


The stresses a nd strains are given in this section for a rectanguJar thin bean1whose
thickness Ir is small compared with il< width b (Fig. 6.9). We consider only regions
away fro1n the e dges and en1ploy the lan1inate plate theory expressions.
6.2 RECTANGIA.AR, Sot.ID BEAMS SUBJECTED TOAXIAL LOAO AND BENDING 215

.~y1111t1 etrical
layup. A t the neutral plane (\\hich fo r a sy1nmetrical laminate
coincides \\~th the n1idplane). the strains and curvatures are (see Eqs. 3.31. 3.32.
and 6.20)

(6.37)

In a ply at a distance t from the midplane, the strains a re (see Eq. 3.7)

fx
fy } = <.~
{ f~ } + .Z { Kx }
Ky (6.38)
{
Y.~y Y:v Kry

The ply stresses are given by (Eq. 3.1 I)

<fx }
(fy = ~II 021 Q
g21 Q11 16] {<, }
Qu. fy (6.39)
Q~il
{
fxy Q61 Qu, Yxy

U1Jsy111n1etr ica/ layup. TI1e strains and curvatures of the axis passing through
the centroid (which is in the "neutral" plane) are (see Eqs. 3.22 and 6.20)

<"x af, 0
f;
Y.~
=
. .1 flt,
"1
0
16 p61 {~'' }. (6.40)
Kx 4r1
Ky fJf, $r1
KN}' fJf. $r6

where at? and pe are evaluated al the ''n eutral .. plane (Fig. 6.9). The location of
the "neutral" plane is given by Eq. (6.31).
In a ply a t a distance z fron1 the ''neutral'' plane, the strains and stresses are
given by Eqs. (6.38) and (6.39).

6.J Rxantple. An L = 0. 2~111 ./ong and b = 0.02n11vitfe bea1n, 1vith the cross section
sho1v1t in Fig. 6.10, is 1nade of graphite epoxy. The 1na1erial properties are given in
Tttble 3.6 (page 81). The layup is (45V011/4S1J. The beam, simply supported at
eacl1 end, i.s loaded tu1ifor111/}' ( p = 1 CX>O 1\ 11111). Calculate the 1naxinuan bending
1non1ent, 1/Je 1uaxin1un1 de.flection, aud the ply srresses t111d srraius.

p = JOOONlm
tttttttl
;;;. i:
~ l =- 200mm IJ = 20nun

Figure 6.10: The beam in Example 6.1.


216 BEAMS

Solution . The maximun1 bending mo 1n ent a t the n1iddle o ( the bean1 is (Table 7 .3.
page 332)

-
J\1v=-=5N
pL'
nl. (6.41)
. 8
The maximum de.Hection of t he correspond ing isotro pic beam is (Table 7 .3)

- 5 pL'
U/ = 384 El . (6.42)

The n1axin1un1 deflection of the composite beam is obtaine d by re placing El


by J,, (see page 212)
_ 5 p L'
w = 384 . (6.43)
du

With the value of d 11 =33.10 x 10 -1 ,_, 10 , (Table 3.8, page 85), the maximum
d e.ftection is

w= 0.0345 m = 34.5 mm. (6.44)

The axia l force is zero (iJ = 0). Thus, \\'ith the co n1pliance n1atrices given in
Table 3.8, the midpla ne s tra ins and curvatures a re (see Eqs. 6.37 and 6.41)

-~
; } {""
0 }
{ Yxy = <tt6
" bFl = I I {o} 0
0
(6.45)

Kx }
K)' = {d11
d12 } 7:-
_ I = {- 33.10}
25.59 10- l I0.02 I
5
{
K.xy d16
1 Q

8.28}1 (6.46)
= { -~40 ~

\Vhere Ks and Ky a re illustrated in Figure 6.11. In a ply, at a d ist ance l fro n1 the
midplane, the s tra ins are give n by E q. (6.38) as follows:

"<y } =l { "'
Ky } =z { - 8.27}
6.40 . (6.47)
{
Yxy Kxy 0

Figure 6.J I: Uluslration o( the curvatures o f the beam tn


E:xample-6.J.

"'
6.3 THIN-WALLED, OPEN-SECTION ORTHOTROPIC OR SYMMETRICAL CROSS-SECTION BEAMS 21 7

: : :
=
i 4Sr

E o,, 0
E
N
0.5 t, o.s .
t%)
j 4S'
-1
""' - 1--
Agu.rc 6. 12: The: nonuro Sll"ts:ie'.s and stTatOS m the be-am an Example: 6.1. lbc un11 of o ts
10' Nim'.

Tiie stresses in the kth ply are (E<j. 6.39)

" } [<:!" Q,, Q,.] {<, } (6.48)


{""f'~y = Q,,
k.7iti1
Q,, Q,..
Qiil QM Jr.
l }'
Yxy k

The eleme nu ol 1hc IQ I matrix ol the 0 -degree ply are given by Eq. (3.65) a nd
ol the wove n fobric by Eq. (3.66). The stresses are calculaled with these values ol
the ( Q J matrices and "'i lh lhe strains given in Eq. (6-47). The rcsuhs are shown in
Figure 6. L2. We note agnin that these stresses are valid only in regions a\\ay from
the edges.

6.3 Thin-Walled, Open-Section Orthotropic or Symmetrical Cross-Section


Beams Subjected to Axial Load and Bending
lo this section we present the deflections of. and stresses and strains in. thin
walled. open-section beams. The method is first illustrated via o r1 hotropicT-beams
with symn1c tricnl c ross section and via orthotro pic L-bea1n s.. TI'e nnalysis: is then
exte nded to orthotropic beams \\ ith arbitrary cross section and to nonortho tropic
1

bea1ns '"ith sy1nme trical cross sectio n.

6.3.1 Displacements of T-Beams


\ Ve consider a T..bcnm \\ hose cross section is symmetricnl \vith respect to the
1

<xis (Fig. 6. 13). The layup ol both the Hange ond the web ;., o rthotropic a nd

I.
'~ v
,..igurc 6.13:
Gl
,,
U l u ~1rn 1 1on
iii,
y

of 1hc 'r.ticnn1.
i.....+ ..',=.
' :' :,
218 BEAMS

syn1metrical. An axial force Rand a Dending mon1ent fi.1y act at the centroid of
the beam. The origin of the x- y- .r coordinate system is attached to this centroid.
Jn addition \Ve use the ~ i -t'Ji-('l and ~2 -JJ2-~2 coordina te. systems a ttached to the
midpoints of the flange and the \veb, respectively. The flange and t he web are
designated by the subscripts 1 a nd 2.
In the folJo,ving \Ve derive the replacement stiffnesses. The displacements of
a Tbeam a re obtained by substituting these re placen1ent stiffnesses into the e x
pressions for the displacen1ents of the corresponding isotropic bean1. Because the
cross section is symmetrical \vi th respect to the zaxis and the loads act in the x-z
plane. the beam be nds only about the y-axis. Consequently, only EA and Et,,. are
o( interest.
The calculation proceeds in four steps. In Ste p 1 we deforn1 the axiso( the beam
and calculate the strains in, and the curvatures o~ each wall segment; in Step 2
\Ve calcula te the forces a nd 1noments in e ach \\all segment; in Step 3 \Ve calculate
the resultant (orces a nd n1on1ents acting on the bean1; in Step 4 \\'e determine the
replace111ent stiffnesses.
We treat both the flange a nd the web as thin plates and a nalyze them by the
lan1inate plate t heory.

Tensile stiffness EA and ce11/roid.


Step I. The axis of the beam (passing through the centro id) is elongated, and the
strain of the axis is denoted by f;. The bean1 does not bend, and the axial strains
are the same across the cross section,

(6.49)

'vhere ~;, and f;2 are the axial stra ins in the midplanes of the flange and the \\eb,
respective ly.

Step 2. The axial stra ins result in distribute d axial forces (per unit length) N~ 1 ,
N, 2 in the midplanes of the flange and the web (Fig. 6.14, left). These forces are
(see Eq. 3.31)
" 1 (6.50)
"" = -
au(
)1 .,,


Figure 6.14: 'fbc dL<>lrihulcd forces and lhc (orcc resullanl.s.
6.3 THIN-WAI.LEO, OPEN-SECTION ORTHOTROPIC OR SYMMETRICAL CROSS-SECTION BEAMS 219

whe re (a 11 )1 nnd (a 11 )2 arc evaluate d in the E-11-~ coordin ate system a t the mid-
planes of the fla nge nnd the web.

Step 3. The total forces acting in the flange and in the v. eb arc b1N, 1 and f>i.N1.1
respectively. and lhe 101al force acting on the beam is (Fig. 6.14. righl)
(651)

Step 4. Equalions (6.49)-{6.51) give

fJ = (....L + ...!!L)
(a11 h (011 )? "
(6.52)

nA
The te rnl indicntcd by the bracket is the replacen1ent te ns ile s tiffness !fit
111e coordinnte of the ce ntro id ~ is calculated by a n10 1n cnt bnlance about the
bouom edge of the web (Fig. 6.14, right) as follows:
(6.53)

The dis1anccs Z1 and i 2 are sho\\'ll in Figure 6. l4. By combining this equation
v.ith Eqs. (6.49)-{652). we obiain the position ol the centroid:
";A.+':,~
1
.. t-u)t ....;C-i1h
l.= '11: .. - (654)
~11'1 +~
The coordina1es of the centers of the flange and the v.eb iith re.spea 10 the
centroid are (Fig. 6. 13)

l 1 =~I - t.c (6.55)

Depe nding on whe re the ce ntro id is located. z2 is eithe r positive or negative.


ln Figure 6.13, : 2 is nega tive.
Be11di11g sti/f11e.Y.11 Ei,,. The be an1 is be nt about the y -axis p:u1sing thro ugh the
ce ntroid (Fig. 6. IS). TI1e radius ol cwvature of the beam is dcno1cd by p,.

Step I . The strain varies linearly ~ith z as folloYt's:

(_. = -p, ::. (656)

l~gurc 6 . 1 .~: The rndiu or cunaturc p ~. the s1ra1n distnbution. and lhe tlc(ormed lihllJlCll of lhe
midph1ncs of the Oangc a nd the \\'eb.
At the midplane of the Hange t = z1 and the strain is
1
.;, = -z,. (6.51 )
Pr
The curvature of the midplane of t he ftange about the y-axis (by definition) is
I
Kt l =- . (6.58)
Pr
In the n1idplane of the web, the strain 1 is
1
E.f2 = /Jy l . (6.59)

The n1idplane of the \Veb remains flat. and its curvature is zero (Fig. 6.15) as
denoted by
Kt.1 = 0. (6.60)

Step 2. The axial force No and the bending moment M, , (per unit le ngth) acting
on the flange are (Eq~ 3.31 and 3.32)

M,, = (- II ) " l '


t It I
(6.61)

The preceding fo rce a nd moment a re illustrated in Figure 6.16 (left). The tenns
(a11)1 a nd (d11) 1 a re e valuated in the ~-i-~ coordinate syste m at the midplane of
the flange.
The layup of the \veb is syn1metrical, and Kt 2 = 0. Consequently. M~ 2 is zero.
Thus, the only force acting in the web is N12 (Flg. 6.16, right). and this force is
1 0
1\h = -( l ...,. (6.62)
au . 2
\vhere (a11)2 is evaluated in the ~-rr-~ coordinate sy.sten1 at the midplane of the
\Veb.
Step 3. The resultant bending moment about the y-axis is (Fig. 6.16)

,\i, = b1 N, 1z1 + b1 M., + f


(b1)
Ni 2zdz, (6.63)

\vhere b1, bi, and t are shown in Figure 6.13.

0 + - - -->

11

Figure 6.16: The distributed force.s and moments (per unit lc:ngth) in the flange and tn the.\\eb
of a r. be.am bent about the y-axi.;.
Step 4. Equations (6.57)- (6.63) yield

- =-
1\f, 1 ..,-b- +--+
P,. (011)1
[.,l
b 1
-1-
(d11) 1 (auh
1 (/~ , )] .
- +z;b1
12 -
(6.64)

The term indic>tcd by Lhe bracket is the replacement bending stiffness El,,.
6.3.2 Displacements of L-Beams
\\'e consider an L-beam. The \Val.ls of the flanges are o rthotropic. and one of
the nxcs or
o rthotropy is aligned with the axis o( lhe beam . The layups of the
flanges need noc be syn1me trical ~ith respect to the nanges' midplancs. The beam
is subjected to nn axial force Nand bending n1oments Mynnd M._ acting at the
centroid (Fig. 6. 17).
We use nn .Y- )1- l. coordinate systen1 with its o rigin ntlachcd to the centroid
of the bca1n ond the ~1 -tJ 1 -('1 and ~2 -112 -~1 coordinate syste ms a ttncbe d to the
n1idpoints of the arbilr:irily chosen reference planes in the hori1.ontal and vertical
Hanges, respectively. 111e horizontal and vertical flanges are designated by the
subscripts I nnd 2.
ln this sectio n lhe re placement stiffnesses are de1ermincd. The displacements
are obtained by substituting these replace-m ent stiffnesses into the expressions for
the displacements of the corresponding isotropic be.a m. BcC3usc the cross section
o( the L-bcam is unsymmetrical. all three bending stiffnesses El,,. El,,. and El,_
as \\'ell as 1hc tensile stiffness tii are n eeded to determine the displacements.
We treat the Aang.. a. thin plates and employ Lhe laminate plate theory equa-
tions. The calculation proceeds along the four st eps use d in the nnalysisofT~beams
(page 2 18).
Tensilt ,Yti/f11ts.'I .4. and centroid. The tensile stiffn e$.., is obtnined by consid
ering 1hc elongotion of the beam while the axis of the bca1n remains straight.

Step I. The nxis of t he beam (passing through the centroid) isclongutcd . and the
strain o f the axis is denoted by f;. ln the absence of bending.. the nxial s tra ins are

0 G
, ,;;: . /
.,....
(, :,
?
0 'h

J=igure 6.J7: L~~c::un .


222 BEAMS

Centroid

Figure 6.18: The distributed forces a nd moments acting in an L-bcam with unsymmetrical la)'up
whc.n the axis of the beam is elongated.

the same-across the c ross section.

(6.65)

\Vhere ff1 a nd E"(2 a re the axial strains of the reference planes of the t\vo flanges.
respective ly. The locations of the re ference pla nes n1ay be chosen a rbitrarily.

Step 2. The axial forces 1V~ 1 , N!.2 and the bending moments M!. 1 , M~ 2 (pe r unit
length) in t he ftanges (Fig. 6. I 8) are expressed in tenns of the strains ; , , ; ,.The
derivation of these expressions is discussed subsequently on pages 227- 228. Here
\Ve quote the r esults. \vhich are

1 (ouh o
(6.66)
'' " = (Dl. 2 <p
,~ _ _ (/l11h. (6.67)
. "' - (D), '12
\Vhere Dis defined in Table 6.2, a nd 611 a nd /311 are evaluate d in the ~ -11-~ coor
dinate systen1.

Table 6.2. The axial force and moments inside the wall
(N,, = N,,, = M,, = 0). The elements of the compliance matrix 6u, 6,.,
6", 13,,, and 1316 are evaluated in the wall's ~-tr( coordinate system.
S)mmetrical
Arbitrary cross section cross section
Orthotropic Orthotropic and Albitrary
1.11symmetrical layup symmetrical layup layup

JV! = 4}E; - ~K!


/.f~ = -t!f}E: + !fl-Kt
Aft_,,=0 0

where Ciu = (au - ~) -811 = (Su - ,,~)


D = a11d11 - /Jf1
6.3 THIN-WALLED, OPEN-SECTION ORTHOTROPIC OR SYMMETRICAL CROSS-SECTION BEAMS 223

Step 3. The resultant axial force is (Fig. 6.18)

FJ = b,N,, + l>,N,,. (6.68)

where b1 a nd b2 a re s hown in Figure 6.17.

Step 4. Equations (6.65)-(6.68) gi"e

v- ..f.. b,(~11). <,."


' - L.. (D) (6.69)
k l k
----.--
EA
The term indicated by the bracket is the te nsile stiffness EA The terms ( D); a nd
(811)k are evaluated in the coordinate systems attached to the reference planes of
the horizontal (k = 1) and \'ertical (k = 2) flanges, respectively.
The coordinates o( the centroid Z:, Ye a re calculated by mome nt equilibria
about point 0 (Fig. 6.17). With the symbols defined in Figu res 6.17 a nd 6.18,
n1on1ent equilibria give

z,/il = llb1Nl.1 + l2b1Nl.2 + b1J\1E 1 (6.70)


><N = y1bi /\\ 1 + y,lnN,, + b2M;i. (6.71)

By combining Eqs. (6.65)-(6.71), we obtain the coordina tes of the ce ntroid


(Fig. 6.17):
b ( - !!iJ.h ~)
1 l1 ( DJ1 - ( DJ1 + :b.CA11t
( D)z
le= ~ (6.72)
2
I:
kI
CD)

(6.73)

The coordinates o f the centers of the fla nges with respect to the centroid are
(Fig. 6.17)

J'l = Y1 - Ye ti="it - le (6.74)


Yi= Yi - Ye ti= ti - ti: (6.75)

The locatio n of the centro id detern1ines whethe r y,, y2 .t1 , and <2 a re positive
or negative. In Figure 6.1 7, )"1 and Zt a re positive and Yt a nd t1 a re negative.
Bending stiffnesses Ei,,, Ely:. To detem1ine the be nding stiffnesses EJ,..r a nd
El>':. the bean1 is bent a bo ut the y~axi.s, \Vhich passes through the centroid. The
radius o( curvature is p,, (Fig. 6.19, left). This be nd ing results in mon1ents J\/,,
~ ~- . -
224 BEAMS

z z z

P,

_!_ ";")J "


P: P~
!Ii

Figure 6.l9: The radii of cun aturc-s Py and P: a nd illustration of the axial strain distributjons.

The be-nding stiffness Elyy is detern1ined belo,v.

Step 1. The strains and curvatures of t he l\\'O Ranges are ide ntical to those of the
Tbeam (see Eqs. 6.57-6.60}. Hence. we write

= -I I
E~l z1 K( I =- (6.76)
Py Pr
I
Etz = - z K( l = 0. (6.77)
Pr

Step 2. The axial forces 1V~ 1 , N( 2 and the bending moments /\-1( 1 M~ 1 in t he Hanges
(Fig. 6.20) are expressed in tern1s of the strains E:1 ' ( 2and curvatures Kf. J, Kt l The
derivation of these expressions is discussed subsequently on pages 227- 228. Here
\Ve quote the results, \vhich are

(6.78)

(6.79)

'

N{I
0 0

y .Y
JV!.?

Figure 6.20: fo rces a nd mo me nts (per unh lc.ngth) acting in the t.....beam with unsymmetrical
layup bent about the )'axis.
6.3 THIN-WALLED, OPEN-SECTION ORTHOTROPIC OR SYMMETRICAL CROSS-SECTION BEAMS 225

Step 3. The resulting be nd ing n1on1ent ~.v about t he yaxis is

,l1, = b1N, ,z,+ b, M,,+ J N,,zdz. (6.80)


(b1)

Step 4. Equations (6.76H 6.80) gi"e

'
11
y
= ..!... [ b1 ((J11)1,i1 _ 2 (.811)1 + (<>11) 1) + (S11)z (~ +
Py (D)1 ' ( D)i ti ( D) 1 (D), 12
'b,)].
r, .

(6.8 1)

The tern1 indicated b) the bracket is the bending stiffness ifi\J'


The Dending sti(fness Ef,,,z is de te rmined be low. .

Steps I and 2. ln t he calcula tion o f the bending n1on1ent Mz the first t\vo steps
are identical to those given fo r the calculation of 1Wy.

Step 3. When the bean1 is be nt a bo ut the y-axis, the resultant bending n1on1ent
iW,about the z-axis is (Figs. 6.20 a nd 6.17)
111, = b1N, i}\ + .f N,,)~dz+ .f M,,dz. (6.82)
\bi) \bi)

Step 4. Equations (6.76H 6.79) and (6.82) gi"e

I
11 - ..!... [( (&11)1
' - Py ( D) 1 t i
- (/111)1 )
(D) 1
bY + ((811)2
( D)z >~
- (/l11}2)
( D)z l2 ' .
b]
(6.83)

t:t,t
The term indicated by the bracket is the bending stiffness fl" .
Bendittg sti!Jiresses El,~ El_,. To determine the bending stiffnesses El" a nd
Ef;..v the bean1 is bent about the z~.axis \Vit h a radius of cur\'ature P::. (Fig. 6.1 9,
right). Expressions for these bending mome nts ca n be derived in the sa111e \Vay as
Eqs. (6.81) and (6.83). 111e results a re

,q_ = .!_ [1 (<~11)2


. p, . >i
2 - 2(/l11h , + (<>11h) + (~11)1
(D)z Yi ( D)z Y. (D)i (D)1
(b)12 + Jt 1) ]
)i '

Eit:
(6.84)

11 -_ ,!_ }2 _ (/l11)2) b]
'" p, [((811)1
(D) , z'
_ (/l11) 1)b
(D) 1 ,y,+ ((811
(D)z~ ( Dh l2 2-
(6.85)

The tenns indicated by the brackets are the bending stiffne.sse.s ft::.::. and 1!1;).
(=El,.).
226 BEAMS

Figure 6.21 : Illus tration of thin\\'alled. open-sect ion ll-e-a ms with symme tncal and unsymme trical
C-T OliS liCCt ions.

6.3.3 Displacements of Arbitrary Cross-Section Beams


In this section \Ve treat thin\\aUed, open-section beams. The bea111s are sub
jected to an a..xial force Nand to bending n1oments :'1y. 1W~ acting at the centroid
(Fig. 6.2 1).
Three types of beams are considered:
1. The layup of the \Va ll is o rt hotropic but unsymn1etrical; the beam's cross section
is arbitra ry.5
2. TI1e layup of the wall is orthotropic and symmetrica l; the beam s cross section
is arbitrary.
3. The layup of the \\'all is a rbitrary: the cross section is symn1e trical with respect
to t he z..axis, and the loads Nand M:: are applied in the x- t symn1etry plane
(Fig. 6.21, right).

The re are no tension-bend ing- torsion couplings in any of the preceding three
types o f beams. In the first two types of bean1s these couplings are not present
because the beam is ortho tropic (page 207); in the third type or be am couplings
are not present because the c.ross section is symn1etrical a nd the loads act in the
symmetry plane.
In the following we derive the re placen1e nt stiffnesses. The displacen1e.n ts are
then obtained by substituting these replace ment stiffnesses into the expressions
for the displaceme nts of the corresponding isotropic beam.
We perfomi the analysis fo r beams of the first type, that is, the layup is or
thotropic but is not necessarily syn1metrical and the cross sectio n is a rbitrary. \Ve
then gene ralize the results to the other t \\'O types of beams in t he list above.
We e mploy three coordinate systems (Fig. 6.22). For the beam we use the x- y-z
coordinate system \\ith the origin at t he centro id a nd the i - Ji-i coordinate system
\vi th the o rigin at a n arbitra rily chosen point. We also define a ~-r1-' coordin ate
systen1 \\~lh the origin a t the reference plane of the \Vall. t\t each point in the wall
~ is parallel to the x coordinate.'' is tangential to t he circun1ference of the \Vall.
and ~ is pe rpe ndicular to the circun1fe re nce.
The calculation proceeds in fo ur ste ps. ln Ste p 1 \\'e deform the axis of the
bean1 (axial strain E~ and curvatures I/ p,, and I/ Pz) a nd calculate the strains in,

' J. C. ~1assa nnd E. J. Barbero, A Strcng1h or r..ta1crials F.ormul.a1ion for Thin \Vntkd Composite
Be.ams with Tomon. lr>u111al <1/Co1111J()JiJe ,1,,fotedlllt. Vol. .1'2, 1560--1594, 1998.
6.3 THIN-WALLED, OPEN-SECTION ORTllOTROPIC OR SYMMETRICAL CROSSSECTIOH BEAMS 227

>
y

Figurt: 6.22: The coordm.atc systcmi..

and the curvatures o( the \\all: in Step 2 .'t c.:alcula1e the Corces and moments in
the waU: in Step 3 \\l! calculate the rcsuhant forcc.s and moments acting on the
beam: in Step 4 we determine the s ti(fncsscs.

Step I. The Bernoulli avier h)'POlhCJis s1a1CJ 1ha11he axial slrain varies linearly
with thecwvaturesof the beam. Th~ lhe axial strain'( at a point on the arbitrarily
chosen reference surface of the .all is related to the axial strain': and curvatures
I/ P,. and I/ p, of the beam by
I I
(; = (; + l - + y - (6.86)
Py P,

where z and y are tl1e coordina tes of the point o n the \Valls refe rence surface.
From geom etry, it can be shO\.\'Jl 1hat the wall's curviuure Kt in 1he ~-' plane is,
(Fig. 6.23)
I 1 .
Kt= - COS<t - - s1na. (6.87)
Py P:.
where a is the nngle belwee n the 11 and y-coordinnte nxes (Fig. 6.23).

Step 2. In this step \\'e express lhc axial rorcc /\'t and bending mon1ents 1\1~ and
Aft " in terms of~; and Kt Tu derive the nccCS$nry expressions \\'e observe that
along the free longitudinal edges o f the bca111 the in -plnnc forces and 1noments
(per unit length) are zero: N. = = =
N,. M,1 0 (Fig. 6.24).Since the dime nsions of

FiguTC 6.23: Cun-aturcs of the bc:am (left) and the curn1urc o( the -..-.II (madd~) and 1ts v-cctor
rcprescntatJoo (n ghl).
228 BEAMS

N,1 = 0
<-i9'

Figure 6.24: The:.fore.es a nd momc:.nls along the longitudinal edge of an open-s.echon, thin-,valkd
be-a m.

the cross section are small con1pared with the bea1n s length, these in-plane forces
and n101nents are a pproxima te.ly zero ins ide th e \Vall:

1\~ = Nt11 = M1, = 0. (6.88)

\Ve appl)' only axial strain and curvatures to t he bean1, but no twist. Therefore.
the t\\ist of the 'Nall is zero:
(6.89)
\Ve OO\V recall t he strain- force relationships given by Eq. (3.22) \Vi th the com -
pliance matrix for an orthotropic material given by Eq. (3.37). To apply the re
lationships to the \Vall we re place x by ~ a nd )' by '' The resulting gene ra lized
strain- force relationships are (orthotropic):
,. <>11 12 0 /Jn /312 0 N,
"'" <>12 <Y22 0 /J21 /322 0
..
N,
y~lt 0 0
"0 0 0 N(,, (6.90)
=
/(' fJ11 fJ21 811 (jl2 0 M,
Ko {Jii p,, 0 811 Sn 0 M,
0 0 p.. 0 0 a.. Mf ,,
"''
S ubstitution of Eqs. (6.88) a nd (6.89) into Eq. (6.90) yields

l'tl
'"
= ["" 811
/l11
I [o]
N, J + 0 M, ,
/l11] M, (6.91)

''<i = 0 = 0 x N, +O x Me + 8.. M.,. (6.92)


Fron1 these equations we can obtain N,_ , A1t_, and ,\ft, 11 in terms off; and K(.
The resulting expressions are listed in the left column of Table 6.2 (page 222).

S tep 3. The resultant force and moments in the be am a re (Fig. 6.25)

Fl= J N, dr1 (6.93)


<SJ

!((, = .rM~
(S)
cos akd'I + f
( SJ
t.1Vt.tf'1 (6.94)
6.3 THINWALLED, DPENSECTION ORTHOTROPIC OR SYMMETRICAL CROSSSECTIOH BEAMS 229

fl,
~ (

~
.~/{

p,\/{IJ
Figure 6.25: The rorces and moments ac.ting on the beam and the forces and moments (per uni I
length) at.ting inside the wall.

X1, = J
(SJ
- M; sin a.,111 + J
(S)
yN,d11 (6.95)

f =I
( S)
2M;,,d11 = o. (6.96)

where Sis the length of the circumference. The torque resultant f is zero because,
for orthotTOpic beams. M,, is zero (Eq. 6.92).
Since T is zero for orthotropic beams. P14 , P24 a nd P"' are zero (see Eq. 6.2).
Consequently, tl1ere is no tension- bending- torsion coupling in o rlho tropic
bean1s.

Step 4. We axially elongate the beam such that I/ Py and 1/ p, remain zero. Thus.
from Eqs. (6.86) and (6.87) we have

K( =0. (6.97)

For a bean1with orthotropic and unsymmetrical layup (arbitrary cross section)


Eqs. (6.93) and (6.97). togethe r with the e xpression for N, in Thble 6.2, left column,
yield

-I
1V= 811
D <"
-d11 x
(6.98)
\St
~

EA
The coordinates of the centroid are o btained by taking a mon1ent about point
0 shown in Figure 6.22:

Fly,= J -JM,sin a,d~


(S)
N,yd11
(S)
(6.99)

Nt, = J Ni'idq + J M,cosa,d1/. (6.100)


(S) (S/
230 BEAMS

\vhere Y and l are the coordinates of a point on the wall's reference swface in
thex, y,! coordinate system. By se tting l / Py = 1/ p, = Oand by using Eqs. (6.86),
(6.98), and the e xpression fo r N1 in Table 6.2 (page 222), left column, we obta in
the coordinates of the centro id as follows:
f (z.';; - !!ff oos ")d11
( S)
z, = '-'---~---- (6.101)
! '~a,,
( S)

Next we detern1ine the bending stiffnesses. First, \Ve set ~~ equal to ze ro in


Eqs. (6.86) and (6.87). Then. Eq~ (6.86), (6.87), (6.94), and (6.95) together with
the expressions in Table 6.2 give the following resuJtant n1oments of a bean1 \\ti.th
orthotropic a nd unsymme trical layup (arbitrary cross section):

1,,

+
f[
(-')
811
(D/ t
/J11
+D . au . ] I
(t smet - ycosa) - D cos et smet d11 p,

(6.102)

- f[
M, = 811
(D/ t + D
fi11
(tsma - ycosor) - 'II .
D ooset smet ] 1
d11 Py
( S)

f [D
11 , 2fi11 . Ct11 , ] 1
+ - y- + --ysma + - sma d11 - (6.103)
D D ~
( S)

The tensile stiffnesses, the coordinates of the centroid. and the bending stiff-
nesses o( bean1s 'vith orthotropic and syn1n1etrical layup (arbirrary cross section)
and \vith arbitrary layup (sytnn1etrical cross section) are obtained sin1ilarly. The
main difference is in Step2, \\he re the appropria te stress- strain relationships must
be used ins te ad of Eq. (6.90). The results are given in Tables 6.3-6.5.
Choice oftl1e refere11ce surface. T he expressions of the replacen1ent s tiffnesses
simplify ""hen the properties are oot evaluated at an arbitrary refere.n ce surface
but at a " neutral'' surface. \\here /1 11 ( orthotropic layup- arbitrary cross section) or
fo11(arbitrary layup - syn11u etrical cross section) is zero. The s urface is .;neutral"'
in t he sense that a bending mon1ent M{ does not cause axial strain f t in this
surface. (Ho\vever, it is not a real neutral surface because . unlike in an isotropic
bea111, in this re ference surface the strain perpendicular to t he bean1 s axis EIJ is not
ze ro.)
6.3 IHIN-WALUU, UPtN-SECllUN UHIHUTHUPIC UH SVMMEIHICAL CHUSSSECl'IUN BEAMS 231

Table 6.3. The tensile and bending stiffnesses and lhe coordinates
of lhe eenlJOld ol open and closed-section beams wilh oumd walls.
The CR>$$ section Is >Wtitrary, and lhe layup ol lhe wall is orlholn>ple
and symmetrical; a11 and d11 are evaluated al the midsullace.

arbitrary cross sec.t ion


orlhoLropic
symmetrical l.:.yup

Tcnsllc stiffnc,;s
EA= J J.,,,,
(SI
...
Coordinates of lhc c<:nlroid

1l1e 11 con1ponent of the compliance n1atrix corresponding to the "neutral..


surface is (Eq. 3.48)

(6.104)

\Yhere JJ11and d11 ure the components of the compUancc malricc.s in 1hc arbi 1rarily
chosen reference surface and e is the location of the "neutral" surface {Fig. 6.26).
{J:
At the "neutr3J" surface 1 = 0, and the preceding equation yield.5

-e= -/Jn onhotropic layup


- (6.105)
&11 3tbitrary cross section.

This cqu:llion npplies to orthotropic beams v.i th S)mme tric31 as y,e ll as un-
symmetrical cross sections.
Rn bcoms wil h arbitrary Layup (symmetrical cross section), Eq. (3.48) and the
expression for Pu
in Table 6.2 (page 222) yield

(6.106)
Table 6.4. The tensile and bending stiffnesses and lhe coordinates of lhe
oentroid of open- and closed-section beams with curved walls. The cross
section is arbitrary, and lhe layup of lhe wall is orthotropic and unsymmetrical;
611 . {3 11, and 011 are evaluated in the wall's ( -1)-( coordinate system: D is
defined in Table 6.2 and 611 and 1J are defined by Eq. (6.157).

ttrbitrary cross se.ctlon


o rll101ropic
unsy1nmetricaJ layup

Tunsilc. stiffness
Open seclio n: lA = J ~"" Closed section: VI= J ~d11
(S) (l)
Coordinates of the ce.ncroid
I (r.~+'4f sui ..} '"
>.r -- ~>l~-~---
! ~~"
>!
Bending stiffness.es
1fl-n = J (ift- 1 j!' zcosat + "~' c.os2 a)d,,
E'lu = f~ (ifr + ~ y sin et +!ff sin 1 &) d11
(.<)

1!J1-: = J ( flJiyi + ~ (zsina - yc.os a) - 7J- cos a: sin u) d~


c:n

Table 6.5. The tensile and bending stiffnesses and lhe coordinates of lhe
oentroid of open-section beams with symmetrical cross section. The layup of
the wall is unsymmetrical and nonorthotropic; 0 , 61 1, and ~11 are defined in
Table 6.2. The elemenlS of the compliance matrix are evaluated in the wall's
e-w-<. coordinate system.
z
scy1nmeLric.al c-r0$S s.ec-tion
nonorlhotropic
un.symme1rical layup

Tensile stiffnc.ss
i;,1lhJ
EA = c
Coordinate.s of the centroid

}'t =0

Bending S.liffness.
El_,,,= J ("4t.t- ~ zcosa.t+ ~ cos:a)lhJ
"'
Arbitrary
rcfcrcn1.1c sur(acc (

figure 6.26: The '"neutral- surface -'here j~1 (orthotropw: layup - arhttrary Cft.:m1CCl1onJ or ?,1
(arbrtnry llllyup - rymmc1nul cross section) ts uro.

From the condition~. = 0 we obtain the location of the neutral .. surfac:e of


the v.all a.' follo\\-S:
- 'ii11 arbitrary layup
e= --=-
611 symmetrical cross section.
(6.107)

For selected cross sections the replac.e me nl stiffnesses and the location of the
centroid nre given in Thbles t\ .l - A.4.
~rg11uu1te1/ 'tttt1/I. n 1e bea1ns \\all n1ay consist of several Oat \Vall seg111ents
(fig. 6.27). The thickness of e acJ1 \Vall segn1ent is s1na ll com1,ared \l/ith the \\ idth of
1

=
the segment. The wnll segments are designated by the s ubscript k(k I. 2, ... , K ,
where K is the total nun1ber or \vall segn1ent~). The layup of each wall seg
men1 may be sy1n1netrica1 or unsymmetrical with respect to the \vall seg1nenCs
ntidsurface.
When 1he v.aU consists of segments and each segment i.s flat. the integrals in
Tables 6.J-6.S may be replaced by summations

I ( )d~ = ""'-"'2
L I ( )d11.
K ..fl

(6.108)
t S)

where h is the \\idth or the kth 'vaJJ segment. For each v.all segment 've define
a ~ -11-~ coordinntc system. \1there ~ is parallel to the .r coordinate. 11 is along
the circumference ol the wall. and ~ is perpendicular 10 the wall (Fig. 6.27). The
origin of this coordinate systen1 may be at a.n arbitrarily chosen re fe rence surface
but n1us1 be at the ntiddle of the 'viclth of the \\ a ll scgntcnt. By performing the
1

integrations in Tables 6.3-6.5 we o btain t he expressions given in Tables 6.6- 6.8.

6.3.4 Stresses and Strains


In a thin-\vallcd isotropic beam subjected to an axial load and 10 bending in the
x- <: plane 1hcre is only axial stress a~. In a thin-v.alled composite beam. in addition
10 the axial stress at . transverse nonnal a and she-a r r,. stTesJC.S are also pre.senL

J.igurc 6.27: Ttunwnllcd, op-en-section beams \Yith segme nted wnll.


234 BEAMS

Table 6.6. The tensile and bending stiffnesses and the coordinates of the
centroid of open- and olosetl-section beams with flat walls. The cross
section is arbitrary, and the layup of eaoh wall segment is orthotroplc
and symmetrical; a11 and d11 are evaluated at the wall's midplanes.

arbilrary cross section


+.::..-:--'-11---"'--'"-+t.I orthotropic
S)'mrne.Lric.al layup

Tunsile stiffne.ss

EA=L: ...!!...
t ...1 (ornJ

Coordinales of the ce.ntroid

T hese stresses a re illustrated in Fig. 6.28. Howeve r, the shear flow, defined as the
integral of the shear stress across the thickness. is zero,

q= f ""~
( h)
= 0. (6.109)

\\here Ir is t he thickness of t he \Vall. In the (olJo,ving subsections, we present analy


ses for calcula ting the three stress components <t(. u,,, rf,, inside oon1posite bean1s.
Ortilotropic layup (arbitrar)' cross secti'111). When the layup of the wall is
orthotropic, the axial strain and the curvatures of t he bean1s axis passing t hrough
the centroid are (Eqs. 6.17 and 6.19)

0 -
f.K-
!!.__
- (6.110)
EA
Ei,"~' - Ei,,M,
- = (6.11 1)
Py fl,,fl., - (fl,y
1 - Ei,_M, + Ei,,M,
- =
p, --
El, 1 1., - (El,d ' (6.112)
6.3 THIN-WALLED, OPEN-SECTION ORTltOTROPIC OR SYMMETRICAL CROSS-SECTION BEAMS 235

Table 6.7. The tenslle and bending stiffnesses and lhe coordinates of lhe
centroid of open- and closed-section beams with flat walls. The cross
sectioo Is arbitrary, and lhe layup of each wall segment Is orthotroplc and
unsymmetrical; 611 , {J ,, . and o 11 are evaluated In the wall's {-ri-(
coordinate system; D Is defined In Table 6.2. The parametets 611 and tJ
are defined by Eq. (6.157).

arbitrary cross section


IJ orthotropic
-..--,---11---~--
Ul'bfmmetrical la)'up

T<nsilc stiffness
.
Open <ttlJon: EA=
-

Coordinates of the centroid


...
~ (n},-.
L.. 10,

... )( ~
J: ~
l/llt + '"' ......,,

At an arbitrary point o n the re ference sur(:1cc o( the \\ all, the axial s train ' ;
1

and curvature Kt :ire related to the strain and curvnture$ of the bea111 by Eqs. (6.86)
and (6.87).

,a =
{
fu
x
+ y-L
P:.
+ lp,-I (6. 113)

I . I
Kt = - - s1n a+ - cosa. (6. 114)
p, Pr

or
whe re a is the angle between the tangent the wall (coordinate~) and the y-axis
(Fig. 6-22).
The layup or the Wllll is onhotropic. nnd hence the following fo rces and mo-
ments are zero (see Eqs. 6.88 and 6.92):

(6.1 15)
236 BEAMS

Table 6.8. The tensile and bending stiffnesses and the coordinates of the
oentroid of open-section beams with symmetrical cross section. The layup
of each wall segment is nonorthotroplc and unsymmetrical; 71, d',,, and /i,,
are defined in Table6.Z. The elements of the compliance matrix are
evaluated in the wall's e-r1-<
coordinate system.

symmetric.;tl c.r~s section


nonorLhotropic
y un.symme.trical layup

Tensile stiffness
K -
EA= I: V1_!!~'~
t.I (IJ)t

Coordinatc.s of the centroid

f..
1:.':1h~
- ~~
u (7>)4 - i""l4 (l)O;.ll,\
-'<- = I: C>t(611 >t

""'' (l\)4
Bending. sliffness
.:-, <i~1>.'
EiJY = f;. ID)A.
(bz'?+ bl ~?111) - ?t~1>tb z cos a + fi,;._1>.t b cos? a
'k. ~ I! (0).., k 4- 4- 10),. 'k I(

The axial fo rce N, and the moment M, (per unit length) in the wall are
(Table 6.2, page 222)

611 a /3n
(6.116)
'"' = JS'< - 7)/(1
/311 Uu
Mi = - D<, + f) K! .0
(6.117)

z
u,

Figure 6.28: Jllustration or lhc stresses in an


isotro pic (top) and a composite (bottom) rbc-am
subjcctc-d tu bending in the x-z p-l an c~
6.3 THINWALUD, OPENSECTION ORTHOTROPIC OR SYMMETRICAL CROSSSECTION BEAMS 237

Equations (6.110) and (6.11 3)- (6.1 17) yield

M.= -1 ( z -~II - coset-/J11 ) + -1 ( y-J11 +s1n


. a -/111 ) + -FJ -
811
Py D D P, D D ti\ D
(6.11 8)

M, = ..!...
p_,
(-tfuD +cosa~)
D
- .!...
p,
(yfuD +sinCt~)
D
- EA
FJ fi11 ,
D
(6.119)

where l / Pr and 1/ p, are given by Eqs. (6.111) a nd (6.112). Tiie strain- force rela
tionships a re (see Eq. 6.90)

,. fin
,.l ""
0'12 fi21

l~ l
YE" = 0 0
(6.120)
K~ fl11 a,,
K,J fl12 cS12
Ki. 0 0

By a pplying the results of the laminate plate theory (see Eq. 3.7), we obta in
the follo\\ring expression for the strains in eac.h ply:

(6.121)

The stresses fro m Eq. (3. I I) are ( orthotropic la mina te, Q16 = Q16 = 0)

:: } = ~:: g;: ~ ]{:: }. (6.122)


{
" l ~ 0 Q,,. y,.
Equation (6.120) sho"~ that y;,
a ndK,. are zero. Consequently, y.,1 (Eq. 6.121)
is also zero, and the ply shear stress 't
(Eq. 6.122) is zero.
Ortltotropic and sy111111etrical layup (arbitrary cross .Yection). The layup of the
wall is orthotropic and symn1etrical. For orthotropic layup, the following (orces
and moments are zero (Eqs. 6.88 a nd 6.92):

(6.123)
238 BEAMS

The a xia l fo rce N~ and the n1oment ArfE. (per unit le ngth) in the wa ll are (see
Table 6.2. page 222)

I
Mi =-
,-" (6.124)
<II

Equations (6.110). (6.113), (6.114), and (6.124) yield

11 1 1 1V 1
N, = -z- + -y-+=- (6.125)
Py a11 P~ a11 A a 11
I l I . 1
A.ft,= -COSCt'- - - SJO (Y-. (6.126)
Py d 11 p, d11

Fo r ortho tropic and symmetrical layup the e leme nts of the [a J a nd [~ J matrices
area,1 = aq,Ji;; = d;i, \Vhereas /Ji; = 0. Thus, the s tra in- fo rce re la tions hips(Eqs. 6.00
and 3.28) give

,." rail
0 11
0
0
Yt11" IN,}
"l!""
0
M, . (6.127)
"
... " d11
0

The ply strains and stresses are given by Eqs. (6.121) and (6.122).
Arbitrary layup (syn1n1e.trica/ cross sectio11). The layup of the \Vall is arbitrary,
and the cross se ction is synun etrical about the zaxis. The loads a re a pplie d in the
:c- z syn1n1etry plane. Such a bean1 bends only about the y..axis. Correspondingly.
I/ p, is ze ro a nd we have (Eq. 6.6)

0 Fl 1 /iJ,. 1
. =-=
:r - = 0. (6.128)
EA Py = El;., p,

With t hese substitutions. Eqs. (6.11 3) and (6.114) simplify to

I
Kt. = - cosa. (6.129)
Py

In the \\'aJls o f an o pe n cross-section beam, the following forces and n101n en t


are zero (see E q. 6.88):

1V,, = Nt,,, = M,, =0. (6.1 30)


6.3 THIN-WALLED, OPEN-SECTION ORTHOTROPIC OR SYMMETRICAL CROSS-SECTION BEAMS 239

Tiie axial force N, and the moments M, and M,. (per unit length) inside the
wall a re (Table 6.2, page 222)

(6.131)

(6.132)

(6.133)

Equations (6.128H 6.133) may be rearranged to yie ld the axial force N, and
the 111oment Aft (per unit le ngth) in terms of applied force and mome nt:

1VE.= ~ M, ( l 611 li11 ) IV 611


- = - COSOt -=- +~ -= (6.134)
Elyr D D EA D

M,
M, = _ li11 +cosa 1'11)
( - z= = - -=IV li11
= (6.135)
' E~,r D D EA D

The strain- force rela tionships in the x, y, z coordinate systen1 a.re given by
Eq. (3.22). We apply this e quation in the ~ , 11, t coord inate system with N, =
N E.rt = i\111 = 0. Thus, \Ve \vrite

,. P11 p,.
,.' "" /l21 fl26
y~lt
..,.
Ct'J!

P" p,,. { M,
N, } .
"',,
=
/l11 bu a,. 1\-f(lt
(6.136)
/( f311 b 12 ~26
K ( 11 (3,. a,. a
TI1e ply stra ins are calculated by Eqs. (6.121). The ply stresses a re (Eq. 3.11)

(6.137)

The elen1ents of matrix [Q] are evaluated in the ~ , rt, (coordinate syste.01.

6.2 Exa1nple. An. L = 0.6.111~/ong Tsectio1t bean1 lt1fth the cross section shO\VIJ in
Fignre6.29 is 111ade ofgraphirt~ epoxy. T/Je 1naterial properties are given in Table 3.6
(page 81). The layup is [45\/012/ 45\l- The beam is builr-in lit both ends. The
bea1u is subjected to a u1uj'Otnlly distributed load ( /1 = - 1 500 Nini) acting in the
plane of syn1111etry. Calculate the 111axbnu111 ileftectio11 and the ply stresses and
strtu'ns.
l4U tstAM5

h = 2mn1 71 =- JSOO _I!


m
p =- lSOO Nim
J* * * * *4 *, l1 =- 2 mm
L = 600mm

b0= br= SO mm
Figure 6.29: The Th<-am in Example 6.2.

Solution. Fro n1 Table A.2, 'vit h bri set equal to zero. the tensile stiffness and the
location of the centroid a re

- bn hw ,,,.
EA= - - + - - = 21.22 x Lu N
(a11}n (a11)w
(6.1 38)
b,. b. )
= 1 ( bn-= -(
-)-d + -(-)- - = 0.0441 m.
EA a u n 0 11 w 2

For syn1metrical layup (which is the case he re) , & 11 is re placed by a, l The e le ment
=
11 for both the web and the flange is (a11 )w (a11)n 5.18 x 10- 'j!) (Table 3.8, =
=
page 85). The dime nsions bn br = 0.05 m, b.. 0.06 m, and d O.<l6J m are = =
sho\\'11 in Figures 6.29 and 6.30. Setting be equa l to zero in the expressio n given in
Table t\.2 results in the followi ng bending stiffness:

/VJ. =~ (d - z.o)' + ~ + _1_ (/>!., + h!,) . (6.139)


(a11)n (~11)n (a11)w 3

For symmetrical layup (which is the case he re) ~11 is replaced by d11. The ele
me nt d11 for both the web and the flange is (d11 ),. = (d11)n = 33. JO x 10- 3 " 1m
(Table 3.8, page 85). Equation (6.139), with b., =
z, 0.0441 m and b,., b. - = =
z, = 0.0159 m, yields

El,,. = 8 530 N m'. (6.140)

{ ,,
bwl

b,..2 z,
11
[ I/

1-i gure 6.30: 1.''bc cros..<> sec lion of the beam in E:)'.ample 6.2.
6.3 THIN-WALLED, OPEN-SECTION ORTllOTROPIC OR SYMMETRICAL CROSSSECTIOH BEAMS 241

Ki\l cNn1>
4S ii,
0.6 Length, r (m)

06 Lcnath. z 1m
- 450
Figure 6.31: Bending, moment ,\i, and hc1u force ii, for lhc hc:.m 1n Example 6.2.
The distributed load is p = - I SOO Nim. 'l"M corresponding bending moment
II, and shear force P, diagrams arc given in Fig. 6.31. The maximum values are
-- =
M, pL' =45N m
-U (6.141)

v-
'
=
2
pL = -~SON.
The maximum deftection is (Thblc 7.3. page 332)

jjj
1
= - - f!!:'
384 El_,
= - 0.0593 x 10- > m = - 0.0593 mm. (6.142)

The axial force Nt and the moment M1 (per unit length) in the wall are (see
Eqs. 6.l25. 6.126, 6.l 11 . and 6.112)

1\~ =
,q,
-::::;:::-Z-
l
El>'Y 011
(6.143)
,\jv I
1\1t. = ~cosa -,
Ely,v d11
where i is s hO\\'U in Figure 6.30 nnd cos u is zero for the \VCb and + I for the
flange. For co111pleteness we nlso cnlculatc lhc shear now, which will be djscussed
in Section 6. 7. The s hear Oow is (Eq. 6.2R2)
v ,,
q"cs,) = N1 = - ~'
E/1 )
J(:...!..)
nu
d11. (6.144)


The tt coordinate is sho,vn in Figure 6.$0. We next determine the forces at point
Q, which is at the inte rsection or the Oange and the web (z = d - :,) at the built-in
...... ....._. .- nm
end(.t = 0. M, = M , .and V, = v , ) . At point Qthe forcesand moments are

Jij;u l N Af;:u 1 N- m
N, = --=-(d - ;:.)- = 17 209 -
El,, a,, m
M1 = --=--
1,, du
= 0.1594 - -
m
V~'br I N
q"" = N,. = =F-::1::--(<I
1,., 2
- :,)-
lilt
= :1:2 151 -
m
(6.145)
242 BEAMS

Figure 6.32: T he axial forc.e /I;~. t he moment A4. nnd the s he.tar force JV~., an the. flange a nd the
wc.h a t the builtin end of the. beam in x.amplc. 6.2.

l11e plus and minus values of the shear flo,v refer to the left and right of Q
(Fig. 6.32). We also calculated these forces and n1oments around the entire cross
section a t x = 0. The results are sho\\n in Figure 6.32.
The relevant elements of the compliance matrices are (Table 3.8, page 85)
Jn m m
a 11 =5.18 x 10- -
N
a,,=
- - 3.52 x 10- -N
9 a,. =21.11x10- N
I 1 1
d11=33.!0x10--' - N d,, = - 25.59 x LQ-J - - d,.=48.51x10- 1 - N
nl - Nm . Ill

At point Q the strains are calculated by the strain- force relationships (see
Eq. 6.90 with Ct replaced by a,~ replaced by d, and fj is zero)

a11 0 0 0.0000892
'
f~ 0 11 0 0 - 0.0000606
Y~it
=
0 a.,, 0
1~~1 } - 0.0000597
(6.146)
.."',
K ( I
0
0
0
0
0
()
d11
d,,
0
M,
= 0.005 28
- 0.00408
0

\vhere K t, K 11 , a nd K~,, are in 1/ m. With the preceding strains. the ply stresses are
calculated by Eqs. (6.121) and (6.122). The results are given in Figure 6.33.

z ,
I '
I I
0
"
o.s 0 -' -o.4 0 .4 u, 2 T r.&-

- 1
I - I
I
figure 6_13: The ply s tresses in t he. beam at the buih in end JUSt lc(I of the vertica l symmell)'
axis of t he Range in Example 6.2. ' l'he uni I o( the s tresses is 10< Nhn 2 (Just to t he right of thc-
S)'mmctr)' ll)'.is. r.1.y is ne gati\'c.)
6.4 THINWAllED, CLOSEDSECTION ORTHOTROPIC BEAMS 243

y y

figure 6.34: Thin-walled. closed sec lion be-ams \\ith cur\'e-d a nd straight "'all se-g mcnt:>suhjec-ted
to axiaJ load and bending moments.

6.4 Thin-Walled, Closed-Section Orthotropic Beams Subjected


to Axial Load and Bending
In this section we treat thin-v.aUed, closed -section bean1s subjected to a..xial load
and bending. The beam may consist of flat and curved walls (Fig. 6.34). The beams
are subjected to an axial rorce IV and to beading 1uoments 1fif.v. :fl:..
Ty.o types of bean1s are considered:

1. The layup of the wall is o rthotropic and symmetrical; the beam's cross section
is a rbitrary.
2. The layup of the wall is orthotropic but unsymn1etric.al; the beams cross section
is arbitrary.

'TI1e re a re no te nsion- bending- torsion couplings in either of the latter types


o( bean1s because the bean1 is orthotropic (page 207).
We define a ~ -11-~ coord inate system along the wall. Al each point~ is parallel
to ll1e x coordinate, 11 is along the circumference of the wall. and ~ is perpendicula r
to the \Vall (Fig. 6.34). The origin of thiscoordinatesysten1 is in a n arbitrarily chosen
reference surface.
Displace111e11ts - synutretrical layup. The layup of t he \Vall is orthotropic a nd
symmetrical. Under the action o f t he applied loads N. M,.. and M,, t he shape
of the cross section c hanges. To exan1ine the change in shape we firs t look at an
isotropic beam. The c hanged shape of the cross section of a solid isotropic beatn
is shown in Figure 6.35. The c.hanged shape of a thin\\aJle d isotropic be.an1 is the
same as that of a solid bean1 'vith the san1e initial cross section. The re is only axial
stress in the beam. a nd therefore . the cross-sectiona l shape 'vould not be altered
if the bean1 \Vere cut.
t\ thin-\\alled con1posite bean1 \vith orthotropic and S)'lnme tric-al layup be-
haves sin1ilarly to a n isotropic. beam, nan1ely, under the actions of an axia l load
and bending the change in s hape of the cross section of a closed-section beam is
nearly the sa1ne as the change in shape of t he cross section of an open-section
beam (Fig. 6.36, middle). This implies that closedsection beams respond to the
applied loads in n1uch the san1e \\'ay as opensection bean1s. The refore, \\'e may
treat thin-walled, closed.section bean1s \\ ith orthotropic a nd symmetrical layup
1
244 BEAMS

Undefonncd Deformed
cross sct:tion cross section

Solid
D D
Closed
section
beam D 0
Open
section
beam D 0
Figure 6-1-5: 1'hc changes of the c.ros..; sections orsolid and thin-\\allcd open and closed-section
isot ropic bean1s subjcc.tcd to a:tial load and bending.

in the same way as we treat open-section bean1s and calculate the displacen1ents
with the replacement stiffnesses given in Table 6.3 (page 231).
Displace111e11ts - u11s)'n1"1etrica/ layup. The layup of the \\aJI is o rtho tropic
but unsy1nme trical. \Vhen such a bean1 is s ubjected to loads FJ, J\1.l., and 1 \1z. the
s hape o( the cross se ctions changes significa ntly, as sho\\'11 in Figure 6.36. In this
case \Ve can no longe.r approxin1ate closed-se ction beams as open-section beams.
An open-section beam 'vould "open up .. under the axial load and bending, as
s ho\\n in Figure 6.36 (right). Sin1ilarly, if a closed section beam \\'e re c ut, the two
cut edges 'vould move re lative to each o ther, as shown in Figure 6.37. Jn a closed
section beam these displaceme nts are prevented by a s hear force Nt.t'/ a be nd ing
moment 1\111 , a nonnal force /\~,, a nd a transverse shear fo rce \I" acting alo ng the
c ut. Since t he 'valls are o rt hotropic, (a) the bean1 does not tv.ist (K~., = 0), and
(b) the t\\'O "cut" e dges do not n1ove relative to ea ch o the r in the axial d irection.
In the a bsence o( these Lnotions no s hear force arises along t he cut (1V{,, = 0). The
forces N a nd v, are gene ra lly s mall a nd can be negle cted (N, =
0, V, 0). Thus. =

Undeformed Symmetrical Unsynunctrica l


cross section layup lavup
Open-
section
beam D 0 [J
Closed-
section
beam D 0 D
Figure 6.36: T he changes o{ lhe cross sectionsofthinwaUc.d opc.n- a nd dosed-section co mposite
beams subjected to axial load and bending.
6.4 IHIN-WAl.UU, CLUSEUSECllUH UHTHOIHUPIC H ~AMS 245

Closed-S'bon
bum tSC 0:: 6C
J\~'f ;l/" /\' "

~ t:9 qp a-.~
N,
Figure: 6_17: 'Jhe forte$ :1long the lcnglhwilic-edges of an opcn11eclio1' bctun {lop) nnd ulong the
cut of a c:loi1cd11cction bc:Jm (hottom).

in the ca.sc o( ortho1ropic walls \Ve have

N, = N,, = 0
c.l osed-section. (6.147)
"'" = 0
v. = 0
There is a bending moment Af4 along the cut that prevents rotation of the
edges reh1li,c 10 each other. The slope of the ,vaJJ musl be the same 10 the left and
to the right of the cut (Figure 6.38)

aw"J
-;;;j' 1lx,hl =
aw"J
iJ11 lefl.
(6.148)

where ul 1 is the displnce rnent of t he \Vall perpendicular to its re(ercnce surface.


The change in curvature of the wall is
azul'
I('I = - -
iJ11l
(6.149)

By intcgrnting this equation from an arbitrary point'' = 0 tos1. we have

1""'''' = - [aw]"
---
a,, - o
(6.150)

'I
Figure 6,Jg: k cll'llhc rot11t1on o f l hc cut edges.
"'"
Undeforn1t
cross section Bending ..\.xial load

D 0 [J
0p<n
sec.tion
beant

c'loscd
section
beam D 0 D
or
Figure. 6.39: The change..; of the cross sections thin-wallc-d open and closcd-scctjon beams
subjcc.1cd to axial load and bending.. Each wall hai> t he same o thotroptc unsymmclrical ln)'Up.

By perfonning the integration along the entire circumfere nce (s 1 = S). we ob-
tain (see Eq. 6.148)

(6.1 51)

This compatibility condition must be enforced for close d.section bean1s. An


analysis that ta kes thiscondition into account is presented in Section 6.6. T his exact
analysis is con1plex. Jn t he next paragraph \Ve present a sin1pler 'vay to detern1ine
the replace1nent s tiffnesses 'vhen the layup is the san1e around the circumference.
To esta blish the be nd ing replacen1ent stiffnesses, \Ve observe that bending the
c ross~sectiona l shape of a closed c ross.section be.an1 with uniJonn layup around
the circun1ference is sin1ilar to the shape of an open.section beam (Fig. 6.39). In
this case the replacement stiffnesses EIJ',' El:.~ and El,.:. may be approximated by
the expressions obtained for opensection bean1s (Tables 6.4 and 6.7, pages 232
and 235).
To establish the te nsile replacen1ent s tiffness, we observe that the beam \\ith
uniform layup does not deform under tension (Fig. 6.39). In this case the diange
in curvature is zero along the circun1ference

closedsection
(6.152)
in tension.

By substituting Eqs. (6.147) and (6.152) into Eq. (6.90), after a lgebraic mani-
pulations. we obta in

., = (1")
, ,~ D " (lf")
jj ~t - Kt, (6.153)

(6.154)

(6.155)
6.4 THIN-WALLED, CLOSE.D-SECTION ORTHOTI!OPICBEAMS 247

where &11 #11 .11 1. and 15 a re defined as


-au= ( JI' ~,)
011 - /I- - (/I 11 - /112812)
11---
812
(6. 156)

-a.. = (&11 - &')


:u. &u
(6.157)

By romporing Eq. (6.153) with the expression for N1 given for open-section
beams in Thblc 6.2 (page 222). we see that they differ only by the terms in the
brackets. Therefore. the 1ensile stiffness EA o( thin\\'alled dosed-section beams
(ortbotropic but unsymmetrical layup) may be calculated from the expressions of
open-section beams (Thbles 6.4 and 6.7, pages 232 and 2.lS) by re placing 811 , D by
iu.15.
111e \11, lA':u. \V23( = \\131), \1-' elements of the compliance nlatrix are then
detenuined by s ubs1ituting the replacen1ent stiffnesses (given in Tnblcs 6.4 and
6.7) into Eq. (6. 19).
TI1e expressions for EA. 111 , El~, and Eiy~ in Tables 6.4 and 6.7 are also
reasonable approximations "'he n the layup is not uni!onn but the "'all is .. thin ...
Slrts.s#S a11d .vtrai11.f. As "'ith displacements. the stresses and s1rains in c-losed-
section beams with orthotropic and symmetrical layup can be cnlculated by the
same exprc.ssions used for 1he stresses and st rains in open-.sec1ion beams (see
Section 6.3.4. orthotropic and S)mmetrical layup).
\Vhen a closed-section beam \vith onhotropic but unsymmetric.31 layup is in
bending. the stresses and strains c.an be calculated by the same expressions used
for the s tresses and SITains in open-section beams (see See1ion 6.3.4. orthotropic
layup. arbitrary cross sec1ion).
When a closed-section beam \vith orthotropic but unsymmetrical layup is in
tension , '''C dclcnnine the s1resses and strains as fo llows.. At an arbitrary point
on the reference surface of the 'valJ the axial strain is related to the s train of the
beam's nxis by (Eq. 6.11 3) as follows:
(6.158)
\Vhe n t.hc layup of the "'all is orthotropic, Nit = N(" = i\1t" = 0. a nd the axial force
and Lhe moment (perunit length) in the wall are given by Eq$. (6.153) nnd (6.154).
Thus.. ~'e have
fJ 611 fJ ii11
Ni = Cl i5 M, =
EA D
--=-=- (6.159)

The strnin- foroe relationships (see Eq. 6.90) give the strnins

( au Pu /111
a11 /J12

{~}
/J11

rt.. = 0 0 0
(6.160)

..,
1

i.
/J11
fJ 11
()
811
812
0
6i1
611
0
where Af,1 is give n by Eq. (6.155).
248 BEAMS

1-i gure 6.40: Beam s.ubjected lo lorque f.

The strains and s tresses inside t he wall (ortho uopic, unsy1n1ne trical layup) are
calculated by Eqs. (6. 121) and (6. 137).

6.5 Torsion of Thin-Walled Beams


We consider thinwalled. open and c losed.section bean1s subjected to pure torque
(Fig. 6.40). Becauseo( the torque, the bean1t \\~s ts about an axis that passes through
the center of t\vist. \Vhen the beam is isotropic, the t"ist axis ren1ains straight.
For a con1posite beam the t'vist axis becomes cur\'ed, except \\he n the bean1 is
orthotropic.
The stresses differ in thin ..walled isotropic a nd composite beams. Under the
action of pure torque there are only shear stresses inside t he 'vall of a n isotropic
beam, whereas there are also normal stresses inside the \\all of a composite beam.
Under the action of the torque, e ach cross section rotates (t,vists) as a rigid
body in itso\vn pla ne a bout the centero( twist. In a ddition, the cross sections suffer
\varping displaceme nts nonnal to their (c ross..sectional) planes. We a re interested
in the twist, the warping displace111e.nts, and the slresses inside the walls.
Stiffness and rate of twist. Under the action of pure tors ion (no a:dal con-
straint}, the rate of t\vist of a n isotropic bea1u is (Eq . 6.4)

f
O= - isotropic. (6.161)
GJ,
Sin1ila rly, for a cotnposite bean1 \Ve write

f (6.162)
0=-= composite.
G/1
\Vith these defl nitions the rate of t\vist of composite beams can be o bta ined by
replacing the stiffnesses G /1 by Gt 1 in the expressions for tJ1e torsion of isotropic
beams. ln the fo llowing \Ve obtain expressions for G/1

&.5.1 Thin Rectangular Cross Section


We consider a solid, rectangular, thin beam (Fig. 6.41). T he layup o f the beam is
arbitrary. The " 1dth of the beam I> is large compared with it< thickness /r (b > > Ii).
T he applied torque gives rise to shea r stresses in the bean1 that n1ay be re presented
by a resultant s hear flow q, the unit of \\hich is force per length. This induced shear
6.5 TORSION OF THIN-WALLED BEAMS 249

Figure 6.41: 'Ille shear slrcss distribution. lhc twist moment. and the shear ftow in a solid thin
beam under lorsion.

equilibrates the applied torque (Fig. 6.41),

f = (qd)b' + (qb')d, (6.163)

where d and b' a re the length and width of the path of the resultant shear flow.
The latter is take n to be approximately equal to the width of the beam (b',.. b).
The l\Vist 1non1ent Mxy (per unit Length) in the cross section is

i\(,y = - qd. (6.164)

Equations (6.163) and (6.164), with the approximation b' = b. yield

f= - 2bM,, . (6.165)

When the bean1 is treated as a plate, the out-ofplane cunature K.TJJ is (see
Eqs. 3.22)

(6.166)

TI1e rate of t\\1st D is (Eq. 6.27)


ay, I
t? = - = - -K:t}" (6.167)
ax 2
By combining Eqs. (6.166), (6.167), and (6.165), we obtain the rate of twist

?= a-4b..T- . (6.168)

whe.re 86(1 is evaluated at an arbitraril) chosen re(erence plane. By comparing


Eqs. (6.162) and (6.168), we see that the torsional stiffness of a fiat t hin-walled
bean1 is

Cl, = 4b.
... (6.169)

= =
For a thin-walled isotropic beam, 666 d,,,, 12/ Gi1 3 (Eq. 3.43). a nd the tor-
sional stiffness in Eq. (6.169) becomes Gi, =GI,= G";'.
250 BEAMS

\Vhen the \\ all is not Hat but is curved, the torsional stiffness 1nay be approxi
1

ma ted by
s
- 11....- tf,,,
G/ 1 = 4 (6.170)
0

'vhere '1 is the coordinate along the circu1nference of the \Vall and Sis the length
or the e.ntire circumference.
Although the beam is subjected to pure torque, there are axial ~; and shear
strains Yx0>" and there is a change in the curvature Kx given by (see Eq. 3.22)

(6.171)

TI1is equation together with Eq. (6.165) yie lds

(6.172)

To detern1ine ll.i, \V24 , and l\'4.a elements of the oon1pliance matrix, \Ve rear
range Eqs. (6.168) a nd (6.172) in the form

{ Kx ~:/Py} = {=;~::} f. (6.173)

By comparing this equation with Eq. (6.17), we see that the terms in brack
ets are the IV14 (= - /1 16/ 21>), IV24 (= -~ 16/2b), and 1..,(= ~,,,,/4b) e le ments of the
con1pliance 111at rix.
\Vhen the layup is syn1n1etrical, /366 is zero and there is no shear strain. When
the layup is orthotropic, {J16 and 8 16 are zero. and there is neithe r axial stra in nor
a c hange in curvature. \\'hen the layup is bo th orthotropic and syn1metrical, there
are neither axial nor shear strains and the re is no change in curvature.

6.5.2 Open-Section Orthotropic Beams


The \\ all of an open-section be.an1 tnay be considere d to consist of t hin wall seg
1

me nts (Fig. 6.27). The to rsional stiffness of a thin-walled, open-section isotropic


beam may be approxin1ated by

(6.174)

'vhere the subscript re fers to the kth segn1ent. Similarly, \Ve a ppro:cin1ate the tor
sional stiffness of a con1posite be am \\ith orthotropic \Valls by

Gi, = (Gi,), + (GJ,h + . . . +( Gi,)K. (6.175)


6.5 TORSION OF THIN-WAllfD BEAMS 251

Figure 6.42: \Varping of an opcnchon com


po.site beam due to tor.sion and due to 1hcJ1r
strain.

For Oat and curved segments. (Gl,). are given by Eqs. (6.169) and (6.170).
The rate of l"\\'ist is calculated by

(6.176)

Stras~s and strains. ln each \Yall segment we en1ploy a (-11-{ coordinate sys
tem. The origin or each coordinate system is at the \Vall segment's reference sur
face. The out-of-plane c:urwture of each wall segment is obtained from Eq. (6.167)
by replacing x and y by { and r1 as follows:
(6.177)
We consider only Mh resulting Crom the 1orque. Thus we have (see Eq. 6.90)
(6.178)
(6.179)
Equations (6.178) and (6.177) give
1 2
J\ftq = -KtlJ = --11. (6.180)
a... a.,,
Equations (6.121} and (6. 122) together with Eq. (6. 179) yie ld tlie strains a nd
stresses

{ :: } = {
Yt11
~ } +~ { ~ }
Yt!J "'t u
(6.181)

{ :: } =
f4,,
[J]
Q"
Y!
(6.182)

lVarpi11g. A Lhin-waUed. open-section beam warps "'hen subjected to a pure


torque. Under torsion the ~all of a beam (no shear deormntion) \Varps, as sho\\'ll
in Figure 6.42. (left). The axial displacement 11 relative to an arbitrarily chosen
reference point (~ = 0) is

t1(1l - 11(0) = -2;1.,0. (6.183)

T. H. G. Mcpon.Auc1ft Slmctwafw En_pw111t1 .fOl,kNJ ltd rdrtion. H!llsltd PrtU. Joh.JI \\'iky
& Soos. New Yort. IWCJ. p. 318.
HtAM:i

Figure 6.43: The swc:pt a rea.

\\here fJ is lhe rate of t\\~Sl, and A,;1 is the S\\'ept area f ron111 = 0 to '~ = s 1 (Fig. 6.43)
about the center o( twist (point 0). For thin-\va lled bean1s the center of t'vist and
the shear cente r coincide. The shear center is defined such that a transverse load
acting a t the shear center of an orthotropic bean1 does not cause t \\ist (see Sec
lion 6.7.3).
When there is no she ar strain the angle bet\\een the c.irc.un1fere ntial and the
longitudina l e dges re mains 90 degrees (Fig. 6.42). For a con1posite bea1n t he shear
strain is zero \\he n /JM is zero (see Eq. 6.178). \Vhen {J66 is not zero the originally
90 degree angle bet\veen the circumfere ntia l and the longitudinal edges becomes
4

90 + Y f,, This change in angle introduces t he additional axial displacen1ent along


the circun1fe re nce

11(s1) - 11(0) = J" r<,,d11.


0
(6.184)

111e total re la tive axial displaceme nt (warping) is (see Eqs. 6.183 a nd 6.184)

611=11(s1) - u(O) = l
-2A,,~ + Y~,dry.
0
(6.185)

For a thin ..\valled. open-section bea m with o rt hotropic layup r,, is given by
Eq. (6.178).
\Vhen the layup of each wall segment is syn1n1etrical, y(,, is ze ro, a nd such
con1posite beams warp sin1ilarly to that of isotropic beams.

6.5.3 Closed-Section Orthotropic Beams - Single Cell


For a thin.,valled, cJosedsection bean1, the points '1 =0 and '1 =S coincide
(Fig. 6.44), a nd thus the warping is

611 = 11(0) - 11(S) = 0. (6.186)

fisure 6.44: 'f he rela1ive- dt.<iplacement a t the '"cur of a c losed


sc.cllon be-ant subjected lo pure torque.
6.5 TORSION OF THIN-WALLED BEAMS 253

Figure 6.45: 'fbe torque canicd by 1\\-11 and b)' iWt,,.


where S is the le ngth of the circum(erence. By employing Eq. (6.185), we write
s
Liu= - 2All + J
0
r(.,dq = - 2Ail + f yf,d11 = O, (6.187)

y~,
where A is the enclosed area and 1 is the shear strain. Fro1n this equation the
rate of t\vist is

0=
1 1.
A .f Y,,dq. (6.188)
2
ln thin\\aJled, cl osed~section beams the torque due to the induced t\vist mo
n1e nt :\1~~ is s mall con1pare d with t he torque d ue to the induced shear force NE,,
(Fig. 6.45). In the follo\ving v.e neglect the twist mome nt M~ 11
We introduce the s he ar flov. q defined as the integral of the s he ar s tress across
the thickness

q= f r,,d~
(11)
N,,, (6.189)

where it is the thickness or the waU. The torque produced by q (or N,, ) is (Fig. 6.46)

1' = f qpt111=2Aq, (6.190)

where p is the distance Detween an arbitrary point and the tangent to the \Vall.
Accordingly, the s hear fto\v is

f (6.191)
q = 2A.
Equation (6. 191) is the some as the Bre dt- Batho formula' developed for thin-
walled, closedsection isotropic bean1s.
Equation (6.90) yields
(6.192)
To sin1pliCy this expression '"e select a re ference surCace at 'vhich !J66 (denoted
by ~;0 ) is zero. l11e 66 con1ponent of the con1pliance matrix corresponding to this

1 T. H. G.1''1ci.<SOn, AiYcraft S1n~cturesftJr Engi11nrl118 Studer11S> .lrd cdi1ion. Ha1!>tcd Pr~ Jo h n \Vik)'
& Sons.. New York. 1999, p. 307.
254 BEAMS

figure 6.46: The torque. carried by t he. shc.ar Ao"' 11.

s urface, \\ hich v.e refer to as the "torque neutral'' surface, is (Eq. 3.48)
1

fj~ = IJ66 + '1'5ti6 (6.193)

'vhere f366 and ~66 are the components of the co111pliance n1atrices in the arbitrariJy
chosen reference surface, and v is the location of the "torque neutral., surface
(Fig. 6.47). For fJ;;. = 0, Eq. (6.193) yie lcl<

"= - fJ..
-
8.,,
(6.194)

By applying Eq. (6.192) a t the " torque ne utral"' surfac.e, we have


(6.195)

'vbere the superscript v refers to the ''torque ne utral surface (Eq. 6. t 94). and &66
is given by Eqs. (3.48) and (6.194) as follows:

" =
(l66 Oi66 + 2.
''"'66. + v,,066 = 0t66 - & (6.196)
8
By combining Eqs. (6.188)- (6.195), we obtain

(6.197)

With refere nce to Eq. (6.162), we see t ha t the replaceme nt torsional stiffness Ci,
is
- 4A2
GI,= y-;;--d (6.198)
.'l' CIM '1
\Vhen t he 'valJ"s layup is syn1metrical, a~ = a66 For a thin~v.aUed isotropic
beam,<>66 = a.. = 1/ Gh(Eq. 3.43). a nd the to rsional stiffness (Eq. 6.198) becomes
.- 4A:
GI, = GI, = G/C l/ hld -
The "'4.t element of the compliance ma trix is \V44 = l / G/1 .

Arbitrary ..Torque nc.utral''


rcfcrroc-e surface surfact'

" ,,
Figure 6.4 7: ' Jbc torque neutral sur(ace. \\1hcre. iM is ze,ro.
6.5 TORSION OF THIN-WAI.LED BEAMS 255

,,
1,,. 52 nt1n

-I[]fl
p =- 6SOON/m
bw 68mm
t*******i h 2mm
l = tOOO mm df SO mm
rl 70 mm
. d .
1. t i
Figure 6.48: The beam in Eumplc. 6.3.

Strtssu an.d strain. The strai1U and curvature in each ply a t the torque neu-
trar surface are given by (sec Eq. 6. 90)

0

Yt',
K(
= 0
0
a .. N,,. (6.199)

Ko 0
Kh 0

where N1. q is the sheor flow calculated hy Eq. (6.191).


With the preceding strains nnd curvatures. 1he s1resses are calculated by
Eqs. (6.181) ond (6.182).

6.3 Exantple. An L = l .0..111 ... fong box bel11tJ, udrh the cross section s!JOH'n in Fig
ure 6.48. is 1nade of graphit~ t /) O.f)' 1'1H nJttferlal properlies are given in Table 3.6
(page8/)- Tire layup i" [45~/012/4SIJ. Tlie beam, b11i/1-i11 at eacll emf. is s11bjec1ed
=
ro a t1nifor111/y di,strif111ted load (/1 - 6 500 Nhn) 11ctiug ttt 1/Je ulidplnne of tire lefl
lt1eb (Fig. 6.49). By negl ecting the cjJ'ec1s of 11.rial restraint, C(l/culate tire 1naxiu1111u
dejfettiou and the 111axinuun 1111i.\'I,

Solution. The cross sect ion is doubly syn1mctricnl, nnd both the centroid and the
shear center (see Section 6.7.3) coincide \Vith the center of the cross section. The

p =- 6 SOO Nim
n',,

z
i<---..1
d,
~
2
Figutt 6.49:: Load.mg on 1hc box beam 1n Eumpk 6..3 (kft). and the loaduig .,,h respect lo the
shear ceo.ter (ngbt).
256 BEAMS

~(Nm)
541.7 /1
~ l.O~ength,x (m)

81.25
b:. <:::::::::j l.0 '
Length, :i: ( n1>

Figure 6.50: 'l'he bending moment i\11 . shear force: Ci':. and torque. f diagrams for the: be-am 1n
Ex.ample: 6.3.

loads \\1th respect to the shear center are (Pig. 6.49}


N
p = - 6500 - (distributed load)
m (6.200)
d1 Nm
t = - p x - = 162.5 - - (distributed to rque}.
2 lll
The corresponding bending n1on1ent 1Wy.shear force V~, and torque f diagrams
are shown in Figure 6.50. The n1aximun1 values are

'
nrn~x 2
pL
.u, = - )2 = 541.7 N . m
v"~' = pL = - 3 250 N (6.201)
' 2
f"~' = tL =81.25N m.
2
The compliance matrices for the flange and the 'veb are the san1e., and their
relevant e leme nts are n:,ble 3.8, page 85)
m m _ 1
a11 = 5.18 x J0- 9 - a,. = 27.77 x 10- -N d11 = 33.10 x 10-> - N
N m
The dimensions are (Pig. 6.48) be= 0.052 m, bw = 0.068 m. d = 0.07 m, de=
0.05m.
From Table A .I the bending stiffnesses are

- be d' 2bc
E 1vr = - - - + - - +
2b! 34692N m'
(a11)r 2 (d11)1 12(au }w
(6.202)
Ef .. =~df + 2b.. + 2bi 20924N m' .
(ll11}w 2 (d11)w 12{ll11)1
The torsional stiffness GI, is (Table A.6, with a;:. replaced b y a66)
2
-- 2di d :!
GI, = aa6(, I) = 7 352 N m . (6.203)
fir+ (,
b.b I UK~IUft UI" I "INWALLtU tU:AM~ lbl

br=- '2 mrn


bw s 68mm
h =- 2mm
d,= SOnvn
d =- 70mm

.\- 24000N
Figure 6.SJ : Cron Kct1o n or lhr beam 1n Examplc 6.4 and 1hr l~d 11ppl11:.d on the bram.

The maximum dcAcction is (Table 7.3, page 332)


- l pl.'
w = - "?" = - OJ.I()() 488 m = - 0.488 mm. (6.204)
384
E1,1
111e mnxin1um t\Visl is al x = L/2 and is

'/! =
1.o
1./l 11tfX =1'n" -=-tfX.
f
G/
(6.205)
1

The torque r varies linearly with x. and !he preceding integral yie lds
f-'L
t = ~ = 0.002 76 rad= 0.158'.
4..,,, (6.206)

6.4 Exampk. An L = l .0.111/ong box bta111. witlr d1t cross staio11 sh~11 i11 Fig
urt 6.51, is 11u1dt ofgraphite epoxy. Tlrt 111art.rial proptrtits an glvt-n in Tobit 3.6
(page 81). Tlr. layup i.< I45~/0 10]. 17refabric iso111heo111sid<ofrh mll. 71re b.am,
=
l>ui/1./11 al eaclt end. is subjected 10 a unifortuly dis1rlb111ed loud ( p - 6 500 Nltn)
acling along rht 111idpla11e of rite left iveb and to an axial load N = 24 000 N. By
neglecti11g the effec.~rs of axit1l restraint, calculate tire nuLti11111111 deflection a11d the
111axi11u1111 uvi.rt.

Solution . The c 1os.~ section is doubly S)'llln1etrical. and the ccn1roid a nd the shear
center (see Section 6.7.3) coincide \vith the center of Lhc cross scc1io11. TI1e loads
Yr'ilh respect LOthe she ar cen1er and the corresponding be nding mome nt 1 \.i ,. shear
force V,. and torque f .re given in Example 6.3 (page 255).
The layup or each "'311 is orthotropic and unsymmetrical. The compliance
matrices for the flange and 1he web are the same. and their rclcV3nl ele ments are
(see Table 3.9. page 86).
m
a;:'= 11.65 x t 0-9 -N a:!' = 43. 70 x 10- mN
l I I
tJW = - 13.97 x 10- -
N
>1r = 12 22 x JO-" -
{J12 N tJ-::: = 51.60 x io- -
N
1
6'11' = 34.94 x 10"' - -
1
N in
&~t = - 25.74-NI 6~ =98.83 -Nm
-.
11 m
1
6!',!" = 131.l I x 10- i - -
N m
.......
/ Neutral plane
o,o....... 7.. . . . . . . . t e-
452' t

IE
Figure" 6.52: The neutral plane.o f Lhe beam in Example 6.4.
\vhere MP refers to the midplane of the wall. The dimensions of t he bean1 are
(Fig. 6.51) br = 0.052m, b. = 0.068m, d = 0.07 m, and d1 = 0.05m.
To evaluate the bending stiffnesses, \Ve determine the position of the neutral
plane (Eq. 6.105 and Fig. 6.52) as follows:

(i = - ::r
/3't>.f1'

~II
= 0.0004 m = 0.400 mm. (6.207)

At the O reference plane the con1pliances of the flanges and the 'vebs are
(Eq. 3.48)

Ctl I = a~:r + 2Q{3~:r +


1
e m
st:r = 6.056 x 10- 9 N
(!,.. = f!l:f,'' + Q~~ = 104.04 x 10... ..!.. (6.208)
N
~11 11 =34.9 4 x 1o->- N
=J "" I
. 01
From Table A.3 the be nding stiffnesses are
-, l
-11r = -
E br -d- + 21,,
-- + 2b-., = 29 214 N m' (6.209)
(et11)e 2 (J11)1 l 2(a11)w
-- bw dl 2bw 2bj !
(,=-(-)-~ +-(,
l +
au w 2 011 .....
( ) =17 463Nm,
12 a 11 r
(6.210)

where the dime nsions J and de are (Fig. 6.52) J = d - 2(1 = 0.0692 m, d1 =
de - 2i1 = 0.0492 m.
The torsional stiffness Gi1 is given in Table A.6 Y.~th reference to the Bneutra l
plane as
2
-- - 2dfd - 2
(6.21 1)
Cf,_ a;:.(de+d) - 8726N m.

where a;;. is (Eq. 6.l 96)

~"'' - 10-9 ~.
2
' -
66 - "'66
(f!,';J,")
()~tJ>
-
-
2340 x N (6.212)
" 66

T he accuracy of Ci,, as calculated by Eq. (6.2 11), is improved whe n Ci, is evalu
ated in reference to the "torque neutrar surface. Equation (6.194) gives
11= - f!i:f,P/J't,f," = - 0.394 mm. When d a nd de a re replaced by d - 211 = 0.0708
and de - 211 = 0.0508 m, respectively, Eq. (6.211) results in Ci, = 9 087 N m2
Next, we c:alculmc the tensile stiffness. From Eqs. (6. 156) and (6.157). we have

_.,,.
{J -
II -
(fJ"'' - Pt~'' tl)- - to 79
ll
6
t.tP
6~
- . X
10-< .!._
N
(6.213)
6""
u -
- (1"'
u
- (&~i"J')-- 28
&~' .
1
23 x 10-J - -.
Nm

The location of the referen<e surface is (Eq. A.3)

"""11, =0.000 382 m = 0.382 mm.


o= - ~ (6.214)

'"
At the e reference surface. the complianc.e.s of the flanges and lhe \Vebs are
(Eq. 3.48)

<tu = &~:i + 2Qp~;IP + Q28~:r = 6.067 X 10- 9 ~

fJ12 =fJ~J'' +1/6~l" =2.380 x 10- ~


I
611 =aw =34.94 x io-' --
N m (6.215)
1
1., = 1w = - zs.14 x 10-' - -
N m
I
- = &ti = 98.83 x
'" 10- 3 - - .
Nm
With these rompliana:s from Eqs. (6.156) and (6.157). for both the Hange and
the ~eb \Ve hnve

- ( (/112)') _9 m
'"' = U!! - ~ =6.01x10 N
(6.216)
- 11 ( 11 (612)') l
8 = ~ - &,;, =28.23 x 10- 3
N. n

From Tublc A.3 the te nsile stiffness is


,.,..,
""' =
2b, 211.
- +.,,--) =39.934xl
(-)
o N. (6.217)
U11 f (011 w

The moximum dcOection is (Table 7.3. page 332)


I pL'
iii = - ,,,., = -0.000 579 m = -0579 mm. (6.218)
J8.S cl,.,
The ma.ximum twist is at x = Lf2 and is

>/t = 1 ~tl.r J.
U2
=
U2 f
&, dx. (6.219)

Torque T varies linearly \Vith x, thus, the integral above yields


f"' 11.. L
>/t = ~ = 0.002 33 rad= 0.133'. (6.220)
4(il,
260 BEAMS

\\/here fm:iJr. = 81.25 N~'" (Eq. 6.201 ). The elongation of the bean1 is

1L fii FJL
1
1.
t;. L= s,tlx = -=dx = -= = 0.000 601 m = 0.601 mm. (6.221)
o o EA EA
6.5.4 Closed Section Orthotropic Beams - Multicell
For multicell con1posite beams the rate of t\vist a nd the shear flo\\'Sare detem1ined
in the s..1n1e v.ay as for isotropic bean1s. Accordingly, for a beam consisting or L
cells the torque is
I.
f = I; 2A,q,, (6.222)
,_,
\\here A 1 is the enclosed a rea or the /th cell and q1 is the shear flow in the /th cell
(Flg. 6.53). The shear strain y. in each wall segment iscakulated by (Eq. 6.195)
(6.223)
\vhere q is the shear flov. in the wall segme nt. Then, the rate of twist of each cell
is (Eq. 6.188)

11, = ,!--
_ A,
f ;:.qt111
i:dl l

(6.224)

For a v.aJI that De.longs to one cell only, q is equal to the shear flo,v of the
appropriate cell (q = q1). For a wall that belongs lo two cells, the shear flow is the
sum of the shear Ho,vs of the adjacent cells, as illustra ted in Figure 6.53.
The rate of t\vist of each c.ell is the same
(6.225)
There are L + I equations (Eqs. 6.224 and 6.222) that can be solved for the
L + I unknowns: q1 (I = I. 2, . .. . L) a nd O.
The stresses in the \\'alls are calcuJated a....; for a single -cell beam.

1-i gure 6.53: She.ar flo''' in multicell be-ams.


6.5 TORSION OF THIN-WALLED BEAMS 261

v,

Figure 6.54: Thin-,,alled I-beam.

6.5.5 Restrained Warping - Open-Section Orthotropic Beams


\\'hen an isotropic beam is subjecte d to pure torsion and the cross sections of the
beam are free to warp, the torque is (Eq. 6.161)

(6.226)

When the bean1 is axially constrained a t one of its cross sections (for exan1ple
at a builtin e nd). then. at this cross section, warping cannot occur. The shorter the
bean1, the n1ore important are the effects of restrained \varping. Furthern1ore, the
effects o( restrained \\'arping a.re more pronounce d for thinwalled op-ensec.tion
bean1s than for thin.walled closedsection bean1sand for solid cross.section bean1s.
Therefore. in this section, \Ve consider only restraine d \Varping of open-section
bean1s.
We illustrate lhe analysis through the exa1uple of a sy1nme trical I-beam
(Fig. 6.54).
Isotropic I bea111s. Under pure torque, in lhe absence of ax:ial constraint. the
shear stress distribution in an isotropic l ~beam is as shov"1t in Figure 6.55. When
the beam is axially restrained, axial forces 1V~ (per unit length) a re introduced in
the \\all (Fig. 6.56, botton1). These axial forces ere.ate a bending n1oment Mrg

....... d 2 ur
M 1 = - -d , Et,, (6.227)
x-
where 11 is the moment of inertia of the flange about the taxis denoted by

Ir= h,b/' (6.228)


12
brand II, are the \\id th and the thickness of the Ranges (Fig. 6.54}, respectively; "'
is the displacement of the Hange due to the rotation about the x -axis (Fig. 6.57),

vr =~ 2 " (6.229)

where d is the distance bet\\een the n1idplanes of the Hanges. We recaJI that

11 T. H. G. tiilcgson,~1in:raf1 StnrauY~-'ftJr 11gir1teriJ1~ Sruilenrs.3rd edition. Halsted Pre.s..;.Jobn \ViJcy


& Sons;, Ne"'' York. 1999, p. 467.
262 BEAMS

-~
~
Figure 655: Isotropic lbcam unde r pure. torque with no axial constraint

Ii= do/f/ dx (Eq. 6.1). Thus, we have


dvr d
- = 1? - (6.230)
dx 2
The bending 1uo1nent J\111nay now be e xpressed as
.\1, = _ do Ehrbf ~- (6.231)
. dx 12 2
\Ve nov. introduce the bin1o ment J\1," defined for an lbean1 as
(6.232)

The last tv.o e quations give

-
A/Pl = - d11
- (Ehrbf
dx
--d- .
24
J (6.233)
~
El, .

The term indicated by the bracke t is the 'varping stiffness Eidt.


Owing to the axial constra int, t he be nding n1ome.n t 1\:ic varies along the xaxis
o( the beam. Correspondingly, there is a shear force in the Hanges. The rela tionship
between the flange s hear force and the flange bending n1oment is

v, =aft,_ (6.234)
dx
The s hear force results in a torque fw(Fig. 6.56, bottom) de fined by
f . = Vcd (6.235)

Figure 6.56: Isotropic I-beam unde r pure torque wilh axial cons.traint.
6.5 IUHSIUH U~ THIN WAll.fU KEAMS 263

A'~ ...., '' d


..;.:..,.. \ l'( C::\_I -
.., \
' ,
...... ... ') 2

Figure 6.57: Rocahon of an I-beam about the x-a.ns.

Equation (6.232). (6.234), and (6.235) give

f "'= di\l.... (6.236)


1/x
From E qs. (6.2.111) and (6.233) we have
.... (/2,,
T.11 = - /.., dx2. (6.237)

where the warping stiffness El. is (see Eq. 6.233)

1. = i11bf ,p. (6.238)


24
The total torque is

1'=~ + (I.) - d~l. ( d'") (6.239)


f. -...-
f.
where T,,.. and f ... are referred to as the "Saint-Venant"' and the "res1rained-
warping-induccd" torques. respectively.
Orlltotropic l.ben111.f. The expression for the to rque or o rtho tropic beanlS is
similar to the exprc.sion for isotropic I-beams (Eq. 6.239) '"' fo llOI"':

(;7,,, +
1' = .._,_- d D) '
mW(- -,
1
(6.240)
tfX-
f. ~
f.

The torsional stiffness Gi1 presented in Section 6.5.2 is unaffected by the axial
cons traint and i.s given by Eq. (6.175).
h remain< to determine the warping stiffness fl... By referring to Eq. (6.238).
we see th31 the warping stiffness or an isotropic I-beam is a function of the product
of the Range's modulus and thickness Ii (El.. - Eli). This Ell is related to the
o.<ial fo rce (per unit length) in the Oaoge as follows:

Ni =ilo1 = Ell<; . (6.241)

For a beam with orthotropic la)'Up, N1 is (see Table 6.2. page 222)
,., 811 <u /J1 I
=- (6.242)
' "~
D t - - K1
D
264 BEAMS

Table 6.9. Boundary conditions for


beams in torsion.
\Varping restrained b =0
un rtstrained M. =o
Rotationally restrained \Ir =0
unrestrained f=O

This equation applies at a reference plane chosen arbitrarily. At the neutra l..
plane (see Eq. 6.105) fJ11 = 0, and, he nce (see Table 6.2. page 222), we have that
8 11/ D = 1/a~1 where the superscript e indicates that a f 1 is evaluated a t the flange's
"neut ra r plane. With these substitutions the preceding equation becomes

" I o (6.243)
1 "~ = e~E
Ct ll

By compa ring Eqs. (6.243) and (6.241). we see that an orthotropic beam's ten
sile stiffness 1/crf1 corresponds to an isotropic beanl's tensile stiffness Eh. \ Ve use
this correspondence and evaluate the warping stiffness of an o rt hotropic I-beam
by replacing Eh by l/a~1 in Eq. (6.238). The result is
.l..b'
- r 2
El... --~d24 . (6.244)

Orthotropic bean1 - arbitrary cross section. Equation (6.240} can aJso be used
for thinv.alled bean1s \vi th o the r c ross.sectional shapes by using the a ppropriate
torsional a nd warpingstiffnesses. The torsional stiffness&, is given by Eqs. (6.170)
and (6:t75). TI1e \varping stiffness El~ of an ortho tropic beam is obtained by re...
placing Eh by l /ar1 in the a ppropriateexpressions9 for thecorrespooding isouopic
l:>ean1. The warping stiffnesses El"' of frequently use.cl cross sectio ns are included
in 'fable A.5.
Boundary co11ditio11s. When the 'varping of the cross section at the e nd of
a bean1 is restrained. the distortion is ze ro. When \Varping is not restrained. the
bi11101nent is zero.
\Vhen the e nd may rotate. the to rque is zero. '\Vhe n the end is rotationally
restraine d, the rate of twist is zero.
The preceding boundary cond itio ns a re sun1ma rized in Table 6.9.

6.5.6 Restrained Warping - ClosedSectlon Orthotropic Beams


Warping of c losedsection bean1s is often negligible. ' ''arping n1ay be.con1e signif..
icant. tho ugh, \Vhe n the shear stiffnesses of different v.aJI segments a re ma rkedly
diffe rent. This is illustrated in Figure 6.58.

" S. P. li.mosbcnko and J. Gere. T/umry fJf El'u'tic Srahilitr. 2nd edition. ~tcGraw-HiJI, NC\\' York, 1961.
p. 530.
6.6 THIN-WALi.ED BEAMS WITH ARBITIIARY LAYUP 265

-~ -,

,'
,
'
z
Agu.rc 6-~ \\~rpng, of hox beam sut>,cctcd to 1ors1on ilcn (..). C: (-..)..

6.6 Thin-Walled Beams with Arbitrary Layup Subjected to Axlal Load,


Bending, and TOl'Slon
ln this seclion we 1ren1 thin.walled. open and closed-section bcnnl~. The beams
wall consistsornatscgmenL< (Fig. 6.59) designated by tho subscript k(k = I. 2 .. _. _
K. '"here K is 1hc to ta l nunlber of \\all segments}. The cross section n1ay be
synunctrical or unsy1n1netrical, a nd the layup of the beam is arbitrary. that is. the
layers are not ncc:cssnrily orthotropic.
111e bea1n i5 subjected to an axial fo rce Fi. beading nlon1cnts .W ..,. M:. and
torque f acting nt the centroid.
We employ two coordinate systems (Fig. 6.60) for the beam: the .r -)~t coor
dinate system with its origin at the centroid and the X-f -i coordinate system \\rith
its origin at 3.n arbitrarily chosen poi.nL The location of the centroid with respect
to the orig.in of the 1-)'-:! coordinate system is given in Section 6.6.3.
The displacements of the longitudinal a'<is. passing through the centroid. are
u. v. "''fl (Fig. 6.3). where u is the axial displacement. v and "' or< the transverse
displacements in the y and z directions.. and 1/1 is the twist or the cross section.
lo the x-y-z. coordinate system the re lationships bet\\een these displacements.
the axial strain ':. 1he curvatures l /Py. 1/ P::_ or the x~axis. and the ra1c of t'vist tJ
(Eq. 6. 1) arc

IJU a'w 1 al/I


-=' Cl
- = 1. (6.245)
ax ~ ax2 = - Py ax
In lhe i -Y-i coordinate system these re lationships bcoo1nc

au a2w= - - a~
-=
a:t '
-"'3"
ax- P'J
- --
ax- /), (6.246)

J:jgurc 6.59: Forcell on open nnd closed cros.<>-sec11on. lh1n-wnlkd 11<:anl11 '''llh Onl w::ill $cgments.
266 BEAMS

4:th \VSJI
scgnlt~ nt

;;

Figure 6.60: fbe coordi nalc syste ms emplo)1Cd in the analysiso( thin,~alled be-a ms wilh ar bitra.l)'
layup.

'''here f*,
1/ p y, 1/pz are the axial strain and the curvatures of the longitudinal
axis passing t hrough the origin of the X- )i- t coordinate systen1, and ii, ii. W, and
~ are the displacen1ents o( the X-axis.
The force, n1ome nt, and torque resultants at the origin o( the bar coordinate
system Rx, tWy, ,\1;;, f K are related to the force applied at the centroid by the
expressions,

(6.247)

\\here ~ and Ye are the coordinates of the centroid in the bar coordinate syste m
(Fig. 6.61 ). For bo th open- and c.losed -sectio n beams t he (orce-strain and strain-
force re la tionships in the x- y-i coordinate system are (Eqs. 6.2 and 6.17)

r1['
~..,, =
M,
f
P 11
P,,
P"
P12
Pu
P,_,
p,.,,
Pu

Pn P,, ' ]j;


P,.. Pi.11

f'.14 P.,
,,.,

,,.L
I>
(6.248)

11"' .
.L
P<
/}
[:
w;,
0
\i'22
H'n
\\'Ll
0
"~1
l\~u
l\~a
""]! ~1
\Vi.t
11'.1'
w,.
M ,.
,:;;; .
f
(6.249)

- ~ z

> y ;----1--,.'-'6 : y
y
fi,
./
x
Figure 6.6 1: Scam subjected lo an axial (orce a l the cc nlro 1d a nd the resultant a:ual force-and
n1ome-nts in t he X-Y--Z coordinate. S)'Stc-m.
6.6 THINWALLED BEAMS WITH ARBITRARY LAYUP 267

In the bar coordinate syste111 these re lat ionships are \Vritten as

1~1
,Q,
f.-
=
l'
P11
/513
/514
P 12
P22
Pn
Pu
P2J
Pi., P:u ? ,..
pl4 p .l4 P,14
~"] 11
,,
.L
p;
F
(6.250)

111 [w
.Lw,,,
111

PZ-
F
-
-
lV12

w,,.
-
IV12
\.V22
\..VL,
W2i1
w,_,
l \'2.~
IV.JJ
w.. .
w,.lJ
\.-V1.f
IV:w
W.i.s
r1
!~y
'1< .
T-;
(6.251)

where [ P ], IP], I IV), and [W] a re the stiffness and compliance matrices in the
ce ntroid x- y- x and bar X-Y- Zcoordin ate systems. These matrices are obtained by
reasoning similar to that used in the derivations of the stiffness and con1pliance
n1atrices of o rt hotropic beams (Sections 6.3.3 and 6.4). For beams \\'ith a rbitrary
layup the algebra ic steps are long and laborious lo a nd the ir presentation is beyond
the scope of this book. The results are sun1n1arized in Table 6.10.
The compliances 0t;;.fi1i, ~;;, which a ppear in Table 6.10. are evaluated in the local
~-1~-~ coordinate systen1. For the kth segn1ent " 'e e n1ploy the ~k-'11:-~k coordinate
system \vith the origin at the. midpo int of the re ference plane of the kth segn1ent,
where~ is paralle l to the x coordinate, 1~ is a long the circumfe re nce of the \Vall,
and ( is perpendicular to the circumfe re nce; Yk and Zk a re the coordinates of the
~1r-rr1:--~k coordinate system's o rigin . \\hich is at t he n1idpoint of the refere nce plane
(Fig. 6.60). For c losedsection bea1ns, rJA: points in the oounte rclock\vise direction
(Pig. 6.62), ct1: is the angle betwee n the 111:- a nd )icoordinate axes., and b1r is the 'vidth
of t he wall segment (Fig. 6.59).
We o bserve that for orthotropic beatns the expressions in Table 6.10 yie ld the
stiffness matrix given in Eq. (6.7).

6.6. t Displacements of Open and Closed-Section Beams


In the analysis \\'e n1ust take into account that t hin\valled De.ams with a rbitrary
layup respond to loads differently than bean1s with ortho tropic layup. Forexa1nple,
an o rtho tropic beam subjected to an axial load defonns., as sho,vn in Figure 6.63
(left). A thin-walled beam with a rbitrary layup deforms as shown in Figure 6.63
(right). Thus, the c.ross sections of be.an1s \\'ith a rbitrary layup do not ren1ain
plane, and the Be rnoulli- Na.vier hypo thesis is inapplicable. Nonetheless, in a long
body, such as a beam. the stra ins may be considered to be constant in the axial
direction, a nd \Ve n1ay apply the plane- strain condit ion (Section 2.4) in the a nalysis.
111e displacen1ents are then calculated \\it h this approxin1ation fron1 t he gov.
erning equations together with the force- strain re la tionship given in Table 6.10.

tu L. P. Kollar and A . Plu1..sik. Analysis or 1ltin \Vallcd Composite Bcanls with Arbitrtu)' Layup,
Jt11~nu1I of Rei11forced Plastics t111d C<HU/>O.~ites. Vol. 21 . 1423-1465. 2002.
268 BEAMS

Table 6.10. Stiffness and compliance matrices in tile bar ([PJ , [WD and centroid
([ PJ, ( WJ) coordinate systems of open- and closed-section beams with art>itrary
layup. The elements <>11, {:111. 611 are evaluated at tile {.-q1.--(k coordinate system
of each wall segment. The enclosed area of closed-section beams is denoted by A.
The superscript T denotes transpose.
Bar coordinate systt tn: [W) = [PJ- 1
K
Open-section be.a m:
...
[Pl= I::IR..f'[w,J-'[R..I
K

...
[Pl= E<I 11,J'[.,,J-'I R;I) + [LIT[ FJ- 1[L)

Centroid coordinate system: [111 = [R,]' [IV)(R,,] (PJ = (IVJ- 1

I 'i.
(R,) =
[
0 cor,
0
0
s1 n a~
0
~"]
ki
A.\1 t
=
[""/Jn
/Ju,

[w) = ;;
I
[""
/ju
0
f111
611
0
0
0
__g_
cAuJrt.i
-lf!"] [~.1 [~
-~di(,
= ) '<
0
0 0OJ
0 I 0
-~/.116 -iiSu. 0 !&.t. 0 0 0 I

t. [""
[ii = .1.-...1 P12
f!&l?.. 0 -)~] [w,r' IR.J
0
- i l' l) *
[LJ =[O 0 0 2
0 0 0 :]-111

(F] = ,., t (b~"'" A:] ["flu &2.? I!. - Sn 0 -~Ji.; ,.


J
p,,. 0 - ~ftr;.. [w,i:]- t [ ""
ftr.i
0
p.,
81 2

-l!l.s - !Su ,
0
l)
,,
[~J = [~ [a"p,. .. .
0 /!11
0
0 0 3J (ii,] = P11 ~,,

~
f!~ .. ]
.!,,; ;
[""p,., .,.P"],
1;;,1= /li.1 ,,

6.6.2 Stresses and Strains in Open- and


Closed-Section Beams
We describe the stresses and strains inside the \Vall \vith reference to each wall
segn1ent's ~k-'~k-~k coordina te systen1 (Fig. 6.60). The forces a nd 1noments inside
the \\all ar e sho,vn in Figure 6.64.
By applying the results of the la111ina te plate theory, the strains and stresses
in the kth wall segment are obta ined by re placing x by ~, y by r1, and .t by ~ in
6.6 THINWAllED BEAMS WITH ARBITllARY LAYUP 269

Figure 6.62: The local coordinate system attached to the.kth wall seg.mc.nt (kf1) and the direc.Llon.c;
of the n coordinates (right) in a dose.c:lsection beam.

Eqs. (3.7) and (3.11) as follows:

(6.252)

(6.253)

where IQ'Jk is evaluated in the .t'th \Vall segme nt's ~k-''k-(k coordinate systen1.
In the kth segn1ent's reference plane the strains and curvatures a re calculated
from the strain- force relationships (Eq. 3.22), which. for convenience, are repro~
duced as foLJo,vs:
,.. /111 /112 fJ1 N,
,.' "" "" "'

Y(1t
K!
=
0-12

"'
/J11
etio
et22
Ct26
/J,1 P.1
.
.,
/J,1 lh1 /Ju.
A.1 /J61 !J..
811 612 &t6
N
Nt.,
M,
(6.254)

K,! /J11 /322 p,,, 812 622 s,. M


K<o
/J1. p,. p,.. s,. s,. s.. k M!.,, k

where the k subscript refers to the kth seg1nent and <xq, fJij- &;; are evaluated in the
.(1h \Vall segment's ~k-1~1;-{1: coordinate systen1. The preceding expressions apply
both to open and c.losedsection beanu. The dif(erence in the t\vo types o( beams
is in the expressions used to calculate the forces and mon1ent (per unit length)

Figure 6.63: Displacements of an orthotropic beam {left) and a beam "'ith a rbilraT)' layup (right)
under tension.
2711 BEAMS

ONwM"'
figure 6.64: 'The fore.cs and moments (per unit length) acting 1n lhe .(1h \Vall o f an o pc.n or
dosed-section beam .

inside the \\ all. The relevant e xpressions, derived by Pluzsik and Koll:ir,11 are
1

sumntarized in the remainder of this section.


Ope11seclio11 bean1. A long the free longitudinal edges of the bean1, the in
plane forces and moments (per unit length) are zero: N, = N.,,
= M, = 0 (Fig. 6.24).
Since the dimensions oC the cross section are small compared \\~th the beam s
le ngth, these inplane forces and moments are approximate ly zero inside the 'vall:

(6.255)

The expressions for NE, At(, Mt 11 are

(6.256)

where Ill], IR,,], and IR, ] are define d in Table 6.10.


Closedseclio11 bea111. In a closedsection beam, Nri is negligible; thus,

(6.257)

The re maining fo rce.s and n1on1ents a re

(6.258)

(6.259)

where Ill]. IR,,], [R.], Iv], IF], and [ L] are defined in Table 6.lO.

ll Jbid.
6.6 THINWALLED BEAMS WITH ARBITRARY LAYUP 271

6.6.3 Centroid
The centroid is locrtted s uch thnt lhc bcnm 's axis rein a ins s traigh t when an a xial
force Fi is applied a t the ce ntroid. Allhoug.h this axis ren1ains stra ight, the beam
n1ay t\\ist about the axis of twist. which docs not necessarily coincide '"ith the axis
passing through the centroid.
From E qs. {6.25 1) and (6.247). the curvaiures or the axis. possing through the
origin o f the bar coordin:11e system. are

(6.260)

Since Nx is applied at the centroid. the curvatures of the axis passing through
the centroid are zero (I/ Py = I/ P: = 0). furthcnnore. the curvatures of the a.-Os
passing through the centroid arc the same :tis the curvatures of the axis passing
through the origin of 1he bar ooordinatc system. Thus " 'e have
I I I I
- = - = - = - = 0. (6.261)
Py P: P; P'f
Equations (6.260) and (6.261) yield the coordinates<.. y, o[ the centroid with
respect to the origin of the bar ooordinrnc systen1 as follov.s:

I I [Ill" w,,
z,
w,,1
y, = - 111,,
-rw.lJ LWIJ . (6.262)

6.6.4 Restrained Warping


\\'he n restrained warping i~ ta ken into account and the layup of each \\ all is 1

balanced, the force -s train re lationships mny be nppl'oxi111ated by 12

iii, 1'1 1 1'1 2 p,, f'l4 0


...'
.L
,ff, P12 Jl.i.2 p~, />24 0 '"l.
iii, = P1.1 P,, P.o f';,4 0 (6.263)
f,.., P,. P,, P_,. P" 0 0'"
f. 0 0 0 0 lfl. t1'0
- tfxl

whe re /,,. is to be calc ulated ns for orthotropic beams.. The efects of restrained
warping on 1he displacements and stresses of beams \\'ith unbalanced layup a re
discussed by Bo uld and Tung."

I? A . Ptuzsi.k arid L P. Kollir, Ecc1 Of ShC'ar ~f(l11'11lation and Rcstraiocd \\'arping on the Dis-
plact-mcnts of Compoii.u: lkarm. kHlmal of R~ft>t~ftl l'ftultc.1 #Md Composua. Vol 11. 2002
(mprcss).
u N. R. Boald and L Tttng. A Vbtolbtory !Of' FlbcrR~anlOl"d lk:arru: "1lh llun\\'.alkd ~
Stttions. /111t'm0lttllt#ll.loh1nol of.'WJMJ """ Suutturd. VaL lO. 277-M. 198'.
272 BEAMS

figure 6.65: The:. box beam in axampk 6.5.

FJ = 24 000N
6.5 E.xan1ple. A.IJ L = l n1-long boxbea111, lVit/J 1/1e cross section sho1v1r in Fig. 6.65,
is 1nade of graphite epoxy uuidirecrional plies. Tire tnaterial properties are given in
Tllbfe 3.6 (page 81 j. Tire layttp is [Oto/45 ml, tvirlr the 0-degrce ply 011 rlre outside of
tire 1vall. The bea1n, builtltt at one end and free at the other end, is subjected to a1'
axial load of 24 000 1\1 By neglecting tire effects of axial restrai1tt, find tire position
of the centroid and calculate the 111axin11on axial displttct!lnent and the 1naxi11uu11
11vis1.

Solution. The analysis follows the steps in Table 6.10 (page 268). From Table 6.10
\Ve have

1 t.. >'
(R,,J = 0 c~Slltk - sin 0
ol (6.264)
0 S JDQ'k COS&.t 0 '
[
0 0 0 1
\vhere Y* and z,,. are the coordinates of the midpoints of the \Vall segn1ents, and
Ctk is t he angle bet\veen 11 and the y coord inates at \\ hich ~~ is tangent to the wall
1

segn1ent (Fig. 6.60). The values of these para1ueters for each Range a nd web are
shown in Figure 6.66. \\Cith these values ( R,,J are

,,,, = [j 00
t
0
0.025
- I
0
0
0
OJ
0
1
0.035 - 0.025
- t l
0 0
0 0

The [ Aj matrix for each web and each flange is

Au]
~13 = ["
P11 11 (6.265)
A;u k /316
b.b I HIN- WALLtU tl tAM~ WllH AHtlllKA.Kf LATUt' Z/"J

y, = 0
b~ b 1 50mm :;, = 35
~ aJ = 180"

-o
. 3
E
E ::
y, -25 Y,=25

".... i..
g :. -o " :;,=O
01 =-90"
n, -'lttr
2 q,
.... h 2mm Y1=0
:,=-35
<:r1=0I>

Figure 6.66: Par.1111cten of lhe w;ill.s o f the beam in Example 6.,.

With the cleme nts of the compliance matrix in Eq. (3.56), ii11 for the nanges
(k = I. 3) and ror the webs(k =2. 4) a re
(ii11)1 = (ii11)l = (ii11). =(An)= 1.615 x 10. (6.266)

The distances bely,een the midplanes o f the corresponding wall segment~ are
bi = b, = 0.050 m. I>, = =
b, 0.070 m (Fig. 6.66). The ele me nlS of [wt[ re

0.2688 341.4 0 110.6]


,.,,, =,.,,, = 34~.4 8060350 5~ 21~130 104
[
110.6 214 130 0 974 290

0.1920 243.8 0
78.98]
243.8 575 970 0 1520950 x 10- .
lWJ=W=
I l l Q 0 216.62
[
78.98 152 950 0 695 920
111e [ i ). ( L), uncl ( F[ matrices are

(ii = [ - 107.82 0 0 0.08061 ] J


- 24. 10 0 0 0.187 36 x w-
ILi = [0. 101 s 0 0 0.Cl06919] lo' (6.267)
24.IO 0 0 - 0.187 36 x

(F( = [0.013 39 - 6.474] x 10-<


- 6.474 22 150 .

The stirfncss matrix o( the beam is

(PJ = ([R,f[w1J"' [R1J + (R2JTlwir'IR2) + (R,)'(w,J- ' (R, J + 11.1r1Fr 11L])


309.93
0 0 0.0607 53]
0.029 49 0 l "" (6 268)
= 0 0 0.01798 0 x w, .
[
0.067 53 0 0 0.004 105
...
'vhere P11 is in N~ P22, P33. and P44 are in N . n1 2: and P 14 is in N . n1. The compli-
ance ma trix is
0.025 76 0 0
- 0.423 7]
l WJ= [Pr' = ~ 33091 55063
0
O
...
x JO ,
[
- 0.423 7 0 0 250.56
(6.269)
~ i - - - I - I
\\hereiV11 is in N ~ l.Yu,
W.u , and W44are in N m!; \.V14 is in Nm
The location of the centroid is give n by Eq. (6.262). With the preceding ele
me nts o f the con1pliance ma trix, we have

1;:1 = - [;: ~~:~r'[~::J = l~I (6.270)

The forces acting at the centToid are

(6.271)

;_I [
The strain~ \\ hich are uniforn1 a long the cantilever, are (Eq. 6.249)
1

0 0

I
0.2576 0Jl004 0
,,., = 10- 1 0
x 0
.,l. 0 0.0006
IJ - 0.0042 0 0

= 10.6182 10- 1 .
(6.272)
- 0.01017
The unit of 11 is l / n1. The axial displace1nent a nd the t'vist of the free e nd are

11= ~ x L = 0.6182 x 10- 3 x I = 0.000 618 m = 0.618 mm (6.273)


>/I= IJ x L = - 0.01017 x I = - 0.010 L7 rod= - 0.58.
In cotnparison, u and I/I calculated by the fini te eleme nt method are
11 = 0.616 mm "'= -0.57'. (6.274)

6.7 Transversely Loaded Thin-Walled Beams


We consider a transversely loaded beam (Fig. 6.67). The applied transverse load,
\Vi th components Py and_,Pz:.Jroduces bending mon1en~ Air.:~~' a torque f . and
transverse shear forces v,.,
V~. The bending n1on1ents Mr A1~ give rise to an axial
6.7 TRANSVERSELY LOADED THINWALLED BEAMS 275

-~ ,
~
q c J t{lld(
(')

Figure. 6.67: Internal (orccs a nd stressc-s acting o n 1he cross section of a transversely loaded
thinwalled beam. The. t-1t-{ coordinate sys Iem is shO\\n in Figure 6.60.

stress a(. and the internal fo re.es ~. v~. and f give rise to shear stress r(lf in the
wall. It is customary to represent the shear stress by the shear flo''' q defined as
(Eq. 6.189),

q f
(/1)
r1qd( = Ni.. (6.275)

where Ir is the t hickness of t he \Vall.


The load p, applied along a line A- A may be replac.ed by an e quivalent load
system consisting of a load p, a pplied along line 8-8 a nd a torque load t = - 11,d,
where d is the ho rizontal distance between points A a nd 8 (Fig. 6.68). The load
P:. results in a shear Ho\v q v \vhile the torque load introduces a shear How qr. The
total shear flow is
(6.276)
Both shear flows qT and q" as \\'ell as the axial stress u( cause twisting of the
bea1n. represented by the rate of t\visl 1? as follows:

v=
.'I. due lo
O"(
due lo
+ qT + qV
due to p, a long an a rbitrary
line A- A

The re is a specia l location of lhe line B- B referred to as the shear center.111


When the load is applied a t this location (i.e., a t the shear cente r), the shear flow
q v does not cause l\\isting of the bean1, and lhe rate of twisl depends only on '1~
and q"=

v=
A due lo
0'1
due lo
+ qT
p, a tthe
shear cente r.

I~ E. H. ~1anslicld and A. J. Sobey. The Fiber Con1positc licticopter Blade - Pan J: Stiffness
Properties - Part 2: Pro:>pccls fo r Acrocla..~lic Tailo ring. Atrf>na111iclll Quortetly, Vol 30. 4 13-449,
1979.
figure 6.68: Shear llow in a lrans\'e.rsely loaded thin-walled beam.

Fo r a beam \Vi th arbitra ry layup the a xial stress <IE introduce..-c; a t\\ist even when
the load is applied a t the shear cente r. Howeve r. <1( does not introduce a twist in
orlhotropic bean1s. Therefore. \vhen the bean1 is orthotropic and t he load acts at
the shear cente r, the ra te o ( t \\ist depends only on the shear Ho\\due to the torque:

orlholropic
11~ at the shear center.

In sun1n1ary, whe n the transverse load acts a t the shear center, an o rthotropic
bean1 does not twist, \Vhe re.as a beam with arbitrary layup does t\vist.
The transverse load p~ causes transverse shear forces ~.. ~. bending nlon1ents
,Q,, ,Q,, and torque f (Fig. 6.67). Tiie displaceme nts caused by the shear forces
V,., V: ar e neglecte d. The displacen1ents due to the bending mon1ents Mya nd A?.,
and the torq ue f are de te rmined by the a naJyses presented in Sections 6.3- 6.6.
The no rmal stresses a re determined by the an alyses presente d in Sectio ns 6.3.4,
6.4, and 6.6.2.
111e shear st resses in the wa ll induced by the bending n1ome nts 1fi/y and ,q::
and the torque fa re determine d by the ana lyses prese nted in Sections 6.5 and
6.6. The she ar stresses in the v.aJI due to t he shear forces ~v. ~are d iscussed ne xt.
The total stresses a re obta ined by supe rimposing the stresses caused by bending,
torsion, a nd shear.

6.7 .1 Beams with Orthotroplc Layup or with Symmetrical Cross Section


We consider beams that ha"e either orthotropic layup (page 207) or have sym-
me trical cross section with t he loads applied in the symmetry plane (Fig. 6.4).
In the follo,ving \Ve calculate the shear flow in o pe n a nd cl ose d ~section beams
when the shear forces P, and 1?,. act a t the shear center (Fig. 6.69).
Ope11seclio11 bean1s. There is no load a pplied in the axial direction. Equilib
rium of an ele.me nt in t he axial S x direction gives (Fig. 6.70)

aq + aN, = o. (6.277)
a,1 as
v,

Figure 6.69: Beam subjected to shear forces at the shear


'
'!11:
rSC
~:;;::l""""': - - - - +
cente r.
l
:tjt-
vy

whereq 0 P is the shear flow and N{ is t he axial force (per w1it le ngth) a tan arbitrarily
chosen refe.r ence surface. Integration of this expression )~elds

q P (s1) = - f.' aN; tfr/.


a~ . (6.278)
0

where '1 is the coordinate a long the ,...,all and s 1 is the distance from the free edge
to t he point of interest.
For an orthotropic 'vall segn1ent (unsymmetrical layup) N~ is obtaine d by com-
bining E qs. (6.118). (6.1 11), a nd (6.112) as fo llows:

N; = ~"A!.;' - lfl, _,\i, (z~ cosa~)


EI,, El" - (EI,.,)' D
- D
~II
_ + ifl,,M, ( y-+smu
+ - _ifl,,M, N 811
. -/l11) +-=- (6.279)
El,.,/._ - (El,,)' D D EA D

The relationship bet\\een the s hear fo rces and the bending n1ome nts are
(Eq. 6.3)

-
\I - - -

a!J, (6.280)
' - ax
BycombiningEqs. (6.278)-(6.280), and by no ting that fo r a trans\'erse ly loaded
beam the norn1al force Nis zero, we obtain the fo llowing e xpression for the shear
ftO\\I:

figure 6.70: forces on an element of a tral\S\ ersely 1

loaded, thin \valled opcn sec.1ion beam.


4 4

.,
278 BEAMS

orlhorropic and unsyn1111etrical layup, arbirrt1ry cross section:

q
op ( )
s1
---
El,. V, - El,.,V,
= - "" ,,... _ ""
r..1,.,
c1--
....
--

(r..1,..)
~ ..
2 .
0
r(tD - /!11 )
s1
811 .
oos a D
d
11

---

- r..1,.,
"" c1-- "' - (r..1
--
- El,, V, + El,., V,.
"" .)2 .
1
- --
r(811 _ /!11)
.f1

Y fj + SlnCi D
I
'I/. (6.281)
. '"" .. 0

\Ve e mphasize that V; and ~ act at the shear center and Elyy. Elr:. El:.:. are
the bending stiffnesses in the y-z coordinate systen1 a ttached to the centroid of
the cross section.
\Vhen the layup of each \\!aJI segme nt is orthotropic and symn1etrical, the
expression for the shear How is obtained from Eqs. (6.278), (6.280), (6.111), (6.112),
and (6.125)
orthorropic and sy111111etrical loyup, arbi1rary cross section:

-
-- -

El,.v, - El,.,V,
- -
r(11
El,, El" - (EJ,.,)2
-- ...... .fj

1 )
t - d11
0
.-...-

- E1,,,~1,; - ci1
r(
- El,.. V. + El,., V,
,,i ;;;;-
---.

2
s1
1 ) d
'1 (6.282)
0
\Vhen the OOan1 's cross section is syn1metrical, the shear center is in the x-z
syn1n1etry plane. When the transverse load acts in this syn1metry plane, the fo rce
is given by Eq. (6.131), (nono rthotropic, unsymmetrical layup). This equation, in
combination wit h Eqs. (6.278), (6.280), and (6.111) , yields
arbitrary layup, synunetrical cross section:

qop (S1) = -
-.

-V.'
EI,.,.
f ( ~II
:r1 -

/J11)d I/.
t= - COSCi =
D
-

D
(6.283)
-"
For an isotropic beam l /a 11 = Eh (Eq. 3.43), and the expressio n for the shear
How (Eq. 6.282) becomes
isotropic beaut:

q"" (s1)= -
...._
1.. v. - 1,_v,.
1,.J--
-,
- (/,,.)
-.
J :r1

(t h)dq -
-.
- 1, ..' v.' + 1,,,v,
!,., , __ - (!,,.)
;
-.
JJ1

(yh)d11. (6.284)
..~ ~ (I ~ ~-- ... I)

Illustration of the shear fto\\' in an open-sectio n beam is give n in Exan1ple 6.2


(page 239).
Closedseclion bea111s - single cell. Jn a single -ce ll, closed-section bean1, the
shear Ho\\' n1ay be expressed in t\vo parts. The firs t part is the shear flow (denoted
by q "P) that would exi~t if the bean1 \\'ere cut longitudinaJly and the beam \Vere
open. The second is a constant shear fto,v qi: ind uced by the forces acting along
6.7 TRANSVERSELY LOADED THIN-WALLED BEAMS 279

+
figu.rc 6.71: Supc:rpos1t1on of the open tell and constant she;;ir Rows in a trans"cncl}' loaded.
stng_lc-ccll closed..scct1on beam.

the --cut" edges (Fig. 6. 71) (q' depends on the location or the cut). Thus. we write
the shear Dow in the closed single-cell beam as

q=q"" +q. (6.285)

The shear flow q is calculated by Eqs. (6.278)-{6.2&3). The constant shear


80\v if is determined as follo\\"S. When the shear force.s V,. and V: act at the she-ar
center (which is the case here). the beam does not twist. Hence. from Eq. (6.188)
we have

(6.286)

The shear strain Y(., in euch wall segment is ca1cuhued at 1he "torque neutral-
surface. From Eq. (6.195) we have

(6.287)

From Eqs. (6.285)- (6.287) we obtain

1? = -l
2A,
f u;,q""d'I + - I q'
2A
f a~tf11 = 0. (6.288)

Fro1n this equation '"e have

, fav,.,qVPtfl/
qt;.= 6 (6.289)
f a;.<111
Closed~sectio11 bea1t1.'f - 11111//lctll. To colculatc the shear Row in a multicell
beam (Fig, 6.72), we "cul" longitudinally ench cell. thereby producing an open-
section beam.. The shear flov" q"P in Lhis open-section beam is calculated by the
analysis given by Eqs. (6.278)- (6.28.1).
ln each cell there is a con.stanl shear Roy. qf induced by the fore.e s acting along
the edges of the cut. In a beam with L ciells there nre l constant shear Oo\\'--S. The
shear fto\11' is the sum of the open...scelion and the constant shear Oo\\'S. For a \\'all
that belongs to one cell only. q is the sum or the open-cell shear ftow and the
constant shear ftow in the appropriate cell (q = q"" + qf). For a wall adjoining
t\\-o cells. the shear flow is the sum of the comtant s~ar Ho\\'S of the adjacent cells
and the open-section beam shear Oow. as illustrated in Fig. 6.n.
+

q"" qL' - 1 qL'

F1sun: 6.72: Superpo.sitlon of the open cell and the constant shear Rows in a tranS\.'ersel)' loaded,
multiccll. closed-section tic.am..

\Vhen t he shear forces Vy and ~ act at the shear center (\vhich is the case
here), the be am does no t twist(~= 0). Henre. for e ach cell we write (Eq. 6.188)

1=1,2 . .. . , L. (6.290)

111e shear strain y~,1 at the "torque neutral .. surface in each \Vall segment is
(Eq. 6.195)

(6.291)
where q is the shear ftow in the wall segme nt (Eq. 6.285).
By combining Eqs. (6.290) and (6.291), we obtain

21A, f '"q'
.
" I 1 -- 0 I= 1.2, .. .. L (6.292)
"II I

Equation (6.292) represents L equations. which can be solved for the L un


kno\vns: qf. ql .... qt.
The stresses in t he \\alls a re calculated the same 1Nay as for a singlecell bean1.

6.7.2 Beams with Artlitrary Layup


In this section \\'e detern1ine the shear Ho\\'S (c-ause d by the she ar forces ~and
V,.acting at the shear cente r) in beams consisting of \Vall segn1ent~ 'vit h arbitrary
layup. No load is applied in the axial d irection. Equilibrium in the~ d irection gives
,, ''
''
I'
I

Figure 6.73: Coordinate 11 along the circumference of an open and a closed-section beam.

(Eq. 6.277. see Fig. 6.70)


aq aN1 _ 0
(6.293)
aq + 8$ -
To de tern1ine the shear Row q, the derivati\'e of the a.xial fo rce (per unit length)
aNtfJ$ must be known.
Ope11-sectio11 bea111s. In an open.section beam the shear Ho'v is (Eq. 6.278)
'
q"P (s1) = - .f aa~' "''
0
(6.294)

where 11 is the coordinate along the circumfe re nce (Fig. 6.73, left). The term aN,/3$
is evaluated as follows. By diffe re ntiating both sides of Eq. (6.256) with respe ct to
~ . \Ve have

!.&
t
a1\i;
'I (6.295)
;i,Q,
'I
ll f i
'I
111e re is no a pplie d load in the axial direction clJx = O), and aiVx1a~ is ze ro.
In addition, (see Eq. 6.247) we have

(6.296)

The shear forces are (Eq. 6.3, $ x )

17. -
y -
a.11,
3$ . (6.297)

111e transverse load acts a t the shear center. Hence, the to rque load t is zero.
and we have (see Eq. 6.3)
af
a~ = - t= O. (6.298)

.I,I II
Substitution of Eqs. (6.297), (6.298) and aNx/a$ = 0 into Eq. (6.295) gives
0
::. = (ihr ' [ R,11[1?,JllV] ~. . (6.299)
a,"~ 0
Jt k
282 BEAMS

\Ve are interested only in a1V~/&~ given by this equation since this is used in
Eq. (6.294) to determine the shear flow.
Closedseclion bea111s. ln a close<lsection beam the shear Ho\v is obtained by
integration ofEq. (6.293),
..,
aN
~d11 +q'.
'r /i.
q= - (6.300)

"
'vhere q"P is defined as

qop = - f" iif


&N, d1/ . (6.301)
I)

1~ is the coordinate along the circumference (Fig. 6.73, right} and q' is a constant
shear flow. The term &Nifas is obtained by different iating both sides of Eq. (6.259)
as follows:

....
<J1V1

JJif q

(6.302)
.,
~
af,,
Iii"
Since there is no applied load in the axial direction, aNx/(){ is zero. Further
more, the transverse load acts at the shear center. Hence, the torque load t is
zero a nd, conse que ntly, af/&/f. is zero (see Eq. 6.3). When Eq. (6.297) is 1L<ed, the
preceding equation (Eq. 6.302) results in

(6.303)

where V,.and if, are the shear forces (Fig. 6.2). The tenn aN,/&/f. given by this
expression is used to calcula te q"P (Eq. 6.301).
The constant shear flo\v is denoted by q'. We note ll1at the rate of t\vist" in the
x- y-.t coordinate system is the same as the rate o( t\vist Fin the X-Y-t coordinate
system. Thus, from Eq. (6.1&7) we have

- 2M + f Y!.d11 = 0. (6.304)

Equation (6.151) gives

f K,,d1/ =0. (6.305)


6.7 TRANSVERSELY LOADED THIN-WALlfO BEAMS 283

By introducing Eq. (6.254) into these equations, a nd by no ting that N,. = q,


and ,y4 = 0. \Ve obtain

~;J{~.} + f [;: ~:J L!JJ,,.


11.,

(6.306)

where q = q., + q'. The terms N,. Mi . and Ml are evaluated by Eq. (6.259). and
q by Eqs. (6.301) and (6.303). To determine M, and q' . we observe that these
result from the bending moments Mr. :'1) and 1he shear (orces V,., ~-Thus. \\'e
write

1\f, = M~ + M

q=
-q"

due to
+ q.. + q'".

due to
(6.307)

iWt. M,. V,...


. V.
where Af. and q" are calculated by Eq. (6.258).
The terms V1 and V, introduce only M;.
If"' , and q'". Thus. when these
=
shear forces are considered. we have Af11 ,\1~' q = qor- + tf = q"P + qcr'. and
Nf. = Aft = Aft.,, = 0. The shear forces acl 1.\11he shear center and hence the cross
section does not twist (D = 0). Accordingly, (Eq. 6.306) becomes

0- f \""'I
.q 1+
"Ptf
P2tJ f [""
,~26
/JJ
"I'
O.?l I"'" I
M"'
If
(6.308)

111ese equations a re lo be sol\!cd for qci and 111; as follo\\s:

(6.309)

6. 7.3 Shear Center


By definition. s hear forces ac:ting al the shear center do no1 cause t\\ist. We desig-
nate the location of Lhe s hear cen1cr by 1he dis lances z... and ylli: Crom the c.e ntroid
(Fig. 6.74). To deiermine these distances, we npply a force V, acting at the shear
center and calculate the corresponding shear flo\v q. The 1orque of the. ext ernal
fo rce about the centroid is equal to 1hc torque produced by the in1emal shear flo\\'
(Fig. 6.75) as follows:

-v,.., = Jqptl11. (6-310)


tst
284 BEAMS

Figure 6.74: Shear ccnlcr.

Hence, the z coordinate of the shear center is


f qpd11
(.1
(6.311)

\vhere. q is the shear flow due to t?v.


The distance )'1'c: is de tem1ined by applying a force ~ at the shear center
(Fig. 6.74). By an argun1ent simiJar to that leading to the preceding equation.
\VC have

J qpdr/
( S)
}'1>c: = -.. (6.312)
v,
\vhere q is the shear fto\v due to V;:.
For bean1s \Vith arbitrary layup, the shear flow is detern1ined by the equations
given in Section 6.7.2. For o rtho tropic bean1s the shear Hov"s a re detern1ined by
the equations given in Section 6.7.1 .
Approxin1al~ ca/culatio11 ofthe shearcenter. Approxin1ate expressions for the
location of t he shear center of open.section orthotropic beams can be obtained
as follo\\'S. The n1ajor contributo r to the shear Ho\\' is t he normal axial stress N~
caused by bending. ln a thin-walled composite segment Nt. n1ay be approxin1ated
by Eq. (6.243) and in an isotropic beam by Eq. (6.241). By comparing <hese equa-
tions \\le see that the con1posite beams tensile stiffness 1/af1 conesponds to the
isotropic beam s tensile stiffness 1'. Thus. the location of the shear center of an
open-section orthotropic bean1 n1ay be approxin1ated by replacing Eh by 1/uf 1 in
the expression 15 (or the location of the shear center of the corresponding isotropic
bean1. The coordinates of shear centers of frequently used cross sections are given
in Table A.5.
6.6 Exan1ple. An L = 0.6-n1-long C-sectio11 betun, H'ith the cross section sho1v11 in
Figure6. 76, is 111ade oj'graphite epoxy. The 1naterifll properties are given iu Table 3.6
(page 81). The layup is (45\/012/45~]. The beam is b11i/1-i11 at bo1h e11ds. The
bea1n is subjected 10 a uniforntly distributed lofld (11 = - 1 500 /o,//1nj acting along

u S. P. Tin1oshcoko and J. Gere. Theory tJf Ekuric Srabili1y . 2nd edition. f\'k .G rnw-HiU, New York.
1961, p. 530.
6.7 I HANSVlHSl.LY LUAU~U I HINWALLfU BEAMS 285

'
SC __...v,
Figu:rc 6.75: 1lu: .:hear force ~ and thC' concs:pond1ng shear Bow.

1/re C'f'lllerli11 of1hefla11ge (Fig. 6. 77). By 11eg/ecti11g tire effuts ofaxi11/ rmrnim,fi11d
tire JJOSitio" ofrhe s/1e11r center aud ca/cu/are the 1naxi11uun deflection. tire 111axi111un1
nvlst, and the 1>ly stresse.'i a11d strains.

Solution. Frtlm 'Thblc A. Lthe tensile stiffness and the locntion of the centroid are

~ 2/>r b,,
EA = -
( - )
au r
+ (-au )-w = 30.87 x 10 N
(6.313)

y, =-=
1(2b,br
-( ) - + -(
1\ "n t 2
b..)
-,- d, =0.0340
au .,..
m,

where a 11 for bolh 1he web and the Oange is {au).= (au)1 = 5.18 x 10- mJN
(Table 3.8. page 85). The dimensions bi = 0.05 m. b. = 0.06 m. 111 0.049 m. and =
d = 0.062 mare shown in Fig.6.76. The bending stiffnesses are (Tuhle A. I)

~I,,. =---+--+-~
br d'
(11u)r 2
lbr b!
(du) 1 12(011). (6.314)
fL = ~,,,, _ )'<)' + ...!!!:..__ + _2_ (i + Cbr - )'l') .
" (1111).. (d11)w (011)1 3 3
where du for bolh lhe web and lhe flange is (tf11 )w = (d11 )1 : 33, 10 X l()- l NI m
(Table 3.8, page 85). Equn1ion (6.314) yields

fi,,, = 22 OIS N m' -


/;.z = 8188N ml . (6.315)

The torsional stiffness and the location o f the shear center are given in
Table A.5. For symmeirical layup (which is the cme here). & is replaced by d

6,-=SO mm

p -1 SOONlm
t* ***+i JQI v.sc

ii'"'""' ~ :
/..- 600 mnt
- '

I
- d , 1i I c t t
J::jgurc: 6.76: The: bco1n in Exnmplc 6.6.
N
p=- l soo-
m

Figure 6.77: Loading on the C-sc-cllon he am 1n


Example 6.6 (left) and loading with respect lo
the shear cc.ntcr (right).

and a by a, and we write

GI,= 4 ((!~), + (d~)J = 13.t9Nm2 (6.316)

2!!L
e= fib (au )i d = 0.0207 m. (6.317)
=-+--
{<111Jf ( <111J..

where for both the we b and t he Hange ti,,. is (d66 )w =(ti,,.), = 48.51 x 10 - 3 "'m
(Table 3.8, page 85).
The loacls\\ith respect to theshearcenter are(Fig.6.77,dr + e - ~ = 0.0447m)

N
p= - 1500- (distributed load)
m
(6.318)
N m
(
t= - px dr +e-- =67.08-- be) (distributed torque).
2 m

The corresponding bending n1oment My,she ar force V~, and torque f diagrams
are given in Figure 6.78. The maxin1um values are
2
.-.nax
M1 = - pl
12
= 45 N. m
---m.1111 pl
V. = - = - 450N (6.319)
' 2
"'='
1 = -t L =20.12Nm.
2
The maximum deHection is (Table 7.3, page 332)

w= !
pl' = - 0.023 x 10- 3 m = -0.0230 mm.
3 4 E lyy
(6.320)

The maximun1 t'vist is at x = L/2 and is

.p =
r.
,I)
l./l
Odx=
1 <>
'f2 f
,..., dx .
1.J/ 1
(6.321)
6.7 TRANSVERSELY LOAOED THINWALLED BEAMS 287

.II,
0.6 Length, s. (mJ SC
- 4SO

20.12
K 0.6
<:::::::::::j L::nglh, :r. <m
Figure 6.78: Bending 1noment Jfl~. shc.ar force ~.and torque f d1ag1nnu1 for lhc: henm 1n
Example. 6.6.

where f vttrics linearly ";th x (Fig. 6.78). TI10s, the preceding integral yields
fm~'t:L
!ft =---;;:;--
4v/
=0.2288 rad =13.1. (6.322)
1

When restrained warping is taken into accounL (I is 0.217 (Example 6.7,


Eq. 6324). This demonstrates that restrained warping must be considered when
calculating the rotation of open-section beams.

6.7 Example. An L = 0.6-111/ong Cstttion bemn. u:ith the cross S'-Ctlon slto1.,n in
Figurt 6.79. is nradt ofgrapltitetpox)~ The 1naterial properties are glvt11 in Table 3.6
(page 81). 'nie lfl)'ll/> i.r (45\/011/45~]. The beam is b11ilti11111 both mds. The
f>ea111 is sul1jectetl to It uniforurly distributed load ( 11 = - 1 500 liJ/nr) acting along
tire centel'line of tire flange. By taking axial res traint into account. Cf1/c11/ate the
11uLXi11111111 dejfet:tlou a1rd the 111axi1u111n nvi.ft.

Solution. 11le bending mo1ncnt ,Qy. shear force V;:, torque f, and the nlaxirnum
deflection arc identical to those given in Example 6.6. The '"'i.St is calculated as
follow:1. The warping stiffness is given in Table A.5 (page 457). We replace a by"

H3 ..,_ []~.
b, SOnun N

Figure 6.7'>: 'lhc cros!I ,;ccllon o f the beam in Bxam plc 6.7 and lhe loading on 1hc l~am.
288 BEAMS

Transvc-rse.:.---"'";:::...;
fran1es
Spar \\eb Long itudinal stiffener

Figure 6.80: Illustration of sti(fencd struc tures.

and write

i!i. = l>!d' -'-


r (a11)1 5.847 N m4 , (6.323)
~ 12

\Vhere the dimensions b1 = 0.05 n1 and d = 0.062 01 are s ho\\n in Figure 6.79, and
a11 for both the web and the flange is (a11)w = (ll11)1 = 5.18 x 10- 9 i1) (Table 3.8.
page 85).
The maxilnum t\vist is at x = L/2 and is16

o/f= tL [P L _ tan h({JL) ] =0.00379rad=0.2 17' . (6.324)


21,,,f!,' 4 4
where r is the distribute d torque load (r = 67.08 ~ Eq. 6.318), and (!, is

Rf7. =
(!, =
vm:. 1.502, (6.325)

where (JT, = 13.19N m2 is give n by Eq. (6.316). If we d isregard restrained warp


ing, the maximum twist is '{I = 13.1' (Eq. 6.322}. By comparing this value with
the value give n above in E q. (6.324), \\'e see that restraine d warping significantly
reduces the twis t.

6.8 Stiffened Thin-Walled Beams


Tbinwalled structures a re often reinforced \Vith longitudinal stiffe ners and trans
verse frames. Illustrations o( s uch rein(orced (semin1onocoque) s tructures are air
plane wings and fuse lages (Fig. 6.80). In many applications such structures may be
treate d as reinforced tbin..walled beams. The n, to simplify the analysis, the beam
is idealized by representing the stiffeners as concentrated areas., son1etimes called
flanges or booms (Fig. 6.81 ). It is assumed that the flange carries a ll the be nd ing
load, and the we b be hveen the flanges carries the shear load. In the fo llowing \Ve
analyze stiffe ned beams in which the v.eb is orthotropic-.

16 Ytl. C. YC1ung and R. G. Budyn.as.: Roo.rk's FofJJl/J/as for Str..sstsn11ti Strallu. 7th edition. t-.kGra"'-
HiU, New York, 2002, p. 425.
6.8 STIFFENED THINWAUED BEAMS 289

coco
Figure 6.8 1: An actuaJ and the ideaJi7.cd stiffened t h1n\\'allc-d beam.

The tensile stiffness o( the beam is

EA= I;"' EA,,,. (6.326)


,,._,
y,here ti\.i1 is the te nsile stiffness of the tuth Hange. and J\1 is the total number of
flanges (Fig. 6.82). The coordinates of t he centroid are

)'t = (6.327)

where Zma nd Jini are t he coordina tes of t hetuth Hange in t he Y-l arbitra rily chosen
coordinate syste1n.
The bending stiffnesses are

-
El" = L,, YmEA."'
'\""' l -
(6.328)
,.,...,
"' t.. EA...
nyy = I: (6.329)
m- 1

"'
El,., = L YmtmEAm, (6.330)
11.-1

y,here <ni a nd Ym a re coordinates of the nrtb flange in the coordinate systen1 at..
tache.d to the centroid. The displacements and the radii o( curvatures o f semi
monocoque beams can be obtained by replacing EA a nd El by the preceding
replacen1ent stiffnesses in the e xpressions describing the deflections of the corres-
ponding isotropic be am.
The nonual force carried by the nrth flange is

- (1
1VJ.m = 1 ) EAm.
-Zn1 +-ym+f; -
Pr p,
(6.331)

n
Y,,. - ;
t - ,,, :
:!,., ~C ....~---'---1---->
y

y,,, ii
F"1gurC' 6.82: S1tffC'nC'd th1n-w.aUC'-d bC'-am.
N
/ -
/ ~
- ~~q,_,
:-,; + {)/\'"" d
. ...,. Bx :r:

Figure 6.83: Equilibrium or a ftangc. of a stiffened thin-walled hcam.

\Vhere Py. P:; a re the radii of curvatures of the axis passing through the centroid
and f~ is t he e longation of the longitudinal axis through the centroid (Eqs. 6.17
and 6.19).
111e shear fio\\'S in reinforced (semimonocoque) and unreinforced (mono
coque) bcan1s are d iffe rent. ln reinforced \Valls the shear flow is calcula ted on
the basis of the preceding stated approximation, namely, that the Hanges carry
all the bending load and the v.aJJs the shear load . Force balanc.e in the direction
of the flange results in (Fig. 6.83)

aNJ:m
q1 - {/I- I = - - - (6.332)
ax
\Yhere q1 and q1- 1 are the shear Hov.s in l\VO adjacent walls. and N,,m is the axial
force in the nllh flange. By utilizing the expression given for l".J.r:m in Eq. (6.331)
and employing Eqs. (6.17) and (6.19). we obtain the change in shear flow induced
by a llange as follows:

q, -q,_,= -
ifl._ v. -
...._:_
El,. v, -
. .. ,tmEAm
Elyy El,_ - (El,._)-
- EI,,V, + El,.,V, -
- -- ,YmEAm. (6.333)
El,., El,, - (El,.,)-

Torsion. In the absence of axial constraint there are no axial forces e ither
in open or in closed.section sen1imonocoque beams subjected to pure to rque.
T bere(ore, the presence of boon1s does not affect the shear flow, and the a na lyses
given in Section 6.5 (or the torsion of unreinforced (n1onocoque) thin ~\valled
beams a pply to re inforced (se mimonocoque) thin\valled bean1s.

6.9 Buckling of Beams


In t his section ""e address the buckling loads of ortbo tropic con1posite bean1s. The
buckling loads o f such beams are calculated from the equilibrium equations (which
include the changes in geon1etry during buckling) and the strain-cUsplace111ent
and the force-strain rela tionships. The first two of these are identical for isotropic
and orthotropic beams. Ho\vever, the force- stra in relationships of isotropic and
orthotropic bean1s are d ifferent. The force- strain rela tionships for an isotropic
beam are obtained by combining Eq~ (6.4) and (6.239)
Lw>tropic:
fJ EA 0 0 0 0
,...
,fl, l.
0 Ef.vy Ely, 0 0
At,
f.
= 0
0
1,.
0
El::
0
0
GI,
0
0
..
l.
ii
(6.334)

f 0 0 0 0 El.
"
-r;r
We express the force- strain relationships of orthotropic beams as (Eqs. 6.7.
6.8, and 6.240)
orthotropic:

fJ CA 0 0 0 0 ..
<"
l.
Sf, 0 Ei_,, El" 0 0 p,
IJ, = 0 !fr" t!l" 0 0 ,,l. (6.335)
T~v 0 0 0 Gi,, 0
T..1 0 0 0 1,.D
0
""
- a;i
TI1e equ11tions governing the buckling loads o f isotropicnnd orlhotropicbeams
differ only in their stiffness n1atrices. Therefore. the buckling load of a n ortl1otropic
beam can be obtained by replacing the isotropic stiffnesses with Lhe replacem ent
stiffnesses in the expression of the buckling load of the corresponding isotropic
beam as follO\\'S:
Isotropic beams Orthotropic beams
EA=> EA
(6.336)
E/11 El,,. El., => El,,. El::. Ely:
GI,. 1. => CJ,. 1.,,.
The s1if(ncs.scs of selected ortho tro pic beams are give11 in Tt.tblcs A. l - A.5. The
warping sti((ncss fi.u
is give n only for opensec.tio n bcnn1s bccnu.sc, gene rally. it
ca n be neglected fo r closed~section beams.
J\ sexiunplcs. in the follo wing \Ye present tl1e buckling lo:tds o f beams subjected
lo axia l loads, bending 1n ome nls, and transverse load~.

6.9.1 Beams Subjected to Axial Load (Flexural-Torsional Buckling)


\\'e nov. consider onhotropic beams subjected 10 an axial load. Under this load
the beam may undergo Oexural and torsional buckling..
Doubly sy111111t trfcal cross section. \\1e consider orthotropic beams v.>ith cross
sections symmetrical with respect to both the}' and the zaxes (fl,, = 0). When
1he cross sec1ion of the beam has f\\'o a.'tes o( symmetry (doubly symme1rical cross
section). it may buckle (a) in one of the planes of symmetry, (b) in the other plane
of S)mmetry. and (c) torsionally (Fig. 6.84).
Buckling loads of doubly symmetrical cross-section isotropic bcan1s are given
by Ttmoshe nko and Gere. 17 By e xchanging the isot ropic st iffnesses in the

11 S. P. Tinlm hcnko nnd J. Gere. 771cory t1f EltJJrit: SwhiUry. 2nd edilltm, ~fcOl'nwI hll, New York.
1961, pp, 41) and 2211.
292 BEAMS

y y

Figure 6.84: lllu.strations of buckling in the x-z and .t-y plan e." and o( torsional buckling.

T imoshenkoand Gere expressions witl1 t he replacement stiffnesses (see Eq. 6.336),


the buckJing loads of o rt hotropic bean1s are

V' tr
2
Elyy buckling in the x- z plane
I o y = (kl)'
= 11'21:.:. (6.337)
buckling in the x- y plane
(kl)'

torsional buckling.

\\here f~a6 w is

.-..s 1 rr 2 El.,,
iV
O w
= -(kl)'
- -
;,;
(6.338)

L is the length of the beam. i", is the polar radius of gyration of the cross section
about t he shear cente r, \vhic.h, (or an isotropic bean1, 1g is

(6.339)

\Vhere )':11. and Uc a re the coordinates of the shear center \\ith respect to the cen
troid. For a n o rt hotropic con1posite bean1 v.e define i.., as

-2 ' ' Ef:.:. + EJ,v


'w = i;, + y~ + Ei\ (6.340)

T be effe ctive length facto r k is given in Table 6.11 for diffe rent end conditio ns.
For long beams (Gt, El,,,/ L') the to rsiona l buckling load simplifies to

torsional buckling
(6.341)
long beam.

For short beams ( & , El. / L') the torsional buckling load is
to rsional buckling
(6.342)
short beam.

1$ l hid., p. 233.
6.9 BUCKLING OF BEAMS 293

Table 6.11. The factor k In Eqs. {&.33n and


(6.338). The simple supports in cases
(a) and (b) restrain the rotation about the
x-axis, but the cross section is free to warp
and free to rotate about the y- and z-axes
Geometry, kJading k

()
A
~
L

~ - k=I

(b)

(c)
]

~
~ -
<--
k=0.1

k = 0.5

(d) <-- k=2

Sy1t1111etricol cross sectio11. \Ve consider orthotropic bean1s \Vi th cross sections
sym1uetrical \\~th respect to the t-axis ( l!J,,z = 0). When the cross section of the
beam has one axis of symme try. it may undergo (a) buckling in the plane of
symmetry (x- z plane), or (b) combined flexural- to rsional buckling. The buckling
load corresponding to buckling in the x-z plane is 19

buckling in the x- z plane. (6.343)

The buc.k ling loads ,(JCT2 and Rn3.corresponding to flexural- torsional buckling,
are the roots of the following equation:

I[ ~. MB '
N,r!Jtt;..
J-If;JCT [ 1
<!<"
""'
,,
t;;. JI-
- 0' (6.344)

where I I denotes the de tern1inant. This equation can be ""ritten in the forn1

(6.345)

where ~y 1Y:;
and ~rV are given by Eq. (6.337), and t he e f(ective length factor k
is given in Table 6.11 .
Equation (6.343) yields the value of F/01 , and Eq. (6.345) yields two values of
Rn denoted by Rai and lJcrl The buckling load of interest is the lo\\est of these
three values. \Ve note that tJ1e lJcri and Na3 values resulting fron1 Eq. (6.345)
are approxiluate when one end of t he beam is fixed and the other end is s in1ply
supporte d (case b in Table 6.11 ).

19 Jbid., p. 235.
bn = 48 mnt
=

b.=48m
m
!I ld=SOmm f----.~v ; :,K
11 = 2mm
K- -->I
_.,=== !
11,2=36 ntm
1-igurc 6.85: The cr05s sec lion of lhc beam in E.)'.amplc 6.li and lhc loading on th e:- hcam.

Unsy111111etrical cross sections. \Vhen the cross section is unsymmetrical, a n or


thotropic.bean1 undergoes con1bined flexural- torsional buckling, and the buckling
loads N0 are given by the solution o( the follo\\ing (third-orde r) equation 2D:

0 0
z,, ]
~)' 1 -y~ = 0. (6.346)
0 - y)'(.. i!
Jn the preceding equation the coordinate directions y and .z n1ust be in the
principal directions (page 208) such that Et,, is zero (Et,., = 0): I I denotes
. --u --u ---s .
the deternunant. The terms N,1 Y , Nn~ and N.:r~ are g1ven by Eq. (6.337), and
the constant k is given in Table 6.1L The solution o( Eq. (6.346) results in three
values of Rcr. o( \Vhic.h the lowest value is of inte rest. The resulting R.:r is approx
in1ate when one end of the beam is fixed and the other e nd is simply supported
(case bin Table 6.ll).

6.8 Exan1ple. An L = 0.5111-long I-section bean1, ~"ith t/Je cross section s/rolt1n i11
Figure 6.85, is n1ade of graphite epoxy nuidirectional plies. Tile 111arerial properties
are given i11 Table 3.6 (page St). The layup is (O,.J. The beam is simply supporred at
ead1 e1Jd. Detertuine the buckling load 1vhe111/Je beaut is s11bjected to au axial load.

Solution. The co1npliance n1atrices for the flange and the \Ve b are the sa1ue, and
their re levant eleme nts are (Table 3.8, page 85)

"" = 3.38 x JO - N
m

_, 1 _, 1
d11 =10.14 x JO - N """ = 329.67 x JO . - -
01 Nm

The din1ensions of the cross section are br1 = 0.048 m, bri = 0.036 m, bw =
= =
O.B48 m, d 0.050 m, ii 0.002 m (Fig. 6.85).
l11e tensile stiffness, the location of the c.entroid, and the bending stiffnesses
are gi,.en in Table A.2. For symmetrical layup (511)1and (811)w are replaced by d11

?I) ]hid. p. 233.


and (au)r and {au)w by au , and from Table A.2 we have
-bnbr2bw
EA= - + -=. + - = 39.07 x JO6 N
a 11 0 11 0 11

t.<
1
= - -
!!A
(hr'd + ub. 2~)
11
= 0.027 27m

- + - -1 + - - =3884Nnr
b.,. L b~ 1 b-ii_ ~
/.. = (6.347)
du 11 12 11 12
- br1 ,/Jn,bn
EJ,,.
..
- (d - z,)" + -ll11
= ll11 z; + d- 11

+ b12 +-1- (b~, +b!,)=18074Nm2


du a11 3
where bw1 a nd b., a re calculated with the preceding value of tc (see Table A.2) as
follows:
!J.,.., = Zc - 2/r = 0.026 27 m h., = bw - b,,, = o.o:n 73 m.
The torsional stiffness, the location of the shear center, and the ""arping stiff.
ness are given in Table A.5. For symmetr ical layup (&i)r and(&;). are re placed by
d,,. and (au), a nd (<>u ). by au (Table 3.8. page 85), and we write

Cl, = 4 (b" + bn + .!.!...) = 1.602 N m'


"i1" " d..
11
e = d h; " v.' = 0.035 17 m (6.348)
(I + .t!
"" au
b~-,.

Elw= ~2 ed = 2.024 N . m4
The cross section is symn1etrical \\ith respect to the .taxis~ and the coordinates
of t he s hear center with respect to the cenuo id are {Fig. 6.85)
)'i;c: =0 ~' = e - <.c = 0.000 789 n1. (6.349)
The polar radius of gyration about the shear cente r;. is (Eq. 6.340)

i.., = '
zk +y!O'I.. +
' Ei., +- Ei,,. 0.024 99 m. (6.350)
EA
For a simply supported beam (k =I , Table 6.ll, page 293), Eq. (6.337) gives
2
iv:!, = " f,11'" = 713.55kN x- t plane

- =
N~, "'L'
lfl".. = 153.32 kN x- y plane (6.351)

iv:!~ = ~'N + kGi,


IN
= 130.53kN torsion,

where (Eq. 6.338)


- Elw
,-
N0 = i:!;I tt(kl)' = 127.96kN. (6.352)
296 BEAMS

The buckling load IV"' is (Eq. 6.343),


1V0 1 = N:', . = 713.55 kN buckling in the x-z plane. (6.353)

and N0 2 and 1V0 3 a re the rooL< of Eq. (6.345) as follows:

I~ (i! - Z:~) - 1\J0 (~t;: + l~J) )i! + l~r; ~\\'i! = 0. (6.354)

Solution of this equa tion yields the flexural- torsional buckling loads
N 2 = 208.88 kN
0 R"' = 106.43 kN. (6.355)

6.9.2 Lateral-Torsional Buckling of Orthotropic Beams


with Symmetrical Cross Section
We consider orthotropic beams. The c.ross section of the beam is syn1me trical with
respect to the z-axis (Et,,= 0). The beam is simply suppo rted a t each end. At the
s in1ple support the beam is restTa ined fro111 rotating about the x-axis., but the cross
section is free to rotate about they and zaxes and is free to \Varp.
The beam is subjected to t\vo e qual and opposite bending n1on1ents at the two
ends of U\e beam (Fig. 6.86, top), a uniformly distributed load p in tl1e plane of
symmetry (Fig. 6.86, middle), o r a concentrated force Pin the plane of symmetry
at the midspan of the beam (Fig. 6.86. bottom). The distance between the shear
center and the point where t he load acts is denoted by 6..
At certain applied loads the bea1n buckles laterally v.hile the cross sections
of the beam rotate (Fig. 6.87). This phenome non is called la teral buckling, or
lateral- torsional buckling.

Figure 6.86: Lateral b uckling o(beamssubjec.ted to l\\'oequal and opposite bending mo mc.nts at
the two e nds of the beam (top), a unionnly distributed load pin the plane o( symmetry ( middle),
and a concentrated forc.c- /' in !he plan e of symmc.try at the midspan o f the beam (bottom). The-
dislance betwe.cn the shear center and the-poinl \\here !he l0<1d ac-Lc; is denoted by .1..
6.9 BUCKLING OF BEAMS 297

Figure 6.M7: lllrnllrnllon o f lntc rnl huckling of a simply supported bca1n,

AIJen and Bulson11 presented the foUo\\~ng fom1ula for Lhc buckling load of
an isoLropic bcanl:

OJ, L'
-
Q ,T l "' [
0 = G 1 L' el::
/J1
O,ii + G,
2 (Gii + G,-)
2
/J1
2
+ =-
"'
1,,
2l + =->
""
1. ,,_
( - )] .

(6.356)
where Q 0
is lhc critical value of the bending moment and is related to the applied
loads. as sho""" in Table 6.12. The positive sign before the square root results in a
positive load (\\hich t'lcts up,vard). \\ hereas the negative sign results in a negative
1

load (which nets downward); G 1- G 3 are constants. \Ve recall Eqs. (6.337) and
(6.338). \vhich relate N~: a nd ~\!' to the stiffnesses

N" - "' lfl,, (6.357)


" ' - (kl.)'
fJer~
- .
=fJ"tr+ j'i1 67, t
1 "' 1.
= ji 1 67
.
(kl)? + [2 ,.

where k = l or a imply supported beam. By combining and rearranging


(6.358)

Eqs. (6.356H6.358), we obtain

- =F1N;,,
Q0 ""' (F,A + F,f!t ~(Fili+ F,/l,)' + N::.1;)
~' (6.359)

where F1 F,. F, ore constants (their values a re also listed in Tobie 6.12). and fl1 is
a paran1c1er that depends on 1he shape of the cross sectionZl
(6.360)

21 H. G, Allen i1nd P. S. l.lul~on , Btukg11nu1d 10 Buckling, f\.fcGruw-Hill, IA ndon. 191\fl , p. 492.


1l Jbid . p. 195.
298 BEAMS

Table 6.12. The buckling loaclS and the corresponding


conslants in Eq. (6.359).
Fo fz F,
(a) Moment !'C"lr. =fl1 0 0.5
(b) Oistribuled load pa = is~., l.13 0.45 0.267
t.
I'. _ .. q.,
(c) Conc-entrared force n - I. l.35 0.55 0.2 12

For a beam n1ade of isotropic material 11 and 12 are

11 = - 1[ u' lA
fJ'Y (A)
J, = -1
In
1, \A/
ty-dA (6.361)

For a thinv.alled, open~section isohopic bean1 . 11 and 12 n1ay be 'vritten as

.f, = El/ [ ( Eli) t 3d11


>'Y ( S)
isotropic. (6.362)
J, = E l/
Y>'
1 (SJ
(Eh) ty'd11

\Vhere/J is the thickness of the \Vall and'' is the coordinate along the circun1ference.
For au orthotropic beam Eli is replaced by l/af-1 (Eq~6.243 and 6.241). and El is
replaced by El (af-1 is evaluated at the " neutral" plane; see Eq. 6.105). This gives

,, = - -j
1
l!lyy (SJ
+tdt)
a If
orthotropic. (6.363)
+ty'd~
1
;, = [
Elyy ( S) au
For orthotropic lbean1s whose crosssection is syn1metrical \Vith respect the
midplane of the web the preceding integrals give
l ( I 3 I , l b! 1 - b!2 )
11 = "" .
( -
1:1yy
) bn (ii - z,) - -( ) l>n~ +
a 11 n a 11 a
(au )w 4

], - - 1(11"n
- - El_,, (ar,)n
_ _ _!! (d - t ) - _ _ _!lz,
'
(af,lr. 12
lb') (6.364)

The dimensions are defined in Table A.2 (page 455).


For doubly symn1etricaJ isotropic and orthotropic beams. /Ji is zero.
Equation (6.359) is accura te for beams subjecte d to concentrated be nd ing
moments (Fig. 6.86, top). For simply supported beams subjected to a distributed
load or to a concentrated force at the midspan, Eq. (6.359) gives the buckling load
\vithin about 5 percent.
6.9 Exantple. An L = 0.5111/ong / section bean1, with the cross secrion sholvn iu
4

Figure 6.85, is 111ade of grt1phi1e epo.ry to1idirec1ional plies. The 1na1eria/ properties
are given in Table 3.6 (page 81). The layup is [O,.J. Tire beam is simply .rupporred at
6.9 BUCKLING OF BEAMS 299

L P~
m ~ J, ~

I' ~ SC


.! - - -

l = 500 nun
-

Figure 6.88: 'fbe beam in Example. 6.9.


+

] " i:
eacl1 end. Deten11i11e rite buckling load 1vheu the bea111 is subjected to a distributed
load tilong 1/1e 10p flange (Fig. 6.88).

Solution. For a trans\'ersely loaded beam the buckling load is (Table 6.12)

(6.365)

where (./0 is gi\'en by Eq.(6.359) as

(6.366)

The sign before the square root is negative bec.ause the load is do\Vn\vard.
From Eq. (6.364} with af1 replaced by au , we have

I ( 1 , l , I b! 1 - b!2 )
J, = ,,., - bn (ti - )' - - brit , +
1!1 ,. a11 a1 1 au 4
1 (6.367)
l(lb/,
J, =-=- lb/,)
--(d - z,) - -~ ~ .
- Elyy a11 12 au 12

The parameters of interest in this problen1 are (see Exan1ple 6.8. page 294)

m
"" = 3.38 x 10- 9 -N EI,, =18074N m' h = 0.002 m
bn =0.(}48 m b,,=0.036m d=0.050m
bw2 = 0.021 73 m b..1 = 0.026 27 m t.o =0.027 27 m
z~ = 0.000 789 m

1V~" = 153.32 kN -. e=0.03517 m


Nny =713.55kN
iw=0.024 99 m
N~~ = 130.53 kN.

\Vith the preceding parameters \\'e obtain

11 = - 0.003 770 m J, = OJJOJ 694 m. (6.368)


JOO BEAMS

and f!, 1 is (Eq. 6.360)

(!,, = .1, + J, - 2z" = - 0 .017 86 m. (6.369)

The bean1 is loaded on lhe top flange~ hence, v.e have

!> =d - e+ /r/ 2=0.oJ584m. (6.370)

The \'alue of Q,Tis c.alcula ted \vith t he preceding paran1eters a nd 1Nith F1 =


1.13. F, = 0.45, F, = 0.267 (Table 6.12, page 298). The result is

Q 0 = - 3607N m . (6.371)

Equations (6.365) and (6.371) give


8Q0 N
Pa= - L'- = - 115 407 m
- (6.372)

6.9.3 Local Buckling


ln this section v.e consider local buckling of orthotropic thin~wa lled beams that
arise due to axial compressive stresses introduced by axial forces and bend ing
moments.
Local buckling analyses of beams are generally performed by modeling the wall
segn1ents as long plates a nd by assuming that edges con1n1on to tv.o o r more plates
remain straight. Then the buckling load n1ay be determined either by assun1ing
that every \\'all segtnent buckles sin1ulta neously while the continuity conditions
at each intersection a re satisfied .23 24 or by considering the wall segn1e.nts as in
di,~dual pla tes rotationally restra ined by the a djacent \\all segments.2s-27 Here,
\Ve derive explic-it expressions for the loc.al buckling loads based on the latter
me thod.
The procedure for calculat ing the load at \\ hich local buckling occurs is illus
1

trated via the exa mple of an axially loaded thin.walle d, closedsection rectangular
beam (Fig. 6.89). First, each wall segment is considered lo be simply supported
(Fig. 6.89, a) a nd the load (N... ar
fo r each wall segment is calcula ted using the ex-
pression given in the first ro\vofTable 4.11 (page 136). The axial strain at 'vbich the
wall segment buckles is calculated by ( /\'.,_ 0 )"a11 (Eq. 3.31), whe rea11 is the II ele-
me nt of the cotnpliance n1atrix of the \Vall segn1ent. The segme nt \\ith the lo\vest

ll D. J. Lee. The Local Buckling Coefficient ror Onbotropic StrucluraJ Sections. Aero11fJ1uic11l Joun1fJ/,
Vol 82. .113--320. 1978.
u A. Zurcick and B. Shih, Local BUl' kling of FiberRcinforccd Polymeric Structural ~tcm bcrs
Under Linea rt)'- Varying Edge Looding - Part I . Theoretical Forn1ulntion. C<11ripfJ.f ilt! StnJa11res,
Vol. 41, 79-86. 1998.
2;'I P. Qiao. J. F. Daval06, nnd J. \Vang. Local Buck.L ing or Composite FRP Shnpcs by Dis.:rc1c Piute
Analysis. l fJun111/ 1,/ Strr1c11Jral Erisinceri11g, Vol. 127. 245-255. 2001.
26
E. J. Barbero and J. Raf1oyianni.s. Local Buck:lins of FRP Bca1ns and Columns.Jtn1r1wl t1ff.lt11eria/:1
in Civil E115U1e~rlr1~. Vol. 5, 339-355, 1993 .
27 J. P. H. \Vcbbcr. P. T. Ho ll. and D. A. Lce, ln.""1abilil)' ofC.arbon Fibre Rcin! o rttd Anngcso f IScctietn
Bcan1s and Colun1ns. Cot'1p<JSitc Suuaures, Vol. 4. 245-265. 1985.
6.9 BUCKLING OF BEAMS 301

Figure 6.$9: Local buckling of a n axially loaded liox beam.

critical s train is considered further because the \Vall segn1e.n t (\ve.b or flange) in
which the a.x;aJ strain is the lowest is n1ost s usceptible to buckling. \Ve calcula te the
buckling load of this segme nt by ueating it as a plate rotationaUy resua ined by the
adjoining wall segme nt o r segments (Fig. 6.89. b). The buckling load is calculated
by t he expression given in the fo urth row of Table 4.1 I (page 136).
'TI1e spring constant f depends on the adjacent (restraining) \Vall segn1ent. To
determine the s pring consta nt, we recall Eq. (4.149)
- M,.
k= a.; ' (6.373)
>y

where aw/a y is the rotation of the edge.


t\ conservative estin1ate of k is obtained2~ 1.vhe n the de(om1ed sh ape is cylin
drical ("long-pla te" approximation). For a long plate (K, = Kxy = 0), the moment
is (Eq. 4.3)

(6.374)

where t he subscript rs refers to the restraining segment (Fig. 6.90). The term
Ky = - &2 w0 /8 y2 (Eq. 4.2). and we write

aw I
- -&w I
1. U
L .12
(- <y)dy=-
.._,_., <)y
3y a l L,./l alO
' (6.375)

where aw/ay is zero at L.,/2 (Fig. 6.90), a nd we have

KL.,- - -
.Y 2 -
8wl
(.I )' a l (I
(6.376)

Equations (6.373), (6.374), and (6.376) give

k= 2(D,,)., _ (6.377)
L.,

~ F. Bkich. 811ck li11g bf ,\/etlll S11uc1u~s. '-kGrnw-HilJ. New York, 1952, p. 339.
302 BEAMS

1110 11$urc 6.90: The restraining wall sc.gment of the


OOx beam.

The preceding expression is valid when no axial load acts. The effect of axial
Loading is ta.ken into account by an a mplification factor r defined as29 30
1
r= ~- (6.378)
1- (N
""
)"
..
where (N,)., is the applied axial load (Fig. 6.90) and (Nx."J:: is the buckling load
of the sin1ply supported restraining wall seg1nent; r is unity \\hen the axial load is
zero and is infinity \vhe-n the axial load is equal to the buckling load. Taking this
amplific.ation factor into account, the spring constant is

(6.379)

The axial s trains of the restraining and the buckled \\aJI segn1ents are the same,
and. conseque ntly, (II', )., is (Pig. 6.89, b)

(11).,
(/11,),, = (N,_")""-(_)_. (6.380)
" ' 1. f':I
\vhere the subscript bu refers to the. \Vall segn1ent that buckles and ( 1\~v. cr)bu is the
buckling load of the rotationally restrained buckled wall segn1ent. The value of
(N.ccr)bu is not knO\\'Il a priori. \\'e n1ay approximate (Ns.cr)bu by the buckling
load (1V:r.a-):_. of a simply suppo rte d \Vall. \Vith this approxima tion, r beco1ues

r =1 -- -
(,\\- - - )t-..
..;1)~.(1t11
(6.381)
(N,...r,H1tu),.

If the we bs a nd the flanges we re s imply supported, t he ir buckling loads would


be (Table 4.ll , first row, page 136)

(II', a)~'= ~: (2j(D11lr<Di2)r + 2((D12)1+ 2(D)r))


r (6.382)
"' ~----
..
(11',a)-: = />2. (2j(D11).(D,2)w+2((D,,),.+ 2(DM)w)).

~ Jbid .. pp.33 1-339.


JO S. P. Tin1oshenko and l Gere, Theory of /,uric Swhilil)'. 2nd edition. 1-.kGraw-Hill, New York.
1961. pp. 15 and 424.
6.9 BUCKLING OF BEAMS 303

(t1> (b )

1-igure 6.9 1: LocaJ buckling of an axially loaded I beam.

11ie flanges buckle first when (i\',. 0 )r (a 11 )r < (N.,.a)".: (a 11 )w In this case the
web restrains the rotation o( the flange, and the spring constant is

k= 2(D,,,)w
b.....
(i _(N.r.cr)!:(a11)
" )j' (iln )r ).
(Nx.
...
(6.383)

The buckling load of the flange is then calcula te d wit h this s pring constant by
the exp ression in tlie fourth row of Table 4.11 (page 136).
The v.ebs buckle first \Vhen (Nx.n}("(a11)r> (1~,.a)~(a11)w. In this c.a se the
flange restrains the rotation of the \veb, and the spring constant is

k= 2(D,,)r (t _ (N, , "):!'.(a11)w). (6.384)


br (N, .a)r(a11)r
The buckling load of the v.e b is then calcula te d v.ith this spring constant by
the expression in the fourth rov. of Table 4.11 .

I-beams. We now a pply t he precee ding proccdwe to I-beams (Fig. 6.91).


If the web and the ftanges were s imply supp<>rte d, their buckling loads would
be (Tables4.9, and 4.11, first row, p ages 125 and 136)

( Nx .a )" _ 12( D"')r


f - (br/ 2)2
(6.385)
al:!'.= ~ (2j~(D
-,-,)-..-(J),
- ,,-}-. . + 2(( D12) .. + 2(D)w)) .
(N,
..
The flanges buckle first \Vhe n (Nx. .:r)~"(a1 1)r < ( N.x,n): (a1 1)w In this case the
web restrains the rotat ion of the flanges. and the spring consta nt is

k= (D,,)w
bw
(i - (N,.a)7(a11)1)
(1~T. n)~(a11).....
(6.386)

The factor 2 is omitted because the v.e b restrains two .. half.. flanges. The buck
ling load of the flanges is calcula ted \vith this spring cons tant by t he expression in
the sixth row of Thble 4.1 I.
111e \Veb buckles firs t v.he n (1Vx,n)7(a11)r > ( 1~t. n)~(au)w ln t his case, as
a conservative esrin1ate, v.e take k to be zero (k = 0). The buckling load is then
c-alc.ula te d by the expression in the first rov. o( Table 4. 11. This expression is devel
ope d by replacing the rotationally restrained wall segm ents by s imply s upported
J04 6t.AM5

figure 6.92: The cross sc-cllonol'the beam tn Exampk


6.10.

l1, -=- 9S.2 mm

\Vall segments. Expressions that do no t include this sin1plification but tnke the
rotational restraint into account are given by Kolllir.3 1

6.10 Example. The box-section bea111 shotv1rin Figure6.92 is subjected ro axial co111
pression. Detenniue the load thot results iJJ local buckling. The bean1 is orthorropfr.\
and the bending stiffnesses of both the flange and the lveb are Dil = 444 N Ill,
D21 =461 Nm. D11 = 103 N Ill, D,,6 = 107 N m.

=
Solution. The widths of the Range and the web arebr 0.0952 m and b. 0.1968 m. =
With these values the buckling loads (with the flange and the web considere d as
simply supported plates) are (Eq. 6.382)
ir 2 kN
(Nx.a}~ = ,. , (2J D11Dn+2(0,2 + 2D,.)) = l 674 -
1~ m
(6.387)
1
ir kN
(1\1,,0):! = b' i2J D11 J),..2+2(D12 + 20,,.)] =
~
392 - .
m
The layups of the web and the Range a re identical, (ll11 )1 = (a11}w- Conse-
quently, (N,.a)r (a11)r > (1\1,.n)::(au )w. and the web buckles fi"'l. The web is
restrained by the flanges (Fig. 6.92. right). The spring constant is (Eq. 6.383)

k = 2( D22)1
br
(i _(1\l,.a)~(a11}w)
(N,.a)('(a11)r
= 7 414 N (6.388)

The parame te r of restraint is (Table 4.11, page 136)

~ = (~2l)w =
0.316. (6.389)
kb.-
The buckling load of the web is calculated by the expression in Table 4.11
(page 136, fourth row) as fo llows:

(N,,)w = ~.. (2Jl + 4.139s j(D11 ).(J),.i)w + (2+ 0.624 2)(( D12). + 2( D66).)] .
\Vhere
L
s = I + lO~ = 0.240. (6.390)

Jl L. P. KoUtir. l..ocaJ Buclding orConlp05itc.(FRP) Beams. Journal ofSuuc1ural Ett,1tineeni1s. 2003.


6.9 BUCKLING OF BEAMS 305

IJ, = l02 mot

With this value the buckling Load of the web is


kN
(N,,0 ),. = 490 - . (6.391)
m
111is buckling load agrees closely with the result of a finite elen1ent analysis
(Nx. = 501 kN/m) and with the buckling Load calculated by the numerical solu
lion of the equations given by Lee32 (Nx,a = 519 kN/m}.

6.11 Exan1ple. Tiu! lsectiotr bea111 shown in Figure 6.93 is subjected to axial cotlf
pressiou. Derertuine tire load 1/Jat resulrs in local buckling. The bean1 is orthotropic
and the bending stiffnesses of both tire j/(luge and the 111eb are D 11 = 698 iV Ill, Diz =
326 N m, D12 = 127 N m, D,,, = 103 N m.

Solution. The width of the flange and the we b arebr = 0.102 m and I>,..= 0.0956 m.
\Vhen the flange is sin1ply suppo rted along one edge and free at the other edge
and the web is simply supported a t both edges, the buckling Loads are (Eq. 6.385)

( N )" = 12 ( D.,.)1 = 475 kN


x.a I (br/2)' m
(6.392}
"2 kN
(Nx.a):: = l'" (2/ Di1Di2+ 2(D11 +2 D.. )] = 1 750--
~ m
The layups of the web and the flange a re identical, (11 )r = (a11}w- Con.<e-
quently, (1V.,. n)~"I (01I )r < (Nx. n)~(a1 I )w. and the flange buckles first. The Hange is
restrained by the we b (Fig. 6.93, rigbt). The spring constant is (Eq. 6.386}

k = (D,,)w
bw
(1 - (J\lw)~' (arr)r)
(1~T.n)~(au )w
= 2 484 N
. (6.393}

TI1e parame ters 11, K , i;, and 11 are (Thble 4.11 , page 136}

11
= 2D,,,D.,+ Di2 = 0.381 K- 2 "" + Dn
j D, I D,,
0.698 (6.394}

i; = ~D,,). = 2.57 ,, = -;;==~"'1==;;~=;'= = 0.249. (6.395)


k(br/2) j l + (7.22 - 3.55 11)1;

D. J. Lee, The Local BucklingCocrticicnt rorOrthotropk Structural Sec1ions. Atro1uu111C1d Jo11r11a/,


Vol. 81 , 3 1 ~J20. 1978.
306 BEAMS

The buckling load of the flange is calculated by the expression in Table 4. I I


(page 136, sixth row, K < t) as follows:

JU>11)r(D11)1 { ri-::-:, 7 (I - K) }
Ki1115.h I - " +( I - q)6(1 - v)J + --,./,lO"=
+"'"'.t=i=
(Nu,)r = (br/ ) 2
2 4 2~
= 850 kN/m . (6.396)

This buckling load agrees closely with the result o( the fi nite e le ment analy
s is o( Qiao et al." (Nx." = 824 kN/m), with the buckling load calcula ted by the
nume rical solution of the equations given by Lee34 (1V.,.a = 916 kN/n1), and with
the data o( Tomblin and Barbe ro;" who tested beams of diffe re nt lengths. The
buckling loads measured by Tomblin and Barbero a re P0 = 247, 224, and 222 kN.
which yield N.,. 0 = 841. 763, and 759 kN/m.

6.10 Free Vibration of Beams (Flexural-Torsional Vibration)


In this section \\'e present the natural fre quencies of ortho uopic bean1s. These
frequencies a re de tern1ined from the e.q uilibriun1,strain-displacen1e nt, and force-
strain relationships.. The first t\vo of t hese equations are identical for isotropic and
orthotropic beams.. whereas the force- strain relationships differ, the diffe re nces
being in the s tiffness matrices (see Eqs. 6.334 and 6.335). Therefore, as in the case
of buckling loads., the natural freque ncies of an o rtho tropic bean1 c.an be obtained
by replacing t he isotropic stiffnesses 'vi th the replace1uent stiffnesses (Eq. 6.336)
in the expressio n of the corresponding isotropic beam.
In the next sec.t ions \\'e present t he c-ircular frequenc.ies w of ortho tropic be ams
\Vi th doubly syn1metrical, sylnn1e tric.al, and unsyn1n1etrical cross sections. The oa~
tural freq uencies and the pe riod of vibration are re lated to 1.1J as folJo,vs:
w 2ir
/=-
2,,.
T=-
w
(6.397)

6.10.t Doubly Symmetrical Cross Sections


The beam is orthotropic and its cross section has two axes of syn1n1etry y and
z. The n1ass is a lso symn1e tric.al with respect to these axes, and. accordingly, the
center o( n1ass coincides v.ith the o rigin of the y- .z coordinate system .
A beam 'vith t'vo cross-sectiona l planes of syn1n1etry n1ay undergo flexural
vibra tion in either of the t\VO p lanes of symn1etry and torsional vibratio n (Fig. 6.94).

" P. Qiao. J. F. Oa1Jal0$. nnd J. \Vang:. l..o(:d Buckling or Co1npositc FRP Shl1pcs b)' D iscrete Plntc
Analysis. Jt>un1lll o/ S1r11c111rlll r1,;intering, Vol. 127. 24S-255. 2001.
JJ O. J. Lee, The Local Buckling Cocfticicnl rorOnbolropk S1ruclurnl Sections. Aero1wt11icolloul't1al.
Vol. 82. 313-320. 1978.
-" J. Tomblin and E. J. Barbero. Local Buckling Expcrinlcn1s on FRP ColuLnns. T//iri IVtdled S1n~crurt-s.
Vol 18. 97-116, 1994.
6.10 FREE VIBRATION OF BEAMS (FLEXURAL- TORSIONAL VIBRATION) 307

y y

s
Figure 6.IJ.I: lllustnhon of Ylhniuon 1n the x-~ and x-y planes and of ton:IOt\al ,,hf"a11on.

Expressions for1he circular freque ncies corresponding lo 1hc ncxural vibration


of an isotropic beam are given by \ Veaver el aJ.3b By re placing El by 'ff11hecircular
fr equencies of or1 ho1ropic beams are

(w,, /) ~
")2 -- ifl,., 1...11 vibration in the x- l plane (6.398)

(O>..,")' -- El,, !1L


/J L"
vibration in the x- y plane. (6.399)

\\'here L is the length of the beam. p is the mass per unit lcnglh. nnd 11 & fo r
different end supports nre given in Table 6 .13. The sub.script I = l . 2 . ... indic.a tes
the first. second. and so forth. modes.
Expressions for the torsional circular frequencies <{, of isotropic beams are
also given by Wtavcr et al. \Vith the replacement stiffnesses. the circular frequen
cics of long (GI,
El../ L') and short (GI, ifl..t L' ) orthotropic beams ore
torsional vibration
e i1;,
(w~," )' -- GI, L1 long beam
(6.400)

( IJ ) '
w~,
-
-
ifl..
~> L'
t.1L torsional vibra tion
short be an1.
(6.401)

where ti is the poli'r monle nt o ( m ass per unit le ngth abou1 t he s hear ccnter-n.

t+ = f ( A)
p",.,,.<f' + tic1 A. (6.402)

Avcn.p is mass per unit volume. A is the area of the cross .section. and'' a. and Ila
ore gi"1!n in Tobie 6.13 for dirrerent end supporu.
We approximale 1he torsional circular frequencies of a beam of arbitrary length
by
lorsional vibration. (6.403)

~ s.
\.\T. \.\'tll\'CI'. p T1n1m hcnlco, and 0 . H. Young., Vil>rtUiOIJ Pmbklflf.' .,, e,,g/11trl11;:, 5th edit ion.
John \.\'ilc)' & SCln!'. N ew York , 1990, pp. 422-i32.
J1 )bid .. p. 476,
308 BEAMS

Table 6.13. The conslants ;and,..., In Eqs. (6.398), (6.399), (6.401),


(6.406). The simple supports In cases (a) and (b) reslrain the rotation
about the x-axis, but the cross section is free to warp and free to rotate
about the y- and z-axes.
Geometry I'
L
(a)

iJ H I = 3.927
(h) ,,, = 7.069
8""' (i + 0.25)rr

9, =4.730
(c)
"' = 7.853
,,,."'
(i + 0.5)rr

IJBI = 1.875
(d) ,,, = 4.694
ll<';i = (i - O.S}n-
B>"' (i -0.5) rr

91=4.730 Cl = 6.283
., = 7.853 02 = &.987
( )
ID= 10.996 "' = 12.566
,,,. (i + 0.5)rr a (i + l) rr

By using Eqs. (6.400) and (6.401 ), we have


-- J - 2
")' - Ef., s; Gf,c;;.
(w~, - e
L' + H v (6.404)

\Ve introduce the definition

(6.405)

With this definition the circular frequency is

(w t i l' = (W..,; l' + 0er,!;,


Ll
torsional
'~bration.
(6.406)

Equation (6.406) is exact for simply supported beams (case (a) in Table 6.13).
For beams with the types of supports shown as cases (b), (c), and (e), the equa-
tion underestimates the circular frequencies (and O\'erestin1ates the period of
vibration) by less than 6 pe rcent. For cantilever beams (cased), Eq. (6.406) may
underestimate the circular frequencies (and overesti1nate the period of vibration)
by up to l4 percent.
6.10 FREE VIBRATION Of BEAMS (FLEXURAL-TORSIONAL VIBRATION) 309

_..--t--...Y~.,.........( Shear center

'
Centroid u
Center of mau

Fig.un: 6. 'IS: Coonhnatcs or the ccnlc:r or morn: and the 1hc:u center.

6.10.2 Beams with Symmetrtcal Cross Sections


\Ve consider orthotropic beams with cross scclions symmetrical with respect to
the t axis ( ifl., = 0). The mass is abo symmetrical with respect to the taxis. and.
accordingly. the center of mass is on the z-axis. \Vhen the cross section of the beam
has one axis of symmetry. it may undergo flexural vibra1ion in the x- :_ plane as
well as Oexural- torsional ' ribration.
A>r such a beam 1here are 1brcc sets of circular frequencies.. The first of these
is"'
W(
.
-- w,., vibration in the x-:. pla ne . (6.407)

The second and third sets of circular fre quencies. '"2 a nd w3 , correspo nding to
the flexural- to rsional vibration. arc the 1v.o roots of the follo\\'ing equation:

0 ] 2[
(w~.)' ~ - w - Cto - ~)
I - (la -
!! z.,)]1 -
- 0, (6.408)

'
where I I denotes the dete rminant.p i$ the nlass per unit lcngth, !lliC is the coordi-
nate of the shear ce nter. zc; is the coordinate or the cente r of nlass (fig. 6.95). and
f) is the polar 1no1nent of 1nass per uniL le ngth obout the shear cente r

0 = /, p,. ..r[r1 + (z - z")' ) ti A. (6.409)

ln these e quations w~,, w~. and w~, a rc given by Eq" (6.398), (6.399), and (6.406).
Solution of Eq. (6.408) provides 1hc c xncl values o f "'l a nd "" only for simply
supported beams (case (a) in Table 6. 13).

6.10.3 Beams with Unsymmetrical Cross Sections


The cross section of the beam is unsymmetrical and the layup is orthotropic.
The beam undergoes combined Ocxural-tonional vibration, which is analogous
to Oexural- lorsional buckling, Therefore. we extend Eq. (6.408) and write ii in the

~ Ibid_ p.477.
310 BEAMS

following form, which is analogous to Eq. (6.346):

0 - (lG - lK)]
B )'
( w,;
0
()'G i y") =0,

(6.410)

\Vhere w is the circular frequency, y""' and x~ are the coordinates of the shear
center, p is the mass pe.r unit length, YG and tG ar e t he coordinates of the c.enter
of mass (Fig. 6.95), and H is the polar moment of mass per unit length about the
shear center,

(6.411)

where w~,. w~ and w~; are given by Eqs. (6.398), (6.399), and (6.406). Equation
(6.410) yields three sets of>; the values are only exact for simply supported be ams
(case (a) in T.'ble 6.13).

6.12 Exaniple. An L = 0.5-111-/ong !section bean1, lvith the cross secrion sho~vtr ill
Figure 6. 96, is 111ade of graphite epoxy nnidirectional plies. The 111arerio/ properties
are given in Table 3.6 (pttge 81). Tire de1Lfity oftire con1posite is 1.6 g/cn13. The layup
is [02.,]. The bc:a111 is si111ply supported ar each end. Calcnlate the 1u11ural frequencies
of the bearu.

Solution. The n1a.ss per unit length is

kg - l , . kg
p = P<0mpA = 1600 - , x 0.264 x 10 m- = 0.4224 - (6.412)
m 111

where A is the a rea of the cross section A= h(bn +ha + I>.) = 0.264 x 10--' m1
(Fig. 6.96). TI1e location of the cente.r of n1ass co incides v.ith the centroid of the
cross se.ction

ZG =0. (6.413)

The polar moment of mass per unit length about the shear center is (Eq. 6.402)

to)= 1(A)
p, .,,(y' + (z - i..,)' ) dA = Pa>mp (1, + (Zc; - z,,) A)
2

= 263.7 x 10- kg m, (6.414)

where & = 0.000 789 m (Eq. 6.349) and Ip = /" + I,_, is the polar moment of
ine rtia(/,.= 26.24 x 10- m4 , lyy = 122.12 x lo- m4 ).
6.10 FREE VIBRATION OF BEAMS (FLEXURAL- TORSIONAL VIBRATION) 311

b11 -= 48 nun

'

l>.= 48mm
!I l r-----
c
-
SC
y
>
f
2"'

2,

h= 2mn1

bn=36 mm
Figure 6.%: The cross scctionof lhe beam in Example 6.12.

With the preceding values of p and t.:+, and with the parameters Bl = .c;1 = ;r
(fable 6.13, page 308), E qs. (6.398), (6.399), (6.405), and (6.406) give

w6 -
- f,, 481 =8166!
.vt - P L4 s

(6.415)

-e
El. '
B l
L4
= 3 458 ~
s
(JJ1 l 1
W
*1 -- (w B1 )
2
+- ,GI = 3 493 -. (6.416}
~ e L2 s

The circular frequency >i is (Eq. 6.407)

u 1
Wl = Ul>' J = 8166 - , (6.417)
s

and w, a nd w_,are t he roots of Eq. (6.408) as follows:

(6.418}

""= 6 024 s-I W3 = 2838-.


1
s
(6.419)

The natural frequencies are f = wf2;r

/ 1=1300Hz fi = 959 Hz /; = 452 Hz. (6.420)


312 BEAMS

6.11 Summary
ln Table 6.14 \Ve sun1n1arize the vario\L" bean1 problems considered in this chapter
and the re levant section numbers..
Attention is called again to the in1po rtant fact that t he displacen1 ent~ buckling
loads, and natural freq ue ncies o f a n o rt hotropic or symn1etrical cross-section beam
can be detern1ined by simply replacing the isotro pic stiCfnesses with the a ppro
pria te re placeme nt stiffnesses in the expressions for t he corresponding isotropic
bean1. It is for this reason that, in this chapter, en1phasis is give n to the develop1uent
of replace 1uent stiffnesses.
Fo r bean1s \vith arbitrary layup there is no d irect a na logy \\rith isotropic beams.
T here-f ore., each pro ble-01 n1ust be t reated ind ividually.

Table 6.14. Section numbers !Of solid cross section and for thin-walled unreinforced
(mooocoque) beams. S111fened thin-walled (semimonocoque) beams are discussed in
Section 6.8 .
Monocoque

Thin-waned Thin-walled
Solid open-secOon closed-section
En<l loa<ls (1V. M,. 1f:i,. f)
Orthotro pic lti)'UP
A xial and bending (EA En 6.2 6.3 6.4
Torsion
no restrained " 'tlrpin g ( Gi 1) 6.5. J 6.5.2 6.5.3, 6.5.4
\\ith re-Strained warping ( t!J,.) 6.5.5
Arbitrary layup
Axial, bending. an<l torsion
no restrained " 'arping 6.2. 6.5. l 6.6. 1, 6.6.2 6.6.1, 6.6.2
\\'ith re.s traine-d " 'arp ing 6.6.4
Transvcrse- l oa~
Orthotropic layu p 6.7.l. 6.7.J
Arbitrar y layup 6.7.2. 6.7.3
Buc kling
Axially loaded (orthotropic layup 6.9.1
or symn1elric.a l cross sec-lion)
Lateral buckling 6.92
(symmet1ic.al C-ross section}
Local buc-kling 6.9.3
Free. vibra tion {orthotropic layup 6.10
or symmetrical cross se.ction)
CHAPTER SEVEN

Beams with Shear Deformation

Frequently, bean1s undergoing s1nall deformations may be an alyzed by the


BernouU i-Na,~er hypothesis. name ly, by the assun1ptions that planes of the c.ross
section ren1ain plane a nd perpendicular to the axis (Fig. 7.1 , le ft).
In this chapter we treat beams for \Vhicb the Bernoulli- Na.vier hypothesis is
in valid. In the analysis \\'e employ a n x, y, t, coordina te systen1 \Vit h the origin in
the centro id. Furthern1ore, to sin1plify the notation, \Ve use single subscripts y and
t describing shear in the x- y a nd x -t planes.
In the x- z pla ne the deHection of the beam's axis 1.v is related to the ro tation
o( the cross section X:: by

no shear de(om1ation
(7.1)
x-z plane.
For thick solid cross.section beams, sandy.rich beams, and thinY.'alle d beams
the first assun1ption (that planes o( the cross section re n1ain plane} is reasonable.
The second assun1ption n1ay no longer be valid because c ross sections do no t nee
essarily re n1ain perpendicular to the axis (Fig. 7.1 , right). In this case the deflection
of t he beam is
dw \\'ith shear defonnation
-=x- + y."' (7.2)
dx ~ x-z plane,
where Y;:: is the transverse shear strain in the x-z plane. The theory, based on the
assumption t ha t cross sections re n1ain plane but not pe rpendicular to the a~is is
frequently called first-order shear theory. A bean1. in which shear de(onn ation is
taken into account is calle d a 1in1oshenko beam.
Sin1ilarly, in the x- y plane, \Ve have

du \vit h shear de fom1ation


-dx =xv + Y\ (7.3)
x-.v plane,

313
314 BEAMS WITH SHEAR DEFORMATION

'"
'l. D

- 1- --- !I:
ZL. D

. [ x
.4 .4
Figure 7.1: Ocformalion of a beam 1n lhc x-z plane wilhout shear dc(ormation (lcfl) and with
shear deforma tion (right).

The rotations of the cross sectio ns Xz, Xy are cause d by the bending n101nents
J\iy , M._. and the transverse she ar strains Yz, Yy are caused by the transverse shear
forces~. Vy. The rela tionships be t,veen x and 1Wand between y and Y-are pre-
sented io Section 7.1.2 for orthotropic bean1s.
Jn torsion, \vhen shear deforn1atio n of the \\ a ll is neglected, the rate of t\vist
1

on is re la te d to the twist of the cross section about the beam's axis Vt by (Fig. 7 .2,
le ft)

do/I u
- = IJ no shear deforn1ation. (7.4)
dx
'Vhen shear deforn1ation is not negligible, the re is a n additional rate of t'vist
of the cross section t? 5, as shown in Figure 7.2, right. Thus, in the presence of shear
deforn1ation, t he rate of l'...,ist of t he beam is 1

do/I = " " + ,,, ""ith shear deforn1atio n. (7.5)


dx
For open.section beams. 0 8 is the rate of twist due to 'A'arping \Vhen the shear
strain y is zero, and ,,s
is the rate of twist due to the shear deformation when
\varping is zero (Fig. 7 .2). For closed.section bean1s t he interpretation of 11 8 and
tJ 5 is n1ore con1plicated and is not given here.

7.1 Governing Equations


T he response of a beam to the applied forces is described by the strain-
displace1nent. force- strain. and equilibrium equations. TI1ese e quations are given
in this section for orthotropic bean1s, including the effect of restrained 'varping.

1 X. \Vu and C. T. Sun, Simplified Thcor)' for Composite Thin-\Vallcd Beams. A/AA Journal, Vol. 30.
2945-2951, 1992.
7.1 GOVERNING EQUATIONS 315

figure 7.2: The rale o( t""A1st O6 due to \\'arp1ng when the shear strain ts 7.cro ( left) and the rate
of l\\ist as due to she-a r deformation when warping is zero (right).

7.1.1 Strain-Displacement Relationships


The.re are seven independent displacen1ents, of \vhicb the foJJowi ng four are iden
tical to the displacements of be ams 'vithout she ar defom1at ion (Fig. 6.3): the a xial
displace1nent u . the t ransverse d is placen1ents v and w in they and l directions.
respective ly, a nd the l\vist of the cross section '{I. When the shear defom1ation is
taken into account. there a re three additional displace n1ents, namely, t he rotations
of the cross section Xz Xr in the x - .z and x- y pla nes, respective ly. a nd the ra te of
twist due to v.arping, 0 8 , ""he n the shear strain y is zero. \Ve define the follo\\~ng
generalized strain compone nts (he reafter referred to as strain):

0 au ax~ 1 ax.._ r = - a1? 6


'x = ax p.._ =-~ Py = - ax IJx
(7.6}

We note that P::. and Pr are not the radii of curvatures of tbe bean1 s axis; they
are the radii o( curvatures only \\/hen shear deformation is neglected. Equations
(7.2), (7.3), and (7.5) yie ld

y... = -ddxw - x... dv


Yr=-
dx
- x,. l}s = di/I - 11. (7.7}
dx

7.1.2 Force-Strain Relationships


The force- strain relationships a re first presented in detail for an o rtho tropic
1..bea1u \\lith doubly syn1metrical c.ross section subjected to bending n101u ent :\i1 ,
shear force acting through the plane of symmetry Y,, and torque f (Fig. 7.3). The
relationships thus obtained are the n generalized to beams \\lit h arbiuary c.ross
sections.

figure 7.3: l htn\;alled I-beam.


316 BEAMS WITH SHEAR DEFORMATION

Figure 7.4: Displacement u of poin t C in the axial


dirc-e.tion.

A
- x

f.. bea1t1. In the absence of t\\is t, the displacen1ent u in the x direction at a point
located at d istance ifrom the centroid is (Fig. 7.4, x, "' tan x, = - ~)

u= - zx~ . (7.8)
'vhere X:: is the rotation or the cross section in the x-z plane.
The strain-displacement relationship (Eq. 2.2). together with E q. (7.8), gives
the axial strain
du dx-
':r =- = - z:- . (7.9)
dx dx
The bending n1oment about the yaxis is defined as

x:;, = i z<r, dA. (7.10)

For an isotropic material a., = E~x . and \\'e have

- (- z-
M, =

dx,)
. ;\
zE
dx
dA= - E J,
A
dx-
r dA-'
dx
(7.11)
~

'"
Recalling the analyses in Sections 6.3 and 6.4, we replace El by fl for com
posite be.ams and y.rite

i\11 = Eln ( - ~~') orthotropic bean1. (7.12)


.............--
l jp,

\Vhere Elr.v is the replacetnent bending stiffness. Re placen1ent bending s tiffnesses


for beams are given in Tables 6.3-6.8 (page 231) and A.1 - A.4.
The shear s train varies linearly with the s hear fo rce. ThtL", fom1alistically. \Ve
\vrite

orthotropic beam. (7.13)


\vhere Su: is the shear stiffness.
Next, we consider an orthotropic bea1u subjected to a torque f (Eq. 6.240):
(7.14)
7.1 GOVERNING EQUATIONS 317

The torq ue f,. (Saint Ven ant torque, Flg. 6.56, top) is (Eq . 6.240)
SaintYenant
(7.15)
torque.

The torque f .., (restrained- \varping induced torque, Fig 6.56, botto n1) is de
rived Delo y" fo llo\\ing the reasoning used for an lbeam withouts hear deformation
(Section 6.5.5).
The displacement of t he flange vr is (Fig. 6.57)

(7.16)

where t/t is the tv.~ st o f the cross section about the beams axis and dis the distance
between the midplanes of the flanges. The rate of twist is 1' = do/t/d x (Eq. 6.1) .
and we \\rite
dvc d
dx = 2" (7.17)

On the basis of Eq. (7.3), the first derivative of the dLplaceme nt is written as
dvc
dx =(x), + (r),, (7.18)

where (X ), is the rotation of thecroos section of the flange a bout the zaxis (Fig. 7.3),
and ( Y)r is the s hear s train in the flange.
We e xpress the rate of twist in the fo rm2 (Eq. 7.5)

(7.19)

The first term represents the rate of t\vist in the absence o f shear deforn1ation.
and the second te m 1 is the rate o f tY.ist due to she ar deformation. Equations
(7.17)- (7.19) give
d
( Y)r = _.,s_ (7.20)
2
Recalling E q. (7.12), we write the be nd ing mome nt 1flr for a n o rtho tropic
flange in the presence of shear deformation as

,\1, = El, ( - d (x ), ) = - ifl,~ dtJ" , (7.2 1)


dx 2 dx
where the second e qua lity is written by virtue of Eq . (7.20), and f!J, is the be nding
stiffness of the flange about the zwaxis..
Fo r a n I-beam the bimoment is (Eq. 6.232)

;\1"* = M,d. (7.22)

2 X. \Vu and C. T. Sun. Silnplificd Theory ror Compo!>iteThin-Wallcd Bcanls. A /AA Jounull. Vol. 30,
2945- 295'1 .1992.
318 BEAMS WITH SHEAR DEFORMATION

The last l\\'O equations give

,\1,., = ( ~ El,) (- d:..") , (7.23)


-------E...~ ----.,.--
f

\Vhere the term in the first parentheses El.,.(= 4}- ifir) is the \\'arping stiffness.
which for an isotropic beam is given by Eq. (6.238) and fo r an orthotropic I beam
by Eq. (6.244).
The shear force in the tlange is related to Mr by

-Vi=--.
dM, (7.24)
dx
By referring to Eq. (7.13), we write V, as
v, = (S,,,)r(v), , (7.25)

\Vhere (~y ) 1 is the shear stiffness of the Range in the x- y plane. The v.arping
induced torque is (Eq. 6.235)

f. = Vcd. (7.26)

Equations (7.22), (7.24), and (7.26) result in

f _ dM. _ (7.27)
.., _ dx

From Eqs. (7.26). (7.25), and (7.20) we obtain

restrai ned-v.arp i ng~


(7.28)
induced torque.

\Vbere s_, is the rotational shear Stiffness defined as


.... -. dl
S~, = (S,,)1 . (7.29)
2
Equations (7.12), (7.13), (7.15), (7.23), and (7.28) are the force,,.train relation
ships for an orthotropic lbeam \Vith doubly symn1etrical c ross section subjected
to a bending moment MY a she ar force acting in the plane of syn1metry ~. and a
torque T(= f.,. + f. ).
Arbitrary crosssectio11 bea1t1s. We nov. consider orthotropic bean1s of arbi
trary cross sections \\ ith internal forces Fi. My. M:.. ~"~.and Mw. f.
1

The re lationship between the axial force (acting at the centroid) and the axial
strain is (Eqs. 6.7 and 6.8}

(7.30)
7.1 GOVERNING EQUATIONS 319

111e be nding n1o n1ents and birnome nt for a (symme trical) 1..bean1 are (see
Eqs. 7.1 2 a nd 7.23)

~}
0
0
El.~ ]{ -
-~
- ~::I
d.x

'
(7.3 1)

where the equa tion fo r M;: is ana logous to the equation for .ft,.. For an arbitrary
cross-section orthotropic bea1n. the 12 and 21 ele n1ents are not zero. He.nee. \Ve
write

O J{ -'~;
0 _ <tu } .
dx
(7.32)
Et"' _ dtJ ll
dx

By utilizing the definitions given in Eq. (7.6), t hese e quations may be \\'Titten
as

o]{J.}
0
~~
;;; . (7.33)
Elw r

The re la tionship between f.,. a nd the rate of twist~ is (Eq. 7.15)


(7.34)

The shear forces a nd t he restra ined- \varping..induced torque fo r a (doubly


symmetrical) I-be am is (see Eqs. 7.13 and 7.28)

{ fv,~. } -- [s,.,o0 ~
s,.
o s_
o]{y'}
0 y,
~
' (7.35)

where the first equality is analogous to the second one. Shear is introduced
by f",.. and by ~I' :lOd ";: 111Cf CfQf C, the fOfCe-Slf 3 i0 fC)atiOflShipS 3fC COUp)ed.
\ \1
e no\\' e xtend the preceding equations to include these couplings and (for arbi
trary cross ..section bean1s) write the fo rc.e - strain re lationships as

v, }- [s_,,.
17. - !,.. (7.36)
{r:. s;.:
320 BEAMS WITH SHEAR DEFORMATION

where(~;) is the shear stiffness matrix of the beam. The inverse of Eq. (7.36) is

Yy } {Vy }
{~~ = ['1;1 ~. '
(7.37)

\vhere (S;;J is the shear compliance matrix

(7.38)

Equations (7.30), (7.33), (7.34), and (7.36) are the force-strain relationships
for a beam " 'ith shear deforn1ation. The 'vay in \vhich the eleme nts of the stiffness
and co111pliance matrices a re detern1ined is discussed in Section 7.2.

7.1.3 Equilibrium Equations


In the presence of shear deforma tion the equilibrium equations for a stra ight beam
subjected to the loads sho,vn in Figure 6. 1 a re identical to those of a beam \vithout
shear deformation (see Eq. 6.3):

aiV a(fw + f ..)


-=-Px ~-a-x-~=- t
ax
al?,
-ax -- - py
av.
_a_x, = - p, (7.39)

a,\1,. = 17. aM, _ ~


ax ' ax - JI

\Vhen the bean1 is axially co nstrained, \Ve have the additional equation
(Eq. 7.27) as follows:

nM., = fw. (7.40)


dx

7.1.4 Summary of Equations


ln sun1n1ary, \Ve have 23 unkno\vns: 7 generalized displaceme nts (u, v. uJ, ~. x.-.
x,, 0 6 ), 8 generalized strain components(<;, 1/p,, 1/ p,, r , y_,, y,, !JS, D), and
8 generalized interna l forces (1V, J\1.-, Afy, JW,.., V,,, ~. f ..., f)-v).
TI1e equations that yield the solution a re as folJo,vs:
Nun1be r of equations Equat ion number
stra in-displaceme nt 8 7.6. 7.7
fo rce- strain 8 7.30, 7.33, 7.34, 7.36
equilibrium 7 7.39. 7.40
7.2 STIFFNESSES AND COMPLIANCES OF BEAMS 321

Table 7.1. Boundary conditions for beams with shear stiffnesses


x- zplane x- yptane
Buillin w=O x~ =O v =O Xv = 0
Simply supporled .,, = 0 i\:t1 =0 v =O ff, = 0
Fre.e- ti, = 0 Ki, = o ti, = o !ii:, =0
Axia l1)' restrained tt =0
unrestrain ed iV=O
\Varping restrained i?" = 0
unrestrained 1\/.., =0
Rotationally resLra ine d \b = 0
\Jnrestrained T=O

In genera] the strain- displace1nent. force-strain. and equilibrium equations


n1ust be solved simultaneously to detern1ine the forces in t he beam. The analysis
becomes sitn pler 'vhen t he bean1 is ortho tropic. \Vhen t he b-ean1 is o rtho tropic
the internal forces can be detern1ined by s ubstituting the replace n1ent s tiffnesses
into the corresponding isotropic bean1 expressions. The replacement stiffnesses
are discussed in Section 7.2.

7.1.5 Boundary Conditions


A t a buil t~in e nd t he d is placen1ent and the rotations of the c ross section are 1.ero.
At a simply s uppo rted end t he displacement a nd the n101nent are zero. At a free
end t he n1ome nts a nd the shear force are zero.
When the e nd of the beam is restrained so that it canno t move axia lly, the
axial displacement is zero. \Vhe n the end is not restrained axially, the axial force is
zero. When 'varping of the c ross section is restra ined, the distortion is 1.ero. When
warping is not restrained, the bimoment is zero.
When the e nd may rotate. the to rq ue is zero. \Vhe n the end is rotationally
restrained, the rate of twist is zero.
The preceding boundary cond itions a re s u111ma rized in Table 7.1 .

7.2 Stiffnesses and Compliances of Beams


The stiffnesses that characterize orthotropic bea111s \vith shear deformation are
sho\vn in Table 7 .2. For lhin..walled beams these stiffnesses are give n in Sec
tions 6.2-6.6 except for the shear stiffnesses derived below in Sections 7.2.1 a nd
7.2.2. The stiffnesses of sandwich beams are presented in Section 7.2.3.
The s hear stiffness of t hick solid.section bean1s a re not discussed here; they
are give n by Whitney:'

3 J. r-.1. \Vhitncy, Str1tc111.ral Analy.~is of IA1ni11111t!d Ar1isn1roph P/111es. Tcchnon1ic. Lancaster.


Pc nnsyl\ania . 1987.p. 270.
322 BEAMS WITH SHEAR DEFORMATION

Table 7.2. Stiffnesses of ortholropic beams wilh shear deformation


Tensile stiffncss EA Set.Lions 6.2-6.4
Be nding. sliffncsses El~. El,,. El,, Sectio ns 6.2-6.4
Torsional stiffness GI, Set-Lions 6.5. J-6.6.2
\V;1rping s tiffn ess El.. Sectio n 6.6.4
Shear stiffnesses s,., .s.t~ s. . ..s,.ts. . . .s~ Seel.ions 7.2.1-7 .2.3

7.2.1 Shear Stiffnesses and Compliances of Thin-Walled Open-Section Beams


To determine the shear stiffness and shear compliance n1atrices of t hin ..,valled
open-section beams we consider a n eleme nt of length 6. L of a thin\\alled open
section bean1 (Fig. 7.5). O n the t\vo faces of the ele n1ent there a re equal and
opposite shear forces ~.... equal a nd opposite shear forces V;_, equal and opposite
torques f(= ~v + f ,..), and a..xial stresses a"', \Vhich are d iffe rent at the two faces.
We oeglect the Saint\'enant torque: hence, f = T..,.
The shear stresses are represented by the shear flow q,

(7.41)

'vhere q+,
~
a nd q=~
are
~
shear flows
~
(per unit
~
(orce) and a re calculated by setting
either V,. = 1 and I', = 0 or v, = 0 and I', = 1 in Eq. (6.281) and q;, is t he shear
Ho\\' (per unit to rque) introduced by a unit torque~; q;, is evaluated by replacing
/r by l/af1 (see page 264) in the expression of q:
gi\'en4 fo r the correspond ing
isotropic beams.
In the folJo\ving \Ve take into account only shear deformation ca.used by shear.
Thus, the strain energy is (Eq. 2.200)

(7.42)

'vhere ~ is t he coord inate pe.r pendicula r to the v.alJ, '1 is the coordinate aJong
the circumference of the cross section. and ~ = x. \Ve assume that the shear flow
q( = ,( r.,dc) does no t \'ary along the I:!. L long ele ment and the shear strain Y< is
unifo rn1 across the thickness of the \Vall. Thus. t he strain e ne rgy is

U= J
~ (r;:,q) d~ 1:!.L. (7.43)

'vhere Yt~, is the shear strain in the walr s reference surface and is related to the
shear Ho'v in the!composite v.aU by Ve~ = et:'.t-iq (see Eq. 6.195), where a~ is given by
~6 = a!6 - (~) (see Eq. 6.196). 111e superscript (! indicates that the con1pliances

"' T. H. G. ~1 ci;son, Aircraft Su1uru"s[iJr En,:inrius S1111Je.rus. 3rd edition. Halsled PrcSSi John \Viky
& Sons.. New York. 1999, pp. 465-472.
7.2 STIFFNESSES AND COMPLIANCES DF BEAMS 323

are evaluated in thee reference plane defined in Figure 6.26. When this expression
is used. Eq. (7.43) becomes

(7.44)

The \\!Ork done by the external forces due to shear defonnation is

1~
2V,.y,.t.L +
lllK' lo dl~laci:mtnl d~ l(I ili~plll((f!IC'l'll chle ii the 01rut
IP lbt y dnbutl 111 1hl' : di~ l"A ""hu:ed
dHplk'C'.metll

(7.45)

We neglect the 'vork done by the axial stresses on the shcnr- forceinduced
warping.
111e law of conservation o( energy gives

U=W. (7.46)

By introducing Eq~ (7.44). (7.45). (7.37). a nd (7.41) into Eq. (7.46), and by
performing the aJgebra. \Ve obtain

yl! !. a (q)'d11 + v21J.


11 (S) 66' 1
a (q) 1tl11 + f' ll !.(S) a 66
: l ($) 6tl :
(q)
...
1d11

+ v.V, f 1., a;.q;q;d11 + v, 'F. ft., a;,.q; q;.d11 + ii, f.ft., a;.q;q;.dq.
(7.47)

where S indicates integration 3round the entire circumference. This equation is


324 BEAMS WITH SHEAR DEFORMATION

v,= tj q,, =,. (3 '


w~,
f ..:=- 1

~3
q, q,,,
Figure 7.6: I-beam and the shc.ar Oo\\'S from unit shear forces in the z a nd y direclions a nd from
a unit torque.

valid for arbitra ry values o( ~v. ~. and f.,,. H e.n ee. the eletne nts of the shear
con1pliance n1at rix are

~
Syz =
[ ( S)
' .. ,
a66q:;q,.(, ''

(7.48)

The s tiffness is gi\'eo by E q. (7.38).


I-beam. We consider an I-beam symmetrical about the taxis (Fig. 7.6). The
layup of the web is symmetrical, a lthough the layups of the top a nd bottom flanges
may be unsymme trie-al. Unit fo rces Vzand ~,. and a unit torque f.., are applied at
the shear center.
The shear Ho\\is, resulting fron1 unit shear forces ~ = 1 a nd ~ = 1 acting at
the she ar center and from a unit to rque f= THI = 1. are sho \\n in Figure 7.6.
De tails of the c.alcula tions are not presente d here. We n1erely note tha t the shear
flo,v alo ng the length o f the \Veb due to Y:: is take n to be consta nt. \Ve now define
Yi, ri. 81:. and 8i;c as follo \\s:

I (a11)n d d - t.c
Yt = I + - &i:= - -
3 (a11). b11 {1 +kl 1.c
I (a11)12 d d- e
Yl = 1 +------- 8)';;;= - - (7.49)
3 (et11) . bll(J + ~,)
\Vhere e is the loca tion of the shear center, Zi: is the location of the cent roid, d is
the d istance bet\\ee.n t he midplanes o f the flange~ and bt1 and bri a re t he \vidt hs
of the flanges (Fig. 7.6.)
To ca lculates,, we a pply Vy and pe rfo rm the integration indicated in Eq. (7.48)
(or the we b and for the top and bottom fla nges. \\'he n o nly ~v acts., the re is no
s hear flow in the we b (Fig. 7.6), and hence the integration needs to be perfomied
7.2 STIFFNESSES ANO COMPLIANCES OF BEAMS 325

only for the two flanges as foUo\vs:

(7.50)

By performing the integrations. we obta in

,,. _ J? [ (a;.)n + (or;:.),, ] , (7.51}


>yy - - bn ( l +.S~)' bn(1+t)'

where the subscripts fl and f2 refer to the top and bottom Hanges. The other
elements of the shear compliance matrix are calculated similarly fro m Eq. (7.48)
with the shear flows sbo'vn in Figure 7.6. T he results are

_-1 ,(')', _
s,,. - . or.,; q, "'1 -
(ai:.)w
d
..!_ (ai:.)n bt1 ..!_ (or;:.) 12 b12
+ 12 " 2 + 12 d2 2 (7.52}
~ -~ . ~

s,,.,.lt = J(S)
or'66 (q'
w
}'d' I = ~
d' ((a;:.)n
-
f.. _
r.11
+ (or;:..)12
b )
fl
(7.53)

(7.54)

(7.55)

The stiffness is given by Eq. (7.38).


Arbitrary crosssection bea111. The shear compliances of selected thin\Valled
co mposite bean1s5 are presented in Tables A.8 and A.9. The shear stiffness matrix
is determine d by Eq. (7.38).

7.2.2 Shear Stiffnesses and Compliances of Thin-Walled Closed-Section Beams


\\'hen there is no restrained ,...,arping ('ft!(> = 0, and consequently ,,s = 0. see
Eq. 7.28), the shear-force-s tTain re lationships (Eq. 7.36} become

lv,i?rl= [~' S,_ ~']ly"l


S" y, (7.56)

~ L. P. Kolltl.r, FkxurnlTorsioruil Vibration of Open Section Cetmpositc. BcanlS "''ilb Shear Oc fom1a 4

tion. ln.rerrid1ion(ll Jo11rnttl of SfJlidsdnd S1ruc11.ue:1. Vol 38, 7543-7558, 2<X>l.


326 BEAMS WITH SHEAR DEFORMATION

The shear compliance matrix is (see Eq. 7.38)

(7.57)

The e lements of the shear stiffness n1atrix of orthotropic thin.walled closed..


section be.ams are obtained by reasoning similar to that used for open.section
beams. Thus, from Eq. (7.48) we have

- f ..
Syz = a 6nqzqy"Ifr/,
(7.58)

'vhere q; and q: a re shear fto\\.'S (per unit force) calcula te d by setting either Vv = 1
--- :.t
v,. --
and V;: = 0 or = 0 and V;:: = 1. These shear fto,vs are calculated according to the

analysis of thin .. \valled closedsection bean1s (Section 6.7). Shear compliances of


rectangular a nd c ircularcross ..section thin\\alled beams are given in Table t\.7.
The shear stiffness are given by Eq. (7.38).

7.2.3 Stiffnesses of Sandwich Beams


Like sandwich plates (page 169). sand\\ich beams consist of a foam or honeycomb
core covered by facesheets (Fig. 7.7). T he core acts much like the web in an
lbeam; it separates the \\VO [acesheets, resulting in high bending stiffness.
In this section we consider orthotropic sand,vich beams consisting of an ortho ..
tropic core and orthotropic fuces heets (Fig. 7.7).
To determine the shear stiffness in the x-z: plane. \Ve apply a shear force Y.:.
For an orthotropic sand\vich pla te the re lationships betv.een the shear forces (per
unit length) and the shear strains are (Eqs. 5.15, 5.34)

1 11, I = (s"0 S12o


~y ]lr"l
YJ.:
sand\vich
plate.
(7.59)

The total shear force~ corresponds to bV.r in the plate (\vhere b is the \Vidth),
and v,. = 0 (only V, acts). Thus, we can apply the laminate plate theory expressions

v,
- y

b
Figure 7.7: Sand\\.i,;;h beam.
7.2 STIFFNESSES AND COMPLIANCES OF BEAMS 327

to sand\llich bca111s by making the follov.ring substituUoru:

\/'. -- ii,
b v, = 0, (7.60)

''"here bis the v.idth of the beam. The shear strain in the plate Yt: corre.sponds to
the trnnsverse shear >train Y: in the beam (Fig. 7.1). Thus. Eqs. (7.59) ond (7.60)
yield
sondwich
(7.61)
beam.
The tenn in parc.nthes-es is the shear compliance of a sund,vich beam in the
x- z plone (sec Eq. 7.37)
- I
"" = s,.b. (7.62)

To determine the be nd ing s tiffness in the x- z plane. v.e agnin refer to a


sandwich plate for which Eqs. (5.ll) ond (5.12) yield

{y,,.*"~ }= [a] {N'N }+[/II {M


M,'}
1
N1 y Afxy
(7.63)

I =*_ !tu_ !6=


.Jr ' '
I =[fl)' { Z:}+ ~!: } .
N_.1
(61 {
M,.,
(7.64)

where A1~. ,\f,. ,\f,1 , /\~. 1\11 1'11,;_v are bending moments and in-p1nne forces per
unit lengU1 and 1~ 1- lf!J, and [61 a re evaluated in the x. y. t coordinate syste m.
We now apply only n bending moment M,. to the sandwich beam (Fig. 7.7). The
relationship be t\vccn the be nding mome~t in the pla te 1W\ and the bcarn iWy is
(Eq. 6.20)
R
M, = -t (7.65)

Since only ,fl, acts. "'e have

.J\I,. = A'I,, =N, =1V = N.c =0. 1 1


(7.66)

Equations (7.6.t). (7.65). and (7.66) yield (x,. = x.>

ax =(!!.!.)
_ 3 X
b
Al,. (7.67)

We oomparc Eq. (7.67) with Eq. (7.32) and note lh3l an orthotropic sandwich
beam is sy1nmetrical \Vilh respect to the z~axis and he nce fly: is 1.cro. Thus. the
bending stiffness in the x- t plane is

~ =;-
El,, "
II
(7.68)
328 BEAMS WITH SHEAR DEFORMATION

b
"'
:- ----
Figure 7.S: Sandvoich beam and lhc cqui\'alc nt I-be.am.

The presence of the core has little ef(ect (i) o n the bending stiffnesses in
the x- y plane, (ii) on the te-nsile stiffness. (iii) on t he wa rping stiffness. and
(iv) on every elen1ent but the S::~ element o( the shear compliance n1atrix. \Ve
utilize this o bservation and detem1ine these properties by neglecting the
core.
\Vhen the core is neglected. t he sandwich bea1n be haves like an lbea1n v.ith a
thin web (Fig. 7.8). We no w use the expressions for a n I-beam in Tables A.2. A.5,
and A.8 and substitute b fo r bo th b11 and ha, the superscripts t and b fo r subscripts
fl and f2, a nd set the terms referring to the v.eb to zero. The resulting bending,
tensiJe, and warping stiffnesses a re

- lb' t i>' - 1 l
El._= ;l" 12 + aIJ 12 EA = - -, b+ b"b
ll au au
(7.69)

\Vhere a :1 and a~1 are evaluated in the coord inate syste ms \Vhose origins are at
the " ne utral" reference planes (which is a l e, Eq. 6. 105) in t he lop a nd bottom
flanges, respectively. When the core thickness is so lo\\' as to prevent d istortion of
the c.ross section, the \\'arpingstiffness is lo\ver than the value give n in Eq. (7.69).
t\ conservative estimate of the \Varping s tiffness is its )o\ver limit. which is ifiw = 0.
The locations of the centroid le and the shear center e are (Fig. 7.9)

le= -
1 (~bd) (7.70)
CA ""
"Neutral'' plane
I

\ "Neutral" plane
Figure 7.9: Location of the s hc.ar center a nd the. centroid of a sa ndwich beam.
7.3 TRANSVERSELY LOADED BEAMS 329

The shear con1plian ces are


0
_ 1.2 ( (a;;.)' (a;:.) )
s,,. =b (1+8") + 2
(t +.J;)' (7.71)

(7.72)

7
}w -
_ 1.2 ( - (c.;:,,)'
db l + J~ + 1 + ..L
(a;:,/) (7.73)

s;:,N = o.
"' (7.74)
where a(;; is given by Eq. (6.196) and J" = (d - e) / d. The e lements of the shear
stiffness ma trix are determined by substituting the eleme nts of the shear compli
ance matrix into Eq. (7.38).
Next \Ve prese.n t uppe r and Jo,ver bounds of the torsionalstiffness of a sandwich
bean1. The uppe r bound is t he torsiona l stiffness o f a solid (not sandwich) beant
with the same cross section as t he sandwich be am (Eq. 6.169)
- 4b
GI, = - J,, uppe r bound. (7.75)

The lower bound o f the torsional stiffness correspo nds to the torsiona l stiffness
o( a sand,vich bean1 in \\ hich the shear stiffness of the co re in the y-z plane is
1

neglected. Jn this case 've need to consider only the t\vo facesheets. The to rsional
stiffnesses of the faces heets a re (Eq. 6.1 69) 4b/~;. a nd 4b/8::,,, whe re superscripts t
and b refer to the top and bo ttom facesheets. The to rsionaJstiffness of the sandwich
beam 'vith t\\'O facesheets is
- 4b 4b
GI, =,,-+ lower bound. (7.76)
066 SM

7.3 Transversely loaded Beams


In t his section \Ve present the deflections o f orthotropic bean1s, \Vi t h cross sections
synunetrical 'vit h respect to tbex- .z plane. 1-\ t ra nsverse load p~ (per unit length)
act" in the plane of symn1etry (Fig. 6.1). The relevant equilibriu1u e quations are
(Eq. 7.39)
dii.
- + p. =0 (7.77)
dx
di't, - (7.78)
dx - 11, = 0.

where \f~ is the shear force, a nd !f.jy is the bending OlOlllCDl in the X- l pla ne.
. .,
For be.an1s \\~th synuuetrical c.ross sections. ~v~ = 0, S.v~ = 0, and s~ = 0, a nd
330 BEAMS WITH SHEAR DEFORMATION

Eqs. (7.32) and (7.36) give


-
A1v = - Efvl'~
- dx. (7.79)
dx
For simplicity, \\'e introduce the following no tations:
fJ, . ;:;; v. v
fl,., . fl r,; . r (7.80)
x~ x y, .. y
p, = p.
With these notat ions \Ve have
- - El-
M= -dx - -
V=Sy. (7.81)
dx
Equations (7.78) and (7.81) }fold
Eid'x
Y= - S dx' . (7.&2)
By combining Eqs. (7.77), (7.78). (7.79) and by using Eq. (7.2), we obtain
d x 3
- fl- + p=O (7.83)
dx3
2
....,d
"'' dx'
x + "(dw
, dx )
- X = 0. (7.84)

Equations (7.83) a nd (7.84), togethe r "1th the boundary conditions listed in


Table 7.1 (page 321), provide the deHection w and the rota tion of the cross sec-
tion X

Kno"o bending moment distribution. \Vhe n the bending n1oment distribution


along the bean1 is kno\vn, the deflection n1ay be obtained by the followi ng proce
dure. 111e deHection is d ue to both bending a nd she ar deforma tion
(7.85)
\Ve recall tha t the deflection is related to the ro tation of the c ross section and
to the shear strain by (Eq. 7.2)
dw
Tx = x + y. (7.86)
\Ve combine the tv.o preceding equations and \vrite
dw dwn JwS
- = - + -=x+r. (7.&7)
dx dx dx
Thus, \\'e have
dw 8
-=x (7.88)
dx
Substitution of the firs t of these expressions into Eq. (7.79) gives
diu..o M
dx' = - El. (7.89)
7.3 TRANSVfRSUY LOAOEO BEAMS 331

From the second expression in Eq. (7.88) we obtain

.; = f ytf.t. (7.90)

By replacing y with V/S(Eq. 7.79) and by perfonning the integration. we have

.; = f .iis,., . = c. + ~Ms (7.91)

where C, is an in1cg.rationconstant. Thedeftections w8 and w5 arc nowdcrermined


by the following steps:

1. For the known bending moment M, the deHection w8 is de1e rinined for a
bean1 wilhout s hear deforn1ation by using the expression ro r the Qorrcsponding
isotropic bcnn1 nnd by replacing the isotropic be nding s1iCfncss El by fi.
2. 111e dencction .,,s is calculated by the expression

,
ut =Cl. +._..
M (7.92)
s
The constant C" is determined from the boundary condition al the support
\Vhe re the dc Rcction w L" zero (w5 = w 6 = 0). Thus. Cu is

(7.93)

3. The tollll dcRcction is

(7.9~)

Maxi1nu1n dcnections of selected be.a ms are given in l 'ablc 7.3.


Sa11dtticli beauu: wi1/1 arbilrary layup. The preceding :inalysis (or orthotropic
bea1ns can be cxle nde d to transversely loaded nonorthotropic sand\\ich be ams
provided lhnt the bcnnl is (ree to rotate about its axis. An important practical
exaLnple is a bcn1n built~ in at one end :ind free at the otJ1e r e nd (cnntilcvcr beam).
11>e internal l'o rccs arising from the applie d loads arc M, nnd ii, (Fig. 7.10).
For sand\vich plates the relationships bet\veen the shear forces per unit le ngth and
the shear s1roins arc (Eq. 5.15)

l v J =[~" S11~'] l Y.r:y"}


sand\\rich
(7.95)
V, S 12 plate.

Equatioa (7.95) with v, = 0 yields


.,"..1 -_(s.. s,, - S:,) Y~ (7.96)
Su
The shear forces in a sandwich beam are ( Eq. 7.60)

V, = 17.b v,.= o. (7.97)

\vhere bis the \Vidth of the beanl.


332 BEAMS WITH SHEAR DEFORMATION

Table 7.3. Bending momenls and maximum deflections ol l>eams wilh


bMding stillness fl and shear stilfness 5
Maxiroom deflection
Geometry, loading1 bending moment W= w +w'
t t t t t ''
A L . -n.
IJl
.'i rl~
= ~i!i
1-> pl~ -s = cl:
/PS'' f =- 8
w ilf

jP

~ jP 'E
Al HI -= !!.!;,.
k{l~M'-'=-8PL 8

w .t .e:!
- a -- $ t'I
-s = e!i.
IJJ it

The shear strain in the plate Yxz corresponds to the uansverse shear suain Y;:_
v;
in the beam. and = \l'., b. Thus. \Ve have (or a bean1

sand,vich
(7.98)
beam.
7.3 TRANSVERSELY LOADED BEAMS 333

J-""1gure 7.IU: Sandv.ich hcam subjecte d to bending. and she-a r.

The term in parentheses is the shear stiffness (Eq. 7.81):


-S= s,,s,, - 'Si' b. 1
(7.99)
12

whe re Su. Sn and S12 are given by Eq. (5.32).


To determine the bending stiffness in the x-z plane we again start the analysis
with sand\\foh plates. From Eqs. (5.11) a nd (5.12), for a sandwich plate we have

{ =t }=(~J ~ } +l.8){~: }
_ !!.u_ _ <tx r:
ily .l.t
{
M xv

1V:i}'
(7.100)

{
f,<" } = IP) { M,
My } + [a ] { N,
N, } , (7.101)
Yxy M.Ty Nxy

where :\(,, J\1y. J\(l}" Nx. N_\.. 1\J.,y are mon1ents and fore.es per uoit length, and[~).
(,8 J and IS] a re evalua te d in the x, y, l coordina te system. We now a pply a be nding
n1on1ent M,. to the sandwich bean1. The relationship be t\veen t he be nding n10111ent
in the sandwich pla te M, and the sandwich beam 1Wy is (Eq. 6.20)
M _ 1Wy.
x - b (7.102)

Since only ,ffy acts., \\'e have


Afy = Mx.v = N,1; = 1\~v = N.ry = 0. (7.103)
From E qs. (7.100), (7.102), a nd (7.103), ax,/8x = (8x.,./ax) is
_ax . - (~) M,..
ax - b (7.104)

By comparing Eq. (7.1 04) with Eq. (7.81), we see that the term in parentheses
is the inve rse of the replace ment be nd ing stiffness:
- b
/=- (7.105)
S11
The deflections in the x-z plane a re calculated by the three steps describe d on
page 331 with Sgiven by Eq. (7.99) and EI by Eq. (7.105).
We note t hat a nonorthotropic sandwich beam unde r the action of a be nding
n1on1ent in the x- z pla ne may also bend in the x- y plane and n1ay a lso t\vist.
334 BEAllS WITH SHEAR DEFORMATION

p = JOOOON/m
2mm
~tttt t ttt!

l -= 500 nun
IJ =- IOO mm
f c= 20mn1
2 nun

1-igurc 7.1 1: The sand,\ich beam in Example 7.1

7.1 Exantple. An L = 0.5-111/oug and b = 0.01 nrwide rectangular sandu1ich bellnl


is 111ade of a 0.0211Jthick core covered on botlr sides by graphite epoxy facesheets
(Fig. 7.11). Tire materilll properties are give11 in Table 3.6 (page 81). Tire lay11p
of each faceslieer is [45\/0 12/45~ ]. The rhick11ess of each faces/teer is 0.002 m.
- -
The bea111, buiftin al both ends, is subjected to a unifor11lly distributed load p =
10 000 N/111. Ca/c11/a1e tire 111axi1111un deflection of the bea111. The core is isotropic
(Ee= 2 x HJ" kN!m2 v, = 0.3).
Solution. The bending compliaoce matrix is (see Eq. 5.53)

~
7.96 0 ] _, [ 21.39 - 14.55
1
8~6 - 1~55
3
ll.71 10- = 95.31 ] 10- --
0 () 11 4.21 N.
m

(7.106)
For symme trical layup (~I= (d[. aod the bending stiffness is (Eq. 7.68)
- b
El,, = 8,;' = 2 1.390.1x 10_6 =4 675 N m-., (7.107)

From Eq. (7.62). (with S11 = 18.615 x JO'~, Eq. 5.54) we have

'" = s,',b= 531.11x 10-~ c1.1os)

The beam is orthotropic, and S:,. = s,. = 0 (Eq. 7.74). Thus, from Eqs. (7.38)
and (7.108) we have

! ,, =) = I 862 x 10' N. (7.109)


S;;;

From Table 7.3, third row (page 332), wit h Et= El,, and S = S", the max-
in1um deflection is
_ I pl' pl2 .
w= / + S! = ~~ + 0.000 l68 = 0.000 516 m = 0.52 mm.
384
wilh<ul rltl.'C:ir
dcf<1nm1li~1n

7.4 Buckling of Beams


In this section \Ve present the buckJing loads o( orthotropic beams subjected to
eithe.r axial o r transverse loads.
7.4 BUCKLING OF BEAMS 335

Figure 7. LZ: B uckling in the x-z plane. ,l:-..;.=====,,.,'-1- FJ~ 6 E--

7.4.t Axially Loaded Beams with Doubly Symmetrical Cross Sections


(Flexural and Torsional Buckling)
The bea1u is orthotropic and its cross section is syn11netrical with respect to both
they and the t axes (El" = 0). A compressive axial load JV,., is applied at the
centroid of the be am (Fig. 7. l 2). We wish to dete rmine the axial load a t which the
beam buckles.
When the cross section of the beam has two a..xes o( symn1etry (doubly sym
n1etrical cross section), it n1ay buckle (a) in one of the planes of symn1etry, (b) in
the other plane of symmetry. (c) torsionally (Fig. 6.84). Accordingly, there are
three sets of buckling loads

(7.110)

where IVcry and Ra:. correspond to buckling in the x- t and x- y plan es a nd Ren;
to torsional buckling (in \\ hic.h case the axis of the bean1 rotates but d oes not
1

bend).
Buckling in the .r-zplane. We consider a bea1n of le ngth L that buckles in the
x- t plane (Fig 7.12). The equilibrium equations a re identical to those of beams
without shear deformation and a re 6

av, _ fl a'w _ 0 (7.lll)


dx .TO dxl -

d1il, -
- V. =0. (7.JJ2)
dx '

where uJ is the deflection of the be.an1 's axis in the x- z: synun etry plane, is the V.
shear fo rce. a nd M,. is the bending moment. By subs tituting Eq. (7.79) into (7.1I l )
and (7.112). and by using E q. (7.2), we obtain the following equilibrium e quations
for a composite bean1:

- d' x, - tf'w
3 - NJ;o d 2 =0
- Ely,.- d (7.JJ3)
x x

- - n'x,
- El,.,--,
dx-
- -
S.. ("'"
- - X) = 0.
" dx '
(7.JJ 4)

6 S. P. Tunoshenk:etand J. Gere, Tlieoryof ln.,11cS1nbili1y. 2nd edition. ~tcGraw-H iJI . New York, 1961.
p.3.
336 BEAMS WITH SHEAR DEFORMATION

\ Vith the notation 1!1,..v El. ~.... !, X.: x,these equations n1ay be rearran ..
ged to yie ld
3 3
-d
- E x - N,
r"-- - 0 (dx
- Eld -
-= x) =0 (7.115)
dx' dx 3
S dx
dw =x --::z::--
-
Eld'x (7.116)
dx S dx'
Solution of the fi rst of these e qua tions is7
+ c2 cosLx
. Lx
x = C , sm ,, + c,. (7 .117)

We substitute this expression into Eq. (7. I 16} and integrate the e quation. The
result is

w= ( - C1cos z x + Cism
. 1:x
) ; L(l + !v
7 ') + c,x + C.. . (7.118)

\vhere C1- C4 are consta nts de te m1ined from the bo unda ry conditions a nd is a
parame ter defined as

(7.119)

Eq ua tions (7.117)- (7.119) togethe r with t he bo unda ry conditions are used


to determine t he buckling load. ln the (olJo,ving \Ve derive the buc.kling loads of
bea1us builtin at bo th ends and o f bea111s built~in at o ne end and sin1ply supported
at the other end.
Beams with both end< built in. When bo th ends of the beam a re built-in, the
displacen1ent and the ro ta tion of the c ross section a t each end are zero (Table 7.1,
page 121) as follows:
w( L) = 0 x(L} = 0 w(O) = 0 x(O) = 0. (7.1 20)
By substituting w and x (give n by Eqs. 7.117 a nd 7.1 18) into these boundary
co nditions. '"'e obtain

[
-~cos Cl> sin .
sin cos
(7.121)
- <!> 0
0
\Vhere <l> is define.d as

<!> =- L( + SL27
.
I -:::- ') . (7.122)

\ Vhen the bean1 buckles., the de Hection is nonze ro and, he nce, a t least o ne of
the consta nts C 1-C4 mus t be nonzero. Thus, al buckling, Eq. (7.121) is satisfied

7
E. Krcys:dg,. Adl'Onced Eri>1 in~Yi11g ,\filtheutaric-s. 71h e.dition. John \\'ilcy & Sons., New Y<irk. 1993.
pp. 136-1.!8.
7.4 BUCKLINGOF BEAMS 337

only v. he n Lht dctc m'linant o f t he matrix is zero. which is a require me nt that is


1

met \Vhen
2<1>(1 - COSJ) - Lsin11=0. (7.123)

This equation is sa1isfied \\-ilen is

= , = 2irr. (7.124)

wberei is an integer (i = 1, 2. 3.... }. Wilh these values of,. Eq. (7.119) gives
the buckling loads (N.-0 = N0 , , \

-
1
= ~ + -.
(f )1 I
(7.125)
Nc.ni n 2 E1 S
We arc interested in the )O\Vest value or the buckling load. For the builtin
beam under consideration the lowest value of fJi:,yi is obtained v.hcn J = 1. This
value is de noted by lJ0 y
l I I
--=~ +- (7.126)
N",, dJ. S
ct>'
The firsl term in this expression is the in\'etse of the buckling lo.1d or a beam
"ilhout shear deformation (Eq. 6.337. Table 6.11. page 293):

(7.127)

We now con write Eq. (7. 126) as

(7.128)

111is cquntion s hO\l/S that the buckling load decre ases with decre asing shear
stiffn ess and. correspondingly. with inc.reasing s hear derorma Lion.

Bea.ms with one end built-in the other end simply supporh.i d. Al the buih -in e nd
( x = L) the deftcclion w and rhe rotation x are zero (Table 7. 1. page 321); at the
simply supported end (.r = 0), lhe deftection w and !he first derivative of x are
zero
dx(O) =O.
U'(L) = 0 x(L} = 0 w(O) = 0 (7.129)
dx
By subsututing ., and x (given by Eq~ 7.117 and 7.118) into these boundary
conditions. v.e obtain
- <I> COSjl d> sin
cos,,
_q,
sin /J
0
(7.130)
[
,, 0
Dt:AM ::> Wll n ~t:Art Ut.f'UftMAllUN

where <!> is given by Eq. (7.122). When the beam buckles. the deftection is nonzero,
and. he nce, at least one of t he constan ts C1- W n1ust be no nzero. Thus, a t buckling,
Eq. (7.130) is satisfied only when the determinant of the matrix is zero. This
requireme.n t is met ""he n

ifj ' ) tan= .


( I + ..,_.-- (7.131)
SL'
The values of that satisfy this equation a re deno ted by 11, (i = I , 2, . . . ). The
values of..; nl ust be obtained nun1eric.ally. The buckling loads ,Qnp (= Rx1>} are
obtained from Eq. (7.1 19) a nd a re expressed as
I 1 1
-:::--- = ~ + :=. . (7.1 32)
Nery; ,11~.~I S

The lo\\est value of ; results in th buckling load. The lowest value of ,


may be a pproximated by setting (I +{/; ') equal to unity in Eq. (7.131). With
this approxin1ation the Jo,vest value o f ; is 1 = rr /0.70, a nd the buckling load
becomes
1 I I
<>" "' -;;f," + :;. . (7.1 33)
/Vnv --. .>
(0.1L)'

This expression overestin1ates the buckling load by less than 6 percent.


The firs t te nn in the expression above is the inverse or tJ1e buckling load o f a
beam wit hout shear defomiation (see Eq. 6.337 and Table 6. I1):

,V!_ = "'El . (7.134)


ar (0.7 L)2
\Ve no \\' \\'Tile

(7.135)

This equation sho\vs that the buckling load decreases with decreasing shear
stiffness a nd, correspondingly, with inc.reasing shear deformation.
Different types of end supports. Equations (7.128) and (7.135) show that for
be.a1ns 'vith one e nd built.in and the other simply supported o r \vith both e nds
built-in, the buckling load in the x-z plane may be written in the fo m1 (S = S.,)
as

,var= (-~ +
1\ 'ny
d-)-'
s<.Z
(7.136)

'vhere 1
V!>' is the buckling load of the beam witho ut shear deformation.
By a nalyses simila r to t hose presented a bove, it can be shO\\'U that this ex
pression is al~o a pplicable to beams \\ith both ends sin1ply su~~orted or \\~th one
end built-in and the other e nd free. In applying Eq. (7.136), N';,Yis calcula ted by
7.4 BUCKLING OF BEAMS 339

Eq. (6.337) with the k values listed in Thble 6.11 (page 293). Equation (7.136) is
accurate for c-Jses (a),(c), and (d) in Table 6.1 I and m ay overestimate the buckling
load by up to 6 pe rcent fo r case (b).
Buckling i11 the x-y pla11e. \Ve express t he buckling Load in the x - y plane
similarly to the buckling load in the x-z plane as
-1
- 1 I
Nu,= "" + ,_- (7.137)
( N <:rS,, ,. )
_. ;

where 1V!~ is the buckling load for the bean1 v"ithout shear deforn1ation (see
Eq. 6.337).
Torsional buckling. We consider a doubly sy1nme trical cross-section beam
subjected to a n axial load N.,.,.,. Under this Load the bean1 n1ay rotate about the
axis. For such a beam the equilibriu111 equation isg

df - Rx-0;' d''/I = o. (7.138)


dx "' dx2
where ~ is the t\\~St o f the cross section a bout the axis, f is the torq ue. a nd i.., is
the polar radius of gyration define d by Eq. (6.340),

2 '
i. =z;,+r.o +
' fl;; EA.
+ ifl,y (7.139)

and f is given by Eq. (7.14) as

f= fl .. + f . (7.140)

From Eq. (7.40) we have

J 1W111 -
- d - T,., =0. (7.141)
x
The force- strain re la tionships fo r a bean1 \\~lh doubly syn1n1etrical cross sec
tion are (Eqs. 7.32, 7.34, 7.36)
- - di!"
Af,... = - El.., -d (7.142)
x
where r? , and 11 5 a re (Eqs. 7.6 a nd 7.7)

o =do/I o5 =di/I - 11. (7.143)


dx dx
Equations (7.138)- (7.143) may be combined to yie ld
8 1 1
- (d''/Idr? ) - d '/I - .,d '/I
s_ d'
x - -d x x - N,,~;, d'
+Gl,d' x =0 (7.144)

- ,po
- EI., dx' - -s_ (""1dx - ) = 0.
ii (7.1 45)

11 S. P. Timoshenkoand J. Gere, Tl1etJry of El11.t1icSwbili1y. 2nd edition. New York, ~tcGraw-Hill , 1961,
" ??7 P n .; fh\ ~r..i tih
340 BEAMS WITH SHEAR DEFORMATION

After algebraic manipuJations these equations become


torsional buckli1Jg:
- d'o"
- E l. - , - - (N,.~.2 - GI,)
-
-- (do" - Ei d'o") = 0
,_..---,
dx dx s_ dx
(7.146)

d'/I
-
Et d 2 11 8
= 11 - ,_..--,. (7.147)
dx s.~ dx-
The corresponding equations fo r buckling in the x- z plane \\'ere derived pre
viously (Eqs. 7.115 and 7.116) a nd , fo r convenience, are repeated below
iHplane buckling:

-d'x
- E - (dx
I - - N,o - - Ettt' x) =0
-=- (7. 148)
dx' dx S dx'
dw
--x Etd'x
- - -
dt - sdx 2
(7.149)

It is re adily seen that these two equations are identical to Eqs.. (7.146) and
(7.147) when the followingsuhstitutions a re made:
lnplane buckling Torsional buckling
.-. --. 2 ---
N.fo => NJ.oim- Gf1
Et= El~
(7.150)
s=s-
w=> Vt
x = o.
Fo r in..plane buckling the load 1Yxo at \\hich the beam buckles is denoted
by ,Qnr For torsional buckling, the load at \vhic.h the beam buckles is denoted
by 1Va~ - Thus, the solut ions for buckling in thex- z plane (Eq. 7.136)are applicable
to torsional buckling v.ith the follov.i ng substitutions:
lnplane buckling Torsional buckling
-- -. l -
N~ ~ N.:~ 1,., - Gil
(7.151)
El = EI~
s=s- .
The parameters w and x are not included here be.cause they do not appear in
the e xpression for the buckling Load.
Fro m Eqs. (7.1 36) a nd (6.337) we o btain
1 1 1
- = -.- + - (7.152)
/Ven ~ 'S..
. (kl)? <.<.

By re placing 1Va,.. EI, ands__ (= S) with the expressions in Eq. (7.1 51), we
have
1 I
- 2 - = - .-.- + ,_..-. (7.1 53)
NcrV' - GI , :r~ Et_ S"'"'
(k l.l
/,If DUl..1\1.INU ur Dl:AM;) ....
where ,V:. is defined as in Eq .(6.338)

,V". =~;rl EJw_ (7.154)


a~ 11, (kl)'

Equations (7.153) and (7.154) may be rearranged to yield

(7.155)

where N0 .., is defined as


_,
-
lVa.,, =
(N!r,,.
-
I
1- +~
1
~ S,.,,., ) (7.156)

and \Vherc N!~. isthchuckling load 'vithoul sheardeforn1alion given by Eq. (6.338)
with the k value. presented in 'fable 6.11 (page 293). Equalion (7.155) is accurate
for cases (a). (cl. and (d) in Tuble 6.11 and overestimates the buckling load by less
than 6 percent ror case (b).
The buckling loads or axially loaded beams \\ilb shear deformation are sum-
marized in App<!ndix 8 .

7.4.2 Axlalty l.Daded Beams with Symmetrical or Unsymmetrical Cross Sections


(flexural-Torslonal Buckling)
\Ve consider ortholropic beams subjected to an axial load. The cross section may
be synunelrical or unsymnle t:rical. The coordinate directions ,l ' and z 1nust be in
the principal directions (page 208), and. conseque ntly. Ei,, 0. =
When the cross section o f lhe bean\ has one a.xis o f sy1nmc lf)' (nbout the
znxis). it nrny unde rgo (n) buckling in the pla ne of symmetry (.r-z plane). o r (b)
co 1nbined flcxun1l- torsional buckling. When the c ross section is unsy1nme trical.
an orthotropic beam unde rgoes combined tlexural-to rsionaJ buckling.
ln general. for S)'lnme trical and unsymme trical cross sections. every element
o( the stiffness matrix must be considered in the calculation or the buckling load.
Whe n lhe off-diogonal terms of the shear stiffness matrix (Eq. 7.36) either are
zero (cross section is doubly symmetrical) o r are negligible. the buckling load may
be approximalcd by the solution of the following equations'
tu1S)'lt1111~1rlcol cross Sl'ction:

-~"]
0
(7.157)
:0.
- y:tt' i!

\I L. P. Kollllr, Flcxurnl1'bNional Buckling or Optn Stion Con1p05llc Co hunn ~ whh Shenr Deror
mution. ln1r.11wr1t>1111/ J1>11nwl ofSnlitls tu1d Struc1t11'i!'l. Vol. 38, 7525- 154 1. 2001
342 BEAMS WITH SHEAR DEFORMATION

\vhere 11 denotes dete rmin ant. The buckling load '/V,r co rresponds to ftexural-
torsional buckling. The solution of Eq. (7.157} resull< in three values of 1Vm of
'vhich the Jo,vest value is of interest.
synunetrica/ cross section:

(7.158)

(7.159)

\Vhere 1V0 1 is the buckling load in the x-z plane and N0 corresponds to ftexural-
torsional buckling. Equation (7. I 58) yields t he value of N"' and Eq. (7.159) yields
two values of Ru denoted by R.:r2 and FJ0 3. 111e buckling load o f interest is the
lo\\est o f these three values.
We observe that the preceding equations (Eq~ 7.157- 7.159) a re identical to
thooe given for beams \\ithout shear deforma tion (Eq~ 6.34l-6.346). The differ
ence is in the expressions used to calcula te 1Vaz , R.:ry, and Rcr~- Here we use the
expressions given by Eqs. (7.136), (7.137), and (7.155).
Equations (7.157)- (7.159) give the exact buckling load whe n S;; = 0 (i # j)
and either (a) both e nds are simply suppo rted. (b} both e nds a re built-in, o r (c)
\Vhen o ne e nd is built-in and the other end is free. \\'hen one e nd is built-in and
one is simply supported (case (b) in Table 6.11 , page 293) the equations a bove
yield only an approximate value of t he buckling load.
111e buckling loads of axia lly loaded bean1s with shear de(onnation a re sum
marized in Appendix B.

7.2 Exan1ple. An L = 0.5111-/ong and b = O.l nlH'ide rectangular sa11d1vich bean1


(Fig. 7.13) is 1nade of a 0.0211lthick core covered ou both sides by graphite epoxy
f(lcesheers. The mareri<d properties"" given ill Table 3.6 (page 81 ). The layup of
each face is [45\/012/ 45\ J. and the rlrickness of each faceslreet is 0.002 m. The
bea111, builtin at botlt ends. is snbje!ted to an axial co1npressive load. Ca/cu/tile the
b11ckli11g load. Tire core is isotropic ( ~ = 2 x Hf kN!m', "' = 0.3).

Solution. The beam buckles in the x- t plane. With the values of f!J,., =
4 675 N . m2 , S,_ = ,L = I 862 kN (Eq~ 7.107 a nd 7.109), and L = 0.5 m. the
!I'{;

i: 2 mm
f c= 20 mm Figu re 7.13: The cross section of lhe sandwich

b= IOOnun
f 2mn1 beam in Example 7.2.
7.4 BUCKLING OF BEAMS 343

''n = 48 nun

m .----.~v ,:~
,. =48mm!I
h -= 2 mn1

bn = 36 mm
ld=50m
1- <== == 1
Figure 7.J4: The cross-section of the beam in E:tample 7.3.

buckling load is (see Eqs. 7.136 a nd 6.337)

-
Nay
1
"'' + ,_.-)
= ( IV. S..
1 -I (=
I
-- ., + ,_-
-4.T ,E/, S..
I )-1
try ,,_,,_ 1.: ....

1
= ( 738 + 1862
1 )-I= 529kN. (7.160)

7.3 Exan1ple. Au L = O.Stlllong 1-settion beatu 1vith the cross settion sho~n in
Figure 7.14 is 111ade of graphite epoxy unidirectional plies. The 1naterial properties
are given i11 Table 3.6 (page 81). Tire layup is [O,,.,J. Tire beam lr simply s11pporred at
each eud. Detennine tire buckling load lVhen tire bea1n lf subjected to a co111pressive
axial lottd.

Solution. The shear con1pliances of symn1etrical 1-DeanlS are given in T.'lble A .8.
The layup of the we b and the flanges is identical. For a symmetrical layup, (a;'..;)1
and (a;6 ).,... are replace.cl by a6<.. and v.e 'vrite
1
-s,,. = '--' ( b ( a..
,
+ ... 2 )
)'
nl + K- b12(1+t,;)
= 1.681 x Io-N
...... a6ti 1 arx.b11 1 a66ba. 4 1
"' = d + IT d'rr +IT d'yf = 2 .413 x JO N
l.Z ( 0-"" + a) 1 (7.161)
-s-=-
tf2 bn
-
b12
= 2 564 x 10- - -
N m'
_ = 1.2 (
.r,~ -d - , (
"ll
a,. ,
1 +K
) + (
""' ) = - 16.906 x 10_NI-
1
b11 I +;;;
)
6
m
Sy; =S~ =O

d - Zc: I (au)n d
o, = -Zc- =0.833 YI =1 + -3 ( ) ( ') =1.16
{)'JI w bn 1 + i;"
(7.162)
d- e I (au)ri d
~K = - - =0.4219
e Y1 =I + 3(an)w bn(l + ~,) = J.2S.
344 BEAMS WITH SHEAR DEFORMATION

The paramete r a,. for both the web a nd the Ranges is a66 = 109.89 x 10- ~
(Table 3.8, page 85), bn = 0.048 m. ba = 0.036 m. bw = 0.048 m, d = 0.050 m.
and e = 0.035 17 m (Eq. 6.348). The shear stiffnesses a re (Eqs. 7.38 and 7.161)

0
0 ''"'] - ) [637.0 414.4 4.200
0
]
x 103
'" 0
0 ,_
= 0
4.200 0 0.4177
(7.163)

From Example 6.8 (page 294) we have ,v:'., = 713.55 kN, ,V::. = 153.32 kN,
N'.:." = 127.96 kN (Eqs. 6.351 a nd 6.352), CJ,= 1.602 N m' (E~. 6.348), i,.. =
0.024 99 m (Eq.6.350). The buckling loads are (Eqs. 7.128, 7.137, 7.155, a nd 7.156)
-1
- 1 I
Na,= (
Ff. + S. ) = 123.57 kN
cq JY
-1
- I I
Nay = ( ~ + S.. ) = 262.16kN (7.164)

- - ,_
Na~ = N0
I cry


"'"

+-:; GI,= 109.98 kN .


'
\vhere

- 1

- I 1
Na. = /if + .!.S-. = 107.42 kN. (7.165)
(
l.'.rPI ;z )

The buckling load /ii"' is (Eq. 7.158)

buckling in the x- t plane. (7.1 66)

N"' a nd N"' are the roots of Eq. (7.159),

(7.167)

where z., = 0.000 789 m (Eq. 6.349). Solution of this equation yields

1Va2 = 169.17 kN N"-' = 87.06 kN. (7.168)

Calculations that neglect shear deformation give the buckling loads as


(Eqs. 6.353 and 6.355)

1Va1 = 713.55 kN N 2 = 208.88 kN


0 N"' = 106.43 kN . (7.169)
7.4 BUCKLING OF BEAMS 345

We noY"con1par e t hese values witb the buckling loads calculated above, taking
shear de.(onnat ion into account. T he buckling loads ca lcula te d with a nd \vithout
shear deforn1atio n differ significantly.

7.4.3 Lateral-Torsional Buckling of Beams with Symmetrical Cross Section


\\ 'e cons ider orthotropic beams. T he cross section of the beam is S)'lllmetrica l \vith
respe ct to the <a xis (Et,. = 0). The beam is simply supporte d a l each end. At the
sim ple s upport the beam is restra ined fro1n rotating about the x-ax-is., but the cross
section is free to rotate about the y .. and .t-axes and free to wa rp.
The beam is subjected to l \\'O equal and opposite bending n1oments at the
two ends (Fig. 6.86. top). a uniformly d istributed load p in the plane of symmetry
(Ng. 6.86, middle), or a concentra ted fo rce P in the plane of symmetry at the
n1ids pan of the beam (Fig. 6.86, bottom). The dista nce between the s hear center
and t he po int where the load acts is denoted by a.
1-\ t a certain a pplie d load t he bean1 buckles la terally \vhile t he cross sections of
the beam rotate (Fig. 6.87). T his phe no menon is called la teral buckling or lateral-
torsional buckling.
In general, fo r syn1n1etrical cross sections, every e len1ent of the s hear s tiffness
n1atrix mus t be conside red in the ca lcula tion of the buckling load. \ Vhen t he of( ..
diagonal te rms of the s hear s tiffness n1atrix (Eq. 7 .36) e ither a re zero (cross section
is doubly symmet rical) o r are negligible, the buckling load may be approximated
by t he follo\\ing equa tion 11>:

- = F, N- 0
Q0 ,
( Fil!. Na~;z
. + f'.,p, ( F,a + F,f!,, )", + -=-" (7.170)
N,r;,

whereQ0 is the critical value of the be nd ing mon1ent and is re lated to the applied
loads, as s ho wn in Table 6. I 2 (page 298). The positive sign before the squa re root
results in a positive load (\\hich acts up\\ard}. \Vhereas the negative s ign results
in a negative load (which acts downward). The values of the constants F, , Fi. F.1
are a lso listed in Table 6.12. The parame ter f!, 1 depends on tl1e shape of the c.ross
section (Eq. 6.360).
We o bserve that E q. (7.170) is identical lo that given for beams without shear
deforn1ation (Eq. 6.359). The diffe re nce is in the expressions used to calculate Nn:.
and R" He re we use the expressions given by Eqs. (7.137) and (7.155). These
equations contain iV!, a nd ,v:',,_ which are to be calculated by Eq. (6.337) with
k =I.
7.4 Exan1pl.e-. AiJ L = 0.51nlong !section bean11vi1h tile cross section s/JoK111 in
Figure 7.14 is 111ade ofgraphite epoxy unidirectional plies:. Tiie 1nateria/ properties

tu A. Sa.pk:\.." and L. P. KolJir. Lateral-Torsional Buckling of Co1nposi1c Beams.. t ri1eruatio1t1tl Jo1u1111/


fJ/ S-Olitls 1111d S1ruc11~rt',, Vol. 39. 2939-2963. 2002.
BEAMS WITH SHEAR DEFORMATION

SC

L -= 500 nt m
Figure 7.15: The beam in Exam ple 7.4.

are given in Table 3.6 (pllge 81). The layup i.r [O,.,J. Tire beam, simply supported at
each eud, is subjected toa distributed /oltd along the top flange (Fig. 7.15). Derern1i11e
tire buckling load.

Solution. For a transversely loaded beam the buckling load is (Table 6.12.
page 298)

(7.171)

where {10 is given by Eq. (7.170) as follows:

(7.172)

The sign before the square root is negative because the load is down\vard.
= = =
The parameters a re F1 1.13, F, 0.45, F3 0.267 (Table 6.12, page 298). tJ. =
0.01584m, (Example 6.9, page 298), N", = 123.57 kN. N""' = I09.98kN
= =
(Eq. 7.164), i$ 0.024 99 m (Eq. 6.350). and JJ 1 - 0.017 86 m (Eq. 6.369).
\Vith t hese values Qn is

Q 0 = - 2 979N m. (7.173)

Equations (7.171) and (7.173) give

8Q N
Pa =~
L-
= - 95 320 m- (7.174)

Calculations that neglect shear deformation give the buckling load as /kr =
- 115 407 Nim (Eq. 6.372). The buckling load calcula ted with and without shear
deforn1ation differs significantly.

7.4.4 Summary
In sumn1ary, v.hen the cross section is such that the off.diagonal e lements of the
stiffness matrix are negligible (~; = 0, i # j), the buckling loads may be approx
imated by the expressions derived ror beams without shear de(ormation by the
7.5 FREE VIBRATION OF BEAMS 347

follo\ving substitutions:
No shear defom1ation \Vith shear defonuation

N!:, = ( ;,.
N c:r:.
+;
yy
)-1
R,,. = (~ .)-1 +;. (7.175)

N'.:~ = (~y + *-~ )-1


l.'.ftlil' 1:, -
Numeric.al examples 11 12 sho\v that these substitutions are reasonable even
when S,; 0.

7.5 Free Vibration of Beams


In this section " 'e present the circular frequencies w of o rt hotropic bea1us. The
paran1ete.r w is related to t he period of vibration T and to the natural fre que ncy
f by
T= 2rr
w
"'
!= 21T. (7.176)

7.5. t Beams with Doubly Symmetrical Cross Sections


The cross section of the bean1 has two axes of syn1n1etry y a nd z. The n1ass is sym
me trical with respe.c t to these axes, and. accordingly, the ce nter of mass coincides
with the origin o( the y- t coordinate system.
A be.ant 'vith t\vo cross.sectional planes of syn1n1etry may undergo flexural
vibra tion in e ither of the t\vo planes of symmetry and torsional vibration (Fig. 6.94).
There a re three sets of vibrations n1odesll.
wi=w.: W3 = W if!, (7.177)
where wy and w.: correspo nd to vibration in t he x- z and x- y planes, respective ly,
and w~ 001Tesponds to the to rsional mode.
Vibration i11 tlte x-z pla11e. We recall the equilibriu1u equations o f beams
subjected to trans"el'lie loads (Eq~ 7.83 a nd 7.84):

-
Ef!_
'x
dx' +P=0 (7.178)

- d'x
El dx' + S -(d"' ) = 0.
dx - X (7.179)

11
lbid.
11
L. P. Kotlar. AcxurnlTorsional Vibrntion of Open Section Compmite Beanls with Shear D<::formn
tion. /111t rnatior1dl Journal ofSo/i1l.f will Suuaurt .'<. Vol. 38. 7543-7558. 2001.
u \V. \\'cnvc.r. S. P. Tin10..(hcnko. and D. H. Young. Vi'l>ra1io11 Pmblen1s in 11,t:i11teri11g. 5th edition.
John \ViJe)' & Sons. New York. 1990, pp. 422-IJ2.
BEAMS WITH SHEAR DEFORMATION

In writing these equations we tLed t he symbols specifie d in Eq. (7.80). In the


present context p is the inertia force give n by
p = pw2u:, (7.180)

where p is the mass pe r unit length. Equations (7.178)-(7.180) may be re arra nged
to yield
1 tt'x 1 d'x 1
pw' d.v' +?d x' - El = 0 (7.181)

w = - -.
El d'x (7.182)
pwl dxJ
The solutio n of the first or these e quations is 1'

X= c J OOS h z" x+ c2 s 1nh r x + c3 cos r x + ci1SIU


IX.
(7.183)

We substitute this e xpression into Eq. (7.182). The result is

w
El,.._ (
= pwi C, L' smh
VJ .
zx + C, L' cosh zx+ C, Ll sm Lx -
I.I VJ \I .3 . 3
C, L' cos zx ,
)

(7.184)

\vhere L is the le ngth of the bean1, C1- Ci are constants de termine d fro n1 the
boundary conditions. a nd and v a re parame ters define d as

= L prd' ) + (pw')'
(-:::- pw
-:::- + -=
2
(7.185)
2S 2S EI

pw')' pw'
"= L - ( 2~ + ( 2s +
p ')
EI .
(7.186)

Fro m the fi rst of these equations, we have

(!::) El + (!::)' ~]
4
..:!..
w2
= p[

l
S
(7.187)

By combining Eqs. (7.185) and (7.186), we obtain

..2 - !'
SL'
.11.12 - v-2 = 0. (7.188)

Equations (7.183), (7.184). (7.187), a nd (7.188) together with the boundary


conditions provide the circula r frequencies. T he process by \Vhich w is detern1ined
is illustra ted by two exan1ples: a be an1 simply supported a t bo th e nds a nd a beam
built-in a l oolh end~

u E. K rcyszig.Advonced E111;lr1et!rh1g Alot11e11w1ltt 7th edition. John \Viky & Sons. New York, 199J.
pp. l .~138.
7.5 FREE VIBRATION OF BEAMS 349

Both ends simply s up ported . When both ends of the be am are simply supported .
at the e nds the deHection uJ and the first derivative of x are zero (Table 7. I.
page 321) as follows:

w (L) = 0 dx(L) =O w(O) = 0 dx(O) = O. (7.189)


dx dx
By substituting wand x (given by Eqs. 7.1 83 and 7.184) into these boundary
conditions., \\'e obtain

v1 sinh v v 3 cosh v 3 sin. 3

v s inh 11
0
v cosh "
11 1
- 11. sin 11
0
- .cos
1t cosl
-'
lc'l
C2
c...
lol
=
0
0 . (7.190)
[
0 v 0 c. 0

For the deflection to be non1.ero, at least one of the constants C1- Ci1 must
be nonzero. Therefore, Eq. (7.190) is satisfied only whe n the de te m1inant of the
n1atrix is zero. This require111ent is met \\hen

(7.191)

This e quation requires that the s ine term be zero. This condition is satisfied
when

. =, =iTr. (7.192)

Equations (7.187) and (7.192) give the fo llowing expression for the circular
frequencies of the bean1 vibrating in the plane of syn1metry:

_!, = p[.2...
w~
(~) + .!.~ (~)' ]
1fT t:r 11f
(7.193)

Both ends built-in. Whe n both ends of the beam are built -in, the displacement
and the rotation of the cross section at each end are zero (Th.hie 7.1 , page 321) as
follows:

w( L) = 0 X (L) = 0 w (0) =0 x (0) = 0. (7.194)

By substituting w and x (given by Eqs. 7.1&.l and 7.184) into these boundary
conditions., \\'e obtain

[ '"''"
'cosh
0
1
L'
v-1 cosh v 3 sin
sinh v
,,,
0
cos
0
1
-=in n
s1n11
- '
0
Ci
c,
c,
= 0 .
0

0
(7.195)
350 BEAllS WITH SHEAR DEFORMATION

Table 7.4. The equations to determine tile circular frequencies .., of beams with
shear deformation. For vibration in the x-z plane fl = fl,,. and = 5,,. For s
vibration In Jhe x-yplane {I = {I,,, and ~ = ~,,.

- ~
..iJ-
- -JJ ti
I ( L )' 4 -
( L )'
+ "' $

Geometry
/,
"'
sin= 0
(OI)
4 (solution :,= ;;r)

11~ tanh11 -~ tan =0


(h) l 2 - JfLi ,; - ,,i = 0
$1)

(<) 3 cos tosh '' + ![(;)3 - (; ).l ] sin s inh '' - 1 = 0


z _ !1 2v1-v2 =
SI}
o
2 + (; - ; ) sin sinh v
(d) l= ==== l
+[(~)' +(~)' c-OS C-OSb" = 0

2-;z2..;._.,z =o

(e)
-coscosh v + !(;- ;Jsin sinh11+ l = 0
! - !7' 2 1,l - ,,z = 0
sV.

For the deflection to be nonzero, a t lea.st one of the constants C 1- C4 must be


nonze ro. T hus, Eq. (7.195) is satisfied only \vhe n the dete nninant o( the m atrix is
zero. This require1n e nt is me t 'vhen

cos cosh v + ~[ (;)


3 3
- ( ;) ] sins inh '' = 1. (7.t 96)

We now recall Eq. (7.188) as fo llows:

~ Et,, 2
,,.- - ~ . v- - v = 0. (7.197)
2
SL
Equations (7.196) and (7.197) C'1n be solved numerically !or. and v; the
re.s uiting values are denoted by , and l 'i The circular frequencies w; are then
calculated from Eq. (7. 187).

Different types of end supports. The equations needed lo calculate (and the c ir
cular frequencies, Eq. 7.187) of beams wit h diffe re nt end supports can be obtained
in a s im ilar m anner. T he results are sumn1arized in Table 7.4. The calculation of
the circular frequencies requires a nun1e rical procedure except ""he n both ends
are simply supported.
7.5 FREE VIBRATION OF BEAMS 351

Approximate solut.ion. The analysis just described requires a numerical pro -


cedure to calculate the circul ar frequencies. In the following nn approximate
solution is prese nted. which results in a closed-form expre55ion for the circular
frequencies.
We apply FClppl's theorem developed for estimating the buckling load of elastic
structures.." and write the circular frequency w M

.!... :: (.!. .)' + (.!...)'. (7.198)


wl "'" w5
where wu is 1he circular frequency of a beam undergoing bending deformation
only, a nd ws is the circular freque ncy of a beam undergoing s hear dcforn\ation
only.
We illustrate the use of the preceding equation through the cxnmplc of a bea1n
with bo th e nds built-in. \Ve then gene ralize the resu lts to beams \Vith diffe re nt e nd
supports.
To c.alcula tc (J)n we recall that, \\*hen there is oo shear de(ormation t he shear
stiffnrn is infinity (S - oo). In this limit. Eqs. (7.185) and (7.186) yield

. :;;:: " (7.199)

By denoting this value or by 1'8 and by substituting l'B = = ,, into


Eq. (7.196). we obtain

(7.200)

Numerical ~olution o( this equation yields

,,., = 4.730
62 = 7.853
(7.201)

1111 :: (I+ 0.5)11'.

With the.se approximate values of 81 the c.ircuJar Erequcncics are (Eq. 7.l87
wilhS- oo)

( _I ) '
.,..
= P.1_ (J::...)'.
if7 ''"'
(7.202)

To determine wS we note that v.1hen there is no bending defonn:uion the bend


ingstiffness is infinity ( ifl- oo). In this limit Eq. (7. 186) yields v = 0. With this
value of v and with the substitution= l's Eq. (7.196) gives
{7.203)

u T. i;imai, Sun1mn1lnn 111corc1ns Conttrning Qilkal Loads or Oirurcat1011. In: Struautal Sta1'iluy
i11 E11;:u1ct!rifl!( Prualctt. L, Koll:ir, ed., E & FN Spon. London, 1999. l'P 2.1-~&.
352 BEAMS WITH SHEAR DEFORMATION

Table 7.5. The circular frequencies ol beams with shear delormation only. The
equations tor calculating"'" (lelt column) and the values ol 51 (right colUmn).
For vibration in the x- zand x- yplanes S is s,,
and S1 y ~ectively.
{w~~ = ! <'',_"')2
Geometry I'S/

l

,,;; ~
.sin s =0

~ ~ tans = 0 5; = i;r

~ ~ sins = 0

t OS5 = 0 s, = (i - O.S);r
5; = 6.283
/.ts2 = 8.987
-c-OSs+ 1 - 'Tsin s = 0 ,, = 12.566
Appr-0xi1na1e.:
5, ~ Ci + l )7r

So lutions to th is equat ion are

i = 1, 2, . (7.204)

With t hese values of !ts Eq. (7.187) becomes (III - oo)

( w15 ) ' = fJ ( llSi s


L )' I (7.205)

Eq uations (7.1 98), (7.202). a nd (7.205) give the circular freque ncies in the x-
t pla ne o f symn1etry of orthotropic beams 'vith buih in ends:

(7.206)

T he subscript y is introduced to ind icate tha t the bean1 vibra tes in the x- t plane.
It can be shown that Eq. (7.206) is a lso applicable to beams \\1th end s upports
other tha n o nes builtin a t both ends provided th at the a ppropriate va lues of Bi
and J.tsi are used. \ ta)ues for 11i (or differen t e nd support" a re given in Table 6.IJ
(page 308). Values fo r .5; are determine d fro 111 tbe expressions given in Table 7.4
(page 350) by se tting v = 0. (Reca ll t ha t whe n the re is no be nd ing deformation
t!l -> oo a nd v = 0.l The resulting equa tions are liste d in Table 7.5. Solutions of
these equations a re also given in this table. By comparing the results in T.1 bles 7 .5
and 6.l 3 we see that . 5, = JJ.GJ .
7.5 FREE VIBRATION OF BEAMS 353

Table 7.6. The errors in the circular frequencies (Table 7.4)


calculated by the approximate expressions (Eqs. 7.206 and
7.2()7),

Geometry Er = "'""":~ 100


L
()
,{,);.

l
-
6 E r =O

;= t 0 S Er < 4.2
(h)
~ i=1 0 S Er < 3. l
; > 2 0 S Er < 2.4
I= 1 - 0.32 < Er ::: 0
(c) ~ t i=2 0 ::: Er < 4.2
; > 2 0 ~ E r < 3.5
;= t -1 .0<Er:;:O
(d) ;=2 0 S Er < 10.3
; > 2 0 S Er < 5.L
I= 1 - 0.43 < Er ::: 0
{c) i=2 -0.32 < Er :5 0
; > 2 Os Er < 2.2

Circular freq uencies of diffe re nt type s ot be an1s \Vere calcula te d nun1erically by


the accurate (fable 7.4) and by the approximate solutions (Eq. 7.206 or Table B.2).
The errors in t he a pproxin1ate solutions are sumn1arized in Table 7.6.
Vibration in lite x-y pla11e. Like '~bration in the x- z plane \Ve n1ay express
the circula r freque ncy in the x- y plane as

..!... -
w~ - (w~) 2
_ 1_ + _ 1_ 2
(w~)
- ..!_
- E fzz
(.!:.. )4+ S.vr
si
,t.. (.!:.. )2'
s;
(7.207)

whe.r e w 6 is the circular freque ncy of a beam unde rgoing be nd ing deformation
only. a nd ,,}> is the circula r freq ue ncy of a beam undergoing shear deforn1ation
only.
Torsional vibration. For a doubly symn1etrical bean1 vibrating about its axis
the equilibrium e quations a re (see Eqs. 7.39 and 7.40)
t1f
-+t =0 (7.208)
tfx
aidxw. _ Ii' _
' w- 0 (7.209)

where t is the ine rtia force defined as 16


r= aw'l/I. (7.210)

16 Y.'. Wcnlcr. S. P. Tin1oshcnko, and D. H. Young.. Vi1'" uio11 Prnble1t1s iu 11gi11eeriJ1;:. 51h edition.
John \Vik)' & Sons, New York, 1990. pp. 40 1~02.
354 BEAMS WITH SHEAR DEFORMATION

'vhere f> is the polar mon1ent of n1ass per unit length about the shear center
(Eq. 6.402). If! is the t\vist of the cross section about the beams axL~. and M,, is the
bimoment: fis given by Eq. (7.14) as follows:

(7.211)

The force-strain relationships are (Eqs. 7.32, 7.34, 7.36)

8
- - d/1
:\1,.., = - /,,,Tx f'llV = GillJ (7.212)

where II and ,,s are (Eqs. 7.6 and 7.7)

d'{I s dY, n
11= - II = - - II (7.213)
dx dx

Equations (7.208)- (7.213) may be combined to yield

-S~ (d'.P
- , - -dll
8
)
+ 67,-d !/I2 +ew-"1
2
, =0 (7.214)
dx- dx dx
2 6
- d 11 - -S,..,.. (""'
- Elu;dXl -;; - t? ) =0. (7.215)

After algebraic manipulations these equations become

\It= ~ [n"'111 _
Hw- dx
Gi,(d11
dx
_!:I."'11,")]
s_ dx (7.217)

Short beam. For short beams (GJ,L' iflm) Eqs. (7.216) and (7.217) simplify to
torsional vibration:

l d 4 /1 6 1 d 2 11 6 1 B
- - - + ,_..---- - =c-
9w' dx' s_ dx' El., 11 = 0 (7.218)

Ei~ d 311 8
\!!= - - (7.219)
f>wl dx1

The equations fo r vibration in the x- t p lane are given by Eqs. (7.181) and
(7.182) aod, for convenience, are repeated here
7.5 FREE VIBRATION OF BEAMS 355

ilrpla11e vibration:
1 d'x 1 d'x 1
pw' J;;' + ~ Tri - f!TX = 0 (7.220)

u; = El
- ,if'-x
3
. (7.221}
pw- dx
ll can be seen tha t these equations are identical lo Eqs. (7.218) a nd (7.219)
when t he follo\\ing substitutions are n1ade:
Vibration in the x- t plane Torsio na l vibration
P=EI
El= E l., (7.222}
s=s~
w=o/t
x=U"
For in-plane vibration o( short bean1s t he circular freq uency is denoted by
wy, a ndfor torsional vibration by w . Thus, the solution (or vibration in the x - t
plane (Eq. 7.206) is also a pplicable to torsional vibra tion of short beams with the
foJJo,ving s ubstitutio ns:
Vibration in the x- t plane Torsio nal vibration
Wy => W~

r>=EI (7.223)
El= El..
'S= -s_,
The paran1eters w and x are not included in this equation because they do not
appear in the expression (or the circular freq ue ncy.
For bean1s with dif(erent end supports the circular frequency is given by
Eq. (7.206). By replacing p , El(= El,,) , and S (= S;;) with the expressions in
Eq. (7.223), we have

1 e ( L ) ' e ( L )' torsional vibration


(7.224)
w~, = EI,,. Bi + S......, 11..s; short beam,

where u; and 5; (= w) a re give n in Tables 6.13 and B.2 (pages 308 and 463).

Long beam. fur long """ms (Gt,L' El,. a nd S~ L' ifl~) Eqs. (7.216) a nd
(7.217) simplify to

Gt, d1 06 B
(7.225)
x-, + o =0
"aw-,-d
do/f = o. (7.226)
dx
TI1ese equa tions a re independent of the torsional shear stiffness So\0111 a nd are
identical to the d iffe re ntial equa tions of long beams \vitbout shear deforn1ation.
356 BEAMS WITH SHEAR DEFORMATION

Hence, the circula r frequency is identical to Eq. (6.400):


- 2
( wt),)'-GI, ~ to rsional vibration
- 0 Ll (7.227)
Jong beam,

Arbitrary length. \Ve no\\.' approxiluate the torsional circular freq uency of a beam
of a rbitrary Length by

( w J>) 2 -- (wo/I)2 1~hofl + (w 'r) 2 jJons torsional vibration, (7.228)

By using Eqs. (7.224) and (7.227) we have

e ( - L )' + .,_...-
(w~;)' = [=- e ( -L ) ' ] - ' + -Ci/.- ' .Gi'1 . (7.229)
EI.., u, s. ..." s; f) L
\Ve introduce the notation

(7.230)

where w!; is give n by (Eq. 6.405)

(w")' = El,., .ti,. (7.231)


"'' e t..-4
W hen the notations above are used, Eq. (7.l29) becomes

( W J) i )
2 2
= w,,,; + e(J},~
Ll torsional vibration. (7.232)

where .o; and .s; (= ..c;;) are given in Table 6.13 (page 308).
Although Eq. (7.232) is an approximat ion, it is accurate when both ends are
s imply supported.
The circular frequencies are sun1marized in Table B.2.

7.5.2 Beams with Symmetrical or Unsymmetrical Cross Sections


We consider free ly vibrating orthotropic beams. TI1e cross section n1ay be syn1
me trical or unsyn1n1etrical. TI1e coordinate directions y and ? a re in the principal
directions (page 208), a nd, consequently, Ely; = 0.
\Vhen the c.ross section of the bean1 has one a.xis of symn1etry (about the
t~:i.xis), it may undergo vibration in the plane of symn1etry (x- t plane) orcon1bined
Hexural- torsional vibration. \Vhe n the cross section is unsymn1etrical, the beam
undergoes con1bined flexural- torsional vibration.
Jn general, for syn1n1etrical and unsymn1etrical c ross sections. every e len1ent of
the s tiffness n1atrix must be considere d in the calculation of the circular freque ncy.
When the off-diagonal terms of the shear stiffness ma trix (Eq. 7.36) e ither a re zero
(cross section is doubly symme trical) or a re negligible. the circular frequency may
figure 7.16: The
Example 7.5.
CJOQ SCC110ft o( lhc beam 10
~--IJDdSOmm
h = 2mm -t
~ ~

bn l6mm

be approximated by the solution of the (ollo\\~ng sets of cqu:Hions 11:


unsynJJIU!tric:lll cross section:

i z")]
0 - (UJ -
(Yo y.,) = 0.
(yo -y,.)
(7.233)

v.here 11 denotes determinant and w is the circular frequency corrc.sponding to


flexural- torsional vibra1ion. Equation (7.233) yields three sel5 of w.
sy111111etrical crossstt1io11:
(7.234)
"'1 = "''
I[ W:o' 0 ]
"'~' - .,- -
'[ I
(ZG - <..,)
-(ioII-

<><>JI -- 0. (7.235)

Equations (7.2.1 4) and (7.235) )fold three sets of w. The firs! set is w 1 which
corresponds to the vibr:uion in the x- z plane. The second and third sets o( circular
frequencies, corresponding to the flexural- torsional vibration, ore the t\VO roots
of Eq. (7.235). and are dcnoled by wi and w 3
We observe 1hnt the preceding equations (Eq~ 7.233-7.235) arc ide ntical to
those given for beams without shear deformation (Eqs. 6.407- 6.410). The differ-
ence is in the expressions use d to calculate wy. w.,. and w~ . Herc these circ.ular
frequencies nrc to be calculated by the expressions that include the shear defor-
malion (Eq._ 7.206. 7.207, and 7.232).
Equations (7.233) and (7.235) give the exact circular frequency when S, 1 = 0
(i 7' j) and the two ends are simply supported (case (a) in Table 6.13. page 308).
When both ends arc built-in. when one end is built-in and ooc is simply supported.
or -hen one end is built-in and the other end is free. the cqw1ions above yield
only approximalc values of the circular frequencies.
7.5 Example. An /_ =
O.S-111-long 1-sttcion bea1'J u>itlr the cross stttion slro..,rn in
Flgure 7.16 is nradt! of gra1>lrire epoxy un;dir~c,;onal plies.. Tire nrarerlal properties
are given ;,, Ttrblt J.6 (page 81). Tire denslty of the con1po.rlt~ is I.6 g/c111". Tire

n L. P. Ko114r, l'lexurn1T<11'11ional Vibra1ion of Open Stion Com1m.1;ilc Dcan1~ whh Shcnr Oeformn
""ti
lion. /Jllt!1Jl{1t/01111I J f111rnfll ofSolid.v Suuaurt!S, Vol. 38. 7S4l-7SSS. 2<lOI,
358 BEAMS WITH SHEAR DEFORMATION

layup is [010J. The beatn is situply supported at each e1Jd. Calculate 1he natural
frequencies of the bea1n.
Solution. The circular frequencies are (Eqs. 7.206. 7.207, 7.230, and 7.232)
- 1
1 I
- - , + -::::---r-)
(
(w )-
yl
!.: !!a
P L?

- 1
I 1
--.,+~ (7.236)
(
("'~.,I i- ~l's-1
P LJ
)

Wru1= ( (w! )' + s_l\, ) -1


.... 1 e t}

'
"'" = \ w;;,, + e7:F
-
G/l.GI '
The parametersarew;, =8166!,w~ = 3 785 ! ,w!1 =3 458 l (Eq. 6.4l5 ), GJ1 =
1.602 N m' (Eq. 6..148), ;,,. = 0.024 99 m (Eq. 6.350), s 1 = c1 = lf (Table 6.13,
page 308), p = 0.4224 ~ (Eq. 6.412). e = 0.000 263 7 kg' m (Eq. 6.414), lo= 0
(Eq.6.41.>), ~" =0.000 789 m (Eq.6.349).S" =414.4 x 101 N. = 637.0 x 101 N. S,,.
S.~.. = 0.41 n Nm' (Eq. 7.163). With these values we have
I I 1 1
W~I "'~' = 3 206-
. = 4 950 s-
w,1 w ,1 = 3 398 s- = 3 J69-s
s
(7.237)
The circular frequency w1 is (Eq. 7.234)
1
"'' = w, 1 = 4 950- (7.238)
s
where w2 and w3 are the roots of Eq. (7.235):

l[w; 0
1 0
~1 p
.,]-w'[ (Zo - ~,)
I (7.239)

Solution of this equation yields


1 I
w,=5428 - "''. = 2 563 s- (7.240)
s
The natural frequencies are (f = w/2lf)
/ 1 =788 Hz /i = 864 Hz h = 408Hz. (7.241)
Calculat1ons that neglect the shear deformation give the natural frequencies
as (Eq. 6.420)
/ 1 = l 300Hz /i = 959 Hz /1=452 Hz. (7.242)
7.6 EFFECT OF SHEAR DEFORMATION 359

We can compare these \\ith the circular frequencies calculntcd above taking
shear deformation into account. The circular frequencies cnlcuh'llcd with and
Yrithoul shear dc(ormntion differ significantly.

7.5.3 Summary
In swnmary. Y.hen the cross section is such that the offadiagonal elements of the
stif(ness malrix are negligible (~; = O. i # j), the circular frequencies may be
approxi1nated by the expressions derived for beams without shear deformation
by the folloy,ing ~ubstitutions:

No shear dc forn111tion \Vith shear de fo rn1ation


- 1
, I 1
-=> - - + - -
,... n
( ,) ( (w)' f;,u~ )
.V '' I } _,
0
(w,)
2
= (--, l
(ai!')
..
J
+ """""""Z
s,.. ~
P Ll
)_, (7.243)

CUR-=>
(J
'
( -I- + -I -
(wl')'

L L
8 Lt
)
.. -
where ,,}'. "'~. and ,,,a are giYl!n by Eqs. (6.398). (6.399). and (6.405).
Numerical cxamples18 sho\\' that the substitutions abo,c 11.rc rcasonab1e even
for the cases when # 0.S.,
Circular frequencies of beams \vith shear deformation are summarized in
Appendix B.

7.6 Effect of Shear Deformation


ln this section. \Ve s hO\\' some of the conditions under \\ hich shenr clcfornlation 1

need to be considered in calculating the maximun1 deflection. ma:<i1nu1n t\\ist.


buckling load. and natural (requeocy. We take into account s hear deformation
through a parameter CJ by e xpressing W, ift. Ra. and win the forn1s
_ Uta vs
u_i = --
1 - a.
v=-- I - "' (7.244)
Na =,Q!(I - a.v) .., =(w (1 - a. ).
2 8 2
)

A5 before. W,8 , 1/f 8 , N:. and w 8


are the values '11itb 00 Shear deformation. The
effect of shear de(onn:uion is indicate-d by the magnitude of u . When a is small
compared wi1h unily (a I) shear deformation may be neglccied. and 1/1, Na. w.
and w n1ay. rt!!>pcctivcly, be a pproximated by UJB, t/J H. ~.and wn. Jn the (ollo\\ing
we present expre5$ions (or cstinlating a...,., a~. a N and a,..

Ill. Jbid.
360 BEAMS WITH SHEAR DEFORMATION

The maximun1 deHec.tion of a beam \\ith shear deforn1ation is


-.. -B -s jjjB
IAJ=UJ + ut = I (7.245)
1 - ---,,,
l +:J'
By comparing Eqs. (7.244) a nd (7.245), we observe t ha t
l
<xw = - -,-,. (7.246)
1 + ~s
Tiie ra tio w8 fw' is given in Table 7.3 (page 332) for diffe re nt types of beams.
By using the results in this table \\'e may \Vrite Ctw as
1
<tw = I - ' (7.247)
l + (JC~j'Ckll
k
4
:T ~ El

where l!T and 'S re present fl,, and 'S., in the x-z plane and f!l" and 'S,.1 in the
x- y plane and k is t he equivalent length factor t ha t depends only on the type of
suppo rt. Values of k a re given in Table 6.11 (page 293); C is a constant in front
of L2S/Ef in Table 7.3 (p age 332). For the six cases listed in Table 7.3, JCJT/k
ranges fron10.79 to 1.0"1. For o ther types o( supports and loading, this ratio varies
over a sin1ilar range. \Ve are interested only in the magnitude of a,..,. Therefore,
\Ve set JC-:r/ k equal to unity, and \\'rite
1
<tw = . cw' s (7.248)
L+78)
The laleral buckling load of a beam is given by Eqs. (7.175) and (6.337). These
equation may be rearranged to yield

Fia= ( ~+~)- = 1\1! (1 -


1

Na
:Lis)
1 + (;;-)?t
(7.249)

Equations (7.244) and (7.249) show that a,v is


1
""' = - . (7.250)
1 + (~) 2 4
61
The circular freque ncy of a la te rally vibrating beam is given by Eq . (7.207).
This equation may be written as
2 2
= [ -1
l + (~)'(~J'i
1 1
2
+ - -I = (w11 ) 2 ( 1 - )
w
(wn) C,s) ]
,

(7.251)
where 8 a nd , a re defined in Tables 6.13 a nd B.2 (p ages 308 a nd 463). For the
types of supports given in Table 6.l 1 (page 293), the ratio Jr,/ k~ ranges from
0.7 to 1.0. We se t this ratio equal to unity and approximate"~ by (see Eqs. 7.244
and 7.251)
1
a,... = , ~ (7.252)
1 + ('L)-
11'
4t:J
7.6 EFFECT OF SHEAR DEFORMATION 361

Equations (7.248), (7.250), und (7.252) show thnt the effect of shear deform
tion on the n1a.xi1nun1 deHccLion. lateral buckling. and lateral \ibration may be
=
estimated by the para1neter a (a = a,.. aN = a,..) :

dcllection
lateral buckling (7.253)
1:.ncrnl vibration.

where f7 and Sare


i7 = i7,, in the .r-: plnnc
(7.254)
fl= fl,, in the x-y plnne.

The preceding expression for a applies to solid cross sections as well as to thin
walled open and closed-.section beams. Next. we present a for torsional buc-k-
ling. torsional vibration and t\\ist ol shon. thin-walled open-section be.ams. \Ve
only consider shon open-section beams bce.1~c 1wist and 1orsion are significantly
larger for open-section beams than for closcd-sec1ton beams. and shear deforma
tion is more pronounced for shorl than for long bca~
The torsion".!.!>uckl~g load is given by Eqs. (7.154)-(7. 156). For short, open
section beams. GI, EI../ L'. and those equations yield

(7.255)

Equations (7.244) and (7.255) show lhot <x.v is

(7.256}

The para111eter a c.an be derived similarly for torsional vibration and (or the rate of
twist of sho rt, open-section bcanls. For such beams the follo\\~ng inequality holds:
Cl, fl..t L'. With this inequa lity, :ind with the approximations /Cir / k =I.
and tr,/k~ =I the expressions for fi~ nnd 0-111 are identical to that given by
Eq. (7.256). Thus. for torsional buckling. torsional vibration and t"1st of short,
open-section bean1s. a is (a =Oto; = a N = a,,.):

l \viSt
torsional buckling (7.257)
torsional vibration.

We note the similarity ina gi\'en by Eqs. (7.253) and (7.257) and use this simi
larity to establish the conditions under \\ hich shear deformation may be neglected
1

in calculating the deDection. twist. buckling load. and frequency. The e rror intro-
w.
duced into Y,. fJ0 and., by neglecting shear deformation is less than 5 percent
when a is smaller than 0 .05. We now set a= O.OS in Eqs. (7.253) and (7.257). This
L = 500mm

Figure 7.17: 'fbe sand\\'ich beam in Exa mple 7.6.


~I 1
1 ========='=Y=:l
::

b=IOO mm
! 2mm
20mm
. 2 mn1

results in the expression


I
0.05 = , . (7.258)
1+ (U)-
R
4t:I
By rearranging this e xpression. we o btain (a= 0.05):

14 ~/ shear deformation negligible


L~ - -::::- (7.259)
k S shear deforn1ation not negligible.

7.6 Exan1ple. consider an L = 0.5.nrlon.g silllply supported sa1,divich bean1.


~Ve

The core is isotropic ( E.: = 2 x tcf k1V/in1 , ''c- = 0.3), and tlrefaceslreetsarenu1de of
graphite epoxy. Tire material properties are given in Table 3.6 (page 81). The lay11p
of each facesheet is (45\/012/45j). The dimensions are shown in (Fig. 7. 17).
Tra1tfverse or axial load 111ay be applied in she x - t pltnre. Deter111ine lt1hether or
not shear deforn1atio11 needs be taken into account 1vlren calc11/ari11g the 111axb1111111
dejfettion, buckling load,, and circular frequency.

Solution. The bea1u is orthotropic, the c ross section is syn1n1etrical about the
t a.xis, and the load is in the x-z syn1metry pla ne. The re(ore, 've o nly consider
8

" in the x- t plane. The bending stiffness is EI,,


~ (see Eq. 7.68 and
= -}-11 "' o.lln
Table 5.1, page 174). and the shear stiffness is s"
= S11b "'C';,bd (see Eqs. 7.62
and 5.32): a~ 1 is the 11 e le n1ent o( the (acesheet 's compliance ma trix and c;s = G
is the shear modulus of the isotropic core. Wit h these expressions. Eq. (7.253)
gives (a =a.,,.,= aN =a..,.):
2 (kL)' a'II c)-' in the x-z plan e.
<t=
( 1+ 1T'! d
(7.260)

For the 145if012/45j] facesheets we h ave a:,


= 5.18 x 10 -m!N (Table 3.8.
page 85), a nd for the isotropic core G = E;,f2 (1 + v,) = 769 x 10' kN/m 2 For a
simply supported beam, we have k = I (Table 6.l I, page 293). With these values,
Eq. (7.260) becomes
a.,,.. = Cl. = a,.. = 0. LO in the x-z plane. (7.261)
For the s.and,vich bean1 in this proble m \\'e bave

14 (ii rI. in the x-z plane.


Tf'f = IOV ~ =0.74m
A.
L = 500 nun
: 4 'iF--
b= 32mm

figure 7.18: The I-beam ln E.~ample 7.7.

Thus, shear deforn1ation is negligible '"hen L.is greater than 0.74 n1. The length
or the bean1 is 0.5 01. There(ore, shear <leCormat ion n1ust be considered in calcu-
lating the deflection. buckling Load, and c ircular freq uency.

7.7 Exan1ple. Au L = 0.5nrlong doubly synunelrical l-bean1 is rnade of graphite


epoxy. The nutleria/ properties are given ill Table 3.6 (page 81). The dilne11sio11s of
the cross secrion are b = 32 nun and d = 50 nun (Fig 7.Jll). The layup of borh the
flange and tlle veb is (45i/012/45~]. Detern1ine lvhet/Jeror 1101 sheardefortnatiou
needs to be considered in cttlculating the 111axin11on deflection, twist, bucklilrg load,
t11ul circular frequency.

Solution. The cross section of the lbeam is doubly sym111etrical, and the layup
of each Hange and each web is o rthotropic and syn1n1etrical '"ith respect to the
nlidplane of the flange o r t he web. We a pproximate the replacen1ent bending a nd
shear stiffnesses by (Tables A.I , A.5, A.8)
- b d1 l
"< d - -
El,-, "' - (
) -, in the x- z plane
au c - (a.. )w
-
El-- ~
"
-b'-
6{11)1 s,,. - l.2(a2b..)r in the x - y plane (7.262)

-- l d 1f,-' - d b 1
Elw = ----
(a11 l1 24
s,- =-----
2 x 1.2(a )
in to rsion.
66 1

The din1e nsion b and d are sho'"" in Figure 7.18, and t111 and a<io are the
eleme nts of the compliance matrices given in E q. (3.29). The subscripts f a nd
w refer to the flange and the ,...,eb. \Ve obtain et by substituting the preceding
stiffnesses into Eq. (7.253). For ma..xin1um de flection, buckling, and vibration in
the x- y and x - z planes, Ct (et= Cfw = a .v =a,..) is

2 (kl)' (11)1 ) - '


Ci = ( I + tr 2 bd (a66)w in the x - z pla ne (7.263)

Ct= (i + tr'10 (kl)' (a(a11)r)


b' ..)r
-' in the x- y plane, (7.264)

For the t\\'ist, torsional buckling, and torsional vibra tion a (0t = et!/! = CtN =et,..)
is (see Eqs. 7.257 and 7.262)
10 (kl)' (11),) -I
a= ( 1+ 1 --,--- about the x -axis. (7.265)
tr - b- (a66)r
364 BEAMS WITH SHEAR DEFORMATION

Fron1 Table 3.8 (page 85) the relevant ele ments of the compliance n1atrix are
(a11 )c =(an),.= 5.18 x 10 - m/N, (a66)c =(a..).= 27.77 x 10- m/N. For a sim
ply s upported beam we have k = 1 (Table 6.11, page 293). With these values
Eqs. (7.263)-(7.265) give
<tw = CiN =Ct$ = O.l4 in the x-z plane
<tw =a.iv= CJ.,.;, = 0.02 in the x- y plane (7.266)
<xi' = Ct N =Ct,.,. = 0.02 a bout the x-axis.

Fo r the l-bean1 in this probletn \\'e. have

in the x-z plane

in the x- y plane (7.267)

14
k
ff}= 4.4b
f't a bout the x-axis.

Thus, shear deforn1ation is negligible when Lis greater than 1.3 n1. TI1e length
of the beam is 0.5 n1. Therefore, shear deforn1ation n1ust be taken into account
\vben ca lcula ting the deHection, t\\dst, buckling load, and circula r frequency.
CHAPTER EIGHT

Shells

ln this chapter we consider thin composite shells. which we analyze on the basis
of the main assumptions e mplo)'ed in the theory of thin plates. Ho\\"e\ter. there
is a major difference in the bch.a,1ior of plates and shellc; subjected to external
loads. Plates resist trans,erse loads by bending and by transverse shear forces. On
the other hand. thin shells resist 1hc transverse loads mostly by membrane forces.
which. a t any given point. are in the plnne 1ang.ential to the reference surface
(Fig. 8.1 ). These membrane forocs arc dctcm1ined by the membrane theory of
shells... '"hich neglects bending mon1cnts. The rcsu1ting stresses. strains. and de-
formations are re3sonablc except near supports and in the \~cinities of abrupt
c-hanges in loads. For thick shells (\vhose thickness is con1parable to the radii of
cu1vature) o r when regions near supports or concentra ted loads a re of intere.st.
n1ore con1p1ex analytical solulions or finite clc1ncnt 1n e thods n1ust be en1ployed.
The decision as to which n1ethod to use rests \vith lhc individual and depends on
his or her experience with analytical solutions and finite e len\e nt calcula tions.
Here in \\'e treat thin s he lls whose thickness his s1nnll co1npared \Vith all othe r
din1en.sions and \Vith the radii o( cut'\1nturcs (Fig. 8.2). The me 1nbra ne fo rces .Nx,
N,v. NxJ" and N1 x acting a t the reference s urfnce of an infinitesimal e lcn\ent are 1
,,,
N, = f (1 +;,)ti: N,=-,J n,(1 +;,)dz
- 1111
<r,

(8.1)

N,1 = f_., (1 + ;,) ti: =_Jr,,


r, 1

., (1 +;.)a:.
N,,

"''here ~and R1 are the radii o( curva1ure in the .Y- ! and y-::: planes. and x . y. t
are local coordinates 'vith x and >' in the plane tangential and ::: perpendicular to
the reference surface at the point or interest (Fig. 8.2). The origin of the coordi-
nate system is at the refercnc:e surface. \\ hich. conveniently, may be taken at the.
1

1 \V. Auge-. Suasa m Slulb. l.ad cchlN)O Spnn.s,cr. lkrbn. 197.l. pp. s-6.

365
Figure S.l: Mc.mbran e forces in a shell.

mids urface. For thin she lls t he quantit ies 1./ Ry and z./ Rx are sn1aJI \vith respect to
unity, and these expressions re duce to
,,, ,,,
'"
N, = f cr,dt 1Vy = f aydl 1Vx.v = 1Vy.t = f T.1:y d 1.. (8.2)
- /ti. - lu, - / 111

In the "me n1bra ne theory of shells" the men1brane forces de pe nd o nly on the
geometry. o n the boundary cond itio ns., and o n the applied loads a nd are inde-
pendent o f the properties of t he ma te ria l. He nce, the me n1brane fo rces ca n be
detern1ine.d by the equations of s tatic equilibriun1.
The force-strain relationships are (Eq. 3.21)
N, A11 A 11 A 16 811 812 81
,.l~
''~" Aw Bu
A 11 A 21 812 B,_. y
A16 A 26 8 ,. B,. B,,.
1VJI)'

A1x
=
8 11 812 81 '"" D11 Du Di
y:}'
K,
(8.3)

M,.
M.,,.
8 11 822 Bu.
B,. 8,. B,,.
D12
D,.
Dn
o,. '
D..
Ky
Kxy

One of the n1ajor assun1ptions o f the me111brane theory is that c hanges in


c urvatures do oot a ffect the stresses. With this assumption, from the preceding
equation, the stra ins are

f ( I } = [ A11
{ ';
Yx~
A ,,
Ai~
A 11
A 11
A 26
A16
AM
A 66
]-I{ } Nx
Ny
N.1:y
. (8.4)

\vhere f;, l;.. and y.J.~v are the strains of the refere nce s urface. T his set of e quations
applie.s to symn1etrical as \ve il as to unsyn1me trica l layups even tho ugh the for1n of
the equa tions is the same as for syn1me trical laminates (Eq. 3.26). ln the me n1brane

R, R,
Figure ~.2: 1lte mcmhrane forces and Lhe radii o( cuntaturcs of an c lement.
figure: 8.3: S tresses in un isotropic (le(t) and
composite she ll ( rig.ht).

theory the strains are indepe ndent o f the n1oments because the effect~ o f changes
in curva tures a re neglected. In S}'llln1etric-'..sl lan1inates the stra ins are independent
of t he n101n e nts beca use the [B] n1atrix is zero.
We oeglect the variations of the s trains across the thickness of the shell. He oce.
the stra ins are

(8.5)

The s tresses in each layer a re then calculated by E q. (2.126) as follows:

{"' } [Qlg 1l2 Q11


dy
'l'.ty
= Qi] {<x }
Q22 g26
Ql6 Ql() Q66
y
Y.ty
(8.6)

Note t hat the stress distribut ions differ in isotr opic a nd con1posite s he lls. In an
isotropic she ll the stress distribution across t he thickness is uniform, and the resuJ..
tant of the stresses is in the mid plane (Fig. 8.3). In a composite she ll the stresses vary
from layer to layer, a nd the resultant of t he stresses gene ra lly is not in the nl idpla ne.
The stresses and s trains resulting fron1 the preceding a nalysis are used in the
design of the membrane section.
Me mbrane forces for isotro pic shells c-a o be found in texts1 and ha ndbooks.
These n1e1n brane forces also a pply to composite she lls. In the next section, '"e
present results for thin composite s he lls o f practical interest.

8.1 Shells of Revolution with Axisymmetrical Loading


A s hell of revolution is obtained by rota ting a cune, called the n1eridian. abo ut an
axis of revolution . We consider a n e len1ent o f the shells reference surface formed
by t wo a djacent me ridians a nd two parallel c ircles (Fig. 8.4).
The load is a.xisymn1etrica l, a nd the re fo re the re are no shear forces (Nxy = 0),
and only 1 V., and N,. normal forces (pe r unit le ngth) act. Force balance in the
t direction (_perpendicular to the surface) gives3

A'x N11
R + R. =p,. (8.7)
' y

where R.t is the radius of curva ture of the n1eridian (Fig. 8.4) and Ry is along a line
normal to t he n1e.r idian \\ ith a le ngth tha t is the d is tance be t\ve.e n the refere nce
1

i Ibid.
3 Ibid., p. 23.
n1c ridian
n1c ridian .---a.'\is of revolution
::.;:>,;:,.,,;"--.::::::

Figure S.4: Shell of revolution.

surface and the point \Vhere the line intersects the axis of rotation; Pz is the coin
ponent of the load normal to the surface a t the point of interest (Fig. 8.5, left).
We now consider the portion o f the she ll above the parallel c ircle defined by
q, (Fig. 8.5, right). We de note by F the resultant of all the loads acting on the shell
above the paralle l c ircle. A fo rc.e balan ce a long the axis of rotation gives

F + 2:rroNxsin J = 0, (8.8)

'vhere ro is define d in Figure 8.5. From the pre ceding equation, 1V., is

F
1V., = 2.irrosinq,
(8.9)

The fo rces Ny and N, are calculated from Eqs. (8.7) a nd (8.9). Expressions for
Nx and Ny are given in Tuble 8.1 for selected proble ms.

8.2 Cylindrical Shells


We conside r thin-v.alled circular cylinders subje cted to pressure JJ:. (v.hich does
not vary circumfe re ntia lly), axial load N, and torque f (Fig. 8.6).

8.2.1 Membrane The<>ry


By neglecting edge effects, o ne may calculate the membrane fo rces (Fig. 8.7) by the
me n1brane the ol')'. Force balances in the x and z directions and nloment bala nce

1-igure 8.S: Load on nn c lc menl and the frcchody diagr am for a shell of re.\olutjon.
8.2 cnlNDRICAl SHELLS 369

Table 8.1. Membrane forces in spherical domes subjected to intemal pressure (8),
self-weight (b), and cones subjected to internal pressure (c); p, p,, Pm are in N/m1 ;
p,, Is in N/m3.

(a)
~ n\: '(>
'/,
p, f\'t = ~P: R
/\j = ! P.: R

p N-
\ - -~
l+ro1~
(h) R
__.....'
'<P Ny = pR( 1 ..,!,9 - cost,6)

(e) ."'I~ = ;:~s[p( (s.1+ j} + p~1s (T+ j ))


Ny = {P:<> + -"P.:1) {s + s,,.) cot a

1'

Figure 8.6: 1'bin cylinder subjcctc-d to radial pressure p~ ("'hk h does not
LI P,'
vary circumferentially). axial load lJ. and torque f.

1'
v.~

,----_., !V,{B~' Figure8.7: The mcmbrandor<es ina thincylinder.

'N y '
. '
~N,

Y.
Figure $.8: The loads a nd t he mc.mbranc. forces on a cylinder.

about t he x-axis yield (Fig. 8.8)

N,2.Jr R = /i1
2N, = p,2R (8.10)
(N..,,R)2.Jr R = f.
'vhere R is the ra dius of the 'vaus refere nce surface. Fro1n these equations, the
me1nbrane forces are

N = --
R f
-' 27' R N, = p,R N~.v = 2:rR1 (8.1I)

The strains conesponding to these 1nen1brane forces are calculated by Eq. (8.4).
T he axial rl', radial 1n.,, and circumferential t1 d is placemenL" are

(8.12)

'vhere u~ and u: represent rigid -body 1notion.

8.2..2 Built-In Ends


1\ s we no te d previously, ne ar boundary suppo rts t he n1en1brane theory is inaccu
rate, and the forces, mon1ents, and d is placen1ents of the shell n1ust be ca lculated by
other 1neans.. In the follo,ving, \Ve consider thin-walled c ircular cylinde rs built-in
at e ach end. The cylinder is s ubjected to pressure P:. axial load 1V. and torque f

figure 8.9: Cylindt:rJ?uilt-in at both ends subjected to pressure p~ . axial


load Fl. a nd torq ue 1.
8.2 CYLINDRICAL SHELLS 371

j'v
V

o,
,,_-
r~ ---~

JVY

F'tgurc $.JO: foKcs a nd mome nts inside lhc wall of a thin cylinder.

(Fig. 8.9). The pressure may vary linearly a long the cylinder's axis,

p, =Poi+ xp,,, (8.13)

where P:fJ and P::i a re specified constants. (For the pressure d istribution sho\vn in
Fig. 8.9, p, 1 is negative.) For the applied loads (Fig. 8.9), neither the stresses no r
the strains vary circumferentially. Accordingly, the equilibrium e quations a re 4

dN_, = O (8.14)
dx
d
dx (RN,,.+ M, , ) = 0 (8.15)

(8.16)

(8.17)

(8.18)

whe.r e the forces JV.,, A ',. N.ty the nlon1ents M.t , 1\-/l.Y and the t ransverse shear
forces Vx and "'), are illtL'trated in Figure 8.10.
The stTains and t he curvatures of the reference surface are5
du1> w d u"
"=-
x dx f; = R
y" -
xy - dx (8. I 9)

d 1 w" w" 2 d v0 (8.20)


Kx= - - -
dx2 " = - Rl Kxy = - R dx '
where nl). u", a nd uJ0 a re the axial, circumfe re ntia l, and radial displacements of
the re fe rence s urface.
The force-stra in rela tionships are identical to those of lan1inated plates a nd
are given by Eq. (8.3).
The e quilibrium equatio ns (Eqs. 8.14-8.18), the strain-displacen1ent relation~
ships (Eqs. 8.19- 8.20), and the force-strain re lations hips (Eq. 8.3) prm1de the
fore~ mon1ents, a nd displacements of the '"all. In the fo llowing \\le reduce these
equations to readily usable forms.

' Ibid.. pp. 205-206.


~ lbid.. p. 2 11 .
:>ntLL:>

Table 8.2. The parameters required In lhe equations


.,,
[al) = [A
8
22- 7
o,.
u- ~
812]
Du
A11
'"'I= [ "< + !\\'
""
(azJ=
[Ar --;t

A~- U:
o,. 811
Bu, + ~
0
]

/111
[/'I] = [ Nu /112] = (t1o] - [u, ][a,1-'[ii
flu

ft =l-lu
Ji = - ~(fln+ flu) f J=7
Uu f. =11:0- '-;' r
,:i=/1:1

The starting point of the analysis is the integration of the first tv.o equilibrium
equations (Eqs. 8.14 and 8.1 5). Tiiese integra tions yield

(8.21)

\\here Di . Di are as yet unkno,vn constants. By substituting the force-strain reJa..


tionships (Eq. 8.3) into Eq. (8.21) a nd by introducing Eqs. (8.19) a nd (8.20) into
the resulting equatio ns, v.e obtain

{'I
Di = lil { - !''~"'.'
!:'. } + l>J
dx
. { .1
dot;.' } .
,f.T
(8.22)

The matrices [ail and [a.1] a re given in Table 8.2.


Fro n1 Eq. (8.22) the derivatives o( u" and v 0 are

{.J,"...} - [.ir'l"'l { . . }+ l>r ' !'I


dx
= - ~-;
dJ.~
/), .
-
(8.23)

The internal forces Ny a nd M, may be expressed as (Eq. 8.3)

B,_.] (8.24)
Do
8.2 cnlNORICAL SHELLS 373

By substituting Eqs. (8. 19) and (8.20) into Eq. (8.24), we o btain

I ~ I= (a i] {-::f"!' } + (a,J { jf}.


Jr d.t
(8.25)

The ma uices (ai ) and I] are given in Table 8.2. Substitution of Eq. (8.23) into
E q. (8.25) results in

(8.26)

where I HJ and g a re given in Table 8.2. This e quatio n can be v.ritten as

(8.27)

(8.28)

where Hit. /-/11, fhi, H11.gl, gi are t he e le1n entsof the matrix { H) and the vector g.
By in troducing Eqs. (8.27) and (8.28) into tbe tbird equilibrium equation (Eq. 8. I 6)
we obtain
d 4 w0 tf2w 0
x + fi-d
f 1-d, .'(2 + /1w = f< +xfs, (8.29)

where / 1 . . . . . fs a re given in Table 8.2. \Ve note that f4 contains the l\\'O as yet
unkno\\'D constants D 1 and D1 .
Solution of t his fourth-o rder differential e quation yields the rad ial d isplace
n1e nt of the re fe re nce s urface,,

w = l -c. [C1 cos (.8.r) + C2 sin (.llx)J + , ->\L- l[ C3 cos(/l(L - x))

+ c, sin (fl(L - x))]I + [;,cr.+xts)]. (8.30)

wbere L is tbe le ngtb of tbe cylinde r and I. and /I are tbe real a nd imaginary parts
of t he roots of the characteristic polyno n1ia l,

>- = Re(y)
\vhere y=
- Ii+ JIf - 4/1f; (8.31)
p = lm(y) 2/1
E quation (8.30) is the solution of inte rest. This equation contains six unknown
constants Di. D,, C 1- C._ The constant D1 is given by Eqs. (8.21) and (8.1 I)
fJ
D1= N,=-- (8.32)
2" R
The second equality in this equa tion is ,Vfitte n by \~rtue of the fact that N.c is
per unit le ngth, Nis the to t.al force, and 271' R is the circun1Jerence~ Di is given by

6 B. Krcysrig.. Ad11a11ced ,.f:{uttring i'.1arheJJ1ariu. 7th ed ition. Jo h n Wi.ky & Son$.. New York, 1993,
pp. l36-t44.
374 SHELLS

O O"'" N,, - Fis ure ~.I I: The shear force and the 1wist momcnl.

Eq. (8.21 ). The total torque acting at the edge of t he cylinder is (Fig. 8.11)

f = (N.,,.R)2;r R + M, 1 2tt R. (8.33)

Thus, from Eqs. (8.21) and (8.33) Di is

f
[),= - - (8.34)
- 271' R2

The constants C1-C.t are obtained fron1 the boundary conditions, \\ hich state 1

that at a buih -in end the radial displacen1ent and its slope a re zero as follo\\'S:

d w0
1JJ
0
=0 - =0 at x=O
dx
(8.35)
dw0
w =0
0
-=0 at x = L.
dx

The derivative of ul' is given in T.'lble 8.4. 'Vith the displacement given in
Eq. (8.30). the boundary conditions a bove gi\'e

i n
h.
_, Yb

['
0 h
Y,, Ci 11
fJ Yi.1 /j
(8.36)
Y,, lS1 I 0 C; = - (1+/.k
Ii
Y.11 X.1 ). fJ c, ,,
fJ.

\Vhere }i1 are Listed in Table8.3. Equations (8.36) provide C1- C.a. Thedisplacen1ent
w(Iis calculated with the constants Di. l>i. C1 - C;1 thus determined.

Table 8.3. The parameters in Eq. (8.36)


)'t3 = e._;,Lcos /1 L Yj, = e->-LsinftL
Yu = r" (I. cos ji L + /l sin fJ L) )14 = e - J. l. (-ft CO'S /j L +A sin/j L)
l'J1 = e-:.Lcos PL })2 = e- ;,LsinftL
~. = -e->.L {Ac-OS fJ L + /j sin fil) }~1 ~ = e - ;, L (/j cos fJ L - l sin Pl)
8.2 cnlNDRICAL SHELLS 375

Table 8.4. Displacement .,. of a c~llnder, Its derivatives, and ils Integral
IV~ = i <fi1 + Xf.i}
+ e-u cos ({Jx )C1
+,-sin ({lx)C,
+ , -HL- cos({l(L- x))C,
+,-ML- sin (fl( L- x))C.,
rlw" f
d;- = f; + e-M cos(Px)(-AC1 + ftC2)
+ e -Al sin (ftx)(-/.IC1 -AC2)
+ , ->.CL->l cos (/I( L - x ))(;C, - jlC,)
+ , -'<'--')sin (fl(L - x))(JlC, + ;c,l
niw"
f
1
l = e-" cos (flxl((I.' - fl')C, - 2A/IC,)
" +.-'-' sin (/fr )(21./lCi + (;.' - fl')C,)
+ , -M1- cos (/l(L - x))((.<2 - jl 2 )C, - 21.jlC,)
+,-Ml.- sin (/l(L- x))(21.jlC, + (l.1 - fl')C,)
d Jw ;, ) , ? ,
T' A)C1 + {1(31. -{l-)C2)
= , - ' cos({lx (1.(3/12 -
"" +,- sin (/Jx )(-/J(3A2 - fJ'lC1 + 1.(3/11 - A2lCz)
+,-Ml- cos (fi(L- x))(- 1.(3{12 - A1)C, - {1(31.' - /l 2)C.)
+,-<-" sin (fl(L- x))(/1(3J.2 - {1 1 )C, - '(3fJ' - ;.2)C, )

f w'dx = kU' + ~ /)
-M - i.C1 - i1Ci
+e cos(/tx) ;,!+1'i
+ e-" sin (/lx) pcl-"f2>.+fl '
+ e - J.( I.- >.} COS {fi(L - x)) i.<-z.11:
;.-rJI
,
S. ("(L- )) -ftC~di.C
+ e -A(l..-.1.} .1n ,, .t ;.l.._JI!

The expressio ns for the axial and circun1ferential d isplacements are obtained
by integrating Eq. (8.23)

(8.37)

v:
The integra l of w 0 is included in Table 8.4, and u:, are rig.id.body motions.
Once the displacen1ents a re kno,vn, a ll o the r parameters of interest can be
ca lcula te d. The strains are calculated by Eqs. (8.19) and (8.20). (The derivatives
appearing in these equations are liste d in Table 8.4.) The forces and n1ome nts per
unit length are calcula te d by Eq. (8-3).
Orthotropic cylinders. For orthotropic cylinders. A16 A26 8 16 826 = = = =
D 16 = Di6 = 0. and the parameters in Table 8.2 become simpler, as sho'vn in
Table 8.5. With these parameters the radial displacement is given by Eq. (8.30).
376 SHELLS

Table 8.5. The pai-ameters required in the equations for Ofthotropic


cylinders

[;} =[~l

and the axial and circ.umfe rential displacen1ents becon1e

u0 = - 1 [ - ( A1 2 -
An
13,,) ---+
--
R
J wdx B
R
d w" n] + u,,.,
1 1 - - + :cu-1
dx
(8.38)

x
0
v = ~ io... Di+ u;. (8.39)
At.6 - 7 - 7
A de tailed examination o f E q. (8.37) s hows that cylinders with arbitrary layup
rotate (v" # 0) when s ubjected to an axial load (Fig. 8.12). On the other hand,
ortbotropic cylinders do not rotate ( v" = 0; see Eq. 8.39) whe n only an axial load
acts.
Efld effect.<. We c.an see from Eq. (8.30) that the e ffects of the ends are confined
to a narro'v "boundary layer.. and decay exponentially 'vith Ax. For example,
'vi th in a dista nce x = 4/ A f ron1 the end. t he value ot u/1 is \\'ithin 2 pe rcent of the
value given by the me mbrane theory.
For Jong cylinders the boundary layers exte nding fro111 each e nd do not meet
(Fig. 8.13} and there is a central region. \\ hich may be treate d as a 1nen1brane. In
1

this membrane region Eq. (8.30) reduces to

la <X < L - Lo. (8.40)

Figure 8.12: C)hnder subjected to axial load. A rbilrary la)'UP


(left); orthotropic layup (right).

.
8.2 CntNDRICAL SHELLS sn

Boundary Layer

Boundary Layer

figure 8.J3: lJlusLration of lhe boundal)' ln)'ers.

When the two boundary layers do not interact, the support at x = 0 does not
affect the boundary at x = L, and vice versa. Now, Eq. (8.30) becomes simpler,
and within each boundary layer the radial displacen1ent is gi\'en by

w0 = , -'-'[C, cos(flx) + C2sin(flx)] + [ ;, (/. + .r/,)]


0 < x < La (8.41)

w" = , ->(L-zl[C,cos/l( L - .r) + C,sin/l(L - x)] + [ ;., ([., + .r.fs)]


L- La < x < L, (8.42)

where La is the length of the boundary layer (say, Lu = 4/ J.). When the length
of the cylinder is L > 2Lu, the boundary layers extending from the two ends
do not meet. For Lu= 4/ J., this give.s L > 8/i. or i.L > 8. When J.L > 8, the
parameters li; (Table 8.3) are negligible, and the constants C1 and C2 are de
termined from the first ty,o ro\\'S, and C3 and C4 fron1 the second t\vo ro'vs of
Eq. (8.36).

8.J Exautple. An L = O.S-1n -lo11g cylinder (inner radius Rt 11 =0.2111) is 1nade of


grt1phite epoxy unidirectional plies. The n1areria/ properties are give1J i1J Table 3.6
(page 81). The layup is (0111/4510]. The Odegree plies are on the imide and are
parttllel to tire cylinder:f axis. The thickn~ss of the tvall is /J = 0.002 111. The cylinder
i.r built-ill tlf botlr ends (Fig. 8.14). An axial load fii = 20 000 N is applied to the
cylinder. Ca/cu/are tire displacentents along the length.

,V= 20000N
n_ = 200 ntm

r
k=2 mm
e
g
V"I x
~ ~--}-~ .. .----...

Figure S.14: The cylinder in Example 8.1.


Solution. The radius of the reference surface, chosen as the n1idsurface. is

R = R.;0 + /r/2 = 0.201 m, (8.43)

whe.r e Ir is the wall thickness (Ir= 0.002 m).


The axial load is fl= 20 000 N, and the torque fis zero. The constants D1 and
D, are (Eq~ 8.32 and 8.34)

fJ N f
D1 = N, = - = 15 836 - [), = - -, =0. (8.44)
2ttR m - 2:r R-
There is no radial load acting on the cylinder, and''"' = p,1 = 0 (Eq. 8.13).
The elements o( the (A], [BJ, and [DJ matrices of the wall are given in Eq. (3.54).
With the values in Eq. (3.54) t he matrices in Table 8.2 (page 372) become
.,,
[a,j = B[ lb1 -
12 - R
-;r
o,,
812] = [55.269 x 10 I 6 820]
D" 16 806 64.842
(8.45)

1a,1= [A"
A16 + 8~
A,. - :~ 'Rf
'D"] = [194.52
A.. - "'If - 34.88
3~.62] x w
4.66
(8.46)

[a 2) = [A" -
Ai& -
!
R'
B"
816 + 0
~'
] = [ 39.05
34.79
- 0.051 61 ] x 106
0.017 45
(8.47)

[ I-
-
[A" Ai - ~~ ]-
811 s,. - ~'
2

2 -
[ 39.46 34.62] 10"
- 0.051 61 0.017 28 x
(8.48)

[ HJ= '"l - [a,J [Jr ' l2l = [


26 306
~ ~~ 3~.~~~] (8.49)

g= (a,Jla.1r 1 J ~J = l -!~~J (8.50)

The constanL / 1- /, are (Table 8.2, page 372)

/ 1 = H,, = 31.76 [, = - ~ (H.?1 + H12) = - 69 931

b = ~ = 651.13 x JO" f, = ''"' - g~ = - 5 288


Is = p,, = o. (8.51)

With t hese [; values the root of the characteristic polynomial is (Eq. 8.31)

- ti +/tf - 4f1 h .
y= 2[, = 5>.05 + 41.401. (8.52)

The parame ters ), and fJ are

J. = Re(y) = 53.05 p = lm(y) =41.40. (8.53)


8.2 cnlNDRICAl SHELLS 379

The expressions in Table 8.3 (page 374) give


Y, , = - 0.8258 x 10- n Y, , = 2.9021 x 10- 12
Yn = 76.319 x 10- ii Y,, = 188.15 x 10- 1i
(8.54)
y11 = - 0.8258 x 10- 11 Jiu= 2.9021 x 10- 12
v,, = - 76.319 x 10- 11 Y.,, = - 188.15 x 10- n

By substituting the preceding values of f; a nd Y;; into Eq . (8.36), we obtain

- I i. fJ0 Y, ,
Y,, Yi1
y,, 1 Ct
Ci 1_1 8.1211
0 _6
[
Y11 >'ii 1 O J c, - 8.121 x 10 (8.55)
v,, y,, ), {J c, 0
Solution o f this set of e quations gives the constants
c, = 8.12 1 x 10- Ci= 10.41 x 10-
(8.56)
c, = 8.121 x 10- c, = 10.41 x 10- .
The radia l displacement is (Eq. 8.30)

w0 = (e- '"'(C1 ons(fJx) + Cisin (fjx)]


+ , -l.\L- )[C,cos({J(L - x)) + C4 sin (fJ(L - x))J}

+ [ ~ (/, +
3 xf,)l (8.57)

With the rigid.body n1otions neglected, the axial and circumferential displace
me nts are. (Eq. 8.37)

I~:) = - [tor'laiJ 1-::}


,,, x[a_.r' I~ J.
+ (8.58)

where ,( w"dx and du..'0 /dx are calculated by the expressions given in Table 8.4
(page 375).
The axial, circu1nferential, and rad ial displacen1ents are calculate d by
Eqs. (8.57) a nd (8.58) together with the constants I.. {J, C1-C,. [., fs given in
Eqs. (8.51), (8.53), a nd (8.56). TI1e results a re plotted in Figure 8.15. T he length of
the boundary laye r is (page 3n) Lo = 4/1. = 0.0754 m.

8.2.3 Temperature - Builtln Ends


\Ve consider a cylinder\\~th builtin ends on 'vhich lhe n1echanical loads f'i, lJ, and
Tact (Fig. 8.9) and that is subjected toa tempe ratureehange t. T. This t. T mayvary
in an arbitrary n1anner in the radial direction and linearly in the axial direction,
t.T(x. t) = t. T;(z)(t. To +xt. 7)), (8.59)
where a T;(z) is the temperature variation across the wall and ~ 7i'1i a 7i are
constants.
380 SHELLS

x (m)
o.s x ( Ill)
:i: (Ill)

~---r o.s o.s

_ .!)_,_ _ _,___,_,
- 20 - 10 20 40
"
Figure 8.IS: 1 be radial w0 , circ.umfercntial if', and axial u" dLo;placcmcnts o( the cylinder \\'all in
x.ample8. L The unil of the displacementli is 10- 6 m.

111e equilibrium and strain- displacen1ent rela tionships are given by


Eqs. (8.14)- (8.20), a nd the relationships betwee n the strains, curvatures, in plane
forces, and moments for plates are given by E q. (4.251). The radial displacement
can be calcula te d by t he steps en1ployed in Section 8.2.2. The resulting expression
for w 0 is identical to Eq. (8.30), but now fi. fl, (which appear through J.), and /J
are those given in Table 8.2 (page 372), and /, a nd fs are those in T.ble 8.6. The
expressions for the a.xial and circumfe re ntial displacements become

T he vector b-r is defin ed in Table 8.6. Once the d is placements a re kno\\'11, all other
parameters of interest can be calcula ted. The strains are calculated by Eqs. (8.19)
and (8.20), the forces and moments (per unit length) by Eq. (4251}.

8.3 Springback
When an unrestrained isotropic she.II is heated to a uniform te mperature, its size
changes but not its shape. \Vbe n a laminated shell is heated unifonnly, both its
size and shape change. This diffe rence in the beh avior of isotropic a nd lan1inated
shells is illusua ted in the next section for a cylindrical shell segment. 7

8.3.1 Sprlngback of Cylindrical Shells


To describe the springback phe no n1enon we examine a cylindric.al shell segn1ent
whose temperature is changed uniformly by l1 To (Fig. 8.16). Because of this tern
perature c hange the arclengths L1 a nd /4 , the tl1ickness II, a nd t he subtended
angle y change to L~ . L:i. h' , a nd y' . The springback is defined as the change in

7
N. Z:a.h1an andl ~'t. O ' Neill. Design and Fabrication of Composite C-0n1po ncnts: thcSpring-Forwnrd
Phcnon1cnon. Crnnpo!itt!.f. Vol. 20. 77-81. 1989.
8.3 SPRINGBACK 381

Table 8.6. The p81amete" required In f<I. (8.60)

fa=p~ - ~ - ar..~ /, =p~1-ll7i'1f


where
b= 1::1 (llI-' bT- <T and [a, ).(... ) .g1 arcgiYtn in Table 8.2.

l>r =[~:+!t ~+!f ~+ '; ::+ ~ i,:+!'f ~+~]n


<T = [Au ""
Bu Bu
A.."
Btr,
B,, Boi B,.]n
Du 0.! D11.>
n is Lite vector oC hygro1hermal Slrains caused by fl J; (z) defined by Eq. (4.2.SO)
n = 1.~...,
I
~"\' 111 r'" ....,
1) l
"''11
)'
"' .. ,I'
I}

the angle during heating or cooling as (oLJo,vs:


1

springback =Y y - Y. (8.61)

The angle y i related to the arclengtbs and the thickneH by (Fig. 8. 16, left)
Li - L,
Y= (8.62)

Similarly. y ' is "


l.j - Lj
(8.63)
y = "' .
When tl1e scg1ncnt is made of a single layer, the thermal strain , h is aa r.
Thus. afte r deCormation, the a rclengths or the top and botto1n surfrtces and the
thickness ore (Fig. 8. 16. right)

Ii = Ir + Ci', a 7;111 . (8.64)

where a,.
and Ci; arc the the rmal expansion coefficients in they and z d irections.
respectively. Substitution o f Eq~ (8.62)-(8.64) into Eq. (8.61) gives the springback
.
spnngbock =(ii,I +a,a
- ii.)aTo
_ ~ (8.65)

When heated. an isotropic material (for which Ci', = ii,) ex:panili equally in

_D
__
every direction. Thus. \\'hen heated uniformly. the springback of an i~otropic

_'i.____;
,

1
\'i __ r?--~
' .., '
~>'
-y
!'
Figure 8.16: Cylindrical tihcll scgrnenl. Unde-
formed clement (lcO) and the ckmcnl deformed
due Lo a unL(onn ch;1ngc In tc 1npc ra1 u ~ ( right).
..., :>ntLL:>

LlT= O LlT> O

Figure 8.17: Springback of <1 composite scg


mcnt made of unidirectional 6bcr-rcinforced
matrix.

v
cylindrical segment is zero; the size of t he seg1nent changes but not its shape.
On the o the r band, in a con1posite mate rial the thern1al expansion coefficient of
the n1atrix differs fro m that of the fibers. Thus, for exan1ple, a heated segme nt n1ade
of identica l layers containing fibers in the circu111(erential direction (Fig. 8.17) e x
pand" more in t he t direction than in they direction (ct.v < Ci.::) ln this case the
springbac.k is nonze ro a nd the segment bends. (A simila r argun1ent holds when
the segment is cooled.)
The thermal strains a re generally sn1all con1pared with unity, and the spring ..
back may be approximated by
. back = (-
spnng a,. - 'z
- ) u ~ hi hi
10 = Ey - Ez . (8.66)
This expression, derived for a single layer, may a lso be used for laminated
con1posites with appropriate \alues of E~ 1 and E~1 . For a Jan1inated composite the
average hyg.rothe m1al stra in perpendicUJar to the surface is a pproxin1ated by
h' - h !>h
-~ = - Ir - = T," (8.67)
where the change in thickness t;./r is given by Eq. (4.287). The c ircumferential
hygrothermal strain is approxin1ated by the inplane hygrothern1al strain of a Hat
lamin ate given in Eq. (4.250)
1\~l
1V!!'
E~l = E~h = r 11 12 0-26 P 11 /311 ~uJ ~~ x
cs.6s)
M'.'
y
M'"xy
\ Vhen the layup is S)'lllme trical. the springback of a cylindrical segment is
given by Eq. (8.66) with the strains in Eqs. (8.67) a nd (8.68). When the layup
is unsrmme trical, an additional te1m n1ust be included in the springback. This
tern1 is arrived a t h)' observing that a fla t plate \Vit h syn1me.trical layup does not
b-end when the plate's temperature is c hange.cl uniforn1ly. On the other hand a
Hat plate with unsyn1me trical layup bends \\'he n the ten1pe.ra ture of the plate is
c hanged uniformly. The change in curvature is K;'.
Hence, the total springback of
a seg1nent " 'ith unsymn1etric.al layup is

springback = &-;+ {E;hl - E~')


unS)'Tllme trical
segme nt.
(8.69)

\Vhere 11:~' is a pproxin1ated by the hygrothermal curvature of a flat laminate given


8.3 SPRINGBACK 383

Figure $. JS: L-shaped stiffc.ne-r in Example 8.2.

by Eq. (4.250) as follows:

iv.:
'N;;"''
y

(8.70)
J\1!''
M"'
y
j~!,
When a cylindrical shell or circular cylinder is analyzed, this springback effect
n1ust be included in the hygrothermal force-strain relationship of laminated plates.
This is accomplished by making the following re placement in Eq. (4.251):

Laminated plate Cylindrical she ll


(Eq.4.251) (8.71)
.-h
y
= <hi
y ,, _ ~ht)
+ .!.R (fo.hl :; .

8.2 E.xample. An L slwped stiffei1er (Fig. 8.18) is made of graphite epo.<y unidi
reclional plies. The n1areria/ properties are listed iii Table J.6 (page 81 ). The lttyup
i.r (45/0J,. Tire ().degree plies lire along tire length of 1/ie sriffe11er. The 1hick11ess
of the wall is h = 0.002 in. The tenrperature is raised by 80 C Derern1ine the neu1
shape of the stiffener.

Solution. The springback is (Eqs. 8.66 and 8.67)


. hi "" _, 5.899 x 10- - l
spnngback = , - /, = 1.653 x JO - x _, = - 1.297 x 10 .
2 10
(8.72)

where his the thickness of the wall (Ir = 0.002 m), /;./r is the change in thickness
due to the temperature change ( /;./1 = 5.899 x 10- m, Eq. 4.295). and is the e
strain in the circumfe re ntial direction (<~' = L.653 x IQ- 3, Eq. 4.273).
The change in the angle of the 90-degree stiffe ner is (Eq. 8.61)

y' - y = springback x 90' = - 0. It7' . (8.73)

This is illustrated in Figure 8. 18. right. The layup of the stiflener is unbalanced
and, the re(ore, the stiffener will also twist.
384 SHELLS

Figure 8.i9: Radii of cur\a lures Rx. t<,. of a shell.

8.3.2 Doubly Curved Shells


We generalize the preceding result (Eq. 8.71) and extend it to doubly curved shells
with radii of curvatures R.,. Ry in the x- t and }'- < planes (Fig. 8.19). In this case,
the follov,.ing replace n1ents must be n1ade in the Jan1inated plate hygrothern1al
stress... train relationships (Eq. 4.251 ):

La minated plate Doubly curved shell


, ht
N. .T
= hi
K1:
l (,.o.hl _ ,ht)
+ _Rx "'x "';: (8.74)

Kr
ht = . ho
"'
+ _1 (,.o.h1 _ .. ht )
Ry "'y "'.::

8.4 Buckling of Shells


We consider a con1posite shell subjected s in1uJtaneously to s urface loads on the
wall and compressive and shear loads at the edges (Fig. 8.20). When this load set
is increased beyond its critical limit, the shell buckles e ither globally or locally
away from the edges. In general, local buckling precedes global buckling. In the
(olJo,ving 1Ne treat local buckling of lan1inated composite shells.
\Ve consider a sn1all element a\vay fron1 the edges. The men1brane forces acting
on this e lem ent are (Fig. 8.20, right)

(8.75)

'vhere A is the load paran1eter. We are interested in the value of A (denoted by


I.a) a t which the s he ll locally buckles. An analysis describing the local buckling
or shells was presented by Kollar. In this analysis the buckling load is de rived by
representing the short waves arising at t he location of the buckling by trigono
me tric functions. The de riva tion is long and involved a nd is not give n he re. \Ve
only quote the results for s he lls that are shallow9 io the region \Vhere the local
buckling occurs. and in this region the s he ll has cons ta nt curvatures (Rx = con..
slant. R,. =constant, and R.'fJ' =constant). The reference surface is design ated by
the function f (x, y) (Table 8.7) and the rad ii or curvatures are l / R = - a' f/ax1 ,

fl L. P. Kolllir, Buckling or Generally Anisotropic Shallow Sandwich Shells. )()unud of Rt inforcttl


Pla.w its a11d Can1p,,siltt Vol. 9, 549-568, 1990.
11 \\'. Flug_i;c:., Stre.f,(~s lr1S/u!ll:t. 2nd edition. Springer. Bertin. 1973. p. 414.
8.4 BUCKLING OF SHELLS 38S

--,,::.--N-,-=---J-. N~,,~
Ji' ~ y
N =- >.N,
!)
N,, = - >uV"JIJ

figure 8.20: Surface and edge loads on a shell and the membrane forces.

1/ R,, = - 82 f13y' . 1/ R,,. = - 23 2 f!axay. For such she lls, the resulting equation is

II!
VJ

([oL 0
L] [M 111,,][o L]T - >-[<t>,[I]
M,, M. L 0 <1>211 J
<l>z[f]])
<!>1(1)
w,
112
= 0. (8.76)

v2

where u 1, v1, 111 , u1 are the in-plane and w1 , w2 are the out-of~plan e an1plitudes of
the buckling waves. The mat rices I OJ, [L]. [1] and t he parame ters <l>i, <l>z (given
in Table 8.8) depend on four constants a , jJ, c,. c1, \\ hich characterize the ""ave
1

patte rn of the buckled shape (Fig. 8.21 ).

Table 8.7. Curvatures of typical shells

f(x. )') R, Ry R,,

~lindec
I= -C,y! 0
.~~' 0

elliptical
~boloid
f = -C1),i - Ci:c2 lC.}' 2~. 0

'
hyperbolic
- - &raboloid
f = -C1y! + Cix2 - :<'!
l
i~. 0
y
hyperbolic
~abolnid
-, f = -Co.ty 0 0 .L
?C1

y
'
386 SHELLS

Table 8.8. The parameters required in Eqs. (8. 76) and (8.89). For a cylinder
1/ Rx = 1/ R,, = Oand Ry= R

[I -2a/!(~+ci,.,)]
0 /! 0 0
- /! 0 0
[0)=
H,
"'
,,,, 0 al + pici pi + ct?cf
..L

re,
~ c,f!')]
0 - a1 0 0
[L] = ~ aq - /Jez 0 0
0 .,,
..L - 2af!ci - 2aftci 2(coa 2
0 811 0
[~ ~] -[ M. ]
Au A11 812
Au Ai.? 0 811 B1i 0 [M.j =
(M,,J =
0 0 ..... 0 0
Bu "'00
[~ ~]
811 0 D11 D1 2 0
812 IJ,_i 0 Do1 Dn 0
0,,,
[J) =
0 0 0 0 [),,,
0
<1>1 = N\o (a1 + piq) + N.,~o2 (alc1 + P2<J) + JV.,0 (a1c 1 + P1)
<l>i = - 2tt/}(N,(lt1 + ,V,.,o( l + c:1c2) + 1'',l)(.'1)

The d irections of the lines passing through the n1axilna of the left and right
propagating waves are (Fig 8.22)

a + fjc,
tan 11 1 =
,., + ac-1
(8.77)

The e igenvalues Aa o f Eq. (8.76) a re the critical load parameters, which are to
be calcula te d for different values of the cons ta nt" Ct , {3, c1 , ci. The lo\\est value of An
resulting fron1 these calculations is the value of interest. In principle, the values of
these constants can not be se.lected a rbitrarily, but must be chosen such that the cor
responding displacen1ents satisfy the boundary conditions. For short wavele ngths.
\Vhich is the case considered he re , the edges do not aJfect the 'vaves significantly
and, hence. the buckling load. Therefore, as an approxin1ation, a rbitrary values of
CJ , jJ, t'1, ci may be chosen for calculating A0

Figure 8.21: Illustration of the buckling pattern at local buc.kling of a shell.


8.4 BUCKLING OF SHELLS 387

y
Figure 8.22: The local b uckling pattern and the corresponding contour plot.

Once the constants a. f'J , c1, a nd c2 a re kno\vn, the dc Hection perpendicula r to


the reference surface of the shell is calculated by
wJj = w1 sin (a(x + c,y))sin (/l(y + e;,x)) + w, sin (a(x + c ,y))cos(/l(y + c,x)).
(8.78)

8.4.1 Buckling of Cylinders


The buckling an alysis presented earlie r in this section for t hin s haUo\\' s hells n1ay
be applied to the buckling of thin cylinder.; by selling t he radius R equal to the
radius Ry of the shell (R = R,.). and R,. and R,, to infinity (l/ R, = 1/ R,, = 0).

8.3 Exam ple. An L = l -111/ong cylinder (inner radius R 11 = 0.2 is nrade of


111)
graphite epoxy unidirecliona/ plies. Tire n1areria/ properlies are givei' i1J Table 3.6
(page SJ ). Tire layup is [- 304/154/02).. The Odegree plies are parallel TO the cylinder
axis. Tire rlrickuess of the wall is Ir= 0.002 m (Fig. 8.23). Au axial load is applied
to the cyliJJder. Calculate the load that ca1'ses local buckling.

Solution. For this cylinder we have

R = R,. = R,0 + h /2 = 0.201 m l/ R, = l/ R,, =0 (8.79)


1~0 = 1Vx,1 =0.
For N.,u we a rbitra rily select lhe value N.i;o = 1. \Vith this value and v.ith the
values in E q. (8.79). the matrices [OJ and (L] a nd the parameters <!>1, <!>2 are

1-lgure 8.23: The cylinder in Example 8.3.


""" :5Htll5

(Table 8.8, page 386)

(~ c1ciJ
0 fi 0 0
[-a
(OJ= ~ - {J a 0 0 (8.80)
4.975 0 Cti + pie~ fl' + a'cf - 2a/i +

~
1 0 - ac 0 0
[(Jc,
lLi = ~ ac1 - pc, 0 0 (8.8 J)
0 0 - 2a{Jcz - 2a{Jc1 2(ciu 2 ci{J' J
tP1 = a 2 + pie~ (8.82)
<1>2 = - 2a{Jc,. (8.83)

The matrices [M0 J, (111,,J a nd (J) (Table 8.8. page 386), with t he ele ments of
the J A) and (DJ mauices giveo in Table 3.7 (page 84), are

235.54 x 10 32.74 x 10' 0 0 0 0


32.74 x JOO 27.79 x JO' 0 0 0 0
0 0 36.01 x 10 0 0 0
(M.] =
0 0 0 65.42 16.29 0
0 0 0 J6.29 11.60 0
0 0 0 0 0 17.39
() 0 - 10.19 x Jo" 0 0 0
0 0 - 10.19 x Jo" 0 0 0
- 10.19 x ttl" - JO.J9 x 106 0 0 0 0
(M. ] =
0 0 0 0 0 - JS.93
0 0 0 0 0 - 7.74
0 0 0 - JS.93 - 7.74 0

[~ ~l
0
0
111 =
0

To find).". we refer to Eq. (8.76):

"'
([Lo l][M M][o L]T- i.[ <t>,<!>2(1]
0 M0 M0
(J)
L 0
<1>,(J ]] .) ,,,, = 0.
<!> 1(1) u,

(8.84)

To obtain ). and the displacements \Ve assume diffe.re.nt values of the para1neters
a , fl, c1 c1 . For example, ""e set a= 40, f3 = 40, c1 = 0, ei = 0. \\'ith these values
8.4 BUCKLING OF SHEi.LS 389

Eq. (8.84) becomes

0 J
II
0 VJ
1600 WJ (8.85)
(GJ -l. =0.
0 "l
0
I 600 "'
Vt'here
""
434.49 109.99 -6.5145 32.601 32.601 - 2.0274
109.99 102.09 -5.53 11 32.601 32.601 - 2.0274
-65145 -5.5311 1.1466 - 2.0274 - 2.0274 0.27312
IGJ = 32.601 32.601 x 109
- 2.0274 434.49 109.99 - 6.5145
32.601 32.601 - 2.0274 109.99 102.09 - 5.5311
-2.0274 -2.0274 0.27312 - 6.5145 - 5.5311 1.1466

Solution of Eq. (8.85) yields

a =40
J.0 = 434 810 for /J = 40
CJ =0
c: = 0 .

We are interested in the IO\.\'CSl vn.luc of Acr- To find this value we repeat
the calculation fo r different sets o f CY. /J. c 1 c1 values. The lowest value o( An
corresponds to a = 37.5,/i = 16.4.tJ = 0. L52.c2 = L.3. Fo r these&,/J, CJ,c, values
the results are

a = 37.5
)." = 345 568 fo r Ji = 16.4
CJ = 0 .152
<2 = 1.3.
The buckling load is

N.,.a = l.aN..o = 345 568 N/ m =346 kN/ m. (8.86)

Figure 8..24: The buckkd aha pc oi tk an1soc.ropK" cyhndcr an EPmplc 8J (left) llnd the boclcled
shape: on the a.ssumptton tba11bc cyhnckr r1 OC1hot..-optC (ng,hl).
J90 5Htll5

-N
-> ,,f:._P:
! N,,

-o~
-> ~
-> [J
- p,

- .Al i
Figure 8..25: C..)'lindcr subJectcd to axial load and pressure.

The d irections or the lines passing through the n1axin1a o( the left and right
propagating waves are (Eq. 8.77)
a+ f3c2 a - /Jez
0 1 = arctan - - - = 69.4 02 = arctan = 56.50 . (8.87)
f3 + ac 1 p - <iCJ

Solution ofEq. (8.84) also gives the a mplitudes (see Eq. 8.78) of the d isplace-
me nts. Fora= 37.5,/3 = 16.4,c 1 = 0.152,andc2 = 1.3 theamplitudesofthe radial
displace1nents are

w, = - 0.707 06 w, = 0.707 06. (8.88)

The buckled shape (Eq. 8.78) determined by these amplitudes is shown in


Figure 8.24, left.
In the following \Ve consider cylinders subjected to an axial load - N and to
a radial pressure - p, (Fig. 8.25). The membrane forces are N.,0 = - N/2tr Rand
N,,, = - p, R.
Orlholropic wall, The layup of the wall of the cylinder is orthotropic, and
one o( the directions of orthotropy coincides \Vith x. For this c-ase \Ve assun1e
that the cylinder buckles in a checkerboard pattern (Figs. 8.26 and 8.27). For this
checkerboard pattern IJ = IJ 1 = t>2 =arc.tan ~- and from Eq. (8.77} we have that
<1 =Ci = 0. Equation (8.76) now reduces to

,,, } =0,
(IOJlM;,](O]r - ll>1[JJ) : ,
{ (8.89)

Figure 8.26: llluslration o( lhe checkc-rboard buckling pattern at local buckling o(ano rthotropic
cylinder subjcct('-d to a:ual load and uni(onn pressure.
8.4 BUCKLING OF SHELLS 391

...11, =;!
....

'. ~
Figure 8..27: The loc;al chcckcrbovd bucthng J>31tcm and the eotTCSpondtng rontour plot of an
ortbotrop.c cyhockr subfcctcd 10 an:al load Md urufonn pr-cuurc.

where the matrix (OJ and the parameter <l>t. given in Tobie 8.8. are evaluated with
c 1 = ei = 0. The eigenvalues of this cqu3lion are

Fu Fu Fu
Fu Fu. Fll
1 F1 3 F,, F,,
Acr= z , (8.90)
N,..,a + N,o/J
IF"fiz ~I
where 11 denotes the dc te nninant. The clements F, 1 (i, j = 1. 2, 3) are given in
Table 8.9 (page 392). The critical load parameter)." (Eq. 8.90) must be calculated
for different values of a and fj . The lcJ\vcst )..(.., is the vnlue o( inte rest.
Orthotropic a11d sy111111etrical wall. When the \Vail's layup is ortho tro pic and
symmetrical ( 8,1 = 0). Eq. (8.90) simplifies to'"

1 "'(t
H! Aj, )
- ~ ]
An = N,,,oa z + N>~,pl {[ .It!.+ crl!Jl (J.. - Apt..tu+ZA..>) + L
~ A.. A.!: At..
Au Au

+ D11a' + 2(Du + 2D.,.){J 2a ' + Du fJ' } , (8.91)

where)"' (Eq. 8.9 1) must be calculated for different values of tt and/I. The lowest
=
Ac-:r is the value of inte rest. When R oo, the term in parentheses is zero, and the
expression above reduce to that for Hnt plat"5 (Eq. 4.12 1).

8.4 Exarnpk. A11 L = 1111-/011g cylintler (inner radius R.,, = 0.2 111) U 1nade of
graphite epoxy tu1id1rect1011a/ pli~s (Fig. 8.28). Tht- 111a1erial pro~rtia are givn1 ilt

10 R. ~l.Joocs. 81.K'tbng o1 Ortulu C)l~.S Sklls 'llll,111 ~luhipk Orthotropit U)>tr.s: lltld Ecttn1ric
Stifftntn. A/AA Jolll'NJI.. Vol. 6. 2.lOl-lJQS, 1968
392 SHELLS

Table 8.9. The parameters required In Eqs. (8.90) and (8.96).


F11 =(Au + J!o.)u#
Fu = -Au~ - Bua 3
- B11"fJ2 - 1~ap?
ro.u = kP? + A t<,a 2
F,, = - BnP' - B11a' P- 2 B;.u'/! - Au~
1
*
F.rJ = D11a J + 2(012 + 2~)P2a2 + Dii/f + (k~ + 2811/l 2 + 2B1~u 2)

Tllble 3.6 (page 81). The layup is [- 304/ 154/ G,],. Tiie Odegree plies are parallel to
the cylinder axi.t Tire thickness of the 1vall is h = 0.002 tu. An axial load is applied
to the cylinder. Calculate the buckling load that causes local buckling.

Solution. The layup follows the JO-pe rcent rule (p age 89), a nd, accordingly, we
may treat the cylinde-r as orthotropic.. The layup is symmetrical and Acr is calculated
by Eq. (8.91)

A.er=
I .,
{[ ~;(1 - A~~~) ]
Nx0a 2 + N,.,fi- .. + al Ml ( __!.__ _ A1(.rlp +2A.>.-.) ) + .!!'..
A.!'! I' A,., Au ,.t,.: An, An

+ D11 a4 + 2(D11 +2D. 6 ) /l 2a1 + D,2 /l 4 }. (8.92)

For this cylinde r \Ve have

R= R,= R;, +h/2=0.201 m 1/ Rr= l / R,,=0


(8.93)
1V,-0 = Nx) O = 0.
For 1\~,o \Ve arbitrarily select the value Nx0 = 1.

With these values a nd with the e le ments of the (A] and [DJ matrices in Table 3.7
(page 84), Eq. (8.92) becomes

},,, = ~' l[ (35.98<> + 13.2~;~'+ 4.24611


4 4) 10-]

+ 65.42a' + I02.l/l a
2 2
+ l l.60/l' I (8.94)

FigurC' S.28: 'fbC' C')'Lindcr in Example 8.4.


8.4 BUCKLING OF SHELLS 393

Table 8.10. The parameters In Eq, (8.96).


F,, = Bua'+ 0..11' F11 ( 811 + O.,, \et/I
f,_, = ( B., + 0..) a{l Fn Bull' + O..et'
F,.. = -0.1a' - (Du+ 20,,.)a,8' - ~ Bi:a
+
F,, = -Dv.fl' - ( O.i 2 0.o)a' ll - ~ Bu/I
F.i.i = Dua1 + A,ttjfl +Su f4, (Du+ ~)"/t
Fss = Dufl' + o..a1 + S,,
\\'here ).0 i.s to be calculated or different of a nnd /J values. \Ve are intere.sted
in a and jJ that result in the IO\\'CSl wlue of .\0 In this case, l.a is minimum for
a = 54.5 and fJ "' 0 and has the value ).,, = 388. Thus. 1he buc.kling load is

N,. 0 = l.aN..o = 388 kNlm. (8.95)

We also calculated the buckled shope. which is axisymmetricoL as shown in


Figure 8.24. right. The buckling lood caleula1ed wi1h 1he assumption thal the wall
is ortbotropic agrees within 12 percent with the result of 1he "exact.., analysis
( Nx.a = 346 kN. Eq. 8.86).
Sandttich C) li11der. When the cylinder's \Vall is of onhotropic sandwich con
1

struction (page 176). >.., is11

F11 F., F,) F,. Fu


F12 F22 F,_, F,. F,,,
f "'1'! Fz, F F.H F_,~
F111 F1a F'."M F.4 F..-.
l P., F,, F.~ P,s F,.,,
Ac, = 1\~,ual + 1\~vo/J2 IF F I IF. F. I
(8.96)

F:: P~ F: J.:::
where lhe eleme nls F,; a re given in ll 'l'dblcs 8.9 nnd 8.10 and ).0 (Eq. 8.96) musl
be ca lculated for diffe rent vulucs of u and /J. 111c lo, vcst resulting value o( Acr is
=
the value o f interest For R oo Lhi.s expression reduce$ to tJ1at for flat sa ndwich
plates (Eq. 5.9 1).
Wben the cylinder y,all ls o f isotropic o r quasi-isotropic sand,vich co nstruct ion
(page 176). Eq. (8.96) simplifies LO

- 1
'"' - N,..,, 2 + N,.,,
{ ""( - ("")' )
A I
(~)l , 2
l v (a-, + /J)

(8.97)

11 L P. Kol!r. BIK'thng ol GcncnlD)' Anu.olropic ShlilMtt Sandtntb Shclh. .Jot.rnal of Rmtforttd


PWoa111tJComptntfO. Vol 9. S49--S6& 1990.
394 SHELLS

\vhe.re .,u(I. Ai~1 , and [j~ are given in Table 5.2 (page 178) and ~ is given by
Eq. (5.39). When the cylinder is s ubjected to axia l compression only (N,., = O. see
Eq. 8.11). the cylinder bucklesaxisymmetrically 12 ({J = 0) a nd Eq. (8.97) becomes

l.n=-N 1 [A""(J ( ;w)') I l (


.rl)
- v ~--,+
/\- Ot-
l ,+><
-D.ll'UQ'- I)-'] .
..)
(8.98)

The lowest value of ),.:r is obtained from t he condition

()An = O. (8.99)
aa
12 J. Hcscd(fa, Buckling of Axially Compressed L)lindrkal Sand.ichShclls. Aa11 Tecl1nica H1111garica.
Vol. 89. Jn-387. L979.
CHAPTER NINE

Finite Element Analysis

The finite cle1ncnt 1ncthod offers a practical means of calculating. the cle fornlations
o(. and stresses nnd strains in, complex structures. A de tailed description of the
finite eleme nt me thod is beyond the scope o f this book. Ln:ilcad. v.e focus on those
fea tures specific to composite ma te rials.
Finite e lement analysis c.onsists of the follo\\'ing major steps:

1. A mesh cncompa5.<ing the structure is generated (Fig. 9.1).


2. The stiffness matrix [k[ or each element is determined.
3. The stiffnc.s matrix [Kl or the structure is determined by assembling the ele
ment stiffness matrices.
4. The loads applied to the structure are replaced by an equi,alent force S)'Slem
such 1hru the forces act a t the nodal points.
5. The displacements or the nodal points d are calculated by

!Kid = f. (9.1)

'''here f is the force vector re presenting the equivale nt applied nodal forces
(Fig. 9.1).
6. 111e vector dis subd hided into subvectors d. each J representing the displace-
me nts of the nodal poinL'i of a particular eleme nt.
7. The displaccmenlS nt a point inside the element are calculntcd by

u =[N]6. (9.2)

where the vector u represents the displacemenlS and [N[ is the matrix or the
shape vectors.
8. The strains at a poin1 inside the elements are calculated by

c = [Bl6, (9.3)

'vhe re ( /J) is the strain- displacement n1atrix.

395
396 FINITE ELEMENT ANALYSIS

noda~},, no.dal
fore.($
~ pornts

clements
Figure 9.1: Struc.turc and its llnite element mc$b.

9. The stresses at a point inside the e len1ent are calcula ted by

"= [ ], (9.4)

" 'he re [ EJ is the stiffness n1atrix characterizing the n1aterial.

The ele ment stiffness matrix, re fe.r re d to in Ste p 2, is defined as

(9.5)

\vbere f1: represents the forces acting at the nodal points of the ele1uent. The
element stiffness n1atrix is 1

[k] = 1
(V)
[BJT(E)[B]dV. (9.6)

'vhere Vis the volu1u e of the eleme nt.


111e preceding ste ps apply to structures made of e ithe r isotropic or compos-
ite ma terials. The only difference bety,een isotropic and composite structures
is in the n1aterial stiffness n1atrix I El. In the following \\'e present expressions
for [ ).

9.1 Three-Dimensional Element


The stress-strain re lationships (or a three-din1e nsional e le n1ent are (Eq. 2.20)

<1x C11 C12 Cu C111 C 1s c,. <x


(fy c,, Cn Ci.1 C2s c,. C24 <y
c" c.. c...
C:o
C12 c,. C.u C.,o
"'
ry;:
= Ct11 2 C4s l

c., Cs2 Cs.i c" Css Cs6


c 44 c46 "
Yy;:
(9.7)

-r.T;: Y.t;:
'rxy c61 c62 c.,
c6..l c64 cii6 Yx.~

11

'vbere [ ] is the stiffness matrix for a th ree-din1ensional element.

I R. D. Cook, D.S. ~ta l k.us. and ~1. E. Plesha . Cor r~pr.~ at1tl Applicatiotu of Fi11i1e Ele1t1e111 Autll)'.~is.
3rd editi on. John Wile)' and Sons. New York, L989. p. 110.
9.3 BEAM ELEMENT 397

9.2 Plate Element


In the absence o! shear deCormation, the force-strain relationships (or a thin~plate
element are (Eq. 3.21)

N,,
Ny
A11
A12
A12
Ai1
A1
A16
811
Br!
B,2
811
816
IJ,_. ,.,;
y
1V.,,. A1 Al 116 81 816 &., y;~.
:\(l = 811 Bu 8rn Du D11 D,. K,,
(9.8)

J\~v 812 8,, a,. D 12 [),_, Di. Ky


A1xy Bi. 8 16 a.,, D,. Di. Dt.6 /(JI}'

11
\\.'here l 1 is the stiffness n1atrix for a plate e len1e nt 'vithout s hear deformations. In
the presence of shear deformation, the force-strain re la tionships a re (Eqs. 5.13-
5. LS)

1V., A11 A16 Biz 8,. 0 0 ,.,


1\ ), A12
A 12
A12 A16
Bi1
811 8n B,_,, 0 0 ,.y
1Vxy A1 Ai& ..... 8,. 816 a.,, 0 0 Y.:'y
J\(l 8u 812 816 Du D12 D,. 0 0 Kx
1'1y = 8,, 0 0
812 816 D 12 [),_, Di. Ky
A(,>, 8,. 8,. a.,, Drn Dz. D66 0 0 K.ty
\I 0 0 0 0 0 0 Su 511 Yx~
"
v,. 0 0 0 0 0 0 :512 S11 y,.~

11
(9.9)

where I EJ is the stiffness matrix for a plate e le 1uent with shear deformations.
Frequently, the be havior of s hells can be described by replacing the curved
surface \Vith sn1aU, Hat e lements. The stiffness n1atrices above a re applicable to
such Oat shell ele ments.

9.3 Beam Element


For a bean1 element, the force- strain re lationships are
arbitrary layup, no shear defornuttion, no restrained K'arping (Eq. 6.2)

rI['"
NI,
B'
T
=
P12

Pu
P"
Pr1
P11

P13

P1:1

IE!
Pr3
p,J
P,,
P,,
~1 ri1
_4
Pl"
P:1,

Py

1..
1?
(9.10)
398 FINITE ELEMENT ANALYSIS

orthotropic, 110 .rh~ar deforrnario11, 110 restrllhu.>d warping (Eq. 6.8)

rl l"
0 0
ti, _ o El,., El,- (9.1 I)
~
~: - ~ Elr.
0
El
0
IEJ
onhorropie, no sltear dtfonnation, re.straintd M.'arpi11g (Eqs.. 6.8 and 6.233)

R EA 0 0 0 0 ,.
'
M, 0 El,, ..
Ei,. 0 0 .L

M, = 0 El," El" 0 0 ,,.L (9. 12)
NI... 0 0 0 Ei. 0 _ <Jt

f,. 0 0 0 0 Gi, ,,'"


11
ortlrotropi<, 1Jrear deformario11. restrai11td warpi11g (Eqs. 7.30. 7.32. 7.34, 736)

R EA 0 0 0 0 0 0 0 <:
My 0 E/11 El" 0 0 0 0 0 -,,,
...
!l!.

ii, 0 El,, El" 0 0 0 0 0 - -;;;


M. 0 0 0 1.. 0 0 0 0 d
- Tx
f,.,.. = 0 0 0 0 G/, 0 0 0 ~

~!1
...s,., :r,. . .
~ 0 0 0 0 0 Yy
0 0 0 0 0 ~t s,, S:..- Y:
"'
f;,. 0 0 0 0 0 g>"'1 S:;i. s~ iJS

lt.'J
(9.13)

'vhcrc I 1 is lhe stiffness matrix for a beam element.

9A Sublamlnate
A laminate consisting of several plies may be analyzed by either plate (Oat shell)
or tbree-dimcnsional elements (Fig. 9.2). For tbick laminates neitber of these

1----~i@ Figure 9.2; Ttuck lnminalc (lop). anaJysL'>


"'1t.h plalc c lements (le(t). an al)'Sts wiLh

- - ~? ( "'-. .q~ 1hrccdi111cru-1onnl clements (righl).


9.4 SUBLAMINATE 399

L:.in1in1a1c Sublamjnatcs FE ~tt"sh

p1 ...
Figure 9.l: 1bld: b.mmatc (left). sublam1natcs (nuddlc). and the 6n11c ckrMnt rnc11h (nght}.

is practical. Plate clements give inaccurate results. 1l1ree-dimcnsional clements


re quire thal lhc 1n111erinl be uniform througho ut the elen1e nt. nnd. he nce, an ele
n1ent must con1ain a single layer or adjacent identic--Jl lnycl's. TI1is n1ay result in a
very large number of e le 1ncnts., n1aking the numerical con1puta tion difficult and
often inc~1sib lc.
We can ovcrcon1c these djfficulties by dividing the la n1ina tc into s ublnn1inates
(Fig. 9.3). Each loyer in the sublan1inate may be monoclinic. onho tropic, trans-
versely isotropic. or isotropic. The thickness of each e lement is the same as the
thickness of 1he corresponding sublaminate. The stillness mairix I 1of s uch a
sublaminale is defined by the relationship

;;,
a,
"ii:
r,i: = IEJ
.,"
l,
(9.14)
f,,
f_.: Y.:
'f",. Y....

111e bar de notes ~\\1 crnge st resses and strains. It is conve nient to re present this
expression in te rnls o f the con1pliance matrLx (J]

111 112 lu ]14 } 15 1,. a,


1: 1 21 111 ln lu lis lz a,
1,, Jn Jn }35 J,,, ;;..
Y,:
= J., le
1.,
1., '"'
1.. J., 1..
1.. J., J,,.
r,o:
r_. ..
(9.15)

Y.: Jn Jn
Y. .,. 1., J., J., 1.. J., J.,, r_.,
,,,
where

1
IEl = l.1 1" (9.16)
400 FINITE ELEMENT ANALYSIS

= ~ ,..1 1:~ ,,
E3 I .:,1, I

Figure" 9.4: Illustration or a sublaminate.

The x,y ,t coordinate system is sho,vn in Figure 9.4. 111e average stresses are
defined as

(9.17)

The second equalities in the left-hand column are written by '1rtue of Eq. (3.9).
The average strains are

(9.18)

Y.~y = Y.t1

where II, is the thickness of the sublaminate (Fig. 9.4). The terms in the right-hand
columns or Eqs. (9.17) and (9.18) show tliat the stresses"' ry:. r,, a nd the s trains
f x. ~,. Yxr do not vary across the thickness.
Jn the folJo,ving ,..,e derive the e len1ents of the compliance n1atrix.

9.4.1 Step 1. Bements of [ J] due to In-Plane Stresses


ln this step 've detern1ine the ele111ents in the first. second, and sixth colun1ns of
the ma trix[.! J. To this end, \'le in1pose the average inplane stresses <fx. Uy. and
i".ry on the sublan1inate (Fig. 9.5, top). Since a~, f"yz T x.t are zero. the stress-strain
9.4 SUBLAMINATE 401

, ,.+t. ,<
p;
"'
/
Figure V.S: 1Jlu11ra1ion of Step I. The ply stress a nd the corrc-~o;pon ding tivcrilgo s1rc-ss on t he
su bl anun ate~

relationship (Eq. 9. 1S) may be written as

(9.19)

(920)

I~,,} = [J"
Y.1.: l si
(9.21)

The strains arc uniro nn across the thickness (Eq. 9. l8). Under these cond itions
K..,, Kv. 1<.1>' are zero. a nd \Ve have (see Eqs. 3.2 1and 3.7)

{
N, f.~
N.. } = IAI { } =(A) { ,.
<, } . (9.22)
JV.,, Y:ry Yxy

where. [ AJ i.s the tensile stiffnes.\ matri.'< of the sublaminatc. (The summation in
Eq. 320 is performed from I 10 K.. where K, is lhe number of layers or ply groups
in the sublaminatc: see Fig. 9.4). For lhesublaminate Eqs. (9.17). (9.18). and (922)
yield

(9.23)
402 FINITE ELEMENT ANALYSIS

By comparing Eqs. (9.19) and (9.23), we have

(9.24)

O wing to the Poisson e.ffect, the inplane forces introduce a no rn1al strain ;.
in the z d irection. For a single layer this strain is (see Eq. 2.133}

a,. } ,
= [s., Si., s,.J { a., (9.25)
f.t y

'vhere t'1x , <Ty, and -r.t }' are t he stresses in t he plies. The average no m1al stress
iixacross the lan1i nate and the normal ply stress <1x are iJlustrated in Figure 9.5
(b<Jllom). The stresses in a single layer a re (Eq. 3.13)

: : } = [QJ { :: } . (9.26)
{ 'Cx)' Yxy

Equations (9.25), (9.26}, (9.23). a nd (9.18) give

(9.27)

By combining E qs. (9.18) a nd (9.27), a nd by replacing the integral with a s wn-


mation , for t he s ublan1inate v.e obtain

K,
<, = z)!Su
k I
s,, s_,.](u - u-1HQJHAr1 { ; : }. (9.28)
. r.t>'
[I,, J.,, J...J
By comparing Eqs. (9.20) and (9.28), we have

K..
p,, J,, hl = L:<rs., si., s_,.J.cz. - z.-1)[QJk)lAr 1
(9.29l
k I
9.4 SUBlAlllNATE 403

where z1c. Zt- 1 are illustrated in Figure 9.4. Since fy~ = fx~ = 0, \\'e have that y)'<:
and y." are zero (Eqs. 2.26 and 2.21 ). a nd E q. (9.21) now becomes

{~'~;,.
IOI
0 = ls1
[;., } 42
ls1 ls6
;,,.] }

(9.30)

To satisfy t his equation, t he preceding e le n1ents of the compliance n1atrix must


be zero:

''] = (00 0() O


1,. 0J. (9.31)

9.4.2 Step 2. Elements of [ J] due to Out-of-Plane Normal Stresses


Jn this step 've determine t he eletne nts in the third colun1n of the matrix (1 ). To
accon1plish this \Ve consider a sublamin ate in \vhich there a re only a;;. stresses. To
form s uch a s ublaminate, first \Ve consider a sublan1ina te restraine d along its edges
(<x = <y = Yxy = 0) a nd apply a uniform stress"'= ii, (Stage I, Fig. 9.6), which
inuoduces inplane stresses 'fix, dr, ~x.v Second. 1Ne apply inplane stresses - ti;w.
- 'iiy, - fxy (Stage 2, Fig. 9.6). 111ird, \Ve superimpose Stages 1 and 2 a nd arrive
at the sublaminate inside \\hich the inwplane ave rage stresses U., , U.v, f.ry a nd
the transverse shear stresses Tyz T.~ .- are zero and the stress-strain relationships
(Eq. 9.15) are

~ } = [ 123
{<, '"] 'ii:: (9.32)
Y.~J' J{ll

(9.33)

(9.34)

Stage 1. FolJo,ving the outline above, '"e a pply"::. to the sublaminate., 'vhich is
restrained along the edges(<, = ,. = Yxr = 0). For a single laye r. Eqs. (2.27) a nd

o,
t t t "t t
!'!. ~.~:!. ~~I~~I~ I I
J, ~~~J, + ~~J,J,J.
Stage I Srage2 Stage 3
Figure 9.6: Illustration of Step 2. S uhlamina tc restraine-0 along its c.dges .subjected to a~ and the
resulting stress ~t (Stage I) : unrestrainc.d subla mina1c. subjected to a., (Stage 2): unrc-s trained
sublaminatc. subjected to at (Stage 3): (<f1 and f'">' arc. not sho\"'n).
404 ~I Nll E HEMENI ANALYSIS

(2.20) give

<1: =c=,,., (935)

{";:~ } {c"}
~ = ;_.
(936)

These cqu3t.ions nlay be rearranged to yield


I
, . ==:--a. (9.37)
C.u

{""} {~u} c
C',,
r,lf
= ~23
Ctil
I
33
ai. (9.38)

By combining Eqs. (9.37) and (9.18). and by replacing the integrals by sum-
maLions. ro r the sublan1inate v..e obtain

-
I : = 111 k-1
I t (z. - t-1)
(CDlt ".:
(939)

Equations (938) and (9.18) yield

;;,
{ 7,,
1 "
-a ,, } = - "-
t-
II, -
({Cu}
C.,
-Cn :tk - U-1
(C.ul.
)
a.:. (9.40)

TI\is equruion gives the inplane stresses in the rcs1r11ined sublaminate.

S tage 2. We apply 1he e qual and opposile or the strcssc"' calculated by Eq. (9.40).
to the s ublaminnte. T he correspondingstrains are ob mined from Eqs. (9.28),(9.23),
and (9.40). Equa1ions (9.28) and (9.40) result in

, = - 1.1,, 1 .., J,.J {;: }


T)'

= - [1,, ]32 (9.41)

Equations (9.23) and (9.40) give

{"Y.,.}= - [Ju ln 11] {r._


a,} 111
1, 111 lu. Or
,. l tt1 J., Jfi:>

=-
[111 11] I ({Cu} lr:.
)
Ji. ;; L: ~2.' z.<~ :;-' ""
111 Jn
K, (9.42)
1., 1 61
J66 J k I Cti.l .t
,\.1 k
9.4 SUBIAMINATE 405

Stage 3. We combine Eqs. (9.39). (9.41). and (9.42) to obtain the strains of a sub
laminate subjected only to t he outOfplane stress <1;.

_ 1
= ( J,' L(
kI
K,
< - <-1
(C)33 k
)-In "J J 1 "
1...1- L..,
h, k-1
"' ({c"}
c,, -
CnJ k
< - ~
(C.u )k
)) '"
(9.43)

{ ~:
_
y JI)'
} = - [',:: ',:
16, 162
',:]-11t ({~::}
166
1,
k-i
-
CbJ /r
<cc,.
- <-) <1,. (9.44)

Comparisons of Eqs. (9.32) and (9.33) wit h Eqs. (9.43) and (9.44) yield

(9.45)

1_,, = ~ ( ' (C- )


h1 L.., <-) - 11,, hi ,... 1-
1 "'"' ({c"}
L.., -c,, < - ~1 ).
llkl 33 k h, ,,._, c., (C.u)k
(9.46)

Since <" =<,, = 0, we have that y,, and y., are zero (Eqs. 2.26 and 2.21), and
Eq. (9.34) becomes

IOI
0 = ['}53']-(/ ; (9.47)

To satisfy this equation, the preceding e le ments of the con1pliance n1atrix must
be zero

[~::] = [~]. (9.48)

9.4.3 Step 3. Elements of [J] due to Out-of-Plane Shear Stresses


In t his step ,...,e detern1ine the elements in t he fourth and fifth c.olun1n of t he matrix
[J).
To accomplish this \Ve apply the shear stresses r,,.~ , r.t;: o n the sublaminate
(Fig. 9.7). Since ii1:. Uy. Ci;. i'x>' are zero. the stTess-strain relationships are

F'igurc-9.7: lllustralion or Step 3. Stress r,, and slrain


v.u on a sublaminale subjected to trlln.o;\'erse she.ar
loads ( le.ft) and the corresponding average stress r ,:
a nd average.strain Y:...~ (right).
FINITE ELEMENT ANALYSIS

(Eq. 9. 15)

(9.49)

(950)

For a single layer, we ha"e (see Eqs. 2.26 and 2.21)

(9.51)

By combining Eqs. (9.18) and (9.51) and by repladng the integrals by summa
tions. (or the sublaminate we obtain

l-r_,, I= - I:
y ..-:
1 .... (
h~ .t--i
<:-.. - ::..-.> [s-...
S.u
(952)

By comparing Eqs. (9.49) and (9.52). we have

(9.53)

l'or the subla1ninate subjected to transverse shear stresses resulting in Yy::. and
Y.~: \vi th
all other strains being zero. Eq. (9.50) beconH!S

(9.54)

To satisfy 1his equation. the preceding elemenLSof the eompliance matrix must
be zero as follO\\'S:

Ju
h ' "
1,.l [ool 0 0 (9.55)
[ J,, '" = 0 0 .
J,111 l ns 0 0
9.4 SUBLAMINATE 407

Table 9.1. The equations used for calculadng the nonzero


elements of the ( JJ compliance matrix
E11U1tion oomben
e9.24 9.24 9.45
----
9,24
9.24 9.24 9.45 9.24
9.29 9.29 9.% 9.29
[1'1,,..' ... '"]
.
) 1i4
~
9.SJ 9.53
9.53 9S3
,9.24 9.24 9.45 9.24

9.4.4 Step 4. The Stiffness Matrix


By combining the results or the preceding three steps. we obtain the following
strain-s~ relationship:

' Ju Ju Ju 0
0
0
0
J,.
J,. "
"' Ju J.,,
fy 'iir
f,. J31 J,, Jn 0 0 J,. 'ii,
= (9.56)
Yy;. 0 0 0 J., Ju 0 r,.,
Y.,. 0 0 0 J,. J" 0 f..-:
y)' 1., J., J., 0 0 J.., r..,
IJl l t1"

where IJJ is the con1pliru1ce 1nntrix of the 5-ublaminatc. 111e stiffness 1natrix is

IEJ= llr ' . (9.57)

We note that both [J J and IEJ arc symmetrical. The equations to be tL<ed for
calculating the ele111ents of(J I arc sum1nnrizcd in Tnble 9.1.
Under planestress condition. the strain....,strcss rclntionship is

(9.58)

Therefore. under plane-stress condition we need to determine o nly those elements


that appear in this expression.

9.1 Example. Calculolr 1/rr stiffntss 11ft1tri.r (JI o/" s11bll11nina1e 111tule of graphite
tpoxy wlidirec1io11a/ plies.. Tiit 111attrial properrits art givtn i11 Table 3.6 (page 81).
111' layup is [01/ 451/(),/451) (Fig. 9.8). 71,. 1illck11.s1 of th s11blami11ate is /1, =
0.0008111.
408 FINITE ELEMENT ANALYSIS

.z = 0.2mm z~= 0.4 mm

I
4 5i.
3

h,= 0.8 mm r'--~----:o~~~-1--.--~~~=_o:..__,__._


45 ~ z1=- 0.2 mm
0 .zo=- 0.4 mnt
Figure 9.8: 1'be sublaminatc in Example 9.1.

Solution. First. \Ve determine the stiffuess and con1pliance n1atrices. The stiffness
matrices (QJ" aod [Q]'5 for the plies are given by Eq~ (3.49) and (3.52) as follows:

-.
[Q] =
[148.87 2.9 1
~.91 9.71
0 4.55
~
] 10 :,
[ 4565
36.55 34.79]
[Q]" = 36.55 45.65 34.79 JO9 m'
34.79 34.79 38.19
N

(9.59)

The compliance matrix for the 0-degree ply is (Eq. 2.224)

6.76 - 2.03 - 2.03 0 0 0


- 2.03 103.63 - 62.18 0 0 0
,
- 2.03 - 62.18 103.63 0 0 0 10- 12 01- .
[SJ= [SI"= 0 0 0 331.61 0 0 N
0 0 0 0 219.78 0
0 0 0 0 0 219.78
(9.60)

The compliance matrix for the 45-degree ply is (Eq. 2.194. IS' J is replaced by
(SJ) as follows:

[S)4' = [f:J [SJ[f.iJ- 1 . (9.61)

The transfomiation matrices [ f%J and [ fi[ are given in Tables 2.15 (page 51)
and 2.16 (page 53). The angle is 45~ hence. we have Cr = cos 45 = Sr = sin 45 =
0.707. The elements of t he [SJ" matrix are

81.53 - 28.36 - 32.10 0 0 - 48.44


- 28.36 81.53 - 32.10 0 0 - 48.44
,
- 32.10 - 32.10 103.63 0 0 60.15 10- n m- .
ISJ4 ' = 0 0 0 275.69 - 55.91 0 N
0 0 0 - 55.91 275.69 0
- 48.44 - 48.44 60.15 0 0 11 4.44
(9.62)
9.4 SUBLAMINATE 409

The stiffness matrices a re ([CJ = rsr')

152.47 7.46 7.46 0 0 0


7.46 15.44 9.41 0 0 0
7.46 9.41 15.44 0 0 0 109 ..!:!._
[CJ" = (9.63)
0 0 0 3.0 16 0 0 m2
0 0 0 0 4.55 0
0 0 0 0 0 4.55

50.26 41.1 6 8.43 0 0 34.26


41.16 50.26 8.43 0 0 34.26
8.43 8.43 15.44 0 0 - 0.98 N
[CJ"= 0 0 0 3.783 0.767 0
10 - (9.M)
n>'
0 0 0 0.767 3.783 ()

34.26 34.26 - 0.98 0 0 38.25

The stiffness matrix [A] is (Table 3.9, page 86)

15.79 13.92] N
[ 7781
[AJ = 15.79 22.14 13.92 to'-. (9.65)
13.92 13.92 17.10 m

We now proceed to determine the eleme nts of the [1] matrix. Equation (9.24)
yields

J11 J,.] [ 12.38 - 5.10 - 5.92] 2


Jn J,,, = 1r, [Ar ' = - 5.10 76.08 - 57.78 10- 12 ~.
J,2 J.. - 5.92 - 57.78 98.65 N
(9.66)

From Eq. (9.29) we have

K.
[h 1 J31 / ,.] = z)
k I
[S1. s,, S.,.Jk(z.. - z..- 1)[QJ.llAr1, (9.67)

where K, is the tota l number of ply groups ( K, = 4) a nd t k is the d istance from the
midpla ne to the ply (Fig. 9.8). Substit ution of the nume rical value.s into Eq. (9.67)
yields

2
[J.11 J32 .r,.j = l- 3.608 - 43.162 39.554] x 10- 11 ~ . (9.68)
410 FINITE ELEMENT ANALYSIS

Equation (9.45) is

J16] 1 L ({C"}
~23 uc~:;: )
[JIJ] = - ['"
1
111 K,
113 121 121 }y, ,,,
16] 161 161 166 t-1 c63 k

- 3.608] 12 m-,
= - 43.162 10- - (9.69)
[ 39.554 N

Equation (9.46) gi"es

,
- 12 n1-
= 92.80 x 10 iT (9.70)

Equation (9.53) yields

['" ] 45 ]
J., 1.S..'. = I1 L - '->[S"
K, (
f" k-1
s 4S
s.,]
Sss
)k
(9.71)

2
= [303.65 - 27.96)10 _12 m (9.72)
- 27.96 247.74 N
The [J J matrix is
12.38 - 5.10 - 3.61 0 0 - 5.92
- 5.10 76.08 - 43.16 0 0 - 57.78
,
- 3.61 - 43.16 92.80 0 0 39.55 10- 12 !!!:..
[J) = 0
0 0 303.65 - 27.96 0 N
0 0 0 - 27.96 247.74 0
- 5.92 - 57.78 39.55 0 0 98.65
(9.73)
The stiffness n1atrix o( the sublaminate is
101.35 24.32 7.95 0 0 17.1 4
24.32 32.84 8.92 0 0 17.11
7.95 8.92 15.44 0 0 - 0.489
ie1 =[Jr ' = 10 -N
0 0 0 3.33 0.376 0 m'
0 0 0 0.376 4.08 0
17.14 17.11 - 0.489 0 0 21.39
(9.74)
CHAPTER TEN

Failure Criteria

Failure of fiberreinforced composites may be caused by fiber buckling, fiber


breakage, n1atrix cracking. delamina tion, or by a co1ubination of these fac to rs
(Hg. 10.1 ). Local fiber buckling, or microbuckling. reduces tl1e compressive stifl
ness and stre ngth o( lhe la1ninate. ~1icrobu ckling does not necessarily lead to
immediate faiJ ure because the surro unding matrix supports t he fibers. The prop
erties of t he fibers and the n1atrix greatly affect t he onset and n1agnitude of fiber
buckling a nd the resulting losses in the compressive prope rties of the lan1inate.
One of the n1ain roles of the fibe rs is to carry te nsile loads. \Vhen d ry fibers
(\vit h no matrix surrounding t he n1) break, they, of course , can no longer carry ten-
sile loads. \ Vhen t he fibers are embedded in a matrix, the ma trix acts as a bridge
about the break a nd transn1its the load across the gap created by the breakage as
well from the bro ke n to the adjacent fibers. Fiber bridging, as this phenome non
is called, is the main reason that the tensile stre ngths of unidirectional, continu-
ous fiber-reinforced co n1posites are higher tha n t he tensile stre ngths of dry fiber
bundles.
Ma trix cracking freque ntly occurs in composite la n1ina tes. ln itself, ma trix
cracking generally does no t result in ultima te fail ure of a laminate. Nonetheless.
n1atrix cracks have n1any detrin1ental effects: they facilitate n1oisture absorbtion.
reduce the matrb::-dominated stiffnesses or the lamin ate and, last but not least, n1ay
propagate into the interface be t\veen adjacent layers., init iating delamination.
Delamination is a separation of adjacent layers that may be introduced either
during n1anufactureor subsequently by loads applied to the la minate. For exa1nple,
loads d ue to transverse impact by an object on the la1nina te are a fre.q uent cause
o( delamination. De.l a1nination reduces the beading stiffness and strength as \veil
as the load ca.rrying capabiJity of the la n1inate under oon1pression. Significa ntly.
under repeated loading the size of the delan1ination n1ay increase to a critical point.
Like the beha\~or of a crack in metal, o nce the critical size is reached, the growth
o( the delamination becon1es unstable. leading to a rapid loss of compressive
strength.

411
412 FAILURE CRITERIA

Fibc:--r buc.kling Fiber breakage Matrix cracking OcJamjnation

!!! i ll
.........
.......~
1111
........
t!

...::o
1I1
lll !!!
Figure J0.1: 1yPical failure modes of composile:>.

Designers would be well served by mechanism-based (physical) theories that


'vould indic.ate the load at 'vhich failure occurs as \\ell as the mode of failure.
AJthougb such theories have been proposed, 1 2 none is as yet at the stage \Vbere
it could be applied in practical e ngineering design. Inste ad, freq uently, ply- stress-
based failu re theories a re used.3 4 According to t hese theories the criterion for
(ailure in any one of the plies is

no failure
I (rJ1. m, m , ri, , ru, ""Fi. F1 . .. .) { : : failure limit . (10.1)
> I failure

\\here a 1 . , r 12 are the stresses in the ply and F1. Fi . .. . are stre ngth paran1eters.
Thecriterion expressed by Eq. (10. t) is esta blished in every ply. a nd failure is taken
to occur when a ny o ne of the plies fails (first-ply failure).
Here, \Ve present three failure crite ria for composites based on the aforen1en
tioned conce pt: the quadratic, the n1axin1um stress. a nd the 1uaximun1strain failure
c riteria. These crite ria offer results that are sufficiently accurate for many (but by
no n1eans a ll) proble ms of practical interest. For this reason, in s pite of their short
comings, they a re relevant to the e ngineer. Nonetheless, the reader is \\'arned to
be cognizant of the folJo,ving significant lin1its of the crite ria listed above:

Each criterion provides o nly the load a t which firstply failure occurs. that is.
the load at which the linear load-displacement curve first changes (Fig. 10.2).
Under the load set t ha t causes firs tply fa ilure, the la minate does no t necessarily
fail because other undamaged plies ca n still carry load. As the applied loads

1 R. F. Gibron. Priucipl1'..1 of Co1n1>0.tilt Alu1t rial J\1eclla,1ics. ~lcGrnwHill, New York, 1994, pp. 114-
126. 244-249. and 3~J67.
? S. R. 5...,anson.Adw111ced CtnllJ)(Jjl(t Af111trial.t. PrentkcHntl. lJpper Snddle Rjvcr, Ne.w krscy. 1997.
pp. 91-120. 123-147.
~ R. E. Rowland:;, S1tengtb (Failure) Theorie:; and The ir Experimental Correlation. Jn: Hd11dbt>0k 11f
Ctn1Jpositt>-.s, Vol. .3. G. C. Sih and A. !\i. Skudra. eek. Elsevier. Anl:;tcrda1n . 1985, pp. 7 1- L2'i.
-' 1'1. N. Nahas.Survey of Failure nnd Po:;t-Failure Theorie:; of Lami.na1ed Fiber Re inforced Compo:;i1es.
Jountdf t,/ Co111posit~3 Ttt//unlog) a1ul Rtsc-4uc'J1. Vol. 8, L38-15.3, J986.
10.1 QUADRATIC FAILURE CRITl:RION 413

-.
Pirillply fu1lurc

-o-
- -
<
- -
<- -
->

Figure 10.2: Load-dJJplaccmcnt curve or a compot;itc part.

are increased beyond those 31 which first-ply failure occurs. there will be a
sequence of ply failures until the load set is reached at which e>uy ply has
failed. The loads at ultimote failure may be coruiderably higher than al first-
ply failure. Thererore. criteria based on first-ply failure are conservative.
None or the criteria sheds light on the failure mechanism or indicates th.e mode
or failure.
Nooe of the criteria provides acceptable results for e'erycondition of practical
interest.
Each cri1erion requires data. some of which arc diffirult to measure.
Each criterion applies in regions in.side the composite av.ay from discontinu-
ities such as holes., cracks. and edge'- (Criterfa applicable to plates containing
a hole or a notch are ghcn in Section 10.4.)

10.1 Quadratic Fallure Criterion


The quadratic failure c riterion includes st resses up to the second power. In its
n1os1 general fo rn1 the quadratic failure crite rion s tntes that no fa ilure occ.urs
\vhen t he inequality below ( Eq. 10.2) is snlisflcd. Thi~ crite rion :.ind some of its
sin1plifie d forms a re vt1riously refe rred to as T.lini.\Vu, Hill, or Tsai- Hill (ailure
criterion.

F1a1 + Fzoz + F.1<13 + P4 rz:i


+ F, r1l + F,,r12 +
Fuaf + Fuaf + FJ.1a} + f44rb + F,~rf1 + F",,rfi +
2( F12a10'2+ F130'1<13 + Fua1 rZJ + Fa~<11 t1;1 + F1oa1 T12 +
FlJazO'J + Fi.iazru + Fz.1a2r13 + F11102r12 + f'.14olrl.1 +
FJSO'Jn1 + fJ611Jr12 + f.urzJ nJ + F.ioruru + F5an1r1.?) < I . (10.2)

\\'here a 1 a 2 , . r 12 are the components (in the .t 1 .\'1, x1 coordinate system) of


the stress at the point or interest. that is. the stress that resuhs from the applied
loads. and the Fs are. strength p:arnmctcrs thnt depend on the material. No failure
occurs when the left-hand side of Eq. ( 10.2) is le.s then unity. This means that the
resultant str= is inside the failure surface (Fig. 10.3. left). On the failure surface
(Fig. 10.3. middle). "here the stress components arc deno1ed by af. aJ...... rf1 .
414 FAILURE CRITERIA

No faj lure failure failure

a, -~ I
I

..,
I

Figure 10.3: Representation of the failure. surface whc.n only a1 and '1! stresses are applic.d.

Eq. (10.2) is

Fictf + Fiai + f,u1 + F4rb + F.~rf~ + F6rf2 + Fu (0'1~


2
+
F22 (aD' + p_,, (11_:} + F., {ri3) + Fss ( rf3) +
2 2 2

F.,,(rf,}' + 2(F12a[al + Fua[aJ + .. + F,.rf3 rf2) = 1. (10.3)


The strength paramete rs must be de tem1ined by tests. For gene.rally anisotropic
and n1onoclinic. n1aterials, 27 and 17 types of tests are require d, respectively. This
makes the use of the quadratic (ailure criterion impractical for structures n1ade
o( generally anisotropic or n1onoclinic ma terials. The criterion becon1es more
m anageable \\hen the material is o rtho tropic or transversely isotropic. Therefore,
in the (oLJo,ving the criterion is prese nted only for these l'No types o( n1ateria ls.

10.1.1 Orthotropic Material


An orthotropic material has three planes of symmetry (Figs. 2.1 I and 2.12). We
select the x1, xi , XJ coordinate system with a..xes perpendicular to these symn1etry
planes.
First, \Ve conside r only the shear-stress component rz.\ acting in the plane of
syn1n1etry (Fig. 10.4). \\'hen only t'23 acts, and it is in the positive direction, then

Plane of s ymn1c-try
Figure 10.4: The-positive and negah\'C shear strcs.o;cs at (ailure acting in an orthotrop1c material;
x 1 x 2. and XJ are perpendicular to the ortbotropy planes.
10.1 QUADRATIC FAILURE CRITERION 415

al [ailure (ru = r[i) the quadratic failure criterioo yields (Eq. 10.3)
2
F4rlj + F"" (rl_T) =1. ( 10.4)

When only r.u acts in the negative direction. then. at failure (r11 = - r;J) the
quadratic failure criterion becomes (Eq. 10.3)

(105)

Because of symmetry. 1he failure stress for positive shear is the same as for
negative shear (rl,t = ril )- The t\\'o prece.d ingequationss.atisfy thiscondi1iononJy
if f 4 is zero.
By sin1ilar argument it can be shown that F5 , Fti are zero. Thus., \\'C have

F, =F, = F =0. (10.6)

Next, \VC apply the normal stresses a 1 "i a 3 ; the shear stresses r 12. ru; and
either a posilivc or a negative shear stress r23 (fig. J0.5). For a po:;itivc shear stress
at failure. ru = r:[.'. and we have (Eq. 10.3)
2 2
F,a[ + F,o: + F,<Tl + Fn (<7[} + Fu (a:} +
2 2
Fll(ul} + F.,(r).l"J' + Fss(r1,)' + F.,.(r/2 ) +
2 (Fua:aJ + Fua-:al + ) + +
+2 (F140: + Fua: + FMal + F.asr:i + F<l6r~J r;r::: 1. (10.7)

For a negative: shear stress at failure, m = -rlJ. and Eq. (10.3) gives

F,o/ + Fiol + F,u~ + Fu (a/)'+ F,, (ai) +


2

1
F.u(aj)' + F.,(rJj ) + F.u (r:,J' + F,.(r/,}' +
1
2 (F12tt1oJ + F11a"taJ + ) + +
-2 (F141u11+ Futtl + F:\4ctl + f4srf3 + F40Tfz) ril = l. ( L0.8)

Because or sym1n etry. the failure stress for positive OUlOf-plnne shear is the
same as for negative outof-plane shear(t'it = rlJ). TI1is condition, together \Vith

l}lunc of iymntcll)'
J:jgurc JO.S: 'lhc: 11tl'Cli!iC$ I (nilurc ac-l ing o n an orthutropic n1a1cdal.
41ti tAJLUKt liHlltHIA

the two preceding equations (Eqs. 10.7 a nd 10.8), give

F,. = F,. = F" = F., = F,,,, = 0. (10.9)

By applying the procedure jus t given to the o the r t\1i'O sy1nmetry planes \Ve find
that the (olJo,ving s tre ngth parame ters are a lso zero:

Fis = Fis = =
FJs F4s = Fsti = 0
(10.10)
F16 = Fu, = F.u, = F4f> = F,~ = 0.
For an orthotropic n1aterial the q uadra tic failure criterion (Eq. 10.2) becon1es

Fla1 + Fz<12
+ flO'J+
F1 1a~ + F2zO'f + F.1.,af + F44rb + F.isrf3 + F66rf1 +
2( F 120'ia2 + F13a1a., + F23<t2u3) < l . (10.11)

At fa ilure, where t he stress oon1ponents are design ated by t he superscript f,


Eq. (10.11) is

F,a[ + F1cr~ + F3a~ +


2
F11 (<1f} + F21 (a:J' +
l'n (aj)' + F,... (r!_,)' + Fss (<[,)' + F,. (rf2 ) 2 +
2(F11t1fo'l + fuO'fn.f + f23afn.f) = 1. (10.12)

1\'or1i11teractio11 strength para1neters. The noninte raction stre ngth paran1eters


are deno te d by F1 , F2 , F.l, F 11 , Fu, F:u, Fa.t, F55 , F{,6. The values of these para n1eters
are obtained fron1 uniaxial a nd from shear tests.
To obta in Fi a nd F11 v.e s ubject the material to uni axial tension and compres-
sion in the x 1 orthotropy d irection (Table 10.1 , top left a nd middle left). At fail-
ure, the s uesses a re":= a ndst ar -s = ~' \\heres is the strength of the n1aterial
and the s upe rscripts ( + ) and (- ) re fer to tension and 001npression. \Vith these
stresses, t he quadratic fa ilure criterion gives (Eq. 10:12)

(J0.13)

Solution of these two equa tions yields


I I
Fi = - - - (J0.14)
sf sj'
The s tre ngth parame ters Fi, F3 , Fn, F.u a re o bta ined in a. sin1ilar n1anner and
are gi\'en in Table 10.2. The tests to detem1ine sl.sl. s.t.s; are illustrated in
Table 10.1.
To obtain the s tre ngth paran1eter FM we subjec.t the ma te rial to s hear t'23 in the
.r1 - .r1 orthotropy pla ne (T.1ble 10.1, bollom lert). Because of material symmetry
the fa ilure s tress is independent o f the direction of the shear stress. a nd a t failure
r, = S23( = siJ = sij). Wit h this stress. Eq. (10.12) gives

F., (sn)2 = I. (JO.JS)


10.1 QUADRATIC FAILURE CRlnRION 417

Table 10.1. Tests to determine the strenotlls of orthotroplc mate~aJs; ,,., , ""
ua are perpendicular to the planes of orthotropy.

., i

.. :
?~
i..... ":

: - -"m

. .. ~ ..
-u @ .
.
.....
.. .. ' ... _ ,,,
_,,,

which results in
1
F44= - - (10.16)
(su l '

The strength paran1eters F."s f(,6 are obtnincd in a .sin1ilar 1nanner. These, as
\veil as the other noninte raction s tre ngth porametcrs.. are given in Table L0.2.

J11teracl1'011 strength para111aters. T he interaction strength para1ueters F 11. F1.h


and F23 can be detern1ined fro1n tests that result in t 1No or more nonzero stress
oon1pone nts inside the n1aterial. O ff.axis uniaxi;,l tests nnd biaxial tests offe r pos-
sible n1eans for detern1ining the interact ion &trcngth parameters..

Offa.'\:is uniaxial tests. When the interaction s tre ngth pnrnmeters of o rthotTopic
materials are to be de lermined by of(.nxis tests,. \\ft rake test coupons fro1n each
(x 1- xz, x2- x3 , and x 1- x3 ) orthotropy plone (Fig. 10.6).

Table ltl.2, The nonlnteraction strenoth parameters


in tenns of streng1hs

F.= ~--!:
Fu = ~
' .
' '
F...,=-'-
"'.,,tl
418 FAILURE CRITERIA

x,

Coupon 3 Coupon 2

e,

(\
e~
:&

I Coupon I

Figure J0.6: Test coupons in lhe .t1-x2. x1-x.,;. x1-x; o rtholropy planes.

Coupon l , take n from lhe x1- x1 orthotropy plane, is subjected e ither to an


axial tensile or to an axial con1pressive stress. A t failure the stress is design ated
by p0 Superscript 1 ind icates Coupon I. The corresponding stresses in the x 1 x 1
coordinate system are (Eq. 2.182)

rn = p'' cose , sin t:J, . (10.17)

The angle e, is s h O\VO in Figure 10.6. Substitution o f the preceding stresses


into Eq. (10.12) results in the expression
1
/ ( F1cos2 e,
+ Fisin2 f >i) +
(pn)2(F11 cos't~, + F22 sin4 fl 1 + F66 cos'e 1sin2 H 1) +
(pn) 22F12 cos 2 l~1sin 2 fl1 = L (10.18)

This equation can be solved for F11 The result is given in Table 10.3. The F13 ,
F13 interaction stTe ngth parame te rs a re obtained in a sin1ilar n1anner and ar e-also
included in Table 10.3.

Biaxial t ests. When the interaction strength para n1eter F11 is to be detern1ined
from bia xia l tests, the specimen is loaded in biaxial tension, resulting in stresses
cr1 a nd a 2 (Fig. 10.7). The load is then increased pro po rtionally such that the ra tio
o( the t\vo stresses re mains constant. At fail ure the stres:ses are de noted by

(10.19)

Table 10.3. The interaction strenglh parameters oblained lrom uniaxial tts1s
(orthotropic material). The angles e,, e,, and 0 3 are shown in Figure 10.6.

Fi 1cos'91 - F22 sin"' 9 1) - F~


1
F22 cos' S2 - Fj3sin" 92) - :

F-.1J. COs'S.i - F11Sin" e .i ) - F;!


10.1 QUADRATIC FAILURE CRITERION 419

Figure I0.7: Tes.t co upon for bia xia l testing ln t he .\1-x 2


ortho tropy pl<lne.

'TI1e s uperscript ( re fers to t he stresses at (aiJure., and the supe rscript 1- 2.identi
fies the applied biaxial s tress co mpone nts in the x 1- x 2 o rthotropy plane. By substi
luting l1:o . .
z) and cr;l l) into Eq. (10.12), and by setting a ll other stress components
equal to zero. we o btain

F1 ":(J ..2) + F 2ct2ftl- l > + F 11 (11f(1 ...2, )2 + F22("'1(1-1))2 + 2F12cr:l1...2)u1<1-2) = l.


( 10.20)

E quation ( 10.20) results in

( 10.2 1)

The other interaction stre ngth paran1eters. obtained in a sin1ilar manne r, a re

1 _ p..1<1;tc;- 1) _ F,a .V-0 _ F.11((fi1<0)' _ pJI( V-0)'


(f .
F.; = i _ c(i ..j) r(i..i) i , j = 1. 2, 3.
UJ; ";
(1 0.22)

Appn>ximate expression.~ for the interaction strength parameters. In practice, it


is difficult to pe rfo rn1 the tests ne eded to gene ra te the interaction s tre ngth para m
eters F;; . To e limin ate the need fo r thes e tests, nume rous a pproxima te expressions
have been proposed (or F;;. One of thes e expressions is obtaine d by observing that
a quadra tic e quatio n is cha racte rized by its d iscrin1inants. 'Vhen all but an y t'vo of
the norma l stresses are zero in the failure criterion (Eq. 10.12), the discrin1inants
are

t:.;; = F;; F1; - F;} i, j = 1, 2. 3 i # j. (10.23)

f o r the q uadratic equation to re present a closed domain (\\hich in our case


is necessary to e ns ure that the stresses ren1a in finite ), eve ry d is crin1in ant must be
positive. T hus. from Eq. (10.23) we have

(10.24)

f o r convenience. \\'e \Vrite

( 10.25)
l:;-tgure J0.8: S tresses in t he. xz-.l) isotrop)' plane of a tra ns \'Cr.>el) isotropic material

f;; are constants tha t may have any va lue \Vit hin - 1 and + l . Tsai and Hahn 5
observed t ha t the q uadra tic failure criterion (or composites reduces to the von
Mises quadratic failure criterion for isotropic ma te ria ls (Section 10.1.3) when
fi; = - 0.5. With this value the interaction stre ngth parame ters becon1e

Fu = - ~/F11f'u Fu= - ~/F11F_,_,. (10.26)

10.1.2 Transversely Isotropic Material


A tra nsversely isotropic material has three planes of symmetry (Fig. 2.15, page 19).
There-fore, the stress pa ra me ters, 'vhich a re zero for an orthotropic material (which
also has three pla nes of symn1etry), a re a lso ze.r o for transversely isotropic n1ate
ria ls (Eqs. 10.9 a nd 10.10). In a ddition, o ne o f the symme try planes is isouopic.
We select this to be t he x2- x1 pla ne (Fig. 2.15). Beca use of isotropy, in this plane
the subscripts 2 and 3 are inte rchangeable, a nd " 'e can " 'rite

F, = F, Fu= F'.1J (10.27)

For the same reason s 12 = s 13 , and from Table 10.2 (page 417) we have
(10.28)

\Ve OO\V a pply te nsile 01 a nd con1pressive cr3 stresses o( e qual magnitude


(.r2 = - <r3) (Fig. 10.8, top le ft). At failure,"'= cr 1 " ' = - 1
<1 a nd Eq. (10.12)
yields
(10.29)

By combining Eqs. (1 0.27)- (10.29), we obtain


1
(<Yr)'= ~~~~ (10.30)
2(f22 - Fn)

5 S. \V. T$ai and H. T. Hahn. burmlucriori u1CO,,JJH'1ite Afa1erial.~. Tcchnontic. Lnnccstcr. Pcnnsytvania.
1980. p. 286.
IU. 1 UUHUftAlll.. rAILUftlC 1..n,1 1cn1un .,,
Next. we apply only the shear stress r 23 {Fig. 10.8. bottom left). At fnilure the
vaJue of lhc s hear stress is ri.1 = tfi. Equation (10.12) noy, yie lds

' ' = l.
F.., ( r..,) ( 10.3 1)

We evalualt F., by observing that {a) the biaxial loading (Fig. 10.8. lop left) is
equivalent to a shear loading r"' in the 45-degree direction (Fig. 10.8. top right).
and (b) in the x,-x,
isotropy plane the magnitude of the failure shear stress is
the same in cerydirection (Fig. 10.8, bottom). Consequently. at fail ure. the sh ear
stress is rf, ="' By substituting "' for rl_. in Eq. (10.31) and by using Eq. ( 10.30),
Y.' e obtain

F" = 2(Fn - Fu). ( I0.32)

With these sim1.,lifications. for a uansverse ly isotropic 111rite rinl the qundratic
failure crite rion (Eq. 10.11 ) becomes

Fr 111 + F, (<1? + O'J) + Fu,,.~ + F,, (,,.f + aJ)


+2 (F22 - F1J) rb + Fu. (rf1 + rfz) +
2 ft 2 (al al+ a1a1) + 2f23<1za3 < 1. (I0.33)

The nonintcraction strength paramete-r s in terms of the strengths are summa


rized in Tobie 10.2 (page 4 17); the interaction strength parameters are given by
Eq. (10.26). The strength parameter.; depend on the strengths. The tests for de-
termining the required strengths st. sJ. s;. s2. s 12 are illustrated in Figure 10.9.

10.1.3 Isotropic Malerial

lsotropic 1nntc riols have nn infinite number of sy1nmeuy p lanes.. and 1he stre ngth
parame ters sivcn in Eqs. (10.9) a nd (10.10) are ze ro. In addition. because each
syn11nctry pli\nC is isotropic, the subscripts 1, 2, :ind 3 are interchangeable. This
results in

F11 = Fn = Fl:J Fn = Fu = F12. (10.34)

.,
o1-~"-_,o, -o4~~....:a:
I I

J::jgurc J0.9: rc~t' to dc1crm1nc the slrcnglhs of a transverse I)' iso1roplc m11lcnnl.
422 FAILURE CRITERIA

Sin1ila rly, s 12 = s 13 = s13 , and fr o n 1 Table 10.2 \\'e have

(10.35)

where F44 is given by Eq. (10.32). Wit h the strength parame ters given in Eqs. (10.34)
and (10.35), the failure c riterion (Eq. 10.1 1) becomes

F, (a, + a,+ a.1) + F11 (err+ al+ erf) +


(10.36)

Let us assume no'v t hat t he oon1pressive and te nsile fail ure strengths ar e equal
(si = s~). With this assumption. F1 is zero (Table 10.2)
F, =0. (10.37)

Furthermore, let us approximate the interaction strength paran1eter by the ex


pression given in Eq. (10.26) as follows:

(10.38)

By substituting Eqs. (10.38) and ( 10.37) into Eq. (10.36) a nd by noting that
F11 = l/(stJ 2, we obtain

(10.39)

T his is identical to the von rvlises quadratic failure crite rion.6

10.1.4 Plane-Strain and Plane-Stress Conditions


Under plane-strain condition the equations and the paran1eters given in Sec
tions 10.1.1- 10.1.3 apply.
Underpla nestresscondition (\vhe re in tl1ex1- X2 planer:13 = 0, t21=0,1'13 = 0.
see Fig. 2.28) the quadratic failure criterion is simplified . For o rt hotropic materials
and (or transversely isotropic n1atelials with X1 no rmal to the plane o( isotropy,
Eqs. (10.12) and (10.33) become

(10.40)

For isotropic materials, Eq. (10.36) reduces to

F, (er1 + <11) + Fu (af + erf) + 2(F11 - F,,)rf, + 2F12er1<r2 < l. (10.41)

The strength paran1eters a nd the strengths re quired (or t he quadratic failure


c riteria are sum1narized in Tables 10.4 and 10.5.

6 N. E. Dowling. Alecha11iral Btlu1vloro[,i.taterials. PrentiocHall. Englewood Oiffs. New Jc.rscy. 1993.


p.245.
IU.I UUAUKAllli tAILUHt t;H.ll t K IUN

Table 10.4. The sttength parameters appearing In Ille quadratic !allure criterion for an
orthotropic (Eq. 10.11). transvl!fsely lsottopic (Eq. 10.33), and Isotropic material (Eq. 10.36).
Thtee-dmensianal stresses or pla.1stram condition
Orthotrope Fl Fu Fn Fn F.u F" F.. Fu Fu Fn
Ttant\>crscly F11 Fn F.. Fu Fn
isotropic
Isotropic Fn
Plane-stress condition
OrlbMIO))H: or F1 IS
transvtntly i~H ropic
)SOlt'f>J)iC F1 Fn Fu

10.1.5 Proportional Loading - Stress Ratio


\Ve consider a par1 s ubjected to :i load set L i, L 2...... l.,. Let us inc-re ase (or
decrcru;e) t he loods proportionally by a factor o f Runtil foilurc occurs (Fig. 10. I 0):

t.:r1 = Rl..1
- I!,,= RLi .. .. 4 = RL. (10.42)

\Vben the part bcha\'CS in a linearly elastic manner. the stresses also increase (or
decrease) proportionally. Consequently. each sttess component increases (or de
creases) by the same proponion until failure occurs. This proc:cdurc is expressed as
I - I -
0'1 = Roi rll = Rr!l
o2I= Ro1 - f
r 13 -
= Rr1.1 ( 10.43)

"J = Ra.l r 1r2 = Rr1


- 2.

111e s upc rsc:ript f refers to the s tress components on the fa ilure surCacc, a nd R is
the fncto r (cnllcdstrcss ratio) by which each load (aod corrcspondinglycach stress
con1poncnl) is multiplied suc.h that the failure surCacc is rcnchcd; ii is illu.str:ited
in Figure IO. l I ror linearly e lastic materials. No failu re occurs whe n R> I , a nd

Table 10.5. The s!Jengths required tor evaluating the noninteraction slfength
parameters (see Tables 10.2and10.4)
Thte1>dlmensional stresses or plme-strain condition
Ot1hotropte s[ sf st s;- s; s; .iu so IV
Ttan\:Vtrscly si sj s; s; su
i.souoptc
Isotropic si s;
Plane-stress condition
Or1ho1ro1,ic or st sj si s;
tran:svers..:I)' iwtropic '"
lsot.ropic -l't s j'
rktLunc \tn11 cn1R

Figure 10. 10: Proportional loading..

failure oc.curs when R5 l:

no failure
(10.44)
failure.

By substituting Eq. (10.43) into Eq. (10.12). we observe that the quadratic
failure criterion (or orthotropic n1aterials becon1es

(10.45)

\Vbether or not failure occurs is indicated by the value of the stress ratio; ii. is
gi,.en by the solution of Eq. (10.45) as follows:

-
R=
- b+/IJ + 4a .
(10.46)
2<1

Only the positive sign is applied in fron t of the square root because R, by defi-
nition, is posit ive. The coefficients a and bare given in Table 10.6 for ortho tropic,
transversely isotropic, and isotropic materials.
Under plane--strain condition the expressions in Thble 10.6 apply.
Under plane-stress condition. in t he x1- x1 plane. '1j, r2 h t"JJ are zero (a3 = 0.
rn = 0, and Tu = 0, Fig. 2.28). By setting t hese stresses equal to zero in Table 10.6,
've obtain the a and b paran1eters listed in Table 10.7.

R.> I R=I fi. <. I


r <1t ----------- "
" (
<12 --------I q I
fl

' '
''' '''
' '>
-;__--~.-'-:

Figure JO.I I: Representation o( the failure. surface when o nly 111 a nd t1l stresses a re applied
(R= a:/a1 = t1J/ t11).
10.2 ' MAXIMUM STRESS" FAILURE CRITERION 425

Table 10.8. The slless ratio ii


Material b
Or-thotropic Futrf + FzzoJ + Fna]
+ F.uri_, + &rf, + F66 r~
+ 2( Fua1o! --r Fuato3 + F23a-z0:t)
Fuof + Fu(af +of)
+ 2(F?? - Fn)T~ + F~(ri + rf1)
+ 2Fu(1T10"2 + 0 10,) + 2Fi.taia,
I.so tropic Fu(of + aJ+aJ)
+ 2(Fu - Fu)(rb + rf3 + rfz)
+ 2F11(a1o:z + 01aJ + aza))
R _,,....,,,!bifZ
"'
10.2 "Maximum Stress" Failure Criterion
Frequently. for orthotropic materials and for transversely isotropic materials un-
der plane-stress condition. a se><alJe-d ma."<imum stress" criterion is used. ln ap-
plying this criterion we use the x 1 , :r1 . x1 coordinate system nligned with the direc-
tion o( orthotropy (Fig. 2. I 2, left). The normal stresses in the .t 1 x1 XJ directions
and the shear stresses in the x 1 -x 1 . x1 - xi. :c1 -x 1 pl.ane.s are compared with 1he cor-
responding strengths. Failure does not occur if none of the stresses exceeds the
strengths..
For an or1houopic m;uerial, no failure occurs '''h en all of the follo,ving condi
tions are met-:
-sj < 01 < .r: lti.11 < S21
-.rz< oz< st lt13 I < S13 (10.47}
-.r~~ < tr) < sj lt12J < S 1z .
For a transversely isotropic 1nate rial under plane~stress condition, no failure
occurs \vhen nll of the follo\\ing conditions are mel:
(10.48)

Table 10.7. The s1J1!Ss ratio ii under plane-stress condillon


Material , b
Orlholropic or Funl + F...:a} + Ft1trfi_
transven.cly + 2Fi:aia?
iSOH'OJ>M:
Isotropic
4lti 1-AILUHt liHlltHIA

\vhere st and si are the tensile and cotnpressive strengths in the i direction and
s~ are the she ar strengths in the i j (i. j
= 1, 2. 3) plane (Table 10.1, page 417).
T\\ 0 in1portant points are to be made he re as follows:
1

I. The "maxin1un1 stress criterion d iffe rs fron1 the quadratic fail ure criterion.
2. The ''maxin1um stress failure crite rion for composites differs fro1n t he maxi
n1um (norn1al or shear) stress criterion used for isotro pic n1aterials. The reason
is th at for isotropic mate ria ls the maximum (nonnal or shear) stress fa ilure cri
te-r ion is based on the maxin1um values o f the stresses a t a point. On the other
hand, for composite 1naterials t he "maxin1um stress failure crite rion is based
on the stress components in the o rt hotropy directions \vhere the stresses are
no t necessarily the highest.

Despite the aforen1e ntioned shortcon1ings, the 111aximun1 stress criterion for
composites (Eqs. 10.47, 10.48) has the advantage that it does not require koowl
edge of the interaction strength para meters.

10.3 " Maximum Strain" Failure Criterion


Frequently, for orthotropic n1ateriaJs a nd for transversely isotropic n1ateriaJs un
d er planestress condition, a so ..called maximum strain criterion is used. In apply
ing this criterion we use the X1, X2 , X3 coordinate syste m aligned \Vit h the direction
o( orthotropy (Fig. 2.12, left). The normal strains in the x1 , x 2 , x 3 directions and
the shear st rains in the x 1- x 2 .x3- x 1 , x1 - x3 pla nes are compared v.1th the corre
sponding maximum allowable strains. Failure does not occur if none of the strains
exceeds the a llo\vable strains.
For an orthotropic n1aterial, no fail ure occurs 'vhen all of the folJo,ving condi
tions are me t:

(10.49)

IY12I < 11~.

For a transverse ly isotropic 111aterial, unde r plane..stress condition no railure


occurs \\'he n all of t he follov.ing co nd itions are me t:

(10.50)

\Vhere 1~t and 1~( are the allowable tensile and con1pressive st rains in the Xi direc-
tion, and 1~~ are the allo\vable she ar strains in the x;- x; (i, j = l , 2. 3) planes..
In general, the strain con1ponents a bove do not necessarily correspond to the
maximun1 strain a t the point. There fore, this is not a t rue n1aximum stra in failure
criterion and is not e quivalent to the n1axin1um strain failure crite rion (or isotropic
10.3 ' MAXIMUM STRAIN" FAILURECRITERION 427

...
4--
-
,, -
figure 10.12: The cyhndcr subJCClcd to 1ntcmaJ ptts.surc
and a:oal load in Example 10.1.

materials, where fo.ilure is based on the maximum value of the s1rain components
at the point.
Furthe nnorc. the " 1naxinlun1 strain.. ( Eq. 10.50) fa ilure criterion above is not
equivnle nt lo the " niaximunl s tress .. fai lure criterion (Eq. 10.48).

JO.J Exarnplc. A cylinder (iuuer radius R ,., = 0.2 1n, a11d 1vall 1/1itkn e.ts /J =
0.002 nr) is lltlltl~ of 1vo11t>11 glas.f epoxy. T/Jt 111aterial properties 11re givt11 iJJ
Tables CJ a11tl C.4 (page 466j. The cylinder is subjected to an t1.xlal load lJ =
500 QCK) N anti to <tll infernal pressure p = 2 000 000 N / 1112 (Fig. 10.12). Detennillt
1Jre stress ratio ii.
Solution~ The radius of the reference surface. chosen as the micburface. is

R = R. + /r/2 = 0.201 m. (1051)

Using the membrane theory of shells., \\.-e determine tb:u 1hc membrane forces
are (Eq. 8.11. with p p,)
fJ N N f
N, = - ;r R = 395 908-m
2
N1 = pR = 402 000-
m
N,,. = z;if- =0.
( 10.52}

The stresses in the \vnll are


,,,, N
a, = -N,1 = 197.95 x lu ?
' m
a1 = !:!J.
t
,.. N
I =201.00 x l <r - ;
m-
( 1053)

N,, 0
r.-,=--;;=.
The wall of the cylinder consists of a single layer. The coordinates of this
layer are x1 .fl, XJ. and they coindde \\~th the x. y. i coordinates o f the cylinder
(Fig. 10.12). Thus. the stresses in the layer are

o 1 = 197.95 >< 10 N,
m-

a, = 201.00 x 10N
-;
m-
(I0.54)

"' = o.
428 FAILURE CRITERIA

With the strengths listed in Table C.4 = 367 MPa, (s:


= 549 MPa, = s; s;
= =
367 MPa, s1 549 MPa . .fll 97 MPa), the required suength parameters are
(Table 10.2, page 417 and Eq. 1026}

p, +1 - -=
= s, 1 - l'
s, =903 x 10 -
1 1 - I'
Fi= -
sf sz = 903 x 10
- - - (10.55)

1 1 r;;-r ,.
F.,, = --, = 106 x 10- ts F12= - zvF11F22= - 2.48x 10- .
(snt
The stress ratio from Table 10.7 (page 425) is

a= Fu<tf + F12af + F66rf2 + 2F12<t1ct2 = 0.1975 (10.56)


b = F1<r1 + p,,,, =0.3604 (10.57)
- - b +/b1 +4a
R= = 1.52 > 1. (10.58)
2(/
'"here R is greater than unity and the cylinder does not fail.
Next, we establish the failure envelope. A t failure the Tsai- Wu criterion is (see
Eqs. 10.40 and 10.3)

(J0.59)

'"here the superscript f refers to the stresses at failure, which are (Eqs. 10.52 and
J0.53)
-r
1
Nx N ...._,
"1 = h = 2tr Rh = 395.9N
N prR
,,4 = --1:.
h = - ,, = 100.5p1 (10.60)

c N.i:v f O
t'r,- = -/-I = "ar R'- I l = .
__ , ,
......
where N and p' are the applied loads at failure. In terms of N and p', Eq. (J0.59)
gives

3.58 x 10- 1 Fi' + 9.08 x 10-p' + 7.78 x 10-"<N')'+


5.01 x 10- "(p')' - 1.97 x 10-1.1 iii',,. = I.
The failure envelope along \vhich the fl' and p 1 values satisfy this condition is
plotted in Figure 10. I 3.
The "maximun1stres.i'' failure criterion is (Eq. 10.47)

- s] < O'J <st (10.61)


-sz < '12 < s;-. (10.62)
10.3 "MAXIMUM STRAIN FAILURE CRITERION 429

"'maximum stress,.

F'tgurc i0.J3: 'fbc .. maximum stre5.'i.- .. maximum strain._


- and the quadrattc (Tsai-Wu) failure
cnvc:.lopes for the cylinder in Example-10.L

The valueof<Y1 =197.95MPais between -s~ = - 549MPaandsi = 367MPa,


and <11 = 20 1 MPa is between - s2 = - 549 MPa and s[ = 367 MPa. Hence, the
cylinder does no t fail.
To establish the failure envelope, we no te that failure occurs 'vben any of the
folJo\ving conditions are met:

-s, = - 549 = a
- r
1 = 395.9N
-.,
s1+ = 367 =a:= 395.9N- r (10.63)
- s2 = - 549 = <1) = JOO.Sp' st = 367 = t1f = 100.5p1

The failure envelope represented by these equations is included in Figure 10.13.


The "maximun1 strain" failure criterion is (Eq. 10.49)

_ ,,, < f1 < tti


(10.64)
- 11; < f2 < flt.

\\C,th the engineering constants 1 = 1 = 29.7 x 109 N/m2 (Table C.3, page 466).
the 1naximum allowable strains are
- s(
q1 = E, = 0.0185
+ si
q1 = E, = 0.0124
,- s+
(10.65)
q' = ~2 = 0.0185 tti = ~2 = 0.0124.

With S11 given in Table 2.7 (page 15), the strain- stress relationships are
(Eq. 2. 132)

:: } = [ - \
{ Y12 0
-1 ~ ]{:: }.
0 cf;; t 12
(10.66)
430 FAILURE CRITERIA

The e ngineering cons tan t" are 1 = 2 = 29.7 x 109 N/012 , G 12 = 5.3 x 109
Nim'. "" = 0.17 (Table C.3). Thus, we have

,, } [ 33.67 - 5.72 0 ] { 197.95 }


,, = - 5.72 33.67 0 x 10- 12 201.00 x 10
{
YI! 0 0 188.68 0

= { ~ ~~ ~~} (10.67)

The value <1 = 0.005 51 is between - 11) = - 0.018 5 a nd it=


0.012 4. and
<z = 0.005 63 is between - 112 = - 0.018 5 and~"[ = 0.012 4. Therefore. the cylin-
der does not fa il.
To establish the failure envelope, we note that fail ure occurs \\/he n any o f the
folJo,ving condilions is n1et:

- 11f = lf r~t = l~
(10.68)
- r +_ r
- 112 = lz 1~1 - fi2

\vhere l { and '~ are the strains at failure, \vhich, in terms of i~r a nd pl, are
(Eqs. 10.66 a nd 10.60)

<l = (13.3N-r - 0.575p') x 10-


(10.69)
.~ = (- 2.21 R' + 3.3&p'J x 10- .
Fro m E qs. (10.65), (10.68), a nd (10.69) we have

- o.0 1ss x 10 = 13.31v' - o.s15p' 0.0 124 x 10 = 13.31v' - o.s15p'


.-r r --r
9
- 0.0 185 x 10 = - 2.27 N + 3.38p 9
0.0 124 x 10 = - 2.27 N + 3.38p.r
(10.70)
The fail ure e nvelope re presente d by these e quations is included in Figure 10.1 3.
Figure 10.13 shows that the failure e nve lopes are markedly different for the
quadratic (Tsai- \''u). the " maximun1 s tress," a nd the "n1aximu111 strain" failure
c riteria. Thus. altho ugh in t his example aJI three fa ilure c riteria indica te that there
is no failure, the re n1ay be conditions under which they give contradictory results.

10.4 Plate with a Hole or a Notch


T he quadra tic, ma xin1um stress. and n1aximun1 strain failure criteria are inappli
cable when failure is initia te d by localized d is turbances such as in1perfections.
c racks. ho les. no tches. o r free edges. lf any of these criteria \Vere evaluated US
ing the s tresses near the distu rbance (v.he re the stresses are high), the criteria
\Vould un<lerestin1ate the fail ure load. On the othe r hand. if the criteria were eva )..
uated using the stresses in the furfield (v.he re the stresses are significan tly lower
than the near field stresses), the criteria v.ould overestin1ate t he failure load . To
10.4 PLATE WITH A HOLE OR A NOTCH 431

;; ;;
t t t t t t t t t t t t t t t t t t

Figure 10.14: Untdirectionally loaded plate contaning a circ.ular hole. and the. ave.rage. stress
dis tritiution along the-x-axis near t he. hole-.

predict the failure in the presence of a distu rbance 've 'vouJd need an understand..
ing of the frac ture beha,~or of the ma te rial a nd appropriate fracture n1ode.ls. To
date, there is no simple, comprehensive model that describes failure of differ-
ent con1posite systen1s largely be cause different systen1s exhibit different dan1age
n1echanisms a nd failure n1odes. One n1odel, \vhich is applica ble in e ngineering
design, is '\\1hitney a nd Nuisn1e.r"s 7g mode l for predicting failure of lan1inates
with local stress concentrat]ons. ln the follo\ving \Ve illustrate t he concept un-
derlying this model via the fail ure of la n1ina tes containing a c.ircular ho le or a
notc.h.

10.4.1 Plate with a Circular Hole


let us consider a large pla te containing a circular hole of radius R (Fig. 10.14.
left). The plate is subjecte d to a uniformly distributed average tensile stress 'ii.
lJnde r the action of this suess the t hrough-t he-thic.kness average of t he norn1al
stress along the x axis is a',, (Fig. 10.14, right).
Whitney and Nuisme r proposed two failure crite ria, one referred to as the
pointstress the other as the average stress criterion.
1\ ccording to the point stress criterion, fa ilure oc.curs \vhen, at a fixed distance
do from the hole a long the xaxis (Fig. 10.15), a'" is e qual to or greater than the
stress at failure of the "solid" plate (i.e.. the plate ivirlrow the hole),

a',, ( R + do, 0) ;:: i1J; failure. (10.71)

where;;:; is the stre ngth of the solid plate (Fig. 10.16).


,;.\ ccording to the llverage stress l'riterion, failure occurs when the stress fiy.
averaged ove r a fixe d distance llo along the x -axis, is equal to or gre ater than the

7 J. ti.1. \ Vh itncy and R. J. Nuismcr. Stress Fract'Urc Criteria for La1n ina1cd Composites Contain ing
Suess Concentrations. Jounwl ofCoJJ1poJite 1\larerials, Vol. 8, 253-265, 1974.
8 R. J. Nuismcr and J. ~I. \\' hitncy. Uniaxial Fuilurc o r Co n1 posi1c Lnminatcs Containini:; Suess Con
ec1ra1ions. Jn; Fr'1(.tlu~ ,lt/~Juu1irJ of C(nt1po.~ilt!S. AST~I STP 593, American Sol.'icty for Testing and
~,tateri als. Ph iladclph ia.1975. pp. 11 7-142.
432 FAILURE CRITERIA

"- =q-' - > (1 (1 -' -


(1 <"J
ttttttttt tttttt t tt ttttttttt

;;, - r
-I---- 11'()
_._

failure
~ failure

figure 10.15: I llustr ation of lhe failure of a plate wiLh a circula r hole accordjng lo the point stress
failure criterion.

failure stress of the "solid" pla te:

-no1 J,R+
R
oa, (x, O)dx?:. '1f, failure . (10.72)

To apply eithe r or the crite ria above. the s tre ngth uf, o f the solid plate
(Fig 10.16) and the stress distribution 0-y (Fig. 10.15) mus t be known. The strength
is to be obta ined by tests. The stress distribution must be determined nwuerie-ally.
Ortltotropic plate. For ortho tropic plates (with the x- and y -axes in the direc-
tions or orthotropy) the stress distribution a long thexaxis is give n by Eq. (4.305).
With the definitions i1y = N1 / Ir and i1 = N{:'/ ii, we write Eq. (4.305) as

uy(x, O) = ~ \2 + (~)' + 3 (~)' - (K7 - 3) [ 5 (~)' - 1(~)']} ,


(10.73)

\vbere 7i is t he applied uni(om1 tensile stress (Fig. 10.14) a nd Kr is the stress


inte nsity facto r (Kr= u,.(R, O)/ul. From Eq. (4.304), K,. is

K r= 1 + -
2 (\ I~
A11 An - 1t11 + A11Ail
2
- Af,)
- ' (10.74)
A 11 A66
'vbere Ai; are the eleme nts of the stiffness n1atrix io the x- y coordina te syste m
(Eq. 3.20). A t foilure i1is denoted by U' (F\g. l0.16). \\Cith this notation, Eqs. (10.7 1)

- J ii = ifr
u = a0
~t.,t'tt1'1'1' ,.t1't,ttt

J
figure J0.16: '['he applied stress al failure o( a plate withoul and \\'ith the hole.
10.4 PLATE WITH A HOLE OR A NOTCH 433

-.. 1~~
.<i
~
~
"'

0 Hole Radius, k
F"tgure J0.1 7: The strength of a composite plate wilh a hole as the func.t ion of the hok size.

and (10.73) yield

ii1 2 orthotropic plate


(10.75)
u~ = 2 + r' + 3r' - (Kr - 3)(5r- 7r) with ho le.

The paranteter r is defined as

R
r= - - - (10.76)
R +tfo
The expression using the average suess c riterion is not given because \\'hitney
and Nuis1ner have sho\vn t ha t the point stress a nd average stress crite.ria yield
very similar results.
Equation (J0.75) shows the hole size effect, which has also been ob<erved
experimentally: ""hen subjected to tension. pla tes \Vith la rger holes have lower
strengths than p lates with smaller holes (Fig. 10.17). To explain this phenomenon.
we observe that the high stresses in the vicinity of the hole are confined to a
region that is narrower for smaller holes than for larger holes (Fig. 10.18). One
possible explanation fo r the hole size effect is that, in this narro\ver region, there
are fev.er Jla,vs (failure initiation sites) that could initia tedan1age gro\vth. resulting
in reduced strength.
It has a lso been argued that plates 'vith smaller ho les a re stronger because
stress gradients near t he hole can bette r distribute for small holes (\vhere the stress
gradients are high) than for large holes (v.here the stress gradients are sn1aller).

Figure JO.JS: The slress d1s1r ibution aJong the xax:is for l \\'O dilfere.nl hole. sizes.
434 FAJLURE CRITERIA

i'i
ttttttttt

2c
I~.
l l

Figure J0.19: Unidirec.tionally loaded plate contaning a notch and the stress distribution nlon$
the x-.axis near t he-notch.

10.4.2 Plate with a Notch


We no\\' apply lhe Whitney- Nuisn1e r point s tress criterion to a large synunetric
orthotropic plate containing a through-the-width notch of le ngth 2c (Fig. 10.l 9,
left). The notch is aligned with one of the o rthotropy axes. \Vhe n this plate is
subjected to a unifo rm tensile stress '7, the througb ..the.-thickness average of the
nonnal stress along the x -axis 'iiy (Fig. 10.19, right) is given by Whit ney and
Nuism e r9 as
- (
<1 y x ,
0)= 'iix
~ (10.77)
..;x - cl
S ubstitution of Eq. (10.77} into Eq. (10.71) ( R replace d by c) yie lds

:~ = /1-c:~.Y
pla te
(10.78)
\vith notch.

The average stress criterion can be obtained similarly. It is not given here
beca use it yie ld-; results sin1ilar to the po int stress criterion.

10.4.3 Characteristic length


The use fulness of the \Vhitney- Nuis111er po int stress and average s tress failure
criteria is pre dicated o n the assun1ption that the c haracteristic lengths d0 or a0
are independent of the ho le o r notc.h size - a t least for a given laminate n1ade
o( a particular n1aterial syste m. In s uch a case, the characteris tic le ngths can be
detern1ined through o ne test fo r one hole or one no tch s i1..e. ObviotLo;ly. t he nlodel
\Vould be even more useful if t he characteristic lengths \Ve re to remain constant for
all lamina tes (layups) made o! diffe re nt typeso! fiber-reinforced composite~ There
is evidence that t he values of t he characteristic lengths do not vary appreciably

11 J. ti.ii. \Vhitnc)' and R. J. Nuismcr. Stre5$ Frac1ur<:. Criteria for La1ninatcd Composites Containing
Stress Conccntrutions. Jour1111l of Contposire i\1atdit1/s. Vol. 8. 253-265. 1974.
10.4 PLATE WITH A HOLE OR A NOTCH 435

over a '"ide range or conditions. For exan1ple, Nuisn1er a nd \.\'hitney 10 have fo und
= =
that the values do LO mm a nd au 3.8 mm fit the data for cross-ply a nd quasi
isotropic glass-epoxy and graphite epoxy laminates containing a circular ho le
or a notch ,...,ell. Altho ugh t he existing data are not all conclusive, the Whitney-
Nuisme r point stress and average stress criteria. togethe r with the characteristic
length values just cited, should be useful at least fo r the purpose of preliminary
de.sign.

10 R. J. Nuismcr and l f\.f. \\'hilncy, Unia:icial Failure of Comp0$itc Laminates Containing Suess
Concetralions. l n: Frocrure 1\1://atiic.vofC'nnposite:f. ;\STh1STP593. A 1ncrican Society for Tc.sting
and ~1atcrials. Philadclphia, 1975. pp. 117- 142.
CHAPTER ELEVEN

Micromechanics

Micro 1nechanics is used to estimate the mechanical a nd hygro the rn1al pro pe rties
o( con1posite n1aterials fro1n the known values of the properties of the fibe r and
the matrix. TI1e re are t hree major categories of micro 1uechanical a pproaches:
(i) mechanics of ma teria l models based o n si1nplifying assum ptions that n1ake it
unnecessary to specify in detail the stress-strain d istributions, (ii) e lasticity 1nodels
requiring that the stresses a nd s tra ins be determined at the n1ic.ro n1echanical leve l,
and (iii) e mpirical e.xpressions resulting fro 1n curve-fit ting elasticity solut]ons or
data.
It is not o ur intent to d iscuss the nun1e ro us available n1odels. Instead, we focus
on two mechanics of materials n1odels, name ly o n the '"rule- o ( nlixt ures a nd the
n1odified rule of n1ixtures: The rule of nlixtures is the s implest and most intuitive
approach and is useful fo r introducing concepts. Ho\\ever, it fa ils to representso1ne
of the properties \Vith reasonable accuracy. The n1odified rule o f n1ixtures is an
improveme nt over the rule of nlixt ures, and predicts the propert ies \Vilh be tter
accuracies.
Methods to predict strength - tho ugh available - are not presented because
they are less accurate than the mode ls predicting elastic and hygrothern1al
properties.

11.1 Rule of Mixtures


We cons ider the volume V of an element s hov.n in Figure 11.1. T he volume of this
eleme nt is

V = V, + Vm + V,., ( 11.1)

\Vhere the subscripts f, m and v refer to the fiber, the n1atrix, a nd t he void. It is
convenie nt to introduce the volun1e fractions as follo\\s:

vr -
Vr (11.2)
V
436
11.1 RULE OF MIXTURES 497

Fi~ t I. I: lllus1.n1t0n ol lhe m:atnx. 6bc:r. and rold \-olumes.

Equations (I I.I) and (11.2) gi"e

ttr+u.. + 11.., = l. (1 13)

Whe n tlie void fraction is negligible (v,. = 0), we hove


(11.4}

The n1ass o f the elc1n e nt in Figure 11.1 is

M = M1+ M0 ,+ M,. ( l 1.5)

By neglecting the mas.~ o f the \'oid, \Ve c.an \\'rite this equation as
JW ::. p(V, +p,,.,Vm. (1 1.6)

v.ilere Pr and p. nre the fiber and the maui~ densities. respectively. The density
o( the composite is
M
""'""' = v = ~Pr + """' {11.7)

ln the following. we treat unidirectional. tiberreinforccd composi1es without


'"oids. \Ve dcri\'C the pmpen ics using t\vo t)rpes of elcn1e nts (Fig.. 1t .2). E.lemenl 1
conta ins a single fiber bundle o f circular c.r oss section. Eleme nt 2 consists o f a
fiber l:iye r s:ind,viche d be t\\'een ''''o layers of ma uix 1nate rial. ln Eleme nt 2, the

! ~fiber
~ Lr= 11L
..........._
1,1 .!!i. 'r t1rL1l
L1 ~
..igurc J 1.2: Rcprc:llt:OIOll\'C elements. E.lcmcnl J {lcfl ) and E lc111cnl 2 (right),
438 MICRO MECHANICS

F,
....____

Figure 11.3: Element l subjected to a force in the llbcr direction ( left). and Element 2 subjected
to a force in the tra.nsversc dircclion (right).

fiber a nd n1atrix \'olumes a re t he same as in Elen1ent 1. Hence, the thickness of


the fibe r layer in Element 2 is urL

11.1.1 Longitudinal Young Modulus E,


Ele n1ent 1 is subjected to a fo rce F1 in the fi ber direction. The force., distributed
over the surface, is (Fig. 11.3)

F1 =U1A , (1 1.8)

\Vhere 111 is the average nom1al stress across the e ntire c ross-sectiona l area A
(A= L2). Part of the force is carried by the fibers and part by the matrix. Thus.
\Ve write

(11. 9)

Ar and Am a re the c ross..sectional a reas ot the fiber bundle and the n1atrix, respec..
tively. When t he Po isson effect is neglected, the norma l stresses in the con1posite
<11, in t he fi ber bund le '1 11, and in the ma trix C1m1 a re

<tn = (n En O'ml = l ml Em. (11.10)

'vhere 1 and 11 a re lhe con1posite and the fiber longitudina l Young moduli and
Em is the ma trix Young modulus, respectively. Equations (11.9) and ( 11.10) give
Ar Am
E=1E1 = Afn E n + Afm1Em. (11.1 1)

The e longations a Land, hence, the longitudinal normal strains of the con1
posite 1. the matrix fm1. and t he fi be r E"n are e qual:

(11 .12)
The volume fractions are
Ar
ur=A (1113)

Equa tions ( 11 .1 I )-(11.13) result in the following expression fo r the longitud-


inal Young modulus of the con1posite in tenns of the fiber and n1atrix n1oduli:
(11.1 4)
11.1 RULE OF MIXTURES 439

11.1.2 Transverse Young Modulus E2


\Ve consider a rcctnngular e lement \\rithsides Lmade up of three Inye rs (Element2.
Fig. 11 .3). The inner layer is a s heet o( fiber with the same volume as the fiber
bundle in Element I. The length and the transverse (x2 direction) Young modulus
o( this inner layer are L1 and Ea. respectively. and tbe length and the Young mod-
ulus o( the outer lay<rs are L . = ( L - L1) / 2 and . The element is subjected to
a force F2 The force is distributed uniformly across the surface A ( A = L'). The
normal stress in 1hc transverse direction is a 2 = Fil A which can be expressed as
( 1115)
where 2 is lhc tronsverse Young moduJus of the e1einent nnd ':z is the average
transverse norn1al strain
Ai.
f 2 = - (L l.16)
I.
111e cha nge in the le ngth o( the e lement is

11 L =2 Lmfml + L cfc2. (1 l.17)


\lt'he re f.,z and 1 12 nre the transverse normal strains in the matrix and fiber layers.
By neglecting the Poi"->n effect, we have
a_, '1(?
(11.18)
f-Z = E.. it?= -
Et2

By introducing Eqs. (11.1 7) and ( 1US) into Eq. (11.16). we obtain


AL 2Lm am2 Lr an
f2= -=----+ - - ( 11.19)
L L Em L 12
TI1e transverse normal s tresses in the composite o2 the n1otrix o.,.z, and the
fibe r t71z lnye rs ore e qual as Collo\\'S:

( l 1 20)
111c n1a trix and fiber volume fractions are
Lr (11.21)
vr= -
L
By uhslituting Eq.. ( I L.20).(11.21).and <z = az/ Ei intoEq. ( 11. 19). we obiain
the trans-ve rse Young modulus

1= ( -
<1 +"
- ) - ' = ( -1 I --
+- "')-' . ( 11.22)
Et1 . Ea 10

11.1.3 Longitudinal Shear Modulus G, 2


\Ve consider Eleme nt 2. TI1e length a nd the longitudinal shear 1nodulus of the
inne r laye r arc L 1 nnd C n 2 respectively, and the le ngth nnd the s hear modulus
o( the oute r layers ore L 111 = ( L - L 1)/2 and Gn The c len1c nt is s ubjected to n
MICROMECHANICS

L1L
- >1-k;- F,,
J - !_,.k
F,,! / "l ..1'
c:fi,7,l'7.t ~
L
/m
w::. '1
tl,;....,....
I
1 Fu

i ~ l
F,,
Figure 11.4: E lement 2 subje-e4cd to u .shear force (left) and deforma11on of the .. top.. (ijkl)
-surface {right).

shear force F12 (Fig. 11.4) distributed uniformly across the surface A (A= L1 ).
T be shear stress r12 = Fri/ A is

(11 .23)

\vhere G-12 is the longitudinal shear modulus of the e lement a nd y12 is the average
shear strain.

Y12 =tan Y11 =


llL
L ' (11 .24)

\\'here 6.L is due to the shear deforn1ations of the matrix and fiber layers,

(11 .25)

\vhere Ym12 and Ynz a re the shear stra ins in t he n1atrix and fiber layers as follows:
'Cml2 rn2
YmJ2= - Yn2= - . (11.26)
Gm Gn1

The shear stresses in the con1posite, the n1atrix, and the fiber layers are e qual:

(11 .27)

By combining Eqs. (11.21) and (11.23)-(11.27), we obtain the longitudinal


shear n1odulus:

ur
G.,= ( -- +- Um )-I = (--+--
ur 1 - "')-I (11.28)
- Gn 2 Gm G112 Gm

11.1.4 Transverse Shear Modulus G23


We again consider Ele.me nt 2 n1ade up of three layers (Fig. l 1.4). The transverse
shear moduJus G23 of this e len1ent is derived in the same \\'ay as the longitudina l
shear n1o<lulus G 12. The result is

ur I -
Gn= ( -- +- -
u,)-1 (11 .29)
GaJ Gm
11.1 HUL~ U~ MIXI UHtS 441

F1guTc I LS: Ocormation oC Element I Juh~c:ted to


an a:ua.l force LD tbc ;r1 <luc<hon.

11.1.s Longitudinal Poisson Ratio v, 2


Ele ment t. shown in Figure 11.2, is subje.c ted to a force F1 in the longitudinal X 1
direction. Because of this force the e lem ent deforn1s, as illustrated in Figure l t .5.
The sides or Ll>c deformed eleme nt are (L+ t>.L1 ). (L - t>.L,). a nd(/, - t>.L,).
The c'hangc in the cross-sectional area Ao( the fa ce ijkl is

( 1l.30)

By definiljon. the no rn1al s trains in the x 2 and x3 directjon.s nre


t>.L;
IJ = - -L- ( 1131)
.
Owing to symmetry. !>. L1 = !>. [_, and. consequently. we hove

(1132)

By neglecting higher-order tenns, we find tbot Eqs. (I 1.30H 11.32) give

( I 1.33)

By sin1ilar argume nts. the changes in the n1at:rix and fiber areas arc

t>.A, = 2Ar<rz ( Ll.34)

111e c hange in the tota l area is the sum of the changes o f the fibe r and matrix
areas:

(1135)

=
Under the action of the longitudinal normal stress o 1 Fa/ A the transverse.
nonnal strains are rt lated to the longitudinal normal strain by

(1136)

By virtue of Eq$. (11.32) and (1 136). we have

( 11.37)

From E qs. (I l.36) and (I 1.33) we obtoin

: t>.A 1
v 11 = -- = -2A
- - (1138)
11 f1
442 MICRO MECHANICS

By combining Eqs. (11.35) and (11.38) and introducing Eq. (11.34) into the
resulting expression, 've obtain
2A1E11 + 2AmEm2 1 A1 E11 Am fml
"" = - 2A <1 =-AT, - 'AT," (11.39)
The changes in the lengths of the fiber, the 1uatrix, and the composite are.equal.
Consequently, in the:c 1 direction the normal strains in the con1posite, in t he fiber.
and in the n1atrix a re equal as (olJo,vs:
(11.40)
With t he definitions
Ar En f m2
ur= A Vf12 = -- I'm = - - , (11.41)
En fm 1
Eqs. (11.39)- (11.41) give the following expression for the longitudinal Poisson
ratio:
(11.42)

11.1.6 Transverse Poisson Ratio 1123


For a transversely isotropic n1aterial (v.hic.h is under consideration he re), the
transverse shear and the Young n1oduli are rela ted by the following expression
(Eq. 2.34):

c,, = ,- (11.43)
- 2(1 + vn)
By rearranging this e xpression, we o btain the transve.r se Poisson ratio
,
vu= G, - I.
2 .J
(11.44)

More accurate expressions for 2 and G2 1 are obtained by the n1odified rule
of mixtures (Section 11.2). These expressions are included in Table 11.1 , below.

Table 11.1. Expressions for lhe engineering constanls


Longitudinal Young modulus 1 = L't 11 + um Em

Ttansverse Young moduJus 1 = (~ + l~:ft")-1


where Et.1 = v1FrE12 + ( l - JUr)E..,
-I
Longitudinal shear modulus Gu =(~+ 1-;t)
where GM!= JiijGoz + ( l - Jiii)G.

Transverse-she.ar mod ulus Gr = (E


Gt,:) + 1-Gm" ') -'
where Gi.u = Jii;Gn:J + (l - ..JVf)G.

Longitudinal Poisson ratio


Transverse Poisson ratio
11.1 RULE OF MIXTillES 443

11.1.7 Thermal Expansion Coefficients


\Vhen the unrestrained E len1ent 1 is subjected to a 6 T ten1perature change. its
volume changes (Fig. 11 .6). The new length, width, and height are (L+ ll.L1).
( L + ll. L,), and ( L + ll. L.l). and the corresponding normal strains are
ll. L,
1
-
=--
L
(1145)

The longitud inal ii'1 a nd transverse &2. li3 t he rn1al e xpansion coefficients are
defined as
- =ft- - fl
U't <t>= - (1 l.46)
ll. T - ll. T
Owing to syn1meuy, the transverse thermal expansion coefficients a re equal
(<Y2= ii'3), and the t ransverse normal strains are equal (2 = 3).
Equations (I 1.45) and (I 1.46) referto the element.
For the fiber and the n1atrix, the strain-stress relationships are (Eqs. 2.162 and
2.163 and Table 2.7, page I 5)

f n - -an6. T =En
"" (11.47)

(11.48)

These equations a re \vritte n \vi th the assun1ptions that the longitudinal norn1al
stress a 1 dominates and is the only stress needed to be considered.
Longitudinal ther111a/ expansion coefficie111. \Vhe n Elen1ent l is subjected to
a ten1perature change 6.T. in the x direction the fiber and the ma trix elongate the
same an1ount as the con1posite. Hence, the norn1al stra ins in the x1 direction are
equal
1 = ln = fml (11.49)
From Eq. (I 1.47) the longitudinal normal stress (in the x 1 direction) in the
fiber and the matrix may be written as (Eqs. 11 .47 a nd 1 I .49)
(11.50)
444 MICROMECHANICS

The to ta l force acting on the ijkl face (Fig. l I .6) is the sum of the fo rces acting
on the fiber and the matrix:

(ll.51)

Since there is no net force on the ijkl face. \\'e 'vrite

Fr + Fm= O'aAc+O'm1Am = 0. (11.52)

By introducing Eq. (1 l.50} into (11.52) a nd by noting that vr = Ac/ A and Um =


Ami A, \Ve obtain

(1l.53)

BycombiningEqs. (I 1.46, left) and (l 1.53),a nd by noting that En ur + ,.,um =


1 (Eq. I 1.14), we obtain
_ vrEn _ VmEm_
a, = e,et11 + TC/m. (11.54)

Transverse tlter111al expa11sion coefficient. When the temperatureof E lement l


is changed by a T, the cross ..sectional area of the face ijkl as well as the fiber and
ma trix areas change (Fig. 1 I .5). The cha nges io these areas are (Eqs. l l .3l-J 1.34)

(1l.55)

The change in the total area is the sum of the changes of the fiber and matrix
areas:

ll.A=ll.Ar + ll.Am. (11 .56)

By combining Eqs. (I 1.55) a nd (I 1.56) a nd by noting that c = Ar/ A a nd Um =


Am/ A, \Ve obtain

(ll .57)

From E q. (1 I.48), the ua nsverse normal strains in the fiber and the matrix may
be written as (x1 direction)

(11.58)

By introducing Eqs. ( 11.47) into these e xpressions a nd observing that <1 =


t:n = fm 1 = a,6. T. \\'e obtain

11 = - vn2(Ci'1 - n)8T + i.112!1T fml = - lm(a'1 - <Ym)8T + Ciml!t. T.


(11.59)

Equations (1 1.46, middle). (11.57), and (11.59) yield

(I 1.60)

where ii1 i.< given by E q. (11.54).


11.1 RULE OF MIXTIJRES 445

11.1.a Moisture Expansion Coefficients


\\'e assume that the n1oisture content of the interface is negligible. Further. \Ve
co nside r con1posites in 'vhich the fibers do not absorb n1oisture and the moisture is
unifo rmly distributed throughout the ma trix. The n1oisture content of Eleme nt I
is the s um of the moisture contents of the fibe.r and the 111atrix

( tl .61}

where Vis the volume. c t he n10 Lo;ture concentration. a nd, as before, f and m re(er
to the fibe r and the n1atrix. Since here \Ve treat co1nposites in \vhich t he fibe r
moisture content is zero (er= 0), \\'e have

( ll .62)

'TI1e moistureinduced stTains of Eleme nt 1 are

(11.63}

where "Pi is the longitudina l and #2 and P3 are the transve rse n1oisture expression
coefficients. Q\\ing to syn1metry, the transverse moisture expansion coefficients
are equal (iJ, = pl).
The moisture expa nsion coefficients of the fiber and the nlatrix are(i = 1, 2. 3)

- ff; c liti - mi C nv
/Jr; = -cr = - -
cc c
/Jn,;= -Cm = - -
Cm C
(11.64}

We now compare the definitions of the therma l and Lnoisture e xpansion coef..
ficients as fo llows:
-
a - -~..
, - 6.T
- fr;
Uti = 67 (11 .65)
- (; nu
Uni = 6. T
Fro1u these equations \Ve note the analogy be t\veen the t he rmal and n1oisture
expansion coefficients. T he te mperature c.hange 6. T corresponds to the
moisture concentra tion c, and the thermal expansion coefficients correspond to
the n1oisture expansion coefficients as follo".:s:

tt;=>P;
Ct-
'&li => -Pei ( tl .66}
c
- c.,;,
Um=> - Pni
c
MICRO MECHANICS

Since er = 0, Eqs. (11 .66) and (I I .62) give the following correspondence be
tween the thermal a nd moisture expansion coefficients:

Cij =>~I
'iici ==> 0 (I 1.67)
z ____..
""m _,,
iim .
v.,
Longitudi11al 11roisture CXJHlllSio11 coefficient. 111e longitudinal 1uoisture e x-
pansion coefficient is obtained by replacing in Eq. (J l.54) ii 1 by p,,
and ii., by
Pm/um. and by setting Cin = 0 as (ollo\vs:
- Em-
/J1= -E /Jm (11 .68)
I

Tra11si1erse 1noist11re expansion coefficient. The t ra nsverse n1oisture expan


P;.
sion coefficient is obtained by re placing in Eq. (11 .f>()) ii; by iim by Pm/"""
and
by setting a,; = 0:

"iii= Pm+ v,vm(-"i11) +,,_,(,ii., - v.,,ii,). (11 .69)

11.1.9 Thermal Conductivity


T he heat conducted (per unit tin1e) in the longitudinal a nd transverse directions
is given by Fourier's la\v

q, = A(K,) ( - ar).
a.n (I 1.70)

\Vhe.r e A is the c.ross-sectional area, aT/ax is t he te mpera ture gradient, a nd K1 .


Ki are t he longitudinal and transverse heat conduction coefficients, respectively.
The longitudina l and transverse normal fo rces a re re la ted to the longitudinal
and transve rse no rma l stra ins by Hooke"s Jaw as follo\\s:

(11 .71)

Equations (11.70) a nd (11.71) show the analogy between Fourier's and Hooke's
la\\'S. TI1is an alogy is used to obtain the n1ic.ron1echanical expressions for the ther
ma] conductivities. The heat.conducted q corre.s ponds to the force F. the thern1al
conductivity K to the Young nlodulus and the ten1pe rature gradient aT/ax to
the strain f.

q=F
K='> E (I 1.72)

=
ar
ox
I I . I KULt Ut MI X I UH t~ 44/

Long itudinal thern1al cond uctility. The longitudinal lhennal conduc1ivity is ob


tained by replacing in Eq. ( 11.14) En by Kn and Em by K.,. The resulting expres-
sion is
(I 1.73)

Trans,ene the nnal conducti,ity. The tnns,erse tbennal conduc1ivi1y is ob


tained by replacing in Eq. (11.22) Ee by Ka and ., by K. The resulting ex-
preMion is
_,
K i= (-Kr2+Kn,-
111 U n1 )
{11.74)

11.1.10 Moisture Diffusivity


Moisture dif(usion in 1hc longitudinal and transverse directions arc nssu1n ed to
folJo,v Fick 's h1\v

1i11 =A( 0,) ( - ~) a.Y1


iii,= A (D,) (-~)
ax, .
(H.75)

where 1ir d the mass of moisture being uansponed (per unil time) acros.~ the
surface A 8c/3x d the moisture concentration gradienL and O.. Dz are the Ion
gitudinal and transvcne di(fusi"ities. respectively.
Equations (11.70) and (11.75) sbow the analogy between Fouriers and Ficks
Ja,vs. This analogy is used to obtain the micromechanical exprenions for the mois
ture di(fusivitics. The hea1 conduction q corresponds to 1he 1noisture transport 1h,
the thern1a l conductivity K to the diffusivity D. and t he te mpe rnlure gradient
aT/iJX to lhC 1noisturc gradient ac/ax as foJIO\\'S:
q ==> 111
K= D {lL76)
aT ilc
-a.r ===> -
ax
Longit11di11al n1oi.tt11re diJ!u.siviJy. The longitudinal moisture diffusivity is ob
tained by replocing in Eq. ( 11.73) Kn by Du and Km by D,.. The resulting expres-
sion is
Di= t>Dn + -..A.. (11.77)

When the diffusivity of the fibers is negligible. we have


Di= 1,. D,,,. (11.78)

Tra11s11ene n1oist11n diffusi1'if)1 The transverse moisture diffusivity is obtained


by replacing in Eq. ( 11.74) Kr2 by Dr2and Km by D111 The rcsull ing expression is

..,,=
n (-"' +"
D12
-) - 1
Dm
( l 1.79)
448 MICROMECHAMICS

When the diffusi\'ity of the fibers is small ( D12 -. 0), this expression gives that
Di -> 0. and E q . ( 11.79) should not be used.

11.1.11 Specific Heat


When the ten1perature of E le 1nent 1 is raised fron1 a specific datun1 T.1 to T, the
heat contents of the composite, the fiber, and the ma trix are

H = pCV(T - Tol Hr= prCrl'r (T - To) Hm = p,,,Cm V,. (T - To),


(11.80)
\\'here C, Cr. Cm are the specific heats of the con1pos ite. the fiber, and the ma trix,
respective ly. The to ta l heat conte nt o( the composite is

H= Hr + Hm. (11.81)

Thus, the specific heat is


C =Cr f>tlJc + Cm Pm Um. (11.82)
p p
By substituting the expression fo r the density p given by Eq. (11.7), \Ve obtain
C= Cr/>fur+Cm/JmLlm.
(11 .83)
P1V1 + Pmt.'m

11.2 Modified Rule of Mixtures


T he modified rule of mixtures isson1e\vhat n1ore con1plex than the rule of mixtures,
but it gives the transverse properties \\'ith better accuracy. We illustrate the main
fea tures of the n1ode l through the derivation of the lrans\'erse Young modulus..
The basic building block is an e len1e nt in \vhich the fiber bundle is taken to be
of rectangular cross section (Fig. 11.7).' The volun1e fraction of tl1is rectanguJar
fiber bundle is the same as the volu1ne frac tion of the corresponding circular fiber
bundle.
As a firs t step \\'e replace this e le n1ent \\'ith a rectangular box" 'vith three
layers. The two outer layers consist of the m atrix whereas, (or no\\', the middle layer
is taken to consist of a fictitious homogeneous n1aterial. T his "box" is con1pared
with Element 2 shown in Figure 11.2. For the present "box" the m iddle layer
(designated by b) corresponds to the middle fibe r s heet in Element 2. Thus. the
transverse Young modulus of the "'box~ is given by Eq. ( 11.22) \Vi th tl1e subsc.ript
f replaced by b

,= ( -"+1-
- -
... ) - ' . (11.84)
- Eb2 Em

I 0. A. Hopkins and C. C. Chnmis..A Unique.Set of~1 i crom<:ch.nnica1 Equations for HighTcmpcra1urc


}.,1etal ~latrh: Composites In: Tt!s1i11g Tn;/1110/n>;r of Al~Ml J\larri.x Cn1npwi1ts, ASTht STP 964.
American Society for Testing and J.latcrinls. Philadelphia, 1988. pp. 159- 176.
11.3 NOTE ON THE MICROMECHAHJCS MODELS 449

1-igurc l l .7: 1be mode.I tor the. modil'i e.d rule o( mixtures ( lop) a nd the rcpre.scnta lion of the
middJc. .. b.. laycr (bottom).

111e volume fraction o f the middJe laye.r lib is de te nnined as follo \\'S. The \vidth
of the middle layer L is the same as the width o f the recta ngula r fiber bundle.
Thus, vb is

v,, = (L)(~) = Lj'ii'i = j'ii'i, (tl.85)


L- L
where vr is the fi be r volume fraction of the entire box (.,,= ( j'ii'iL) 2/ L', Fig. ll.7,
middle).
The Young n1odulus Ehl o ( the n1iddle layer is obtained by replacing this
homogenized fictional layer by a layer consist ing of the rectangu lar fibe r bund le
i
surro unded by matrix \~ 1.1, bollo m). The volume fraction o f the fiber bund le
in this n1iddle layer is ~~ 4 = ./Vi. \Ve now a pply the longitudina l rule of n1ix
lures (Eq. 11 .14) to the middle layer. The result is

Eb2 = ftrEn + (1 - j'ii'i) E,,. (tl.86)

Equations ( 11.84) a nd (11 .86) give the uansverse Yo ung modulus. O the r prop
erties m ay be obtained sin1i1arly. De tails of the derivations are not given here. The
results are included in Tables 11.1 a nd 11.2.

11.3 Note on the Micromechanics Models


The various availa ble micromechanics a pproaches yield simila r results. except fo r
the rule o f mixtures. \Vhich underestimates the shear moduli and the transverse
properties. This is illustra ted by the e xamples in Figure 11.8, \vhere the transverse
Yo ung n1oduJus and the longitudina l shear modulus are sho'vn as calculated by
450 MICROMECHANICS

Table 11.2. Expressions ID< the hygrothermal properties

Thennal expansio n "'l 1-:, an + ~


- = !I.fa.- -
. Um
1
c.oeffki e nts z = t'zCtu + ViCtm
+v11u2 (Cto -ii1} + L'inPin (Ci,.. - 1)
rvloisture expansion
- ,,.. -
fJ 1 = i:~ flm
coefficients fi2 = flm + u1vn2(-fl1) + 11.,{,8111 - t.tm.8'1)
(co = 0)
Longi1udina l
thermal conduc.tivity

Transverse.
thennal co nduc.tivil)'
Longitudinal
moisture. diffusivily when Du =0

Transverse Di = ( ~ + l~e)-'
moisLure diffusivity where D,,, = /Vo D,, + (1 - jV,)D,.

Di= A. ( -~ -
1
Jiii)- when l>Ji = 0
Specific heat C = C e!!i
f I'
+Cal
~
P
= C1,llfY+C1~A.l\11
1~tt+IW111

o Finite difference

5 !!li s fu
E H-T G,.
4 4
MROM
/
-:::;:::::::::;::= MROM
H-T
3 3
2 ROM 2 ' . . - - - - - - ROM

0 0 '--~~+-~~+-~~+-->

10 100 IOOO .!& JO JOO IOOO fL_


E"' G"'
Figure 11.$: 1b e transverse. Young and shear moduli c-a lculaled h)' the. rule o ( mixtures (RO~l).
the modilied rule of mixtures (MROrvJ), t he Ha lp1rr-Tsai (H-'f) equations.. a nd the finile dif(cr 4

encc-solulions (circles) o ( Adams and Doner ( ttr = 0.55).


11.3 NOTE OH THE MICROMECHAHICS MODELS 451

Table 11.3. The engineering constan!S in Example 11.1


E, (Gl'a) E, (Gl'a) G,, (GPa) ,..,,
Nominal (1.y= 0.6) 148 9.65 4.55 0.300
Back calculated ( '11 = 0.55)
Rule of mix-1ures 136 0.304
f\lodified ru le of n1ixlures 9.00 4.14

(i) mechanics o( ma te rials n1ode ls (rule of n1ixtures, n1odified rule of n1ixtures).


(ii) the elasticity solutions of Adams aod Doner."' and (iii) the Halpin- Tsai'
sen1ien1pirical expressions.
Even \vhen prope rty values predicted by various micromechanics expressions
are by and large similar, they should not be relied upon for design purposes
because of inaccuracies introduced by approximations in the models, by uncer
tainties in constitue nts p roperties, and by n1anufacturing variances. Rathe r, it is
recommended that the ply properties be determined by testing the ply itself.
Micron1echanics expressions are useful for estin1ating the d1anges in ply prop
erties \vith sn1all changes in the \'olume fraction or in the properties of the fiber and
the n1atrix. The procedure fo r estimating the changes in ply properties is refe1Ted
to as back calcula tion .. and is illustrated in the follo\\ing example.
11.1 E.xantple. Tire uo111i11al engineering consta11ts of a unidirecliouttl graphite
epoxy ply are given iu Table 11.3 for a fi ber volu111e fraction of 60 perce11t. The
fiber volutue ls reduced 10 55 percent Estinuue rhe en.gi11eeriug co11sta11rs for this
reduced fiber 110/tune frt1ctlon.
Solution. Generally, the n1atrix properties are known reasonably accurately.
whereas the fibe r properties are not. The refore, the fibe.r prope rties a re back
calcula ted.
The ply's longitudinal Young modulus is (Eq. I 1.14)
(11 .87)

The fiber's longitudinal Young n1odulus with the mauix prope rties in Table C.2
(Em = 4.1 GPa, Gm = 1.5 GPa, ''m = 0.35) is
1 1
En = - (E1 - VmEm) = O ( 148 - 0.4 x 4.1) = 244 GPa.
Vt .6

With this value of En the longitudinal Young modulus for v, = 0.55 is (Eq. 11 .87)
E,0-" = urEn +umEm= 0.55 X 244 + 0.45 x 4.1 = 136 GPa.

2 D. F. Adams and D.R. Doner. Transverse Nornl:ll Loading of a Unidircc1ional Conlpositc. JourntJI
o/Coutpo.~ile Aft11er1of,, Vol. I, 152- 164, 196i'.
J 0. F. Adams and D. R. Doner. Long:i1udinal Shear Loading, or a Unidircc1ional Con1positc. Jo11r11al
of Cou1po.,J'te Aftr1eru1/,, Vol. I, 4--17, 1967.
4
J. C. Halpin and S. \V. Tsai, Eff.ts of 11vi1onuu!11tal Facrors 011 Co1uposite J\1aretiats. Air Force
~fatcrial Lnboratory. \Vright-Pnucrson Air Force Base. Dayton. Ohio. TR-67-423. 1969.
452 MICROMECHAHICS

The ply's longitudinal Poisson ratio un 2 is obtained sin1ilarly by the rule of


mixtures. The result is given in Table 11 .3.
The ply"s transverse Young modulus is (Eqs. 11.84 and 11.85)

-
E, _
(.i6.
E + E
/iii)_,.
1-
(11.88)
bl m

By rearranging this expression. we obtain ( u1 = 0.6)

1
E., = .foi ( E, -
1 - .foi - I 1
E,. ) = .j[6 9.65 -
(
1 - v0.6
4.1
rn-;)-1= 15.92 GPa.
From (Eq. 11 .86) we have
E., = .foiE12 + ( 1- .foi} .,. (11.89)
From this expression the trans\'erse fiber Young n1odulus is (ur = 0.6)

Ea= ~(Et.2 - (1 - JiiilEm)


yllf

= !,(15.92 - (1- M )(4.1)) = 19.36 GPa.


v0.6
With this value of En. Eq. ( 11 .89) gives

E~;" = ../0.55(19.36) + (1 - ../0.55)(4.1) = 10.73 GPa.


The ply's transve"'e Young modulus for u1 = 0.55 is (Eq. I 1.88)

"-
'
" = (../0.55
10.73 +
t-./0:55)-'
4.1
=900GP
a.

The shear modulus G12 is calculated by the modified rule of n1ixtures in a simiJar
manner. The back..calculated engineering constants are tabulated in Table l 1.3.
APPENDIX A

Cross-Sectional Properties of Thin-Walled


Composite Beams

ln the (ollo\\ing tables a11 and du are t he e len1ents of the con1pliance ma trix
o( symme trical laminates (Eqs. 3.29 and 3.30) evaluated a t the midsurface: a1;.
{3;1 , 8;; are the ele n1ents of the con1pliance n1atrix of nonsyn1n1etrica l lan1inates
(Eq. 3.23). These properties a re evaluate d a t each \vaUsegm ent's "neutral .. pla ne.
ln the 1uain text these elemen t~ are identified by the superscript e. To simplify the
notation, in this Appendix the superscript o is 0 1nitted.
The subscripts fl , f2, and w refer l o ftange I , flange 2 and the we b, respective ly.
The modified n1aterial properties are

-
0- 11
(
= au - ({J,.)')
~ -
&11 = (
0'11 - /JI,)
-. -
()11

-fl = (/Ill. - --;;;;-


11
p,.s,.) -/111 = ( /l11 - -
f312812)
-
622

- (
611 = Su - (~>')
-:s;- 111 ( a')
= 811 - ::.14 .
~ll

The location of the "neutral.. plane with respect to the n1idplane is

-
,,.,.
I'l l
ope n section
e = - a" orthotropic unsymme trical layup (A.I}
II arbitrary cross section

open section
arbitrary layup (A.2)
syn1metrical cross section

closed section
orthotropic unsymn1etric.al layup (A.3)
arbitrary cross section.

p~:r and ~~:P a re evaluated a t the n1idpla nes.

453
454 CROSS-SECTIONAL PROPERTIES OF THINWALlED COMPOSITE BEAMS

The stiffnessu;:. is (Eq. 6.l 96)

a6t, = a 66 -
P'
~. (A.4)
11"6
\Vhere a66, /J66. and $66 are evaluated at the "'neutral plane.

Table A.1. The tensile and bending stilfnesses and the roordinates of
the centroid. The layup of each wall segment is orthotropic and
symmetrical. The properties are evaluated at each wall segment's
"neutral" plane, Which Is at the midpJane.

dI i:
)~ ..'~I~- \\'
<I

. . . 2

,,,
tA= lb
""
ID>.,,= ft:t~ =" (:,: + d~ )
Table A.2. The lenslle and bending stitfnesses and lhe coordlnales of lhe
cenlrold. The layup of each wall segment is orthotroplc and unsymmelrlcal. The
properties "" end 611 are evalualed at each wall segment's "neulral' plane,
e.
"'1ich Is at

Table A.3. The lenslle and bending stiffnesses. The layup of eacn wall
segment Is ortholroplc end unsymmelrlcaJ. Doubly symmmelrical
aoss secllon. The property & , Is evaluated at each wall segmenl's
"neulral" plane, "'11ch Is al (I. The properties a11 and 811 are
evalualed al each wal segmenrs neu1rar plene, which Is at ;.
456 CROSS-SECTIONAL PROPERTIES OF THIN-WALLED COMPOSITE BEAMS

Table A.4. The tensile and bending stiffnesses and the coordinates of the
centroid. The cross section Is symmetrical about the z-axls. The layup of each
wall segment is arbitrary. The properties a 11 and 611 are evaluated at each wan
segment's neutral" plane, which is at 'ii

~ " - - ...r,
b.,. !I
...\\~ 0

~' b_,jz,
" - "'
l<-:-->lb r,
fl

El.= _hu (d - z)2 +....l!Lz2 + ..JJL + ~+-'-(t>!, +b!: )


JY (au )n c @u)f! (.tu)n (1111) (uu)..,.
t J
11

EA- ~ ~
- + {ii11)
{U-11) 1 ..

-
Z:- -
.L ( 2h, ~ ....f!:t._d)
fA (iiu )1 2 + (UuJ,. f

2
-
El yy =
1 ,,.;
(Uu ), ll
2
+ {iru), ( ..
br y-Zc:
)
+ (iiub...) ... (dr-Zc:) 2 +{in)"
h ,.
CROSSSECTIONAL PROPfRTIES Of THINWALLED COMPOSITE BEAMS 457

Table A.5. The warping and torsional stillnesses and the location ol the
shear center. The layup ol each wall segment is orthotropic and
unsymmetrical. The properties 666 and 611 are evaluated at each wall
segment's neutral" plane, which is at (;.

,,,

drbm:
:~ i.' . ~
It c y
- 11 4(~ ~
G

-1
= t&..l>n + (il,..,)a + ~
"h
. )
w E '" -- (ulq
12 e
d
~r,
bn

_M_:_
- 1I =
G 4( 2
\.W.)1 + )
(&u,)_.
e = ~~<~~I~-
~+-!L.
.l!J._ + -.'M-
(11)1 l""'
(vu ) 1 ( 11 11 ) ...

~+-'
(11)1 ("uh

I ii
. . f

SC
d b.,,,
,. .' --->
w y ;f
..
IC ~1

CJ, = 4 t.!:,. + (~.)


i =0
111
458 CROSSSECTIONAL PROPERTIES OF THJNWALLED COMPOSITE BEAMS

Table A.6. The wi!Jplng and torsional stiffnesses of


closed-section beams. The layup of each wall segment
is orthotropic and unsymmetrical. cxS8 is given by
Eq. (A.4).

Table A.7. Shear compliances of closed-section beams. The layup of eacil


wall segment is orthob'Opic and unsymmetrical. The properties 666 and j 11
are evaluated at each wall segment's ..neutral" plane, whlc-h is at (,). a&&
is given by Eq. (A.4).

.... (a~}.., (u:.,),r.~


S.;.; = -r;,- + (;1 ,l~y}

..... (u;.), I (..,;'.,).,.d


s.~Y = ~+;;: dfr!
s,, = 0
Y = 1+ ! (uu J, !!. _ l + ! (au),. !!!_
.. J (au J,. J1 Yy - 3 (uu ), d

....._ ..- UM
S.;.; = S yy = lftr

s,, =0
CROSS-SECTIONAL PROPERTIES OF THIN-WALLED COMPOSITT BEAMS 459

Table A.8. Stiear compllan<es. The layup of each wall segment Is orthottopic and unsymmetrical. The
coordinales of lhe centroid y,, z. are given in Tables A.1 lhrough A.4, and lhe coordinate or lhe shear
center e In Table A.5. The properties are evaluated at each wall segment's neutral" plane. whiell is
at{>. a 06is given by Eq. (A.4).

t;
.,f

r- I
11
0

,,
!-'-\\'

11->
!!.
2

~
f
11

b" l;
~:::t==~r,

er1't: ~~ l
~w I
b.

1~ 11 fa:
h.,2

I 1 ca111n " _ 1 ! {UU ) c _ d _


YI = + J {u11)., 11i1(l-t~ ) Y2 - + J (a11),.. bii(l +llc)

'if dr ~ I
!- ..... ~ I-' \ \ '

r .
I
SC
1
0 ->:-i-!4 .
r,
...........
'r f7'1
le iJ ii
Ye Ya:

8.., =-
, '-
7-1
I J - 36.,
q,..lt = lid; 1'='C"
460 CROSS- SECTIONAL PROPERTIES OF THJN-WALLEO COMPOSITE BEAMS

Table A.9. Slleat compliances. The layup of each wall segment Is orthOtrOj)lc and
unsymmetrical. The coordinate of the centroid zc is given in Table A.2. The properties ate
evaluated at each wall segmenfs neutral" plane, which is at 11- u;. is given by Eq. (A.4).

r'
-
$
"
\V -u~~:::::::::-:.]!-..

\.\'here

fJ = 4y - 3

\\here

6 - d - ;..,

"
APPENDIX B

Buckling Loads and Natural Frequencies of


Orthotropic Beams with Shear Deformation

The buckling loads N.:r o( bean1s \\~th doubly syn1metricaJ cross section are given
in Table B. I.
The buckling loads FJ0 of beams \\'ith unsyn1n1etrical cross section, \Vhe re the
coordinate directions y a nd .z are in the principal directions (EZv:. = 0, page 208),
are given by the solution of the follo\\ing equation:

(B.J)

where 11deno tes determinant. The buckling load~ Ro:ry. Na-.. Fin~ a re calculated
by the expressions in Table B.1. y5, and t,..; are t he coordinates of t he shear center
with respect to the centroid, a nd i,.; is t he po lar radius o f gyration

;2 _ ,.2 + , + ifi:.~+ B,v


... - """( .v;; EA (B.2)

The circular frequencies w of beams with doubly syn1metrical c ross sect ions are
given in Thble B.2. The circula r freque ncies w of bean1s wilh unsymn1etrical cross
section. where the coordinate directions y and z are in t he principal directions
(/1,, = 0, page 208), are given by the solution of the following equation:

-(zc-z..)]
[~
0 0
w'y
0
0 ] - w2 [
w~~
01 1
- (tc - z~) (yG - y,J
"i
(Ye Y~) =0,

(B.3)
where wy. w.,. a nd w~ are given in Table B.2, Y~ and z~ are the coordinates of
the s hear cente r, p is t he nlass per unit le ngth, Ya and le are the coordinates of
the c,e nter of mass (Fig. 6.95). and e is the polar moment of mass about the shear
center (Eq. 6.411).

461
462 BUCKLING LOAOS ANO NATURAL FREQUENCIES

Table 8.1. Buckling loads of beams. For buekling In the >-zplane 'ii", = 'ii,., fl = Byy,
and S = Szr For buckling In the >-yplane N"' = N,,, ff = Ei,,. and S =
. .... --. 1 ---- , ........ , ....
s,,,. For rorsional
bucklRlg fie,~= N" + lfGI,, El = e E(_,, and S = j! S~.
Bucking load
No shear With shear
Geometry, loading deformation deformation

()
n..- N: s
-- - -'-+..!.

(b) j:;.====o.~ -

() ..~ ---"'E~ -
(d)
BUCKLING LOADS ANO NATURAL FREQUENCIES 463

Table B.2. Circular frequencies of beams. I = 1, 2. 3.. '..:.: where i = 1 ~lonas to


the fU'St mode. For vibration in the x-z plane "''' = w1, EI_= El'" and S = s,,.
For vibration in the x-y plane WzJ = WJ. {7 = 'f1 zz, and S = Sy,.For torsional
Ct Bit ~ i
vlbration(w.;1)2 =wf + if(~)2 , p = 1f.and ~ = 71.
1 1
- --- + -1-
wi - (wrJ' (wn'
No shear deformation No bending deformation

Geometry (w~)' = ~ (1';') (wf)' = ~ (~g)'

====>.~ ., = 3.927
(b) ;=j
., = 7.069
s.; =ai= i lr
Bt =:::: (i +0.25) ;r

(c) ~ii====!
., = 4.730
., = 7.853
s; ::: (i + 0.5) n

Put = J.875
(d) /J.SJ =GI = (i - 0.5) 1f
lU =4.694
u; ::: (i - 0.5) n

., = 4.730 51 = 0 1 = 6.283
(e) ., = 7.853 = 1-'0l = 8.987
., = t0.996
$1
sJ = o.i = 12.566
.., ::: (i +0.5) n s. = /1-(iJ =:::: (i + l) ;r
APPENDIX C

Typical Material Properties

The properties of typical n1aterials are given in Tables C.1- C.5. These prope rties
are non1inal only in that they may vary ov..ing to variations in the n1aterial and in
the n1anuJacturing process.
Typical thickness of a graphite epoxy ply is 0.127 mm (0.005 in.).

Fiber
En longitudinal Youngs modulus
Gn 2 longitudinal shear n1odulus
' "112 longitudinal Poisson's ratio
sn tensile strength
r~n n1axin1un1 elongation
<in longitudinal thermal e xpansion coefficient
i1-r1 transverse the nual expansion coefficient
Pr density

Matrix
Em Young's modulus
Gm shear n1odulus
Vm Poisson's ratio
Sm tensiJe strength
r~m maximun1 elongation
Pm density

Composite ply
Vf fiber volun1e fraction
Poon~p density
,. , longitudinal and transve rse Youngs n1oduJi
G12. longitudinaJ shear modulus
longitudinal Poisson's ratio

464
TYPICAL MATERIAL PROPERTIES 465

1'23 transverse Poisson's ratio


+ - tensile and con1pression strengths in the fiber direction
S1 , S 1
si . sz tensile and con1pression strengths perpendic.ular to the fib-ers
Sn shear strengths
a,. a2 longitudinal and transverse thermal expansion coefficients
~, , ~. longitudinal and trans"erse moisture expansion coefficients

Table C.1. Fiber properties

Material
En
GPa
6 112
GPa
...
II Pa
I/fl
%
an
10- rc au
Pl
..L
"" <'
Glass
E-glass 72 33 0.09 310.l 4.3 5-5.4 2.6
S-glass 87 37 0.18 3792 4.4 1.6-2.9 2.49
Aratnid
Kevlar 29 83 29 0.44 3620 4.4 1.44
Kevlar49 124 43 0.45 3620 2.9 -2 59 1.44
Kevlar 149 172 65 0.33 3448 2.0 1.44
Graphite
AS4 234 93 0.26 3930 1.7 - 0.9 LS
IM6 276 109 026 5102 1.8 1.74
IM7 276 109 0.26 5309 1.9 1.77
IMS 303 120 0.26 5447 1.8 1.8
PANEX 228 0.25 3600 l.6 1.72
T300 231 91 0.27 3241 1.4 - 0.6 7-12 1.77
T40 283 107 0.32 5654 2.0 1.8
TSO 393 159 0.24 2413 0.6 l.S
T55 379 142 0.33 1724 0.5 1.99
TIS 517 204 0.27 2069 0.4 1.99
Boron 400 0.2 3160 0.79 4 .5 0.2 2.57

Table C.2. Matrix properties

Material
E.
GPa
Gm
GPa

MPa
IJm
% ""'..L
" '""
Epoxy
Der 332 3.4 l.2 0.35 64 1.9 1.22
934 4.1 l.S 0.35 83 2.0 1.3
Epon 2.8 1.0 0.35 83 .l.O 1.30
3501 4.4 1.6 0.36 69 1.6 1.26
5208 3.9 1.4 0.35 50 l.3 1.27
1962 3.7 1.4 0.35 1.27
Vinylester
1222 3.2 79 2.5 1.3
Polyester
7241 :l.3 79 2.4 l.38
TYPICAL MATERIAL PROPERTIES

Table C.3. Ply properties


., p..,, E, E, G12
Material % ,,.,
...!.. Gl'a GPa Gl'a l'll
""
Graphite/Epox)'
1'30015208 70 1.6 181 10.3 7.17 0.28 0_59
ASl3501 66 1.6 138 8.96 7.1 0.3 0.59
H-IM6/epoxy 203 11 .2 8.4 0.32
AS41APC2 1.6 134 8.9 5.1 0.28 0.66
TIOOl934
tape 148 9.65 4.55 0.3 0.6
cloth ( 13mil) 74 74 4.55 0.05
doth (?mil) 66 66 4.1 0.04
Gl;1ss/Epoxy
Scotchply 1002 45 1.8 38.6 8.27 4.14 0.26
S-glass/Epoxy 50 2.0 43 8.9 4.5 0.27 0.40
\VovenGl/Epoxy 45 2.2 29.7 29.7 5.3 0.17
Ara.mid/Epoxy
Kevlar 49 60 1.5 76 5.5 2.3 0.34 0.37
Boron/EpOx)'
8 (4)15505 50 2.0 204 18.5 5.59 0.23
85.615505 50 2.0 201 21.7 5.4 0.17
E-glassNinylester 24.4 6.87 2.89 0.32

Table C.4. Ply properties

Material
t
MPa
..-
MPa MPa
s: Si
MPa
. ,,
MPa
Grt1phite/Epoxy
1'300!5208 1500 1500 40 246 68
ASl3501 1447 1447 52 206 93
H-IM6/epoxy 3500 1540 56 150 98
AS41APC2 2130 1100 so 200 160
TIOOl934
tape 1314 1220 43 168 48
cloth ( 13mil) 499 352 4 58 352 46
doth (?mil) 375 279 368 278 46
Gl;1ss/Epoxy
Scotchply 1002 1062 6IO 31 118 72
S-glass/Epoxy 1280 690 49 158 69
\VovenGl/Epoxy 367 549 367 549 97
Aramid/Epoxy
Kevlar 49 1400 235 12 53 34
Boron/Epoxy
8(4)15505 1260 2500 61 202 67
85.615505 1380 1600 56 125 62
E-glassfVinylesLer 548 803 43 187 64
Table C.5. Ply properties
c;,
Mata111 "
10 ' tr:'
Graphiti:/Epox-y
T.lOOl5208 -0.7 25.0
"' "'
0 0.6
ASf.l.'IOI -0.7 29.0 0 0.4
Glass/Epoxy 8.6 221 0 0.6
Aramid1Epe>J1y
Kc'tlar.&9 - 4.0 79.0 0 0.6
lloron/Epe>Jly
8(4)/SSOS 6.1 30.3 0 0.6

Conversloo Factors between U.S. Customary Uni1s and SI Units


To colMWt from To Muni!Jly by
inches ( in.) 1nilli1neters (mm) 25.400
inches ( in.) o'leters (1n) 0.0254
foo1 (11) m eters ( m) 0.3048
squart inchei> ( in.1) square ntete rs ( m?) 0.000 645
square (eel (Ct: ) square me te.rs ( m?) 0.092 903
aJbic feet (ft1 ) cubic meterS. (m:;) 0.028 317
C)'Clts ptr second htrtt (th) 1.0
pound-ma"' (lb) kilograms (kg) 0.453592
pound-force (lbf) nc-..1ons(N) 4.4-18 22l
kilopound-loru ( kip) kilonc~'toos (kN) 4.4-18 222
pound-force per s.qu:a.rc DC'4'r10mfmetcr2 47.880
fool (pd) c;,=""l
pound-force per square kllone\\'lOnslmele-r1 6.894 151
inch (psi) (~ = kPl
ki lopound.rorcc per 11qunre 1neganew1onsfme1er1 6.894 757
inch (ki;i) (~= MP)
inch-pound force (i n.lbl) n ewton-meter ( N m) 0.1 12 985
(Ot)lpt)und force ( fllbl) oew1on-1neter ( N m) U 55 SIR

SI Untt Prefixes

MUftipi catfon Factor Pcefix Symbol


I 000 000 000 000 10" ltrra T
I 000 000 000 10' gig G
I 000 000 10' mega M
I 000 10' kilo t
100 - 10' hecto h
10 dtka da
0.1 - 10- deci d
0.01 io-' ~nti c
0.001 - w-' milli m
0.000 001 - lO micro
0 .000 000 ()()I - w- oano n
0.000 000 C)(J(l OOI 10'" 11 pico p
Index

A matrix. 70-3. 80 rcs1rained--warpinj;induccd lorquc. 263,


An1plilication factor. :302 313-19, 339, 354
Average stress criterion, 431-2 sandwich. Ste Sandwich bcan1s
shear center. See Shear center
Back cakuJation or engineering constan1s. 45 1 shear deformation. See Beams with shenr
cxa1nplc. 451 dcfornuition: Sandwkb bea1ns
8-matrix. 70-3, 80 shear rorcc. 203
Bcan1s stiffnc.u. 204-8
applied loods, 203-4 strain-displacem<:-nt relationship, 204
bcndins . 206-JO summary or cqua1ions.. 31 1
bending stiffness. 206. 207. 208. 209. 210 with s~n1mc 1rical cross section, 2~7
bounda ry conditions, 205, 206, 264 tcn:>ilc stiffness. 206. 2U7. 208. 2 10
box-beams.~!'! Box-beams thin-walled be-a.JM. Set aoscdSCClion beams
buckling. Sl'e Buckling of beams; Buckling or under nxial load and bending: Open
bcan1s with shear dcforn1ation section beams under axial load and
C-scction beams. See C-bcanu bending: Stiffened beams; Thin-wuUed
ccntrciid. 203 beams unde r axiaJ load. bending. and
wilh circula.1 cross section. See Beams v. ilh torsion: To-rsion of thin\l.'tltled beams
circular cross section torque neutral surface, 254. 258
compl iance, 209-10 torsional stiffness. 206, 208. 2 10
coordinate systc1n. 203 transverse!)' Jondcd. Ste Transversely londcd
coupling.. See Coupling beams: Trans\crsely loaded be-nms with
curvature. 204 shear deformation
cquilibriu1n, 204 T-sec1ion bcanu. See Tbcams
force-strain re latio nship. 204, 206-8. 209 twist.203
iovcrning equations. 20.l-5 \ibration. Set Vibration
lbcams. See J-bca.ms warping stiffness. Stt> \Varping stirfnes.s
internal forces. 203-5 Zsection bea1ns. ~e Zbcarns
isotropic beams., 206 Bcan1 ckn1cn1, 397-8
L-bcams. See L--bcams Bean1 equations. sumn1ary of, 311
neutral plane. 2Ll. See al.tt> Neutral plane Bean1s with circular cr0$S section, 454. 455, 458
orthotropic bcan1s, 207-8 Bcan1s with shear dcforn1ation, 313-64. Si~ abo
rate of twist. 204 Sandv.k h beams: Trans\crscly loaded
rcctanguJar solid cross section. Se.e beams iAith shc.ar dcfetrn1a1ion
Solid-scC1ion be.ams under axial load and bendini n1on1cnt, 319
bending bendins stifrncss, 322
rc rercncc plnne, 213. Ste also Reference boundary conditions.. 321
pJanc buckling. See Buckling of bca~ wi1h shear
rc-pla-ccmcnt stiffness. 210 deformation

469
470 I NDU

Beams wilh s.hcar deformation (coutiuue.( ) examples. 2.'i5, 257. 272. 304
coordinnlc sys.icm. 313 loe.al buc.k.ling. ~3. 304-5
deflection. 313. 330-4 neutral surface. 258. 259
dcfonnaiio n, 313-14 paran1eter or restrain!. 30ol
crfccts of s hear deformation, .)59-64 reference surface. 159
equations.. swnmary of. 320-L shear compliance. 458
equilibri um . 320 spriog constnnt. 304
force-strain rel ationship. 315- 19 sti.ffne.~ 256. 2.58, 2.i9. 273
governing cqua1ion.s, 314-21 tensile stiffness. 159. 454, 455. 456
lbcams, 315-18, 324-5 torque neuual surfatt. 258
rlltc. of twist, 314. 315, 317 torsion. 255-7. 257-9
rcs1rnincd-warping-induccd torque, 319 torsional stiffness. 256, 2.'i8. 458
rota1ion oft he cross-section, 313. 314 twist. 257, 259, 274
Saint-Vcnant torque. 317-19 warping stiffness. 458
shear compliance. 320. 321-6 Bredt-Batho formula, 253
shear force, 319-20 Buckling an1plilka1ion factor. 302
s.hcar stiffness 310, 321. 322. 325-6 Buckling constant. 292-3
strain-displ.acc1ncnt relationship. 315 Bu.:-kling length. St:e Buckling: of plates
SI rain c ncri,')', .l22 Bucklin.s load parameter. See Load pnrameter
tensile stiffness. 322 Buckling load. Se~ Buckling of: beams: be1uns
torsional s1iffncss. 322 with ~hear dcfornu1tion~ cylinders: plates::
transverse shear strain, 3 I 3 shells-
vibration, 347-59 Buclding of beams. 2<.K>-:306. 461-2
warping stiffness. 322 effceti\e kngth factor. 292, 293
Ben ding-bending coupl ing. 7.l e.xamples, 294. 298. 304. 305
Ben ding momcnl. See ~ton1cnt Oexural- torsional. 2914">
Bcn ding-shc.nr coupling. 73 forcc.-strain relationship. 291
Bending stiffness lateral-torsional, 2~300
of bc.nms v.ith she.ar deformation, 322 loc.al buc.k.ling. .~5
of box-beams. 2.'i6, 258. 454. 455, 456 orthotropic bi:an1s. 290-1
of C-bean1s, 285, 454. 456 spring constant, 301-4
of circular cross-section be.ams, 454. 455 Buckling ofbca1ns with shear deformation,
of closed-section beams. 231. 232. 234. 235. 334~7

246 equilibrium, 335. 339


of I-bea1ns. 295, 454. 455, 456 examples. 342, 343. 345
of Lbctuns., 223-5 Ocxural. 3J5-9
of open-section beams., 230. 231, 231.134. 2.15. Ocxural-torsional, 34 1~
236 l.11ter11l-1orsional. 345
of sandwich beams. 327. 328. 333. 334 s.hear deformation effects. 359-62
of stiffened beams, 289 torsional, J39-l I
ofT-bcams.. 219- 21, 140 Buckling of cylinders. J87-94
Beodiog sti((nC$$ ptiramcler. l(X) buckled shape., 389
Bending-t" is1 coupling,. 72-3. 76 buckling, pattern, 387, 390-1
BcrnouJli-Navier h~pothe.sis. 203, ill. 267. examples. 387, 39 1
3 13 \l.'ith ortbotropk wall, 3~3
Biaxial test. 418-19 with s:tndwich wall, 393-&
Bimomcnt. 262. 317. 318. 319.354 Buckling of plates. 112-tl
Boundary ronditions. 47-S under biaxial load, 112- 1.5, 118-22, 187
beams. WS. 206. 264 buddedshapc, 122.125. 127. 116
beams with shear dcfom1otion. 321 buckling. length, 135
plates. 92, IOI. 113. 145 buckUng: lo.ad. 123. 125. 136
sandwich plate.s, 172-3, 183 wilh built-in and simpl)' suppor1ed edges.
torsion of beams, 264 118-24
Box-beams, 155-60, m~ . ~~3 equilibrium, 132
bending stiffness. 256. 258. 454, 455, 456 examples. 115. 116, 121. 126. 130
centroid. 274, 454, 456 examplc:s. longplatcs.: 13:\. LIB, 139
compliance. 274 examples. sandwich plates.: 186, 188. 195
dcRection, 257, 259 face wrinkling,. I~
elongation. 260, 274 with free edge. 124-7
INDEX 471

Buckling or pliltCS (corifittued) displacements., 243-7


under l i nc-al'ly \':uying load. 140 rorce-stroin relationship, 24 7, 267
load p.aramc1cr, 112. 119 neutral surface. 45"3
long platc.s with various cdg:e suppo-rts, l.1'2-11 tensile s.tiffnc.ss, 231, 232, 2J4. 235, 247
potential energy. I 18, 125 transversel) looded. 278-80, 282-3
potential of external forces. I 13 Con1pliance. Stt also Shear eontplianoc
principle of -"1ationary potential e nergy. 113, of bc--mns. 209-10
118 of d osed-section beanls, 247, 254, 266-..~. 273,
Ritz method, 113. I 18, 183, 188 274
with rotationaUy rcs;traincd edges. 127-31 constraint on, 56. 58-6'1
snndwkh plaic.s., 18.5-8 exa1nplc, 21
shear load.under. 14~ 1 of iso1ropie mntc-riaJ. 12. 14-15, 21. 35, -12
with simply supponcd cdgc.s; 112-18 of generally nni$otropic nu1tcria1. 9-11 . 29, JO.
strain energy. 112 .\6
under uniaxinl load. 122. 123, 124--39 of l:uninnte$.. 71-2, 74, 76. 77-9, 81-8
Budd ing o f sbcUs. 384-7 of n1onod inie mn1crial. 12- 15. 32-3
budded shape., 386, 389 in new rcrcrencc plane. 8 1
buckling pattern, 386, 387, 390, 391 of open-se,.1ion bca~ 266-8
cx:unplcs, .l.87. 391 of ortbotropic 1natcrial. 12.14-IS. 34.41
under plancstrain. 30, 32. 3.\ 34-5
C-bcams. 284-8 under plan<:strcss.. 39, 41. 42
bending stiffness. 285. 4:54. 456 of sol idsec~ion bean\S... 212-13, 2 14. 2.50
centroid. 285, 454. 456 transfornuition, 53-4
coordinate.system, 285 of tmns\erscly isotropic 1naterial. 12. 14-15.
dcftcction, 286 20,35,42
examples. 284. 287 of woven rabric. 44
shear ccn1cr. 286, 457 Consenation of energ:'' 145, 323
shear compliance. 459 Constituti\c cquntions. See Strc.s s-strain
tensile stiffness. 285. 454. 456 rc.lationship. Forcc-straio rcln ti on.~hip
torsional stiirncss, 286. 457 Continuity condition. 48-9
lwiSI. 286-7. 188 Con\'t:rsion factors, 467
warping stiffness. 288. 457 Coordinate s~cm, .~
Center or mass... 309, 310 of beams. 203. 211. 313
Center or g.mvity, 20:1. 213 of Cbcams,.221
Center or twist. 25'2 of closed .section bcanls. 243. 26.'i, 267, 268-9
Centroid. 203 of isotropic mntcrinL 20
of box-bean1s. 274. 454. 456 of laminntC$.. 63-4. 66
of Cbean1s, 28.'i, 454, 456 of nl onocJinic mntcrinJ. 11
of dosed-section bean1s. 231. 232. 234. 235 of open-section beams,. 226-7. 26.i. 2<>6. 267.
of 1-be:uns. 295. 454. 455. 456 268, 269
of Lbcnms.. 223 of onbotropic material, 14. 16
of opensc,tion be.nnu., 230, 231 232. '234. 235. under p-lanc.-s1rain. 22
236 under p1anc-.str~ 38
of sandwich bean1s, 328 of plntes.. 90
of solidsc-etioo beams. 213 ofTbea1ns. 2 I 7
of stiffened beams. 289 translorn1a1ion. 49-50
ofTbc.nms. 219. 240 of tranS\'Crscly isotropic material, 19
of thinV"alJcd be.ams \loitb arbitrary layup, 271 Core., 169. 170. 326
Characteristic Jcnsth, 431-2. 434-5 Coupling
CircuJar frequency, 141. See also Vibration in balanced lnminates. 75
of bc~1ms. 306, 350, 352, 463 in bcanls in 1orsion. 248
of plat~ 141, 196 in bean1s with symmetrical cross sec1io n.
Closedslion beams under axial load and 20<~7
bending. 243-8. s~ al.~o Box bcnnts in C)'Undcrs. 376
bending stiffness. 23 I. 2.l2. 234. 235. 246 in isotropic Lnminates, 78
centroid, 2ll, 232. 234, 235 in lamina1cs. 72. 73
comp111ibili1y condi1ion. 245-6 in monoclinic olalcrial. 12
co1npliance. 247 in orlhotropic beams. 207-8
coordinate system. 243, 269 in orlhotropic closedst<::tion bc3ms. 243
INDEX

Couplins (ro11tiriut d) of laminatC'$., 66-7


in orthotropic laminate$.. 75, 76, 77 of L-beams., 221
in ortbettropic material. 17 of opcn-scc.tion bcanu.. 226, 265-7
in etrthotropic openscciion be.ams., 226, 229 under pJane.-strain. 25-8, 30, 33, 34. 35. .l7
in orthotropic solidse<tion bcnnu.. 213. 214 of plates.. 91
in qunsiisotropk laminates. 80 of sandwich plates. 169, 170, 199, 200
in sandwich beams.. 333 of solid--section beams. 210-14
in sandwich plates. 176 sun1mary. 57-8
in solid-section beams. 212. 2J4 ofTbcams, 217- 18
in thin rectangular cr<>.($SCction bcan1s. 250 Di.splattment function of \ibrating beams.. 119
transversely Joodcd thin-walled bea1ns. 275.
276 Effccthc length factor. 292. 293
Critical lo..'ld paran1eter Engine-c ring constants
ror C~1linders. 386, 388-9, 391, 39'2, JCJ3, 394 constraint on. 6 1
ror plates. 112, 115, 118, 121 example. back calculation, 451
ror sandwich plntcs.. 188 of isotropic material. 13, 16. 20
Curvature. 67-8 1nicromccbanics expression, 442
of be.ams.. 204 of n1onoclinic n1atcrial, 13, 16
of cylinders., 371 of onbo11opie material, 13, 16
of la1ninates... 67-8 ofuansvcrscl~ isotropic.matcri.al.1 3, 16. 19
of opcn-scctjon beams., 227. 234. 235. 251 of wo\en fabric. 44
of pla tes.. 90. 94, 95 Engi neering strajn, 3, 5, 6
of radius.. 365-6 Equilibrium equat ions.~
of shells. 365-6, 385 for beams., 204. 276. !SO
of solid.scction beams, 2 11. 215. 2 16 for benms with shear derormation, 320. 3.15.
Cylindrical defornu1tion. 94. 132. 142.301 339, 35.l
Cyli ndric-al shells, 368-80. See 11/so Bue.kilns of for buckling of plates. 132
shells for cylindrical shells. 371
with built-in ends. 370-5 for long plnh.-s. 95-6. 132
coupling.l76 under plane-strain. 29, 32
curvnture, .l7 l under p lanc~-strc-ss., 39-40
disp-lattment, 375-6 rorplate-s. 91. 142. 197
end cffocts. 376-7 ror sandwich plates.. 179, 197
equilibrium. 371 sun1mary oL 57
examples, Jn. 387. 391 . 427 Exan1plcs
mcn1branc forocs.. 369, 370 b3ck eakulntion or e ngineering con.st.ants;. 451
membrane theory, 366. 368-70 beam under la1cral lond, 215, 239. 255
with orthotropic walls, 375-6, .l90-l boxbc-Jlm. 155. 257, m. 30:I
strain -displacement relationship. 3'71 C-bc1un, 284, U7
temperature eCfeets. .l79-80 consuaints on con1plianccs and engineering
constants. 6 1
D matrix. 70-3, 80 C)lindc r. 377, 427
Deflection cylinder budling. 387. 391
of beams.. 313, 332 fatt. wrinkling, 195
of beams '"ith shear derormation. 313, 3~ failure. 427
of box-beams, 257. 259 hygrothcrmnl cffcc1s. 156. 159. 162. 383
of C-bean1s, 286 J-bcnm buckling., 294, 298. 305, l43, 345, 363
of long plates. 9~100 J-benm vibration, 3 10. 357
of plates.. 91-1, 100-7, IITT-12 laminate co1npliancc and s'iffness. 8 1. 83. 87
of sandwkh plates. 180. 182-4 local buck.ling. 304. 305, 387. 391
shear dcfonnation effects.. 359-62 longplaie buekling..133, 138, 139
of Tbc.ams, 241 plate buck.ling. 115. 116, 12 1. 126. 130
Dcforn1ation under plane .strain. 27, 31, 35, 36 plate under lntcral load, 96, 10.l. 105, 1I l
Dcla1nination, 167. 411-12 plate vibration. 143. 147, 148. 150
Density. 4.l7 ply compliance and .stiffn~ 21. 42, 43
Displacement, .l-4 .sandwich be.nm, 334. 342, 362
of d osed-section beams. 24.l-7. 265-7. 273-4 sand-wich plate. 180
of cyJindrical shells. 375-6 srindwich pla1c. budding.. 186, 188. 195
oflbeams.316 sand-wich plalc-' 'ibration, 198, 201
INDEX 473

Exan1ples (ro111i1111eti) Gene.raliz.cd fCt-rcc. 71


shear center. 284. 294 in bean1s with shc-nr dcforn1ation. 320
!ihcar now, 239 Generalized foroc~s11ain relationship. 71
shear deformation effects on b<:sms.. 362, 3-63 Generalized strain, 71
s pringback. 383 in bean1s with shenr dcforn1ation. 'I 5. 320
sublt1minatc .s1iffncss, 407 Generally nni.-.otro1>ic n1n.1erial
Tbcam. 2.J9 co1nplianec, 9-11, 29, 30. 36
woven fab-ric, 43 description, 8
Extcnsioncxlcnsion coupUng. 73 under plane.-strain. 28--30, 35-6
Extcnsionsbcar coupling. 72-3. 76 under planc..s-trcss., 38-10
Extcnsionlwist coupling, 72-3, 76 stiJfness, 9-11
stress-strnin relatietnship. 8-1 I
fabric. .'i~e \ VO\'CR fabric Go\'crning equations. 57
Fattshccl , 169. 326 etf be-nms. 203-5, 314--21
Face WTinl:Ling. 190-6 of pla1C$.. 90-2
Failure cn\clopc. 428-30 of sandwich plates. 17~2
example, 427 summary or. 57
Failure modcs. 411- 12
Failure surface., 421 Halpi~Tsai, 450-1
Failure theo ries. 4 11-35 Hill failure criterion, 4 12
applicability or. 412-13 Hetlc size e ffect. 433
Fiber failure, 411-12 Honeycon1bcorc. 169. 326
Fibers.. I Huber orthotropy. 108-12
propcnics. 465 1-lygrothcrnuat 44-7. 151--63. 379-84
Filan1cnt wound layer, 4 1 examples. 156, 159. 162. 38J
Pini1e ckn1cn1. 395-410 force, 152. 154, 155. 156
bcanl cle1ncnt stiffness. 397-8 rorcc--s:1rain relationship. 151-6
cxa1nplc. 407 under planc-s1rnin. 47
formulation. 395-6 under plane~rcss. 47
plate clement stifrncss, 397 properties. 450, 467
sublaminatc clcmcn1stiffness. 398-410 strain, 44-7, 153
thrcc~din1cnsionnl clc1ncnt stiffness. 396 strain-displacen1cnl relationship. 44-7
First order shc.nr theory, 313 stress. 152
First p-1>' faiJurc. 412 stress-strain relationship. 45-6. 47. 152
Flexural ri,idity. See Bending stiffness thicknC;S;S change. 161-3
Force-strain relntion.ship
ror beams., 204. 206-8, 209 J-benm.s
ror beams with shear deromt:dion, 315-19 bending: n1omcn1, 316
ror buckling or beams., 291 bendinit stifrness. 295. 454. 455. 456
ror closcd-scdietn beams. 247 bin1on1cnt. 262, 318
hygrothennnJ. 153. 154. 155 buck.Ung. 294-6. 29..~3<X>. 305-6. 343- 5. 346
(or laLninatCS. 7 1, 74, 75 center of mass. 310
ror opcnscction bean1s, 228. 2J6. 237, 238, 239 centroid, 295, 454. 455, 456
ror platCS; 91 displacement, .\16
ror sandwich plates.172 example. effect of shc~1r dcfom1ation, 363
ror shells. 366 cxa1nplcs. bu(kling, 294. 298. 305. 343. 345
ror solid-section bcanlS under axiaJ load and examples. vibration, 310, 357
bending.. 2 1I. 212, 213. 214, 215 local buck.Ung, 303, 305-6
ror Tbeams, 242 paramclcr of res1:rain1, 305
ror 1hin-waUed bcnms under axial lo.ad nnd pol.ar n101ncn1of mass. 310
bending. 266. 269 polar radius or gyration. 29:5
ror torsion of be:uns. 250. 255 rale of 1wist, 317
Free e-dge effect, 166-S. 210 restrained warping.. 26 1-4
Free vibration. Se~ Vibrntion rcslraincd warping induced torque. 263,
Frequency. See Natural rrcqucncy: Cir<:ular J l8
frequency Saini Vcnnnt torque, 263. 317
shear cenlcr. 295. 457
Genernlfa.cd dtsplaeement in beams with shenr shear comp-liancc. 324-5, 343-4, 459
deformation. 320 shear deformation effects. 363-4
474 INDEX

J .~.ums (Ct>111iru1t ,f) strain-displacement. 66-7. 75


with shear dcfornu1tion. 3 15- 18 Site%. 65-6
.shear flow. 241 sym1neuie. 63. 64-5. 74--5. 84, 85
shear stjffnc.ss. 318 transverse shear force. 68
spring constant. 305 unsymn1etric, 86
strain-displnccmcnt relationship. .1 16 L-bcants. 221-5
tcnsilc. i;tiffncss. 295, 454. 455, 456 displacement, 221
transverse shear forcc . 316 centroid. 223
torque., 261. 262. 263, 316 bending stiffness, 223-5
torsion, 261-4, 315-18 tensile stiffness, 223
torsional stjffncs..;. 295, 457 Load parameter, 112, 118, 187. 384. See ul.\'IJ
\ibration. 310-12. 357-9 Critical load para1nctcr
warping stiffncsi:. 262-4. 295, 318, 457 Local budd ing
lncnia forc.e, 142, J48. 353 of cylinders., 387. 390-1
lnpl:ulc--0u1-ofplanc coupLing. n-3. 76 example. box-beam, 304
lntcraction strength parameters. 417-20. example, Jbcam, 305
423 examples. q lindcrs..m . 391
Jntcrlan1inar stresses, 166-8, 210 of thin\\'allcd beams. 300-4
Jsouopic nu11crial Long plates. 94-100. See also Bud:ling or plates:
compliance. 12, 14-15, 21 , 35. 42 Sandwich plates
coordinate S)~c1n. 20 :ipplicabilit)' or thCOf)' in bending, 94-5, 97
description. 20 applicability or thCOf)' in buckling. 116. 132,
engineering oonstan 1~ 13, 16, 20 l<JO
under planc.-strain, 34--5, 38 applicability or thcot)' in vibration, 148.
under planc-Slress.. 42 202
Poisson's ratio. 20 curvnturc, 95
shear mt'.ldulu.s.. 20 C) lindricaJ defom1a1ion. 94
1

stirfn~ 21. 35. 42 dctk-etion. symnlctrkal la)up. 96


Young' s n1odulus, 20 dctkction. unsymmetrical layup, 98-100
description, ?4
Kinctk energy. 144--5 cquilibriutn, 95-6, 132. 142
Kirchhoff hypothcs:i1;, 66, 169 examples, 96. 133. 180. 198
\ibration, 141-4. L9f>-9
Lamina, I. 2
Lantinate plate theor)' 65-72 Mass. 437
Laminates, 6.l-88 r>.fatcrial propcrtiC'S. See Propcrtic.'$
angle-ply. 65 ri.tatrix crae>k.ing, 411- 12
balanced. 65. 75 ti.fat:rix properties. 465
compliance. 71-2, 74. 75. 76. 77-9, 81-8 r>.fnximu1n strain criterion, 426-7
coordinate system. 6~. 66 example. 427
couplins. 72-3, 75, 76. 77. 78. 80 ri.tsxi.mu1n s.trcss crilcrion. 42.'HS
cr~~ply, 65 example, 427
curvnturc. 67-8 ti.fcn1brane rorccs. 365-70
curved. 81 ti.fen1br:ine theory. 365. 366. 368-70
displacements., 66-7 r>.fcridian. .'67
ex.nmples. 8 1, 83. 87 M icromecbani~ 436-52
rorceSltain relationship, 71, 74. 75 engineering constants.. summary or. 442
in-plane rorces, 67- 8, 70, 74 bygrothcnnttl properties, sun1nuu-y o(, 450
isotropi<. 78, 79 ti.fodifi cd ruk or mixtures, 448-9
laminate code. 63 ri.toisture concentration. 44. 445
moments.. 67-8, 70. 74 ri.tois1urc diffusi\'ity, 447-8. 450
neutral plnnc, 66 ti.foisture c.xpansion coerficicnt. 44, 46. 445-6,
nonnal shc.nr rorce. 67 450
orthotropic. 75-8. 207 rt.fo1ncnt
!t/4 laminate, 65. 79-80 in beams. 2 10. 2 1I. 316. 327. 332
qua..tj-isotropk , 79-80, 177 in lantinates, 67. 70. 74
rcrerenC<." plane. 66-7, 68. 69. SO-I in plates. 91. 96. 99-100.108. 109
rotation. 66 rt.fo1ncnt couple. See Bimomcnl
stirrness.. 70-3, 74. 75, 76. 78. 79. 8 1-8 f\.foment of incrti:i, 208, 261
INDEX 475

f\.1onodinic nu11crial stress-strain relationship, 17


compliance. 12-15 , 32-3 Yo u ng's n1odulus. 13
coordinate system, 11 Out-or-plane s hear ro-rtt.. St!e Shear rorcc
description, 11
engineering 1.'-0nstants. 13, 16 Parallel axis theorc.1n, SO
u nder planc.strain . 30-4, 36-S Par:unc1er of rd>trainl, 128, .104-5
under plane-stress.. 40-1 Period of vibrution. 141
Poisson's ratio, 13 Plane~1rain . 22-38. 267
shear modulus, 13 body fo rce. 22. 24
51.iffn~ lo&, 41 compliance. JO. 32. 33, 34-5
"1ress-strain rclationi;bip. I 7 coordinate system, 22
Young's n1odulus. l.l definitio n, 22-5
f\.ionocoquc bca1n, 311 dcforn1a 1ion. 27. JI . 35. 36
d isplacement. 25-8, 30. 33. 34. 35. 37
Natural frequency, 14 1. Ste.> also Vibrntion end loads. 24-5
Ncutrnl plane (surface). 230. 231. 254 end suppo r~ 23
in box-beams.. 258, 259 equilibri um, 29. 32
in doscd-sec:tion lx:am$.. 453 fi ber o rienta 1io n. 2.3-4
in open-section beams, 230. 2J1 , 233. geometry. 23
453 generally aniso tropic material, 28-30.
in sand wich beams.. 328 .!5-0
in ssndv.ich plates, tn bygrothe m1al. 47
in solidscction beams, 2 13 isotropic m aterial. 34-5. 38
Nonintcraction strcni;th parameters.., 416--17, mo nOoClinie m aterial.~. 3f>-8
420. 421 . 423 o rtho tropk material, .14-5. 38
stiCfn..-s1<., 30, 33. 3~5
Off-axi.~ unidirectio nal test, 4 17- 18 strain-displ.nccn1ent re lationship. 2.1~. 2&. JO,
Open-section beams, 2 17-43. s~e also C-bcaLns: 3"3. 35. 37
I-beams; ~bcan1s; Tbca.ms; Torsion or strC$$-Strain relationship, 29-30 .12. :33, 36, 37,
thin-walled beams; Ttansvcrscl) Joodcd .!8
th in-walk-d beams surface force, 22. 24
bend ing stiffness, a rbitrary cross section. 230, transverscl)' iso1ropic malcriaJ. 34-5. 38
2.11, 2J2, 234, 235. 236 Plane-stress. 38-44. 66
centroid. 2JO, 2.11. 2.12, 234. 2.35. 236 compliance, 39. 4 1, 42
coordinate system, 226-7 coordinate systc1n, 38
curvnture. 2!7. 214, 235 defin itio n. 38
displaocmcnt. 226 eq uilibrimn. 39-40
example, 239 examples, 42. 43
rorccstrain rcl.ntion~hip. 228. 236. 2.l7. 238, ~cncrally anisotropic ntaterial. 38-40
2.19 bygrolhc nnal. 47
neutral surface. 230, 23l. 2.13. 453 isotropic m at erial, 42
rercrcnce surface, 230. 2.13 mo nOoClinic m att rial, 40-1
scs:mcnted walJ. 2JJ o rtho tropic material, 4 1
shear llow, 234. 241. 275. 278 stirfncss.. 39-10, 41. 42-3
strain, 234, 235, 237. 269 s trCS$., 38, 39
tensile stiffness. arbiu ary cross section. 229. stressstrain relationship. 39.-10
2.11. 232, 234. 235. 236 transversely isotropk materi.nl. 42-3
torsion. 250-2 Plate c.Jcnte nt. :397
transverse!)' lo:adcd, 276-8. 280 Pla1cs. 89-94, 100-12. Set! lll.(O Long plates~
Onhotropic material Sandwich plates
compliance. 12., 14-15, 34. 41 bending n1on1cnt. 91. 108, 109
coordinatcsys1c1n. 14, 16 boun d.Ju)' cond itions.. 92, 101, I l 3, 145
desl~ription. 14 buckling:. See Buckling of plates
ensineerin g constan ts.. 13, 16 coordinate system, 90
u nder plane- strain, 34-5. 38 cun:-atu.rc, 90. 94
under plane.-str~ 41 deftectio n o r b uiltin plnte.1<., 107-12
Poisson's ra1io. 13 dcftection o r simply supported plntcs.. 100-7
.shear modulus, 13 deflection unde r pure. bend ing and inplane .
stiffness..17, 34. 4 1 loads. 9~
476 INDEX

Plntcs (cor11inue,f) under propo11ional loading. 424-5


cquilibriu1n . 91 ror transverse!)' isotropic material. 420-1
examples, 103, 105, l l J Qunsihomoge.ncous material, 3
rorec-suain rchitionship, 91
governing equations. 90-2 Radius of curvature.365-6
byg.rothcmu1J effects. See Hygrothcrmal Rate. of twist, 26, 248
in-plane foroC$.. 91 of bc-nnu.. 204, 315. 316, 317
potential energy, 107. IOS.109, 118.125 of d osed-section beams. 253-4, 260. 279. 280
potential or external forces, 10 1 of l-bcains.317
principle of siationary potential energy. IOI. ~car deformation effects. 359-62
113, 118 of solidscction beams. 2 12. 214
rckrcncc. plane. 99 of thin rectangular cross section bcan1s, 249
Ri1z method, IOL of transversely loaded beams, 212. 214, 275-6
shear force , 91 Rayle igh energy method, 145, 149
strain-displncc1ncnt relationship, 90 Rayleigh principle. 145. 200
strain e nergy. 92, 93, IOI R<.'duccd bending s1iffn<.'ss. 90
strain energy, plate. buc.kling. L12 Referenoc plane (surface)
strain e nergy. vibrating pln!e. 144. 149-50 ror doscd-sedion bea1ns. 254. 259
twist mon1cnt. 9 1 ror laminates. <'6-7. 68. 69. 80-1
\ibration. 144-51 ror loni pla tes. 99
Pla tt-s "'i th a hole.. 16'3-6, 431-4 ror open-section bcantSi 230, "231. 233
Plate. "'ilh n no1ch, 4.34 rorsandwich pl.nles.. 170-1, 177
Ply. I. 2 ror sotidscction beams. 2 13
c.lUlmplc, compliance nnd stiffness. 2 I. Re inforced beams. See Stiffened beams
42.43 Re pin-cement shear force, 92
orientation, 63 Re placement stiffnC$S.. 210, 291
propcr1ics. 466-7 Re prescntnti ..c dcLncnt. 437-8
S>liffnes:s.. 69--70 R<.'s.traincd warpini. 26 1-64. 271 . 288
strl-:SS-Strain relat ionship. 39. 68-9 Rcstrained-warpinsinduocd torque., 263,
thickn~464 318-19, 339, 354
Ply group, 63 Ritz method, S:H;, JOI. 113. I 18, 183. 188
Ply stress-based failure. criterion. 4 12 Rotation. 26
Poin1 streS5 fnjlure criterion, 431-t of bcnms with shear deformation. 313, 314
Poisson's ratio. 1.3, 19. 20, 178. 441-2 of laminmes, 66
major nnd n1inor, 62 of sandwich pln1cs. 169. 179, 18.\ 199, 200
Pol:tr n1omcnt of mass. 307, 309, 310, Rota1ion.nl shear stiJfne.ss. 318-20
354. 361 Ro1a1iont1I spring constanL &e Spring consln.nt
Polar radius or g,,vra1ion. 292, 295, 339, 461 Ruic of mixtures, 436-9
Potential energy. 55
for pistes. 107, 10&. 109 Saint-Vcnnnt torque, 263. 317- 19, 339, 354
for plaH: buckling. 118, 125 Sandy,;ch beams. 326-9. See also Buckling of
Potential or external forces. 55 beanu with shc.nr deforn1a1ion
rorpla1e.s, IOI, 183 bending stiffness. 327, 328. 333. 334
ror piste buckling. I l3 centroid, 328
Prindplc. dircc1ion, 208-9 core. 326
Pri.ndplc or stntionary potential energy. 56 coupling, J.13
rorplstes. 101, 184 examples. .'J4, 342. 362. 363
ror plate. buckling. 113. 118 neutral plrinc. 328
Propcnics... 464--7 shear center.328
used in the. cxan1plcs.. 81 shear compliance:. 327. 329. 334
Proponjonal loading, 423-25 shear deformation c ffcds. 362-3
shear foro<.\ 326-7, 333
Qlllldratic failure criterion, 413-25 shear s1iffncss. 329. 3".l.l, 3.14
applicability or, 4 12- 13 tensile stiffn<.'ss. 328
e.x.amplc. 427 tomonal stiffnc:ss, 329
ror isotropic n1a1e rial, 421- 2 transversely loaded. 331~
ror 01thotropic material, 4 14- 20 warping stiffness. 328
under plane.-strain, 422 Sandwich plates, 169-202
under plane.stress.. 422 boundnry conditions.. 172- 3. 183
INDEX

Sandwich plates (contiuuevl) in open-section beams. 276-&, 281


buddjng of long plates. 185-7 in stiffened beams. 290
buckling of sintpl) supported plates. 187-8 in transversely loaded bcnms., 275-83
core. 169, 170 Shear force
core.stiffness, 175 in bean1s. 204
coupling, 176 in bean1s with she.or deforn1ation, 319-20. 3JI
dcth.'<tion of long plates, I 78-80 in lantinates. 67, 68
dcRcction of simply supported plates, 182~ in plates. 9 1
deformation, 171. 175-6 in sandwich plate.s. 171-2. 175. 176
displaocmcnu., 169, 170. 199, 200 in sandwich be.ams., 126-7, 3'33
equilibrium, 179, 197 Shear modulus. 13, 19, 20. 439-40. 442
example. 180 Shear strain in thin-walJed beams.. 251. 253-4,
examples. buckling. 186. 188. 195 2:19
examples. vibration. 198. '201 Shear stiffness
faccshcct, 169 of be.ams wit h shear dero1mation, 320. 321,
Catt. wrinklins. 190-6 322.325-6
for<~cstrain relationship, 172 of lbenins.318
govcrningcquntions. 170-2 of sandwich bean1s. 326. 329. 333. 334
isot ropic plates, 176-8 ofsandwieh platcs. 172. 176. 178
long plates.. 178-82 Shells. 365-8. St!e also Bud::Uns of .shells:
neutral plane. 1n Cylindric-al shells
orthotropic plates. I 76 coordinate system. 365-6
Poisson's mtio, 178 curvature. 385
potential of external forces, l&.1 force-st.rain relationship. 366
q uasiisottopic. 178 men1brane rorcc-s. 365-8, 369
rcrerc-nccplnnc.1 70-1, 177 men1brane theory, 365, 366
rotntion.169. 179. 183, 199.200 meridian. 367. 368
.shear for.x. 171- 2. 175.176 radii of curvature. 36.~
shear stiffness. 171.176. 178 strain. 367
stiffness.. 174--S stress-strain relationship. 367
strain, 182 Shells of rc,olution, 367-8
!>traindi..~placcment reh1tionship, 170-1 Sl unit prefixes. 467
strain cncrg:~. 17~, 182-3, l 99-200 Solid-section beams under axial lood and
stress-strain relationship. 17l.175 bcnding..2 1CJ...15
transverse shear strain, 170 axia l rorce. 2 1I
\ibration, 196-202 bending n1on1cn1, 211
Sentimonoooquc beams. See Stiffened bean1s cenlroid. 213
Shear center, 252. 283-J compliance, 2 12- 13. 2 14
of Cbean1s. 286. 457 coordinate system, 211
example, 284 curvature . 2 11. 2 15. 2 16
of I-beams. 295. 457 displsc.cmcnt. 211- 14
of sandwich beams. 328 equations. sumnu1ry of, 31 1
of transverse!)' loaded beams. 275. 28J-4 example, 215
Shear con1plinntt rorcc-slrain relation.ship, 2 1l.212, 213. 214,
of bc.nJru: with shear deronnation, 320. 32 1 ~ 115
of box-beams. 458 neutral plane. 2 13
of Cbean1s. 459 rate of twist. 212, 214
of cir~~ulnr cross-section benms., 458 rcrerence plnne. 21.1
of lbenms, 324-5. J4J-l. 459 s:lrain, 215
of sandwich beana. 327.."\29, 334 sotr~215
ofTbcams.460 strC'$.."strain rclstionship. 215
of Z-bean1s. 460 Specific heat. 448. 450
Shcnr derormation. erfeets of, 359-<>4 Sprinsback, .~
cxa1nplcs. 362. 363 exa1nplc, 383
Shear ftov., 234, 275 Spring constanl. 128, 301-1. 305
in dosed-section !><:ams.. 253, 260. 278-80, S1iffencd hcan1s, 288-90
282-3 axiul ron~e. 289
example. 239 bending stiffness. 289
in I-beams. 241 centroid. 289
478 INDEX

Stiffened bcan1s (continued) Suess. 3 , 5. 6


shear Oow. 290 doscdsection beams. 268--70
tensile. stiffness. 289 hygrothemt11l. 152
torsion. 290 lnminates. 65-6
Stiffn<:s...,. See dlso Bending stiffness; Shear no tatio n . 5
stiffnes.1: lbrsional stiffness; \Vnrping open -sect ion bcan1s. 268-70
s tiffness planc.-strc:u., 38. 39
of bc.'lms, 204-8 sign in\ention , 4
o f box-beams, 27.l-4 solid-section beams under axial load and
consttaint o n. 56. 58-61 bending.2 15
example. 2 1, 42 sun1mary. 5 7-8
of ft.lament wound I.ayer, 41 tran sforn1ation, 4 9-Sl
of gcncrall)' anisotropic material, 9, 11 Stress intensity factor. 16.'i. 432
of isotropic nH1tc1ial, 21. 35, 42 Stres..., ra1io. 42:.~S. 428
of laminates.. 70-3, 74, 75, 76. 78, 79, 8 1-8 Stress-strain relat ionsh ip. See alto Fo rce-strain
of n1onodinic nu1tcria1, 14, 41 relatio nship
of o rtbo tropk material, 17, 3.:1, 4 1 eq uations. sumn1al)' of, S7
under planc.-.strain , 30, 33, 34--5 generally aniso tropk material, 8-11
un der plan cstres:s;, 39-l, 4 1, 42-3 h yg.rothe nnal. 45-6. 47. 152
of p ly. 69-70 p-lancstrain. 29--30. 32. 33. 36, 37. l8
<f sandwich p lates. 174-8 planc:~stress.. 39-40
of thin>A'allcd bc:uns with arbitrary layup, s.and\\ich pl at~ 171. 175
266-8 she!~ 367
transfornu1lion. 53-4 solid-section beams. 2 15. 2 17
of uaMvcrscl~ iso tretpH: material, 20, 35, 42 torsion o f be.ams.. 251
of WO\'CO fabric, 41 mo noclinic material, 17
Stiffness paran1ctcr, JOS. 129 onh o1ropic materiaL J 7
Strajn, 3 ply, 39, 68-9
dosed-section beams. 269. 274 Su blan1inatc c lement , J 98-4 10
hygrotbcrmal. 44-7. 153 example, 407
no tation , 6 Surface ro rec, 22. 24
opcn scction bcan1s. 234. 235. 2J7. 26&-70.
274 Tbea~217-2 1
f.ltndwich p lates. 182 bending stiffness. 2 19-2 1, 240
shell~ 367 centroid , 2 19, 240
solid-section beams. 2 15 coordi nate systeln, 217
to rsion of betuns., 251 dcOe.c-t:ion . 24 l
sun1mary, 57-8 d isplaoc:mcnt, 2 17-18
trans forn1a1io n . of. 52-.l example, 239
S1rain-dis1>laccmcn1 relationsh ip. 4-6 ro rce-stra.in re latio nship, 242
beams. 204. 3 15 shear compliance. 460
C)'Undric11l shell$. 371 shear llow, 24 L
equations. sum1nar)' of. 57 tensile ~iffncss, 2 19, 240 . 456
l bcams.. 316 torsional stiffne.ss, 457
l.nminatcs. ~7 warping stiffness. 457
plane-st rain, 25-6. 28, 30, 33. 35, 37 Ten percent rule, 89-90, 117. 149. 392
ptates, 90 Tensile stiffness
sandwich p lates, 170-1 <f be: ams, 206. 207. 208. 210 , .1'22
thi nwallcd sectio n beams unde r axin l load. o f boxbean1s. 2.'i9, 454, 455. 456
bending and 1orsion, 265 o f Cbean1s. 28.'i. 4 54 . 456
Strain energy. 55 of circular crosssecti on be.ams.. 454 , 455
o f beams "'i lh shear dero nnation. J22 o f closcdscc1ion hean1s, 231, 232. 2J4,
of b ue.kling of p lates, 112 235, 247
o f p laies.. 92. 93. IOI o f 1beams. 295. 45 4. 455. 456
of sandwich plates. 17J-4, 182-3 of Lbe.tuns., 2:?3
of \'ibrati ng p la t'--S., 144. 149-50, 199--200 o f o penscctio n beams, 229, 2:31, 232. 23-1. 235.
S1Tcngth, 416, 417, 421. 42J 2-16
Strength parameter, 4 12. 4 13, 416. 423 of sandwich beams. 328
INDEX 479

Tcn.~ilcstirfncss (co111U. 11e,f) thin rectangular cross-section beams. 248-250


of stiffened bc:uns, 289 to rq ue . 253
ofTbc-<ims. 219, 240 to rq ue ne utral surrace, 254, 258
Tensorial suain. 3, 6 torsional stiffness. 249. i.'iO. 254
The rmal oonducti\ity. 446-7, olSO "''arping,. 2.'il-2
Thermal expansion oocfficicnt, 44, 46, 152. warping stiffness. 262-4
443- 4. 450 lransronn.ations.. 49-54. 69
lhicknes.s., change in. 161-3 Tran.cwerscl)' iso tropic material
Thin plntcs. Su Plates co1n pliancc, 12, 14-15. 20. 35. 42-3
Thin"-'alkd beams unde r axial load. bending coordinate system, 19
nnd torsion. 265-74 dei;c:ription. 19
centroid. 271. 274 engineering constan ts., 13, 16, 19
ccunpliancc. 266-8, 273-4 under plane.-strain. 34-5. 38
c<>ordinatc systcm. 265, 266. 267. 268 under plane-s tress., 42-.~
displaocmcnl, 26.'i-7, 274 Poisson's ratio, 13. 19
example. 272 shear modultl$., 13. 19
for<~e-suain rch1lions.hip, 266, 269, 271 stirfncss 20, 35, 42-3
rnte or twist. 274 Yo ungs n1odulus, 13, 19
restrained warping. 271 liaru;verscl)' loaded beami; with shear
segmented waU. 265 deformaijon .329-34
stiffnc~ 266, 267. 268 bending n1oment. 332
sirain-disp1J1ccn1cn1 relationship, 26.'i bending stifrness. 3.33
strain, 268-70, 274 deflectio n. 3.30-4
stre5S, 268--70 equilibrium . 329
Thrcc~din1c rui on.a l clcmcnl. 396 exampk , 334
limoshcnkobcam, 313 sandwich beams.3.31, 334
Torque. 20.,, 253, 261 . 262. 263-4 s hear ro rce. 331
Torque ncutrid surface, 254, 258 shear s tiffness.. 333
Torsional stiffness Transverse I)' loaded t hin-walled bean1s. 274-80
of bcnms. 206, 208, 210. 3!2 applied loads. 274
of box-hean1s. 2.'i6, 2.'i8. 458 :txial ro rce. 1TI
of C-bean1s. 286, 457 C-beams. 284-8
of circular cross:-seciion be.ams., 458 doscd-i;cction beams. muhiccll. 179-80
of closed-section bean1s. 2.'i4 <l~d-scction be.ams. i;ingJe.cell. 278-9.
of J-bca1ns. 29:'i. 45 7 282-3
of o pensec-tio n ~ams.. 250 eq uilibrimn. 276, 280
of sandwich bea1ni;. 329 examples, 284. 287
of solid stion beams., 249, 250 internal forces, 275-6
of Tbcams. 457 open-section beams. 276-8. 281
of beams in torsion. 249. 250. 254 rate of twist. 275-6. 279. 280
of Z.bean1s, 457 s hear center, 283-4. 275
Torsion of1hin\\1alled bean1s. 248-265 s hear Bow. 275-83
boundary cond itions.. 264 shear strain, 279
box-beams. 255-60, 272-4 lransvcrsc shear fo rce. See Shear force
dosed-section. mulliceU. 260-1 Tsai-\Vu railurc criterion. 413
closed-sectio n, s ingle. cell. 252-60 Twis1
co1npliancc. 2.'iO. 2.'i4 of box-bcan1s. 2.~7, 2.'i9. 274
curvature, 251 of C-beanlS~ 286--7. 288
exaLnpki;. 2.'i5, 25'7, 272 of solidsectio n bca~ 203
ro rce.-strain rel n tion.~ip. 250. 255 Twisl n1o mcnt. See f\.1o mcnts
lbenms, 261-4. 3 1 ~ 1 8
open-section bcan1s, 2.'i()-2 Unidirectional test, 4 17-LS
rate or twisi. 248. 249, 253. 254. 260
restrained warping. 261~ Vibratio n
shear Bow. 253. 260 of beams. 306--10
Wear strain, 251, 2.53-1, 3 14, ."\22 of benms wilh shear deronnation, 347-59
stiffened beams., 290 circular frequencies o r bean1s "''ilh shear
S>tress-s1rain relationship. 251 deforn1ation, 350 .352. 353, 359
480 INDEX

Vibration (co111i1111eti) \Varpins. 25 1-2


dcftcc1ion of \'ibrating plntcs. 145, 199 \Varp-ing stiffness
equilibrium of \ibrating be.ams. .353 of be.ams "';th shear deformation.
equilibriu1n of ~i brating pistes.. 142, 197 322
cxumplcs, I-beam. 3 10 .357 of box-bcan1s. 458
examples, plates. 143. 147. 148.. 150, 198, 201 of C-bcan1s. 288. 457
forcc--i:train rc-lntions.hip. be.am. 354 of c-ircuhlr cross-section be tum, 458
I-beams, of, Jltl-1 J. 357- 9 of I-beams, 261-4, 295. 318. 457
inertia force., 141. 348, 353 of sandwich bca1ns. 328
kine-tic energy of plate, 144-5 ofT-bc.inns. 457
of Ion~ plutcs. 14 1-1 of Z.bcan1s. 457
of long sandwich plates, 196-9 \Vhitncy-Nuis1ncr failure. criterion. 430-4
shear deformation effects. 359-62 \Vork done hy external forocSi 323
of simply supported plntc.s. 144-51 \\'oven fabric. I. 2. 41-2, 43-1
of si1nply supported sandwich pl.ntcs.. cxamplc. 43
199-20 1
strain energy of vibrating plate-Si 144. 149-50, Young's n1odulus, 13, 19. 20
199-200 m>cromcchanfos expressions, 438-9, 442.
Volume.. 436-7 448-9. 450. 451-2
Volume. frnction . 436-7
Von ~Uses failure criterion. 420. 422 Z-bcams. 457. 460

S-ar putea să vă placă și